Sie sind auf Seite 1von 442

P

ICAP
Practice Kit

Advanced accounting
and financial reporting
First edition published by
Emile Woolf Limited
Bracknell Enterprise & Innovation Hub
Ocean House, 12th Floor, The Ring
Bracknell, Berkshire, RG12 1AX United Kingdom
Email: info@ewiglobal.com
www.emilewoolf.com

Emile Woolf International, May 2016

All rights reserved. No part of this publication may be reproduced, stored in a retrieval
system, or transmitted, in any form or by any means, electronic, mechanical, photocopying,
recording, scanning or otherwise, without the prior permission in writing of Emile Woolf
Publishing Limited, or as expressly permitted by law, or under the terms agreed with the
appropriate reprographics rights organisation.

You must not circulate this book in any other binding or cover and you must impose the same
condition on any acquirer.

Notice
Emile Woolf International has made every effort to ensure that at the time of writing the
contents of this study text are accurate, but neither Emile Woolf International nor its directors
or employees shall be under any liability whatsoever for any inaccurate or misleading
information this work could contain.

Emile Woolf International ii The Institute of Chartered Accountants of Pakistan


Certified Finance and Accounting Professional
Advanced accounting and financial reporting

C
Contents
Page
Question and Answers Index v
Section A Questions 1
Section B Answers 161

Emile Woolf International iii The Institute of Chartered Accountants of Pakistan


Advanced accounting and financial reporting

Emile Woolf International iv The Institute of Chartered Accountants of Pakistan


Certified Finance and Accounting Professional
Advanced accounting and financial reporting

I
Index to questions and answers

Question Answer
page page
CHAPTER 1 REGULATORY FRAMEWORK
1.1 GENERAL PURPOSE FINANCIAL
1 161
STATEMENTS
CHAPTER 2 ACCOUNTING AND REPORTING CONCEPTS
2.1 DEFINITIONS 2 163
2.2 CONCEPTUAL FRAMEWORK 2 164
2.3 CARRIE 2 166
CHAPTER 3 PRESENTATION OF FINANCIAL STATEMENTS
3.1 CLIFTON PHARMA LIMITED 3 167
3.2 BSZ LIMITED 4 169
3.3 YASIR INDUSTRIES LIMITED 6 172
3.4 FIGS PAKISTAN LIMITED 7 175
3.5 FAZAL LIMITED 9 178
3.6 BABER LIMITED 9 179
3.7 GOLDEN LIMITED 10 179
3.8 METAL LIMITED 10 180
3.9 ENGINA 11 181
3.10 SHAZAD INDUSTRIES LTD 12 183
3.11 AZ 12 184
3.12 J-MART LIMITED 13 187
3.13 QALLAT INDUSTRIES LIMITED 14 188
3.14 SKYLINE LIMITED 15 189
3.15 WALNUT LIMITED 15 190

Emile Woolf International v The Institute of Chartered Accountants of Pakistan


Advanced accounting and financial reporting

Question Answer
page page
CHAPTER 4 IAS 8: ACCOUNTING POLICIES, CHANGES IN
ACCOUNTING ESTIMATES AND ERRORS
4.1 WONDER LIMITED 17 191
4.2 DUNCAN 17 193
4.3 MOHANI MANUFACTURING LIMITED 18 193
CHAPTER 5 IFRS 15: REVENUE FROM CONTRACT WITH
CUSTOMER
5.1 PARVEZ LIMITED 20 195
5.2 SACHAL LIMITED 20 196
5.3 BRILLIANT LIMITED 21 197
5.4 WAQAS LIMITED 21 199
5.5 ANABELLE 22 201
5.6 REAL CONSTRUCTION COMPANY LTD 23 202
5.7 GLADSTONE LTD 24 205
5.8 SILVER CONSTRUCTION LIMITED 24 207
5.9 XYZ LTD 25 209
5.10 ABC LTD 26 209
5.11 DX LTD 26 210
5.12 PL LTD 26 210
5.13 FX LTD 27 211
CHAPTER 6 IAS 16: PROPERTY, PLANT AND EQUIPMENT
6.1 FAM 28 212
6.2 GUJRAT CONSTRUCTION LIMITED 29 213
CHAPTER 7 NON-CURRENT ASSETS: SUNDRY STANDARDS
7.1 SPIN INDUSTRIES LIMITED 30 216
7.2 QURESHI STEEL LIMITED 30 217
7.3 IMRAN LIMITED 31 218
7.4 KATIE 32 219
7.5 ALNUS LTD AND BUTEA LIMITED 33 221
7.6 VICTORIA 33 222
CHAPTER 8 IAS 38: INTANGIBLE ASSETS
8.1 BROOKLYN 35 224
8.2 RAISIN INTERNATIONAL 35 225
8.3 OXTAIL LIMITED 36 226
8.4 SKY LIMITED 37 228
8.5 COMFORT SHOES LIMITED 38 229

Emile Woolf International vi The Institute of Chartered Accountants of Pakistan


Index to questions and answers

Question Answer
page page
CHAPTER 9 IAS 36: IMPAIRMENT OF ASSETS
9.1 CHARLOTTE 39 230
9.2 ABA LIMITED 39 232
9.3 HUSSAIN ASSOCIATES LTD 40 234
9.4 IMPS 41 235
CHAPTER 10 IFRS 5: NON-CURRENT ASSETS HELD FOR SALE AND
DISCONTINUED OPERATIONS
10.1 SAUL 43 237
10.2 SHAHID HOLDINGS 44 238
10.3 PRIMA 45 240
CHAPTER 11 IFRS 16: LEASES
11.1 X LTD 47 243
11.2 PROGRESS LIMITED 47 244
11.3 MIRACLE TEXTILE LIMITED 47 245
11.4 ACACIA LTD 48 246
11.5 SHOAIB LEASING LIMITED 48 248
11.6 FLOW 49 250
11.7 PINUS LIMITED 49 250
11.8 LODHI TEXTILE MILLS LIMITED 50 251
11.9 AUTO CONSTRUCTION PAKISTAN
50 252
LIMITED
CHAPTER 12 IAS 37: PROVISIONS CONTINGENT LIABILITIES AND
CONTINGENT ASSETS
12.1 ROWSLEY 51 254
12.2 MULTAN PETROCHEM LTD 52 255
12.3 VIOLET POWER LIMITED 53 256
CHAPTER 13 IAS 19: EMPLOYEE BENEFITS FLOWS
13.1 LABURNUM LIMITED 54 258
13.2 JABEL LIMITED 54 258
13.3 KAGHZI LIMITED 55 259
13.4 LASURA LTD 55 259
13.5 UNIVERSAL SOLUTIONS 55 260
13.6 DHA INTERIORS LTD 56 261
CHAPTER 14 IFRS 2: SHARE BASED PAYMENTS
14.1 TOSHACK LTD 58 264
14.2 IFRS 2 58 264
14.3 SAVAGE LTD 59 266
14.4 YORATH LTD 59 267

Emile Woolf International vii The Institute of Chartered Accountants of Pakistan


Advanced accounting and financial reporting

Question Answer
page page
14.5 QUALTECH LTD 59 267
14.6 BRIDGE LTD 60 268
14.7 CAPSTAN LTD 60 268
14.8 NEWTOWN LTD 61 269
14.9 SINDH TRANSIT LTD 61 270
CHAPTER 15 IFRS 9: FINANCIAL INSTRUMENTS: RECOGNITION
AND MEASUREMENT
15.1 AJI PANCA LTD 62 271
15.2 PASSILA LTD 62 272
15.3 FINANCIAL INSTRUMENTS 63 273
15.4 ESPANOLA LTD 63 275
15.5 SANDIA PL 64 275
15.6 GEO ALLOYS LTD 64 276
15.7 CASCABEL LTD 65 277
15.8 FAIR VALUE HEDGE ACCOUNTING 65 278
15.9 CASH FLOW HEDGE ACCOUNTING 66 279
15.10 WATERS LTD 66 280
15.11 ARIF INDUSTRIES LIMITED 67 282
15.12 QASMI INVESTMENT LIMITED 68 284
CHAPTER 16 IFRS 7: FINANCIAL INSTRUMENTS: PRESENTATION
AND DISCLOSURE
16.1 SERRANO LIMITED 69 286
16.2 POBLANO LIMITED 69 286
16.3 PIQUIN LTD 70 287
16.4 AJI LTD 70 288
16.5 CHILTEPIN LTD 71 289
16.6 HABENERO LTD 71 290
CHAPTER 17 IFRS 13: FAIR VALUE MEASUREMENT
There are no specific questions in this area.
CHAPTER 18 IAS 12: INCOME TAXES
18.1 SHAKIR INDUSTRIES 72 291
18.2 DWAYNE LTD (PART 1) 73 292
18.3 DWAYNE LTD (PART 2) 74 292
18.4 COHORT 74 293
18.5 MODEL TOWN GROUP 75 295
CHAPTER 19 BUSINESS COMBINATIONS AND CONSOLIDATION
19.1 HELLO 78 297

Emile Woolf International viii The Institute of Chartered Accountants of Pakistan


Index to questions and answers

Question Answer
page page
19.2 HASAN LIMITED 79 298
19.3 FLAMSTEED AND HALLEY LTD 81 301
19.4 BRADLEY LTD 82 302
19.5 X LTD 83 304
19.6 KHAN LIMITED 84 306
CHAPTER 20 CONSOLIDATED ACCOUNTS: STATEMENTS OF
PROFIT OR LOSS AND OTHER COMPREHENSIVE INCOME
20.1 MILLARD LTD 86 309
20.2 SHERLOCK LIMITED 87 311
20.3 FAISAL LIMITED 89 314
20.4 GOLDEN LIMITED 90 319
CHAPTER 21 IAS 28: ASSOCIATES AND JOINT VENTURES
21.1 JOINT ARRANGEMENTS 92 321
21.2 HELIUM 92 321
21.3 HAMACHI LTD 92 323
21.4 HIDE 94 325
21.5 HARK, SPARK AND ARK 94 326
21.6 P, S AND A 96 329
21.7 H LTD GROUP 97 331
CHAPTER 22 IFRS 3: BUSINESS COMBINATIONS ACHIEVED IN
STAGES
22.1 STEP ACQUISITION 99 333
22.2 A LTD 99 333
22.3 X LTD GROUP 101 336
22.4 PLAIN LTD 102 338
22.5 MANGO LTD 104 342
CHAPTER 23 COMPLEX GROUPS
23.1 PARVEZ LTD 106 345
23.2 HASAN, RIAZ AND SIDDIQ 107 349
CHAPTER 24 DISPOSAL OF SUBSIDIARIES
24.1 PATCHE LTD 109 353
24.2 DISPOSAL 110 356
24.3 PART DISPOSAL 110 356
24.4 THE A GROUP 111 357
24.5 BARTLETT 112 359
CHAPTER 25 OTHER GROUP STANDARDS
There are no specific questions in this area. The topic is covered as parts of
other questions.

Emile Woolf International ix The Institute of Chartered Accountants of Pakistan


Advanced accounting and financial reporting

Question Answer
page page
CHAPTER 26 IAS 21: FOREIGN CURRENCY
26.1 DND LIMITED 113 361
26.2 STARLIGHT LIMITED 113 361
26.3 PERCEPT LTD 114 362
26.4 ORLANDO 115 363
26.5 MANCASTER AND STOCKPOT 116 364
26.6 A, B AND C 118 367
26.7 OMEGA LIMITED 119 370
26.8 PARENT COMPANY LIMITED 120 371
CHAPTER 27 IAS 7: STATEMENTS OF CASH FLOWS
27.1 EVERNEW LTD 122 374
27.2 BELLA 123 376
27.3 BISHOP GROUP 125 377
27.4 THE GRAPE GROUP 127 380
CHAPTER 28 IAS 33: EARNINGS PER SHARE
28.1 AIRCON LTD 130 382
28.2 CACHET LTD 131 384
28.3 MARY 132 385
28.4 MANDY 132 385
28.5 AAZ LIMITED 132 387
28.6 ABC LIMITED 133 388
28.7 ALPHA LIMITED 134 391
CHAPTER 29 ANALYSIS AND INTERPRETATION OF FINANCIAL
STATEMENTS
29.1 ALPHA LIMITED AND OMEGA LIMITED 135 393
29.2 COOK LIMITED 136 394
29.3 FITZROY LIMITED 137 395
29.4 TRAVELWELL LTD 138 398
29.5 SACHAL LIMITED 140 400
29.6 OPAL INDUSTRIES LIMITED 141 402
CHAPTER 30 SUNDRY STANDARDS AND INTERPRETATIONS
29.1 GUJRANWALA FOODS LIMITED 143 405
29.2 WAH AGRIPROD LTD 144 406
29.3 HELIOS GROUP 145 408
29.4 FASHION BLUE ENTERPRISES 146 410
29.5 KHAN LIMITED 147 411
29.6 AFRIDI 148 412

Emile Woolf International x The Institute of Chartered Accountants of Pakistan


Index to questions and answers

Question Answer
page page
CHAPTER 31 IFRS 1: FIRST TIME ADOPTION OF IFRS
31.1 IFRS 1 149 413
CHAPTER 32 SPECIALISED FINANCIAL STATEMENTS
32.1 IFRS FOR SMES 150 415
32.2 AKMAL GENERAL INSURANCE LIMITED 150 416
32.3 MAHFOOZ GENERAL INSURANCE
151 417
LIMITED
32.4 DEE GENERAL INSURANCE LIMITED 152 418
32.5 BANK LATEEF BANK LIMITED 152 418
32.6 SECURED BANK LIMITED 153 419
32.7 AL-AMIN BANK LIMITED 153 420
32.8 BLUE-CHIP ASSET MANAGEMENT
154 420
LIMITED
32.9 A-ONE ASSET MANAGEMENT FUND
154 421
LIMITED
32.10 IAS 26 155 421
32.11 SOGO LIMITED 155 422
CHAPTER 33 INTERNATIONAL PUBLIC SECTOR ACCOUNTING
STANDARDS
There are no specific questions in this area.
CHAPTER 34 IAS 29: FINANCIAL REPORTING IN HYPERINFLATION
ECONOMIES
There are no specific questions in this area.
CHAPTER 35 ISLAMIC ACCOUNTING STANDARDS
There are no specific questions in this area.
CHAPTER 36 ETHICAL ISSUES IN FINANCIAL REPORTING
36.1 ETHICAL ISSUES 157 424
36.2 SINDH INDUSTRIES LTD 157 425
36.3 SOHAIB AND OMAR 159 428
36.4 ABBAS AND BASHIR 159 428

Emile Woolf International xi The Institute of Chartered Accountants of Pakistan


Advanced accounting and financial reporting

Emile Woolf International xii The Institute of Chartered Accountants of Pakistan


Certified Finance and Accounting Professional
Advanced accounting and financial reporting

SECTION
A
Questions
CHAPTER 1 REGULATORY FRAMEWORK

1.1 GENERAL PURPOSE FINANCIAL STATEMENTS


The IFRS are generally accepted as accounting standards in the preparation of
general purpose financial statements in many countries of the world.
Required
(a) Briefly explain the meaning of general purpose financial statements in
accordance with IAS 1 (Presentation of Financial Statements)
(b) Explain briefly any FOUR possible reasons for the prevalence of IFRS in
many countries of the world.
(c) Explain the arguments in support and against financial reporting standards.

Emile Woolf International 1 The Institute of Chartered Accountants of Pakistan


Advanced accounting and financial reporting

CHAPTER 2 ACCOUNTING AND REPORTING CONCEPTS

2.1 DEFINITIONS
A statement of financial position is a snapshot of a business at a point in time. It
shows the assets that an entity owns and the liabilities that it owes. This is all that
is required to convey a businesss performance, position and adaptability.
As income generated and expenses incurred by a business are already reflected
within the assets and liabilities shown in the statement of financial position, a
statement of profit or loss is a superfluous statement.
Required
Briefly appraise the validity of the above statement, defining the words
underlined.

2.2 CONCEPTUAL FRAMEWORK


(a) Explain the term Conceptual Framework in relation to International
Financial Reporting Standards (IFRS).
(b) Define assets and liabilities.
(c) The International Accounting Standards Boards framework for the
preparation of financial statements requires that entities should comply with
certain accounting concepts and underlying assumptions which include:
(i) Substance over form;
(ii) Materiality;
(iii) Comparability; and
(iv) Going concern.
Explain briefly the meaning of these concepts.
(d) Discuss the information needs of the following users of a companys
financial statements:
(i) Lenders;
(ii) Suppliers;
(iii) Customers ;
(iv) Employees; and
(v) Government and its agencies.

2.3 CARRIE
Carrie starts in business on 1 January Year 1. Carries sole shareholder contributed
capital of $1,000. Carrie purchased one item of inventory for $1,000 and sold that
inventory for cash of $1,400. At the end of Year 1 the replacement cost of the same
item of inventory is $1,100. General inflation during the year was 7%.
Required
Calculate the profit for the year and set out a summary statement of financial
position as of 31 December Year 1 under the following capital maintenance
concepts.
(a) Physical capital maintenance
(b) Financial capital maintenance
(i) Historical cost accounting
(ii) Constant purchasing power accounting

Emile Woolf International 2 The Institute of Chartered Accountants of Pakistan


Questions

CHAPTER 3: PRESENTATION OF FINANCIAL STATEMENTS

3.1 CLIFTON PHARMA LIMITED


The following trial balance relates to Clifton Pharma Limited, a public listed
company, at 30 September 2016.
Rs.in 000
Dr Cr
Cost of sales 134,000
Operating expenses 35,000
Loan interest paid (see note (1)) 1,500
Rental of vehicles (see note (2)) 8,600
Revenue 338,300
Investment income 2,000
Leasehold property at cost (see note (4)) 250,000
Plant and equipment at cost 197,000
Accumulated depreciation at 1 October 2015:
- leasehold property 40,000
- plant and equipment 47,000
Investments 92,400
Share capital 280,000
Share premium 20,000
Retained earnings at 1 October 2015 19,300
Loan notes (see note (1)) 50,000
Deferred tax balance at 1 October 2015 (see note (5)) 20,000
Inventory at 30 September 2016 23,700
Trade receivables 76,400
Trade payables 14,100
Bank 12,100
830,700 830,700
The following notes are relevant:
(1) The effective interest rate on the loan notes is 6% per year.
(2) There are two separate contracts for rental of vehicles. A recent review by
the finance department of these contracts has reached the conclusion that
Rs. 7 million of the total rental cost of vehicles relates to a finance lease
rather than an operating lease or rental arrangement.
The finance lease was entered into on 1 October 2015 which was when the
Rs. 7 million was paid: the lease agreement is for a four-year period in total,
and there will be three more annual payments in advance of Rs. 7 million,
payable on 1 October in each year. The vehicles in the finance lease
agreement had a fair value of Rs. 24 million at 1 October 2015 and they
should be depreciated using the straight line method to a nil residual value.
The interest rate implicit in the lease is 10% per year. The other contract for
vehicle rental is an operating lease and the rental payment should be
charged to operating expenses. (Note: You are not required to calculate
the present value of the minimum lease payments for the finance lease.)
(3) Other plant and equipment is depreciated at 20% per year by the reducing
balance method.

Emile Woolf International 3 The Institute of Chartered Accountants of Pakistan


Advanced accounting and financial reporting

All depreciation of property, plant and equipment should be charged to cost


of sales.
(4) The leasehold property has a 25-year life and is amortised at a straight-line
rate. On 30 September 2016 the leasehold property was re-valued to Rs.
220 million and the directors wish to incorporate this re-valuation in the
financial statements.
(5) The provision for income tax for the year ended 30 September 2016 has
been estimated at Rs. 18 million. At 30 September 2016 there are taxable
temporary differences of Rs. 92 million. The rate of income tax on profits is
25%.
Required
(a) Prepare a statement of profit or loss for Clifton Pharma Limited for the year
to 30 September 2016
(b) Prepare a statement of financial position for Clifton Pharma Limited as at 30
September 2016

3.2 BSZ LIMITED


The post closing trial balance of BSZ Limited, a listed company, as at June 30,
2016 is given below:
Debit Credit
Rs.in million
Cash at banks current accounts 7
Cash at banks in saving accounts 22
Stocks in trade closing 90
Accounts receivable 60
Provision for bad debts 3
Advances to suppliers 16
Advances to staff 6
Short term deposits 11
Prepayments 4
Sales tax receivable 12
Freehold land at revalued amount 375
Furniture and fixtures - cost 27
Accumulated depreciation Furniture and fixtures 8
Machines - cost 85
Accumulated depreciation Machines 27
Building on freehold land cost 150
Accumulated depreciation Building 26
Computer software cost 10
Accumulated amortization Computer software 2
Deferred taxation 40
Short term loan 85
Accounts payable 75
Accrued liabilities 7
Provision for taxation 17
Issued, subscribed and paid up capital (Rs. 10 each) 400
Surplus on revaluation of fixed assets 120
Accumulated profits 65

875 875

Emile Woolf International 4 The Institute of Chartered Accountants of Pakistan


Questions

Additional Information:
(i) The first revaluation of freehold land was carried out in 2012 and
resulted in a surplus of Rs. 120 million. The valuation was carried out
under market value basis by an independent valuer, Mr. Dee, Chartered
Civil Engineer of M/s SSS Consultants (Pvt.) Ltd., Islamabad.
(ii) The details relating to additions, disposal and depreciation/amortization of
fixed assets, during the year 2016 are given below:
The company uses the straight line method for charging depreciation
and amortization. The building is depreciated at a rate of 5%
whereas 10% is charged on machines, furniture and fixtures and
computer software.
Construction on third floor of the building commenced on March 1,
2016 and is expected to be completed on September 30, 2016. The
cost incurred during the year i.e. Rs. 20 million was capitalised on
June 30, 2016.
Furniture and fixtures worth Rs. 8 million were purchased on April 1,
2016.
A machine was sold on February 29, 2016 to NJ Enterprise at a price
of Rs. 13 million. At the time of disposal, the cost and written down
value of the machine was Rs. 15 million and Rs. 10 million
respectively.
(iii) 50% of the accounts receivable were secured and considered good.
10% of the unsecured accounts receivable were considered doubtful. Bad
debts expenses for the year amounted to Rs. 1.0 million. An amount of Rs.
1.4 million was written off during the year.
(iv) All advances given to suppliers are considered good and include an
amount of Rs. 4.0 million paid for goods which will be supplied on
December 31, 2017.
(v) Cash at banks in saving accounts carry interest / mark-up ranging from
3% to 7% per annum.
(vi) The authorised share capital of the company is Rs. 500 million.
Required
Prepare the statement of financial position as at June 30, 2016 along with the
relevant notes showing all possible disclosures as required under the
International Accounting Standards and the Companies Ordinance, 1984.
(Comparative figures and the note on accounting policies are not required.)

Emile Woolf International 5 The Institute of Chartered Accountants of Pakistan


Advanced accounting and financial reporting

3.3 YASIR INDUSTRIES LIMITED


The following trial balance related to Yasir Industries Limited (YIL) for the year
ended June 30, 2016:
Dr Cr
Rs.in million
Ordinary share capital (Rs. 10 each) - 120.00
Retained earnings - 10.20
Sales - 472.40
Purchases 175.70 -
Production labour 61.00
Manufacturing overheads 39.00
Inventories (July 1, 2015) 38.90
Administrative expenses 40.00 -
Distribution expenses 19.80 -
Financial charges 0.30 -
Cash and bank - 13.25
Trade creditors - 30.40
Accrued expenses - 16.20
10% redeemable preference shares - 40.00
Debentures - 80.00
Deferred tax (July 1, 2015) - 6.00
Suspense account 30.00 -
Leasehold property - at cost 230.00 -
Machines at cost 168.60 -
Software at cost 20.00 -
Acc. depreciation Leasehold property (June 30, 2016) - 40.25
Acc. depreciation Machines (June 30, 2016) - 48.60
Acc. amortization Software (June 30, 2016) - 12.00
Trade receivables 66.00 -
889.30 889.30
Additional Information:
(i) Sales include an amount of Rs. 27 million, made to a customer under sale
or return agreement. The sale has been made at cost plus 20% and the
expiry date for the return of these goods is July 31, 2016.
(ii) The value of inventories at June 30, 2016 was Rs. 42 million.
(iii) A fraud of Rs. 30 million was discovered in October 2015. A senior
employee of the company, who left in June 2015, had embezzled the funds
from YILs bank account. The chances of recovery are remote. The amount
is presently appearing in the suspense account.
(iv) On January 1, 2016 YIL issued debenture certificates which are repayable
in 2021. Interest is paid on these at 12% per annum.
(v) Financial charges comprise bank charges and bank commission.
(vi) The provision for current taxation for the year ended June 30, 2016 after
making all the above adjustments is estimated at Rs. 16.5 million.

Emile Woolf International 6 The Institute of Chartered Accountants of Pakistan


Questions

(vii) The carrying value of YILs net assets as on June 30, 2016 exceeds their
tax base by Rs. 30 million. The income tax rate applicable to the company is
30%.
(viii) On July 1, 2015, the leasehold property having a useful life of 40 years was
revalued at Rs. 238 million. No adjustment in this regard has been made in
the books.
(ix) Depreciation of leasehold property is charged using the straight line
method. 50% of depreciation is allocated to manufacturing, 30% to
administration and 20% to selling and distribution.
Required
In accordance with the requirements of the Companies Ordinance, 1984 and
International Accounting Standards, prepare the:
(a) statement of financial position as of June 30, 2016.
(b) statement of profit or loss and other comprehensive income for the year
ended June 30, 2016.
(Comparative figures and notes to the financial statements are not
required.)

3.4 FIGS PAKISTAN LIMITED


Figs Pakistan Limited is a listed company engaged in the business of
manufacturing and marketing of personal care and food products. Following is
an extract from its trial balance for the year ended 31 December 2016:
Debit Credit
Rs.in million
Sales - Manufactured goods 56,528
Sales - Imported goods 1,078
Scrap sales 16
Dividend income 12
Return on savings account 2
Sales tax - Imported goods 53
Sales tax - Manufactured goods 10,201
Sales discount 2,594
Raw material stock as on 1 January 2016 1,751
Work in process as on 1 January 2016 73
Finished goods (manufactured) as on 1 January 2016 1,210
Finished goods (imported) as on 1 January 2016 44
Purchases - Raw material 22,603
Purchases - Imported goods 658
Stores and spares consumed 180
Salaries, wages and benefits 2,367
Utilities 734
Depreciation and amortization 1,287
Stationery and office expenses 230
Repairs and maintenance 315
Advertisement and sales promotion 4,040
Outward freight and handling 1,279
Legal and professional charges 71
Auditor's remuneration 13

Emile Woolf International 7 The Institute of Chartered Accountants of Pakistan


Advanced accounting and financial reporting

Debit Credit
Rs.in million
Donations 34
Workers Profit Participation Fund 257
Worker Welfare Fund 98
Loss on disposal of property, plant and equipment 10
Financial charges on short term borrowings 133
Exchange loss 22
Financial charges on lease 11

Additional information:
(i) The position of inventories as at 31 December 2016 was as follows:
Rs. m
Raw material 2,125
Work in process 125
Finished goods (manufactured) 1,153
Finished goods (imported) 66
(ii) The basis of allocation of various expenses among cost of sales,
distribution costs and administrative expenses are as follows:
Cost of Distribution Administrative
sales costs expenses
% % %
Salaries, wages and benefits 55 30 15
Depreciation and amortization 70 20 10
Stationery and office expenses 25 40 35
Repairs and maintenance / Utilities 85 5 10
(iii) Salaries, wages and benefits include contributions to provident fund
(defined contribution plan) and gratuity fund (defined benefit plan)
amounting to Rs. 54 million and Rs. 44 million respectively.
(iv) Auditors remuneration includes taxation services and out-of-pocket
expenses amounting to Rs. 4 million and Rs. 1 million respectively.
(v) Donations include Rs. 5 million given to Dates Cancer Foundation (DCF).
One of the companys directors, Mr. Peanut is a trustee of DCF.
(vi) The tax charge for the current year after making all related adjustments is
estimated at Rs. 1,440 million. Taxable temporary differences of Rs. 3,120
originated in the year million, over the last year. The applicable income tax
rate is 35%.
(vii) 274 million ordinary shares were outstanding as on 31 December 2016.
(viii) There is no other comprehensive income for the year.
Required
Prepare the statement of profit or loss and other comprehensive income for the
year ended 31 December 2016 along with the relevant notes showing required
disclosures as per the Companies Ordinance, 1984 and International Financial
Reporting Standards. Comparatives are not required.

Emile Woolf International 8 The Institute of Chartered Accountants of Pakistan


Questions

3.5 FAZAL LIMITED


Fazal Limited is engaged in the manufacturing of specialized spare parts for
automobile assemblers. During the year 2016, the company has undertaken the
following transactions with its related parties:
(i) Sales of Rs. 500 million were made to its only subsidiary M/s Sami
Motors Limited (SML). Being the subsidiary, a special discount of Rs. 25
million was allowed to SML.
(ii) SML returned spare parts worth Rs. 5.5 million.
(iii) Raw materials of Rs. 5 million were purchased from Jalal Enterprises,
which is owned by the wife of the CFO of Fazal Limited.
(iv) Equipment worth Rs. 3 million was purchased from Khan Limited (KL). The
wife of the Production Director of the company is a director in KL.
(v) The company awarded a contract for supply of two machines amounting
to Rs. 7 million per machine to an associated company.
(vi) In 2014, an advance of Rs. 2 million was given to the Chief Executive
of the company. During the year 2016, he repaid Rs.0.3 million. The
balance outstanding as on December 31, 2016 was Rs. 1,100,000.
Required
Prepare a note for inclusion in the companys financial statements in accordance
with the requirement of IAS 24: Related Party Disclosures.

3.6 BABER LIMITED


During the year ended June 30, 2016, Baber Limited (BL) has carried out
several transactions with the following individuals/entities:
(i) AK Associates provides information technology services to BL. One of the
directors of BL is also the partner in AK Associates.
(ii) SS Bank Limited is the main lender. By virtue of an agreement it has
appointed a nominee director on the Board of BL.
(iii) Mr. Zee who supplies raw materials to BL, is the brother of the Chief
Executive Officer of the company.
(iv) JB Limited is the distributor of BLs products and has exclusive
distribution rights for the province of Punjab.
(v) Mr. Tee is the General Manager-Marketing of BL and is responsible for all
major decisions made in respect of sales prices and discounts.
(vi) BLs gratuity fund is administered by the Trustees appointed by the
company.
(vii) MM Limited is the leading supplier of BL and supplies 60% of BLs raw
materials.
(viii) Ms. Vee who conducted various training programmes for the employees
of the company, is the wife of BLs Chief Executive Officer.
Required
Comment as to whether the above individuals/entities are related parties of the
company or not. Support your arguments with references from International
Accounting Standards.

Emile Woolf International 9 The Institute of Chartered Accountants of Pakistan


Advanced accounting and financial reporting

3.7 GOLDEN LIMITED


The following related party transactions were carried out by Golden Limited
(GL) during the first year of its operation i.e. year ended December 31, 2016.
(i) Inventory costing Rs. 15 million was sold for Rs. 18 million to Platinum
Limited (PL) which owns 60% shares in GL. It is GLs policy to add 30%
margin on cost. Outstanding liability at year end, in respect of these
purchases was Rs. 6.5 million.
(ii) PL provided administrative services to GL. The cost of these services, if
billed in the open market, would have amounted to Rs. 350,000. No entries
were made to record these transactions, as it was agreed that the services
would be provided free of charge.
(iii) A property was sold to Silver Limited (SL), an associated company, at its
fair market value of Rs. 10 million. 50% of the amount was settled prior to
year end. GL reimbursed Rs. 500,000 to SL on account of transfer and
other incidental charges related to this property.
(iv) An interest free loan of Rs. 2 million was granted to an executive director of
the company under the terms of employment. During the year, Rs. 200,000
were repaid by the executive director.
(v) On July 1, 2016 GL obtained a short term loan of Rs. 25 million from one of
its major shareholder, at the prevailing annual interest rate of 12%. The
principal as well as the accrued mark-up were outstanding at the close of
the year.
Required
Prepare a note on related party transactions for inclusion in GLs financial
statements for the year ended December 31, 2016 showing disclosures as
required under IAS - 24 (Related Party Disclosures).

3.8 METAL LIMITED


On 1 July 2015, Metal Limited (ML) acquired 80% shareholdings in Copper
Limited (CL), 90% shareholdings in Zinc Limited (ZL) and 55% shareholdings in
Steel Limited (SL). The following transactions took place among these
companies, during the period up to 30 June 2016:
(i) On 1 May 2015, ML sold a machine to CL at 20% above the carrying
amount of Rs. 16 million. CL paid the entire amount on 15 July 2015. The
useful life of the machine is 10 years.
(ii) On 1 July 2015, ZL awarded a contract of Rs. 15 million to Iron Builders
and Developers (IBD) for the extension of its existing factory. One of the
directors of ML is also a partner in IBD.
(iii) Since the date of acquisition, ML has been providing management services
to CL and ZL. ML did not charge management fee for its services during the
first year. However, with effect from 1 July 2015, management fee has been
charged from each company at the rate of Rs.0.5 million per month.
Payment is made on the 10th day of the next month.
(iv) On 1 January 2016, ML sold goods amounting to Rs. 10 million to Gold
Limited (GL). The wife of chief financial officer of ZL is a major shareholder
in GL.
Required
Prepare a note on related party disclosure including comparative figures, for
inclusion in the individual financial statements of ML, CL, ZL and SL, for the year
ended 30 June 2016.

Emile Woolf International 10 The Institute of Chartered Accountants of Pakistan


Questions

3.9 ENGINA
Engina, a foreign company has approached a partner in your firm to assist in
obtaining local stock exchange listing (or stock market registration) for the
company. Engina is registered in a country where transactions between related
parties are considered to be normal but where such transactions are not
disclosed. The directors of Engina are reluctant to disclose the nature of their
related party transactions as they feel that although they are a normal feature of
business in their part of the world, it could cause significant problems politically
and culturally to disclose such transactions.
The partner in your firm has requested a list of all transactions with parties
connected with the company and the directors of Engina have produced the
following summary:
(a) Every month, Engina sells Rs. 50,000 of goods per month to Mr Satay, the
financial director. The financial director has set up a small retailing business
for his son and the goods are purchased at cost price for him. The annual
turnover of Engina is Rs. 300 million. Additionally, Mr Satay has purchased
his company car from the company for Rs. 45,000 (market value Rs.
80,000). The director, Mr Satay, earns a salary of Rs. 500,000 a year, and
has a personal fortune of many millions of pounds.
(b) A hotel property had been sold to a brother of Mr Soy, the Managing
Director of Engina, for Rs. 4 million (net of selling cost of Rs.0.2 million).
The market value of the property was Rs. 4.3 million but prices have been
falling rapidly. The carrying value of the hotel was Rs. 5 million and its value
in use was Rs. 3.6 million. There was an over-supply of hotel
accommodation due to government subsidies in an attempt to encourage
hotel development and the tourist industry.
(c) Mr Satay owns several companies and the structure of the group is outlined
below. Engina earns 60% of its profits from transactions with Car and 40%
of its profits from transactions with Wheel. All of the above companies are
incorporated in the same country.

Required
Write a report to the directors of Engina setting out the reasons why it is important
to disclose related party transactions and the nature of any disclosure required for
the above transactions under IAS 24 Related Party Disclosures.

Emile Woolf International 11 The Institute of Chartered Accountants of Pakistan


Advanced accounting and financial reporting

3.10 Shazad Industries Limited


Shazad Industries Ltd has recently acquired four large subsidiaries. These
subsidiaries manufacture products which are of different lines from those of the
parent company. The parent company manufactures plastics and related
products whereas the subsidiaries manufacture the following:
Product Location
Subsidiary 1 Textiles Karachi
Subsidiary 2 Car products Lahore
Subsidiary 3 Fashion garments Peshawar
Subsidiary 4 Furniture items Multan
The directors have purchased these subsidiaries in order to diversify their product
base but do not have any knowledge of the information required in the financial
statements regarding these subsidiaries other than the statutory requirements.
Required
(a) Explain to the directors the purpose of segmental reporting of financial
information.
(b) Explain to the directors the criteria which should be used to identify the
separate reportable segments. (You should illustrate your answer by
reference to the above information)
(c) Critically evaluate IFRS 8, Operating segments, setting out any problems
with the standard.

3.11 AZ
(a) For enterprises that are engaged in different businesses with differing risks
and opportunities, the usefulness of financial information concerning these
enterprises is greatly enhanced if it is supplemented by information on
individual business segments.
Required
(i) Explain why the information content of financial statements is
improved by the inclusion of segmental data on individual business
segments.
(ii) Discuss how IFRS 8 requires that segments be analysed. (10 marks)
(b) AZ, a public limited company, operates in the global marketplace.
(i) The major revenue-earning asset is a fleet of aircraft which are
registered locally and its other main source of revenue comes from
the sale of holidays. The directors are unsure as to how to identify
business segments.
(ii) The company also owns a small aircraft manufacturing plant which
supplies aircraft to its domestic airline and to third parties. The
preferred method for determining transfer prices for these aircraft
between the group companies is market price, but where the aircraft
is of a specialised nature with no equivalent market price, the
companies negotiate a price for the aircraft.
(iii) The company has incurred an exceptional loss on the sale of several
aircraft to a foreign government. This loss occurred on a fixed price

Emile Woolf International 12 The Institute of Chartered Accountants of Pakistan


Questions

contract signed several years ago for the sale of second hand aircraft
and resulted from the fluctuation of exchange rates between the two
countries.
(iv) During the year, the company decided to discontinue its holiday
business as a result of competition in the sector. This plan had been
approved by the board of directors and announced in the press.
(v) The company owns 40% of the ordinary shares of Eurocat, an
unquoted company which specialises in the manufacture of aircraft
engines and has operations in China and Russia. The investment is
accounted for by the equity method and it is proposed to exclude the
companys results from segment assets and revenue.
Required
Discuss the implications of each of the above points for the determination of
the segmental information required to be prepared and disclosed under
IFRS 8 Operating Segments and other relevant International Accounting
Standards.

3.12 J-MART LIMITED


(a) Explain the terms adjusting events and non-adjusting events and give
three examples of each.
(b) J-Mart Limited, a chain of departmental stores has distributed its
operations into four Divisions i.e. Food, Furniture, Clothing and Household
Appliances. The following information has been extracted from the records:
(i) The company allows the dissatisfied customers to return the goods
within 30 days. It is estimated that 5% of the sales made in June
2016 will be refunded in July 2016.
(ii) On June 2, 2016, three employees were seriously injured as a result
of a fire at the companys warehouse. They have lodged claims
seeking damages of Rs. 2.0 million from the company. The
companys lawyers have advised that it is probable that the court may
award compensation of Rs. 400,000.
(iii) Under a new legislation, the company is required to fit smoke
detectors at all the stores by December 31, 2016. The company has
not yet installed the smoke detectors.
(iv) On June 20, 2016, the board of directors decided to close down the
Household Appliances Division. However, the decision was made
public after June 30, 2016.
(v) The company has a large warehouse in Lahore which was acquired
under a three-year rent agreement signed on April 1, 2015. The
agreement is non- cancellable and the company cannot sub-let the
warehouse. However, due to operational difficulties, the company
shifted the warehouse to a new location.
(vi) A 15% cash dividend was declared on July 5, 2016.
Required
Describe how each of the above issue should be dealt with in the financial
statements for the year ended June 30, 2016. Support your point of view in
the light of relevant International Accounting Standards.

Emile Woolf International 13 The Institute of Chartered Accountants of Pakistan


Advanced accounting and financial reporting

3.13 QALLAT INDUSTRIES LIMITED


The following information pertains to Qallat Industries Limited (QIL) for its
financial year ended June 30, 2016:
(i) QIL sells all its products on one-year warranty which covers all types of
defects. Previous history indicates that 2% of the products contain major
defects whereas 10% have minor defects. It is estimated that if major
defects were detected in all the products sold, repair cost of Rs. 150
million would result. If minor defects were detected in all products sold,
repair cost of Rs. 70 million would result. Total sales for the year are
amounted to Rs. 830 million.
(ii) QIL has two large warehouses, A and B. These were acquired under
non-cancellable lease agreements. Details are as follows:

Warehouse A Warehouse B
Effective date of agreement July 1, 2011 January 1, 2014
Lease period 10 years 8 years
Rental amount per month Rs. 450,000 Rs. 300,000
On account of serious operating difficulties, QIL vacated both the
warehouses on January 1, 2016 and moved to a warehouse situated
close to its factory. On the same day QIL sub-let Warehouse A at Rs.
250,000 per month for the remaining lease period. Warehouse B was
sub-let on March 1, 2016 for Rs. 350,000 per month for the remaining lease
period.
(iii) On July 18, 2016, QIL was sued by an employee claiming damages for Rs.
6 million on account of an injury caused to him due to alleged violation of
safety regulations on the part of the company, while he was working on the
machine on June 15, 2016. Before filing the suit, he contacted the
management on June 29, 2016 and asked for compensation of Rs. 4 million
which was turned down by the management. The lawyer of the company
anticipates that the court may award compensation ranging between Rs.
1.5 million to Rs. 3 million. However, in his view the most probable amount
is Rs. 2 million.
(iv) On November 1, 2015 a new law was introduced requiring all factories to
install specialised safety equipment within four months. The Equipment
costing Rs. 5.0 million was ordered on December 15, 2015 against 100%
advance payment but the supplier delayed installation to July 31, 2016. On
August 5, 2016 the company received a notice from the authorities levying
a penalty of Rs.0.4 million i.e. Rs.0.1 million for each month during which
the violation continued. QIL has lodged a claim for recovery of the penalty
from the supplier of the equipment.
Required
Describe how each of the above issues should be dealt with in the financial
statements for the year ended June 30, 2016. Support your answer in the light of
relevant International Accounting Standards and quantify the effect where
possible.

Emile Woolf International 14 The Institute of Chartered Accountants of Pakistan


Questions

3.14 SKYLINE LIMITED


The following information pertains to Skyline Limited (SL) for the financial year
ended December 31, 2016:
(i) A customer who owed Rs. 1 million was declared bankrupt after his
warehouse was destroyed by fire on February 10, 2017. It is expected that
the customer would be able to recover 50% of the loss from the insurance
company.
(ii) An employee of SL forged the signatures of directors and made cash
withdrawals of Rs. 7.5 million from the bank. Of these, Rs. 1.5 million were
withdrawn before December 31, 2016. Investigations revealed that an
employee of the bank was also involved and therefore, under a settlement
arrangement, the bank paid 60% of the amount to SL on January 27, 2017.
(iii) SL has filed a claim against one of its vendors for supplying defective
goods. SLs legal consultant is confident that damages of Rs. 1 million
would be paid to SL. The supplier has already reimbursed the actual cost of
the defective goods.
(iv) A suit for infringement of patents, seeking damages of Rs. 2 million, was
filed by a third party. SLs legal consultant is of the opinion that an
unfavourable outcome is most likely. On the basis of past experience he
has advised that there is 60% probability that the amount of damages
would be Rs. 1 million and 40% likelihood that the amount would be Rs. 1.5
million.
Required
Advise SL about the amount of provision that should be incorporated and the
disclosures that are required to be made in the financial statements for the year
ended December 31, 2016.

3.15 WALNUT LIMITED


Walnut Limited (WL) is engaged in the business of import and distribution of
electronic appliances.
The following events took place subsequent to the reporting period i.e. 31
December 2016:
(i) On 15 January 2017, one of WLs competitors announced launching of an
upgraded version of DVD players. WLs inventories include a large stock of
existing version of DVD players which are valued at Rs. 15 million. Because
of the introduction of the upgraded version, the net realizable value of the
existing version in WLs inventory at 31 December 2016 has reduced to Rs.
12.5 million.
(ii) On 20 December 2016, the board of directors decided to close down the
division which imports and sells mobile sets. This decision was made public
on 29 December 2016. However, the business was actually closed on 29
February 2017.
Net costs incurred in connection with the closure of this division were as
follows:

Emile Woolf International 15 The Institute of Chartered Accountants of Pakistan


Advanced accounting and financial reporting

Rs. m

Redundancy costs 1.50

Staff training 0.15

Operating loss from 1 July 2016 to closure of division 0.80

Less: Profit on sale of remaining mobile sets (0.50)

1.95

(iii) On 16 January 2017, LED TV sets valuing Rs. 3 million were stolen from
a warehouse. These sets were included in WLs inventory as at 31
December 2016.
(iv) WL owns 9,000 shares of a listed company whose price as on 31
December 2016 was Rs. 22 per share. During February 2017, the share
price declined significantly after the government announced a new
legislation which would adversely affect the companys operations. No
provision in this regard has been made in the draft financial statements.
(v) On 31 January 2017, a customer announced voluntary liquidation. On 31
December 2016, this customer owed Rs. 1.5 million.
(vi) On 15 February 2017, WL announced final dividend for the year ended 31
December 2016 comprising 20% cash dividend and 10% bonus shares, for
its ordinary shareholders.
Required
Describe how each of the above transactions should be accounted for in the
financial statements of Walnut Limited for the year ended 31 December 2016.
Support your answer in the light of relevant International Financial Reporting
Standards.

Emile Woolf International 16 The Institute of Chartered Accountants of Pakistan


Questions

CHAPTER 4: IAS 8: ACCOUNTING POLICIES, CHANGES IN ACCOUNTING


ESTIMATES AND ERRORS

4.1 WONDER LIMITED


Wonder Limited (WL) is engaged in the manufacturing and sale of textile
machinery. Following are the draft extracts of the statement of financial position
and the statement of profit or loss for the year ended 30 June 2016:
Statement of Financial Position
2016 2015
Rs. m Rs. m
Property, plant and equipment 189 130
Retained earnings 166 108
Deferred tax liability 45 27
Statement of profit or loss
2016 2015
Rs. m Rs. m
Profit before taxation 90 120
Taxation 32 42
Profit after taxation 58 78
Following additional information has not been taken into account in the
preparation of the above financial statements:
(i) Cost of repairs amounting to Rs. 20 million was erroneously debited to
the machinery account on 1 October 2014. The estimated useful life of the
machine is 10 years.
(ii) On 1 July 2015, WL reviewed the estimated useful life of its plant and
revised it from 5 years to 8 years. The plant was purchased on 1 July 2014
at a cost of Rs. 70 million.
Depreciation is provided under the straight line method. Applicable tax rate is
30%.
Required
Prepare relevant extracts (including comparative figures) for the year ended 30
June 2016 related to the following:
(a) Statement of financial position
(b) Statement of profit or loss
(c) Statement of changes in equity
(d) Correction of error note

4.2 DUNCAN
Duncan Company has previously written off any expenditure on borrowing costs
in the period in which it was incurred.
The company has appointed new auditors this year. They have expressed the
view that the previous recognition of borrowing costs in the statement of profit or
loss was in error. The company has decided to correct the error retrospectively in
accordance with IAS 8.

Emile Woolf International 17 The Institute of Chartered Accountants of Pakistan


Advanced accounting and financial reporting

The financial statements for 2015 and the 2016 draft financial statements, both
reflecting the old policy, show the following.
Statement of changes in equity (extract)
2015 2016
Retained Retained
earnings earnings
Rs.000 Rs.000
Opening balance 22,500 23,950
Profit after tax for the period 3,200 4,712
Dividends paid (1,750) (2,500)

Closing balance 23,950 26,162

Borrowing costs written off were Rs. 500,000 in 2015 and Rs. 600,000 in 2016.
The directors have calculated that borrowing costs, net of depreciation which
should have been included in property, plant and equipment had the correct
policy been applied, are as follows.
Rs.000
At 30 December 2014 400
At 31 December 2015 450
At 31 December 2016 180
Had the correct policy been in force depreciation of Rs. 450,000 would have been
charged in 2015 and Rs. 870,000 in 2016.
Required
Show how the change in accounting policy must be reflected in the statement of
changes in equity for the year ended 31 December 2016. Work to the nearest
Rs.000.

4.3 MOHANI MANUFACTURING LIMITED


Mohani Manufacturing Limited is engaged in manufacturing of spare parts for
motor car assemblers. The audited financial statements for the year ended
December 31, 2015 disclosed that the profit and retained earnings were Rs. 21
million and Rs. 89 million respectively. The draft financial statements for the year
show a profit of Rs. 15 million. However, following adjustments are required to be
made:
(i) The management of the company has decided to change the method for
valuation of raw materials from FIFO to weighted average. The value of
inventory under each method is as follows:
FIFO Weighted Average
Rs. m Rs. m
December 31, 2014 37.0 35.5
December 31, 2015 42.3 44.5
December 31, 2016 58.4 54.4

Emile Woolf International 18 The Institute of Chartered Accountants of Pakistan


Questions

(ii) In 2015, the company purchased a plant for Rs. 100 million. Depreciation
on plant was recorded at Rs. 25 million instead of Rs. 10 million. This error
was discovered after the publication of financial statements for the year
ended December 31, 2015. The error is considered to be material.
Required
Produce an extract showing the movement in retained earnings, as would
appear in the statement of changes in equity for the year ended December 31,
2016.

Emile Woolf International 19 The Institute of Chartered Accountants of Pakistan


Advanced accounting and financial reporting

CHAPTER 5: IFRS 15: REVENUE FROM CONTRACT WITH CUSTOMER

5.1 PARVEZ LIMITED


The following transactions took place at Parvez Limited (PL).
(1) On 5 March 2017 PL sold goods to a bank for Rs.18m cash and agreed to
repurchase the goods for Rs.19m cash on 5 July 2017. The goods will be
shifted to a storage facility under banks control and security.
(2) On 31 March PLs car manufacturing division consigned several vehicles to
independent dealers for sale to third parties. The sales price to the dealer is
PLs list price at the date of sale to third parties. If a vehicle is unsold after
six months, the dealer has a right to return the vehicle to PL within next
fifteen days.
Required
Discuss how the above transactions should be accounted for in the books of
accounts of Parvez Limited.

5.2 SACHAL LIMITED


Sachal Limited (SL):
(a) Sells standard computer software package meant for small and medium
sized restaurant management. This software package is sold:
x at price of Rs. 1.5 million payable before delivery,
x with thirty days trial time, and
x without any maintenance support after trial time
As per practice, it takes around six months for the customers to use the
package independent of any support from SL. Practically, SL has to provide
on-site support service for at least six months to almost all customers free-
of-cost. However, in case of customers request for support beyond six
months, SL provides services under a formal paid service contract.
(b) Provides maintenance and support for the above standard software
package at a price of Rs. 0.3 million per annum.
(c) Provides designing and development of customized software to customers.
Payment is made monthly by customers on the basis of chargeable hours
of developers of SL. First year maintenance service is provided free-of-cost.
Subsequent maintenance service is provided at the rate of 10% of the total
contract price. Thereafter, for next three years maintenance service is
provided at 5% of the contract price per annum.
Required
Explain the considerations to be taken into account in determining accounting for
revenue by Sachal Limited.

Emile Woolf International 20 The Institute of Chartered Accountants of Pakistan


Questions

5.3 BRILLIANT LIMITED


Brilliant Limited (BL) manufactures and sells plastic card printing machines with
laminators. A machine-specific card printing software is provided as a must part
of the printing machine. BL also sells plastic cards imported from Thailand.
BL agreed to supply the following to, Proud Learners (PL), a country-wide school
network:
x 15 Card printing machines Available in ready stock
x 8 Laminators Would require 30 days to deliver
x 100,000 Plastic cards Available in ready stock
A lump sum price of Rs.9.2 million for the total contract has been agreed
between BL and school network.
Cost and list prices of the goods are:

Item Price (Rs.) Cost (Rs.)

Card printing machines 800,000 400,000

Laminators 200,000

Plastic cards 12 5

BL does not sell printing machine without laminator. However, in order to get this
order BL went against its policy. There is another supplier of imported card
printing machine of almost similar specification. This supplier sells the machine at
Rs.750,000.
In most recent customers surveys printing machine of BL has been given 7 out
of 10 points as against 9 out of 10 given to competitors imported machine. There
is no supplier of laminator in the market.
Required
Identify performance obligations and allocate the transaction price to the
identified performance obligations.

5.4 WAQAS LIMITED


Waqas Limited (WL) enters into a contract of construction of a reverse osmosis
plant for the manufacturing unit of Ali Chemical Limited (ACL) for Rs.20 million,
for which WL estimated cost is Rs.12 million. This included supply and installation
of plant and related construction work. The project is to be completed within 18
months. WL measures performance on the basis of cost incurred.

Emile Woolf International 21 The Institute of Chartered Accountants of Pakistan


Advanced accounting and financial reporting

At the end of seventh month ACL and WL agreed to modify the contract by
adding construction of an additional water reservoir at a price of Rs.2.5 million,
which will supply drinking water to a sister concern of ACL. The additional cost is
estimated as Rs.1.8 million by WL. At the end of seventh month WL incurred 4.2
million on the project.
At the end of tenth month ACL and WL agreed to modify the contract by
increasing the size of water reservoir that was included in the original design of
the project. ACL and WL agreed to an additional consideration of Rs.1 million, for
which WL will incur an additional cost of Rs.1 million. At the end of seventh month
WL incurred Rs. 7.2 million on the plant project and Rs. 0.72 million on additional
reservoir.
At the end of sixteenth month ACL and WL agreed to modify the contract by
adding pumping and piping facility from plant to the manufacturing unit of ACL for
a consideration of Rs.3 million. This facility was part of the project, but at the
inception this contract was awarded to another contractor, which was terminated
by ACL. The cost to be incurred by WL was estimated as Rs.2.8 million. At the
end of sixteenth month WL incurred Rs.11.7 million on the plant project and
Rs.1.35 million on additional reservoir.
Required
Advise how these transactions should be recognized in the books of Waqas
Limited.

5.5 ANABELLE
On 31 March Year 5 Anabelle had four construction contracts in progress. Details
are set out below.

Contract Contract Contract Contract


A B C D

Rs.000 Rs.000 Rs.000 Rs.000

Contract price 1,850 750 960 800

Costs to date 1,490 590 405 120

Estimated future costs - 25 600 480

Work certified at 31 March Year 5 1,850 695 480 120

Profits taken in earlier years 190 - - -

Revenue taken in earlier years 990 100 - -

Progress billings to date 1,850 690 650 100

Cash received to date 1,850 600 600 100

Emile Woolf International 22 The Institute of Chartered Accountants of Pakistan


Questions

All contracts are expected to take more than 12 months to complete. Contract A
was completed during the year. Contract C commenced on 1 May Year 4.
Contract D commenced on 1 January Year 5. It is not considered possible on 31
March Year 5 to assess the outcome of Contract D with any certainty.
Anabelle recognises revenue in respect of construction contracts on the
percentage of completion basis, based on the proportion of total costs incurred to
date.
Required
Show how the above would be included or disclosed in the financial statements of
Annabelle for the year ended 31 March Year 5. Work to the nearest Rs.000.

5.6 REAL CONSTRUCTION COMPANY LTD


Real Construction Company Ltd. is a major construction company.
The company is half way through a contract to build a new bridge at a contract
price of Rs. 300 million.
Progress report on this contract as at 1 April 2015 is as shown below:

Rs. million

Cumulative sales revenue recognised 150

Cumulative cost of sales to date 112

Profit to date 38

The following information has been extracted from the accounting records as at
31 March 2016:

Rs. million

Total progress payment received for work

certified as at 29 February 2016 180

Total costs incurred to date (excluding rectification costs below) 195

Rectification costs 17

Real Construction Company Ltd. had received progress payments of 90% of the
work certified as at 29 February 2016. The Company surveyor estimated that the
value of the further work to be completed during March 2016 would be Rs. 20
million.
At 31 March 2016, the estimated cost of the uncompleted contract was put at Rs.
45 million.

Emile Woolf International 23 The Institute of Chartered Accountants of Pakistan


Advanced accounting and financial reporting

The rectification costs were the costs incurred in widening the pedestrian access
roads to the bridge. This was as a result of an error by companys architect when
he made his initial drawings.
The Company calculates the percentage of completion of its contracts as the
proportion of value earned to date compared to the contract price.
All estimates can be taken as reliable.
Required
(a) Briefly explain the principles underlying each of the two methods of
recognising revenue and describe the circumstances in which their uses
are appropriate.
(b) Prepare extract of the financial statements for the contract for the year
ended 31 March 2016.

5.7 GLADSTONE LTD


Gladstone Ltd, a company commenced a four year contract early in 2013. The
price was initially agreed at Rs. 12,000,000.
Profit, which was reasonably foreseeable from the year ended 31 December
2013 is to be taken on a costs basis, and revenue is to be taken on a consistent
basis.
Relevant figures are as follows.

2013 2014 2015 2016

Rs.000 Rs.000 Rs.000 Rs.000

Costs incurred in year 2,750 3,000 4,200 1,150

Anticipated future costs 7,750 7,750 1,550

Work certified and invoiced to date 3,000 5,000 11,000 12,500

Required
Show how the above would be disclosed in the statement of profit or loss and
statement of financial position of Gladstone Ltd for each of the four years ended
31 December 2016.
Note Work to the nearest Rs.000.

5.8 SILVER CONSTRUCTION LIMITED


Silver Construction Limited (SCL) was incorporated on July 1, 2015 with a share
capital of Rs. 500 million. It is involved in the construction of bridges, dams,
pipelines, roads etc. During the year ended June 30, 2016, the company
commenced work on six contracts, details of which are as follows:

Emile Woolf International 24 The Institute of Chartered Accountants of Pakistan


Questions

CONTRACTS

I II III IV V VI

Rupees in million

Total contract price 300 375 280 400 270 1,200

Billing up to June 30, 2016 200 110 280 235 205 1,200

Contract cost incurred up to June 30, 2016 248 68 186 246 185 1,175

Estimated further cost to complete 67 221 - 164 15 -

Following additional information is available:


(i) As per terms of Contract IV, the company will receive an additional Rs. 40
million if the construction is completed within a period of twelve months
from the commencement of the contract. The management feels that there
is a 90% probability that it will be able to meet the target.
(ii) An amount of Rs. 16 million was incurred on Contract II on account of a
change in design. The company has discussed it with the customer who
has informed SCL that the amount is on the higher side and needs to be
revised.
Required
(a) Make relevant calculations and prepare appropriate extracts to be
reflected in the statement of financial position and statement of profit or
loss for the year ended June 30, 2016.
(b) Justify your accounting treatment in respect of the additional information
provided above.

5.9 XYZ LTD


XYZ Ltd completes commercial construction projects. These projects generally
take a significant amount of time to complete, ranging from twothree years.
During this time, the customer specifications for the contract may change.
XYZ Ltd has entered into a contract with a customer to build a new 10-storey
office building, expected to take two and a half years to complete. This contract is
for a fixed fee of Rs.25 million and has been determined to be a single
performance obligation.
At the end of year one, when XYZ Ltd determines that it has satisfied 45% of its
performance obligation, the customer requests a change order because it wants
to be compliant with a new environmental standard and therefore will need to
change (among other specifications) all the windows. The total cost to the
customer for ensuring the building is compliant with this new standard will be an
additional Rs.5 million, and the cost to XYZ Ltd will be Rs.4.5 million.

Emile Woolf International 25 The Institute of Chartered Accountants of Pakistan


Advanced accounting and financial reporting

Required
Compute the revenue recognized under original and modified contract for the
year ended?

5.10 ABC LTD


ABC Ltd promises to sell 120 products to a customer for Rs.12,000 (Rs.100 per
product). The products are transferred to the customer over a six-month period.
The entity transfers control of each product at a point in time. After the entity has
transferred control of 60 products to the customer, the contract is modified to
require the delivery of an additional 30 products (a total of 150 identical products)
to the customer. The additional 30 products were not included in the initial
contract
Required:
Compute the amount to be recognized as Revenue when:
x The price of additional products reflects the stand-alone selling?
x The price of additional products does not reflect the stand-alone selling
price?

5.11 DX LTD
DX Ltd owns and operates radio stations. The main revenue stream is
advertising revenue. Contracts are signed with various businesses for the sale of
airtime. The account executives obtain these contracts and are compensated
through a 5% commission on the total contract price for each new contract
signed.
Executive B has obtained a new two-year advertising contract with Company.
Total contract costs related to this contract are as follows:
Legal fees contract drafting = Rs.10,000
Commission paid to the account executive = Rs.7,500
Meals and entertainment incurred during the sales process Rs. 1,750
Creative Directors time salary allocation of Creative Director to develop on-air ad
= Rs.1,500
Actors amounts paid to external actors to record the on-air ad = Rs.750
Total Costs = Rs.21,500
Required
Discuss whether to capitalize or expense out each cost component?

5.12 PL LTD
PL Ltd sells a Rs.100 gift card to a customer, with no expiry date. The entity
maintains statistics on its gift card sales and redemptions and has noted that
historically only 80% of gift cards are redeemed and 20% are unexercised.

Emile Woolf International 26 The Institute of Chartered Accountants of Pakistan


Questions

The entity believes this historical information has appropriate predictive value and
estimates that 20% of the value of the gift cards will not be redeemed. In addition,
the entity applies the guidance on constraining variable consideration and
determines it is highly probable a significant revenue reversal will not occur once
these breakage amounts are recorded in revenue (i.e., using an appropriate
pattern of recognition). Therefore, it is appropriate to recognize these amounts as
breakage in revenue.
Required:
The amount by which the gift cards to be recorded?

5.13 FX LTD
A customer enters into a contract with a heavy duty machine manufacturer for the
purchase of a tractor for Rs.10 million. All pieces of equipment sold by the
manufacturer come with a one-year standard warranty that specifies the
equipment will comply with the agreed-upon specifications and will operate as
promised for a one-year period from the date of purchase.
In signing this contract, the customer also requested to purchase an additional
Rs.200,000 two-year warranty commencing after the expiry of the standard one
year warranty.
Required:
Explain how would you treat the warranty?

Emile Woolf International 27 The Institute of Chartered Accountants of Pakistan


Advanced accounting and financial reporting

CHAPTER 6: IAS 16: PROPERTY, PLANT AND EQUIPMENT

6.1 FAM
Fam had the following tangible fixed assets at 31 December 2015.
Cost Depreciation NBV
Rs.000 Rs.000 Rs.000
Land 500 500
Buildings 400 80 320
Plant and machinery 1,613 458 1,155
Fixtures and fittings 390 140 250
Assets under construction 91 91

2,994 678 2,316

In the year ended 31 December 2016 the following transactions occur.
(1) Further costs of Rs. 53,000 are incurred on buildings being constructed by
the company. A building costing Rs. 100,000 is completed during the year.
(2) A deposit of Rs. 20,000 is paid for a new computer system which is
undelivered at the year end.
(3) Additions to plant are Rs. 154,000.
(4) Additions to fixtures, excluding the deposit on the new computer system,
are Rs. 40,000.
(5) The following assets are sold.
Cost Depreciation Proceeds
brought forward
Rs.000 Rs.000 Rs.000
Plant 277 195 86
Fixtures 41 31 2
(6) Land and buildings were revalued at 1 January 2016 to Rs. 1,500,000, of
which land is worth Rs. 900,000. The revaluation was performed by
Jackson & Co, Chartered Surveyors, on the basis of existing use value on
the open market.
(7) The useful economic life of the buildings is unchanged. The buildings were
purchased ten years before the revaluation.
(8) Depreciation is provided on all assets in use at the year-end at the following
rates.
Buildings 2% per annum straight line
Plant 20% per annum straight line
Fixtures 25% per annum reducing balance
Required
Show the disclosure under IAS 16 in relation to fixed assets in the notes to the
published accounts for the year ended 31 December 2016.

Emile Woolf International 28 The Institute of Chartered Accountants of Pakistan


Questions

6.2 GUJRAT CONSTRUCTION LIMITED


Gujrat Construction Limited (GCL) accounts for non-current assets using the cost
model except for land and buildings for which it has adopted the revaluation
model. GCL makes an annual transfer between the revaluation surplus and
retained earnings each year to reflect the realisation of the revaluation reserve.
The companys financial statements include the following balances for property,
plant and equipment for the year ended 31 December 2015:
Cost/valuation Accumulated depreciation
Rs. Rs.
Land and buildings 1,500,000 315,000
Plant and machinery 1,276,500 879,300
Office equipment 356,400 210,400
Additional information
(1) Land is included in the above at a valuation of Rs. 850,000.
(2) Items of property, plant and equipment are depreciated on a straight-line
basis as follows:
Buildings over 50 years
Plant and machinery at a rate of 25% per annum on cost
Office equipment at a rate of 20% per annum on cost
Land is not depreciated.
(2) On 30 June 2016 GCL purchased plant at a cost of Rs. 135,000 and office
equipment for Rs. 36,500. On 30 September 2016 it sold, for a profit,
machinery which had cost Rs. 104,000 on 1 January 2014 and was
classified as held for sale on 30 June 2016. For the first time GCL utilised
spare capacity in its workshops and started to self-construct a specialised
turning machine for its own use. This machine was almost complete by 31
December 2016. Costs incurred on the machine to 31 December 2016
amounted to Rs. 29,200.
(3) During December 2016 GCL's the foundations of one of the company's
warehouses were found to be insufficient to support some of the machinery
that had been housed there, although this machinery had subsequently
been moved to another site. This workshop had cost Rs. 150,000 on 1
January 2012 and had been revalued to Rs. 210,000 on 31 December
2013. It is now estimated that its fair value is only Rs. 100,000 and that
costs to sell would be Rs. 5,000. Its value in use has been estimated at Rs.
90,000.
(4) On 1 November 2016 the directors decided to sell an item of machinery. At
that date a buyer had been identified and contracts were on the point of
being exchanged, at an agreed price of Rs. 11,000. Selling costs were
expected to be Rs. 1,500. This machine had cost Rs. 50,000 on 1 May
2014.
Required
Prepare the note showing the movements on property, plant and equipment,
including accumulated depreciation, which would be included in the financial
statements of GCL for the year ended 31 December 2016.

Emile Woolf International 29 The Institute of Chartered Accountants of Pakistan


Advanced accounting and financial reporting

CHAPTER 7 NON-CURRENT ASSETS: SUNDRY STANDARDS

7.1 SPIN INDUSTRIES LIMITED


On September 1, 2015, Spin Industries Limited (SIL) started construction of its
new office building and completed it on May 31, 2016. The payments made to the
contractor were as follows:

Date of Payment Rs.

September 1, 2015 10,000,000

December 1, 2015 15,000,000

February 1, 2016 12,000,000

June 1, 2016 9,000,000

In addition to the above payments, SIL paid a fee of Rs. 8 million on September 1,
2015 for obtaining a permit allowing the construction of the building.
The project was financed through the following sources:
(i) On August 1, 2015 a medium term loan of Rs. 25 million was obtained
specifically for the construction of the building. The loan carried mark up of
12% per annum payable semi-annually. A commitment fee @ 0.5% of the
amount of loan was charged by the bank.
Surplus funds were invested in savings account @ 8% per annum. On
February 1, 2016 SIL paid the six monthly interest plus Rs. 5 million towards
the principal.
(ii) Existing running finance facilities of SIL
Running finance facility of Rs. 28 million from Bank A carrying mark
up of 13% payable annually. The average outstanding balance
during the period of construction was Rs. 25 million.
Running finance facility of Rs. 25 million from Bank B. The mark up
accrued during the period of construction was Rs. 3 million and the
average running finance balance during that period was Rs. 20
million.
Required
Calculate the amount of borrowing costs to be capitalised on June 30, 2016 in
accordance with the requirements of International Accounting Standards.
(Borrowing cost calculations should be based on number of months).

7.2 QURESHI STEEL LIMITED


On July 1, 2015, Qureshi Steel Limited (QSL) signed an agreement with Pak
Construction Limited for construction of a factory building at a cost of Rs. 100
million. It was agreed that the factory would be ready for use from January 1,
2017. The terms of payments were agreed as under:
(i) 10% advance payment would be made on signing of the agreement. The
advance paid would be adjusted at 10% of the quarterly progress bills.

Emile Woolf International 30 The Institute of Chartered Accountants of Pakistan


Questions

(ii) 5% retention money would also be deducted from the progress bills.
Retention money will be refunded one year after completion of the factory
building.
(iii) Progress bills will be raised on last day of each quarter and settled on 15th of
the next month.
The under mentioned progress bills were received and settled by QSL as per the
agreement:

Invoice date Amount (Rs.)


September 30, 2015 30 million
December 31, 2015 20 million
March 31, 2016 10 million
June 30, 2016 15 million

On April 30, 2016 an invoice of Rs. 1.5 million was raised by the contractor for
damages sustained at the site, on account of rains. After negotiations, QSL
finally agreed to make additional payment of Rs. 1.0 million to compensate the
contractor. The amount was paid on May 15, 2016. It is expected that 75% of the
payment would be recovered from the insurance company.
The cost of the project has been financed through the following sources:
(i) Issue of right shares amounting to Rs. 15 million, on September 1, 2015.
The company has been following a policy of paying dividend of 20% for the
past many years.
(ii) Bank loan of Rs. 25 million obtained on December 1, 2015. The loan carries
a markup of 13% per annum. The principal is repayable in 5 half yearly
equal instalments of Rs. 5 million each along with the interest, commencing
from May 31, 2016. Loan processing charges of Rs.0.5 million were
deducted by the bank at the time of disbursement of loan. Surplus funds,
when available, were invested in short term deposits at 8% per annum.
(iii) Cash withdrawals from the existing running finance facility provided by
a bank. Average running finance balance for the year was Rs. 60 million.
Markup charged by the bank for the year was Rs. 9 million.
Required
Compute cost of capital work in progress for the factory building as of June 30,
2016 in accordance with the requirements of relevant IFRSs.
(Borrowing costs calculations should be based on number of months) (18 marks)

7.3 IMRAN LIMITED


On January 1, 2016, Imran Limited started the construction of its new
factory. The construction period is approximately 15 months and the cost is
estimated at Rs. 80 million. The work has been divided into 5 phases and
payment to contractor shall be made on completion of each phase.
In the year the company had the following sources of finance available.
(i) Rights i s s u e o f shares amounting to Rs. 15 million on January 1,
2016. The company usually pays a dividend of 10% each year.

Emile Woolf International 31 The Institute of Chartered Accountants of Pakistan


Advanced accounting and financial reporting

(ii) Bank loan of Rs. 32 million carrying a mark-up of 13% was raised on
March 1, 2016. (This loan was outstanding for 306 days in the year).
(iii) On August 1, 2016, Rs. 10 million were borrowed from the bank. Interest
thereon, is payable at the rate of 11%. (This loan was outstanding for 153
days in the year).
Investment income on temporary investment of the borrowings amounted to
Rs.0.5 million.
The details of bills submitted by the contractor, during the year are as follows:
Particulars Date of payment Rs.
st
On completion of 1 phase March 1, 2016 20,000,000
nd
On completion of 2 phase April 1, 2016 18,000,000
rd
On completion of 3 phase October 1, 2016 16,000,000
th
On completion of 4 phase Payment not yet made 17,000,000
On June 1, 2016, the Building Control Authority issued instructions for stoppage
of work on account of certain discrepancies in the completion plan. The company
filed a petition in the Court and the matter was decided in the companys favour
on July 31, 2016. Work recommenced after a delay of 61 days.
The following periods may be relevant:
Period Days
March 1 to December 31 306
April 1 to December 31 275
August 1 to December 31 153
October 1 to December 31 92
Required
a) Assuming that the loans were taken specifically for the project, calculate
the amount of borrowing costs that s h o u l d be capitalised i n t h e
p e r i o d e n d i n g December 31, 2016 in accordance with the requirements
of IAS 23 Borrowing Costs.
b) Assuming that the loans constituted general finance, calculate the amount
of borrowing costs that s h o u l d be capitalised i n t h e p e r i o d e n d i n g
December 31, 2016 in accordance with the requirements of IAS 23
Borrowing Costs.

7.4 KATIE
During the year ended 30 June Year 2, Katie received three grants, the details of
which are set out below.
(1) On 1 September, a grant of Rs. 40,000 from local government. This grant
was in respect of training costs of Rs. 70,000 which Katie had incurred.
(2) On 1 November Katie bought a machine for Rs. 350,000. A grant of Rs.
100,000 was received from central government in respect of this purchase.
The machine, which has a residual value of Rs. 50,000, is depreciated on a
straight-line basis over its useful life of five years.
(3) On 1 June a grant of Rs. 100,000 from local government. This grant was in
respect of relocation costs that Katie had incurred moving part of its

Emile Woolf International 32 The Institute of Chartered Accountants of Pakistan


Questions

business from outside the local area. The grant is repayable in full unless
Katie recruits ten employees locally by the end of Year 2. Katie is finding it
difficult to recruit as the local skill base does not match the needs of this
part of the business.
Required
Show how the above transactions should be reflected in the financial statements
of Katie for the year ended 30 June Year 2. Where any accounting standards
allow a choice you should show all possible options.

7.5 ALNUS LIMITED AND BUTEA LIMITED


Alnus Limited owns 80% of Butea Limited
On 1 December 2016 Alnus Ltd obtained a Rs. 200,000 government grant,
representing 50% of the cost of a depreciating asset which was acquired on 1
October 2016 for a total cost of Rs. 400,000. The asset has a four-year useful life
with no residual value, and depreciation of Rs. 75,000 has been charged in the
year to 30 June 2017. The draft financial statements show the Rs. 200,000 grant
as income in the year.
During the year ended 30 June 2017, Butea Ltd commenced redevelopment of
one of its holiday parks, which was in a state of disrepair and not in use.
Discussions with architects started on 1 October 2016, and work started on
successfully obtaining planning permission on 1 November 2016. The first
expenditure relating to the redevelopment was incurred on 1 January 2017. The
construction work was completed on 30 June 2017, and this was funded out of
Butea Ltds existing loan finance.
The following loan finance was in place during the year:
Rs. 1.8 million of loan finance paying 5% pa interest
Rs. 1.2 million of loan finance paying 8.5% pa interest
Neither loan was taken out to finance a specific purpose or building. The total
cost of construction work was Rs. 800,000 and the directors capitalised Rs.
51,000 of finance costs (being nine months of interest on construction costs of
Rs. 800,000 at 8.5% pa).
Required
Explain the correct IFRS accounting treatment for the above transactions and
explain how Buteas transaction would be reflected in the consolidated financial
statements.

7.6 VICTORIA
Victoria owns several properties and has a year end of 31 December. Wherever
possible, Victoria carries investment properties under the fair value model.
Property 1 was acquired on 1 January Year 1. It had a cost of Rs. 1 million,
comprising Rs. 500,000 for land and Rs. 500,000 for buildings. The buildings
have a useful life of 40 years. Victoria uses this property as its head office.
Property 2 was acquired many years ago for Rs. 1.5 million for its investment
potential. On 31 December Year 7 it had a fair value of Rs. 2.3 million. By 31
December Year 8 its fair value had risen to Rs. 2.7 million. This property has a
useful life of 40 years.

Emile Woolf International 33 The Institute of Chartered Accountants of Pakistan


Advanced accounting and financial reporting

Property 3 was acquired on 30 June Year 2 for Rs. 2 million for its investment
potential. The directors believe that the fair value of this property was Rs. 3
million on 31 December Year 7 and Rs. 3.5 million on 31 December Year 8.
However, due to the specialised nature of this property, these figures cannot be
corroborated. This property has a useful life of 50 years.
Required
(a) For each of the above properties briefly state how it would be treated in the
financial statements of Victoria for the year ended 31 December Year 8,
identifying any impact on profit or loss.
(b) Produce an analysis of property, plant and equipment for Victoria for the
year ended 31 December Year 8, showing each of the above properties
separately.

Emile Woolf International 34 The Institute of Chartered Accountants of Pakistan


Questions

CHAPTER 8 IAS 38: INTANGIBLE ASSETS

8.1 BROOKLYN
Brooklyn is a bio-technology company performing research for pharmaceutical
companies. The finance director has contacted your financial consulting company
to arrange a meeting to discuss issues relevant to the preparation of the financial
statements for the year to 30th June 2016. Your initial telephone conversation
has provided the necessary background information.
1 On 1st August 2015 Brooklyn began investigating a new bio-process. On 1st
September 2016, the new process was widely supported by the scientific
community and the feasibility project was approved. A grant was then
obtained relating to future work. Several pharmaceutical companies have
expressed an interest in buying the know how when the project completes
in June 2017. The nominal ledger account set up for the project shows that
the expenditure incurred between 1st August 2015 and 30th June 2016 was
Rs. 300,000 per month.
2 In August 2016, an employee lodged a legal claim against the company for
damage to his health as a result of working for the company for the two
years through to 31st March 2015 when he had to retire due to ill health. He
has argued that his health deteriorated as a result of the stress from his
position in the organisation. Brooklyn has denied the claim and has
appointed an employment lawyer to assist with contesting the case. The
lawyer has advised that there is a 25% chance that the claim will be
rejected, 50% chance that the damages will be Rs. 600,000 and 25%
chance of Rs. 1 million. The company has an insurance policy that will pay
10% of any damages to the company. The lawyer has said that the case
could take until 30th June 2019 to resolve. The present value of the
estimated damages discounted at 8% is Rs. 476,280 and Rs. 793,800
respectively.
3 Brooklyn owns several buildings, which include an administrative office in
the centre of London. The company has revalued these on a regular basis
every five years and the next valuation is due on 30th June 2018. Property
prices have increased since the last review and particularly for the London
premises. The cost of engaging a professionally qualified valuer is very
expensive and so to reduce costs the finance director is proposing that the
property manager, who is a professionally qualified valuer, should value the
London property and that the increase in value should be included in the
financial statements. The finance director is of the opinion that the property
prices may fall next year.
Required
Prepare notes for your meeting with the finance director which explain and justify
the accounting treatment of these issues, preparing calculations where
appropriate and identifying matters on which your require further information.

8.2 RAISIN INTERNATIONAL


(a) Discuss the criteria that should be used while recognizing intangible assets
arising from research and development work.
(b) Raisin International (RI) is planning to expand its line of products. The
related information for the year ended 31 December 2016 is as follows:

Emile Woolf International 35 The Institute of Chartered Accountants of Pakistan


Advanced accounting and financial reporting

(i) Research and development of a new product commenced on 1


January 2016. On 1 October 2016, the recognition criteria for
capitalization of an internally generated intangible asset were met. It
is estimated that the product would have a useful life of 7 years.
Details of expenditures incurred are as follows:

Rs. m
Research work 4.50
Development work 9.00
Training of production staff 0.50
Cost of trial run 0.80
Total costs 14.80

(ii) The right to manufacture a well-established product under a patent for


a period of five years was purchased on 1 March 2016 for Rs. 17
million. The patent has an expected remaining useful life of 10 years.
RI has the option to renew the patent for a further period of five years
for a sum of Rs. 12 million.
(iii) RI has acquired a brand at a cost of Rs. 2 million. The cost was
incurred in the month of June 2016. The life of the brand is expected
to be 10 years. Currently, there is no active market for this brand.
However, RI is planning to launch an aggressive marketing campaign
in February 2017.
(iv) In September 2015, RI developed a new production process and
capitalised it as an intangible asset at Rs. 7 million. The new process
is expected to have an indefinite useful life. During 2016, RI incurred
further development expenditure of Rs. 3 million on the new
process which meets the recognition criteria for capitalization of an
intangible asset.
Required
In the light of International Financial Reporting Standards, explain how each
of the above transaction should be accounted for in the financial
statements of Raisin International for the year ended 31 December 2016.

8.3 OXTAIL LTD


During the year ended 31 December 2016 Oxtail Ltd entered into the following
transactions relating to intangible assets. Oxtail Ltd uses the cost model to
measure intangible assets.
(1) Research costs of Rs. 70,000 were incurred from 1 January 2016 to 31
March 2016 on the early development of a new product. A project review
was undertaken on 1 April 2016 and on that date Oxtail Ltd assessed that
the development of the new product would be economically viable.
Subsequently, further costs of Rs. 120,000 were incurred to complete the
development phase of the project. This included Rs. 15,000 on promotional
advertising. Production of the new product started on 1 January 2017.
(2) On 1 April 2016 Oxtail Ltd acquired some technical know-how which will
completely change the way its manufacturing process operates.

Emile Woolf International 36 The Institute of Chartered Accountants of Pakistan


Questions

The following costs have been incurred:


Rs.
Original cost of technical know-how 180,000
Legal costs incurred as part of acquisition 4,000
Manufacturing supervisors time to install new process 3,200
Staff training incurred in operating new process 13,000
Testing new manufacturing process 4,800
205,000
The new manufacturing process was available for use on 1 July 2016. It was
believed that the new process would be of benefit for the next four years after
which it would be replaced.
Although operation of the new manufacturing process during its first six months
went well, a breakthrough in the development of improved technology by a
competitor led to an impairment review being carried out by Oxtail Ltd. At 31
December 2016 the fair value (less costs to sell) of the technical know-how was
assessed as being Rs. 152,000 compared with the present value of the estimated
future cash flows expected to be generated by the technology of Rs. 157,000.
Required
(a) Explain the required IFRS accounting treatment of the two issues above,
preparing all relevant calculations.
(b) Prepare an extract from Oxtail Ltd's statement of financial position as at 31
December 2016 and a summary of the related costs that would be
recognised in profit or loss for the year ended 31 December 2016 in respect
of intangible assets.

8.4 SKY LIMITED


During the year 2016, SKY Limited developed two inter-linked websites in
house. One of them is for external users and provides information about the
companys products, operations and financials. It can also be used for electronic
order processing and accepting payments through credit cards. The second
website is for internal use like intra-net, providing and sharing companys
policies, customer details, employees information, etc.
Both the websites were launched on September 30, 2016 and are now fully
operational. The company has received a few online orders which it believes will
increase over time. On the other hand, use of internal website has resulted in
minor reduction in costs of communication and certain other administrative costs.
The management is optimistic that its utility will increase significantly. However, it is
not in a position to estimate the amount of economic inflows that this website can
generate.
During the year ended December 31, 2016, the company incurred the following
expenditure in the development of websites:
(i) An amount of Rs. 0.3 million was incurred on undertaking a feasibility
study and defining hardware/software specifications for the websites.
(ii) Rs. 4 million was incurred on the development of internal website
while an expenditure of Rs. 11 million has been made on development of
external website.

Emile Woolf International 37 The Institute of Chartered Accountants of Pakistan


Advanced accounting and financial reporting

The expenditure on external website includes an amount of Rs. 6 million


paid for linking it with the credit card clearing facilities and installation of
security tools.
(iii) The company acquired two dedicated servers and one backup server
costing Rs. 3 million in total. Operating software for the server was
acquired for Rs. 2.0 million whereas software related to data processing
and front-end development cost Rs. 3 million. The management is of the
view that these costs would not have been incurred if the website project
had not been initiated.
(iv) With effect from October 1, 2016 the company has signed a one year
contract for website maintenance at a cost of Rs. 2.0 million.
(v) Two IT personnel were trained to operate the websites, at a cost of Rs. 0.2
million.
(vi) Rs. 0.4 million were incurred on the promotion of its external website. The
company believes that this advertising will boost the companys online
sales.
Required
Comment on the accounting treatment of each of the above mentioned costs in the
light of relevant International Accounting Standards.

8.5 COMFORT SHOES LIMITED


In 2010 the management of Comfort Shoes Limited planned to acquire an
international trademark to boost its sales and enter into the international market.
In this respect, the management carried out a market survey and analysed the
information obtained to initiate the process. The relevant information is as follows:
(i) The cost incurred on the survey and related activities during the year 2010
amounted to Rs. 1 million.
(ii) An agreement was finalised and the company acquired the trademark
effective January 1, 2011. According to the agreement Rs. 5 million were
paid on signing of the agreement and Comfort Shoes was required to pay
1% of sale proceeds of the related products on yearly basis. The analysis
carried out at that time indicated that the trademark would have an
indefinite useful life.
(iii) The company has developed many new models under this trademark and
successfully marketed them in the country as well as in international
markets. However, in 2015 the company faced unexpected competition and
had to discontinue the exports. It was estimated that due to discontinuation
of exports, net cash inflows for the foreseeable future, would reduce by
30%. As a result the management was of the view that as of December 31,
2015 the carrying value of the trademark had reduced to 90%.
(iv) Due to continuous inflation and flooding of markets with very low priced
shoes, it was decided in December 2016 that use of the trademark would
be discontinued with effect from January 1, 2018.
Required
(a) Explain how the above transactions should have been accounted for in the
years 2010 to 2011 according to International Financial Reporting Standards
(IFRSs).
(b) Prepare a note to the financial statements for the year ended December
31, 2016 in accordance with the requirements of IFRSs. Show comparative
figures.

Emile Woolf International 38 The Institute of Chartered Accountants of Pakistan


Questions

CHAPTER 9 IAS 36: IMPAIRMENT OF ASSETS

9.1 CHARLOTTE
Charlotte Ltd is a company with a 31 December year-end.
The following is relevant to three tangible non-current assets held by Charlotte.

Machine 1
This was purchased on 1 January Year 1 for Rs. 420,000. It had an estimated
residual value of Rs. 50,000 and a useful life of ten years and was being
depreciated on a straight-line basis.
On 1 January Year 6 Charlotte revalued this machine to Rs. 275,000 and
reassessed its total useful life as fifteen years with no residual value.
On 1 January Year 7 an impairment review showed machine 1s recoverable
amount to be Rs. 100,000 and its remaining useful life to be five years.

Machine 2
This was purchased on 1 January Year 1 for Rs. 500,000. It had an estimated
residual value of Rs. 60,000 and a useful life of ten years and was being
depreciated on a straight-line basis.
On 1 January Year 7 this machine was classified as held for sale, at which time
its fair value was estimated at Rs. 200,000 and costs to sell at Rs. 5,000. On 31
March Year 7 the machine was sold for Rs. 210,000.

Machine 3
This was purchased on 1 January Year 1 for Rs. 600,000. In Year 1 depreciation
of Rs. 30,000 was charged. On 1 January Year 2 this machine was revalued to
Rs. 800,000 and its remaining useful life assessed as eight years.
On 1 January Year 7 this machine was classified as held for sale, at which time,
its fair value was estimated at Rs. 550,000 and costs to sell at Rs. 5,000.
On 31 March Year 7 the machine was sold for Rs. 550,000.
Tax is at the rate of 30%.

Required
Show the effect of the above on profit or loss and revaluation reserve of Charlotte
in Year 7.

9.2 ABA LIMITED


Aba Limited conducts its activities from two properties, a head office in the city
centre and a property in the countryside where staff training is conducted. Both
properties were acquired on 1 April 2013 and had estimated lives of 25 years with
no residual value. The company has a policy of carrying its land and buildings at
current values. However, until recently property prices had not changed for some
years. On 1 October 2015 the properties were revalued by a firm of surveyors.
Details of this and the original costs are:

Emile Woolf International 39 The Institute of Chartered Accountants of Pakistan


Advanced accounting and financial reporting

Land Buildings
Rs. Rs.
Head office cost 1 April 2013 500,000 1,200,000
revalued 1 October 2015 700,000 1,350,000
Training premises cost 1 April 2013 300,000 900,000
revalued 1 October 2015 350,000 600,000
The fall in the value of the training premises is due mainly to damage done by the
use of heavy equipment during training. The surveyors have also reported that
the expected life of the training property in its current use will only be a further 10
years from the date of valuation. The estimated life of the head office remained
unaltered.
Note: Aba Limited treats its land and its buildings as separate assets.
Depreciation is based on the straight-line method from the date of purchase or
subsequent revaluation.
Required
Prepare extracts of the financial statements of Aba Limited in respect of the
above properties for the year to 31 March 2016.

9.3 Hussain Associates Ltd


The assistant financial controller of the Hussain Associates Ltd group has
identified the matters below which she believes may indicate impairment of one
or more assets:
(a) Hussain Associates Ltd owns and operates an item of plant that cost Rs.
640,000 and had accumulated depreciation of Rs. 400,000 at 1 October
2015. It is being depreciated at 12% on cost.
On 1 April 2016 (exactly half way through the year) the plant was damaged
when a factory vehicle collided into it. Due to the unavailability of
replacement parts, it is not possible to repair the plant, but it still operates,
albeit at a reduced capacity. It is also expected that as a result of the
damage the remaining life of the plant from the date of the damage will be
only two years.
Based on its reduced capacity, the estimated present value of the plant in
use is Rs. 150,000. The plant has a current disposal value of Rs. 20,000
(which will be nil in two years time), but Hussain Associates Ltd has been
offered a trade-in value of Rs. 180,000 against a replacement machine
which has a cost of Rs. 1 million (there would be no disposal costs for the
replaced plant). Hussain Associates Ltd is reluctant to replace the plant as
it is worried about the long-term demand for the product produced by the
plant. The trade-in value is only available if the plant is replaced.
Required
Prepare extracts from the statement of financial position and statement of
profit or loss of Hussain Associates Ltd in respect of the plant for the year
ended 30 September 2016. Your answer should explain how you arrived at
your figures.
(b) On 1 April 2015 Hussain Associates Ltd acquired 100% of the share capital
of Sparkle Limited, whose only activity is the extraction and sale of spa
water. Sparkle Limited had been profitable since its acquisition, but bad
publicity resulting from several consumers becoming ill due to a

Emile Woolf International 40 The Institute of Chartered Accountants of Pakistan


Questions

contamination of the spa water supply in April 2016 has led to unexpected
losses in the last six months. The carrying amounts of Sparkle Limiteds
assets at 30 September 2016 are:
Rs.000
Brand (Sparkle Spring see below) 7,000
Land containing spa 12,000
Purifying and bottling plant 8,000
Inventories 5,000
32,000
The source of the contamination was found and it has now ceased.
The company originally sold the bottled water under the brand name of
Sparkle Spring, but because of the contamination it has re-branded its
bottled water as Refresh. After a large advertising campaign, sales are
now starting to recover and are approaching previous levels. The value of
the brand in the balance sheet is the depreciated amount of the original
brand name of Sparkle Spring.
The directors have acknowledged that Rs. 1.5 million will have to be spent
in the first three months of the next accounting period to upgrade the
purifying and bottling plant.
Inventories contain some old Sparkle Spring bottled water at a cost of Rs.
2 million; the remaining inventories are labelled with the new brand
Refresh. Samples of all the bottled water have been tested by the health
authority and have been passed as fit to sell. The old bottled water will have
to be relabelled at a cost of Rs. 250,000, but is then expected to be sold at
the normal selling price of (normal) cost plus 50%.
Based on the estimated future cash flows, the directors have estimated that
the value in use of Sparkle Limited at 30 September 2016, calculated
according to the guidance in IAS 36, is Rs. 20 million. There is no reliable
estimate of the fair value less costs to sell of Sparkle Limited.
Required
Calculate the amounts at which the assets of Sparkle Limited should
appear in the consolidated statement of financial position of Hussain
Associates Ltd at 30 September 2016. Your answer should explain how you
arrived at your figures.

9.4 IMPS
A division of IMPS has the following non-current assets, which are stated at their
carrying values at 31 December Year 4:
Rs. m Rs. m

Goodwill 70

Property, plant and equipment:


Land and buildings 320
Plant and machinery 110
430
500

Emile Woolf International 41 The Institute of Chartered Accountants of Pakistan


Advanced accounting and financial reporting

Because these assets are used to produce a specific product, it is possible to


identify the cash flows arising from their use. The management of IMPS believes
that the value of these assets may have become impaired, because a major
competitor has developed a superior version of the same product and, as a
result, sales are expected to fall.
The following additional information is relevant:
Forecast cash inflows arising from the use of the assets are as follows:
Year ended 31 December:
Rs. m
Year 5 185
Year 6 160
Year 7 130
(i) The directors are of the opinion that the market would expect a pre-tax
return of 12% on an investment in an entity that manufactures a product of
this type.
(ii) The land and buildings are carried at valuation. The surplus relating to the
revaluation of the land and buildings that remains in the revaluation reserve
at 31 December Year 4 is Rs. 65 million. All other non-current assets are
carried at historical cost.
(iii) The goodwill does not have a market value. It is estimated that the land and
buildings could be sold for Rs. 270 million and the plant and machinery
could be sold for Rs. 50 million, net of direct selling costs.
Required
(a) Calculate the impairment loss that will be recognised in the accounts of
IMPS.
(b) Explain how this loss will be treated in the financial statements for the year
ended 31 December Year 4.

Emile Woolf International 42 The Institute of Chartered Accountants of Pakistan


Questions

CHAPTER 10 IFRS 5: NON-CURRENT ASSETS HELD FOR SALE AND


DISCONTINUED OPERATIONS

10.1 SAUL
Saul operates its business through a number of divisions. It has a year end of 31
December. Set out below are extracts from the draft financial statements of Saul
for the year ended 31 December Year 1.
Statement of profit or loss for the year ended 31 December Year 1
Rs.000
Revenue 3,900
Cost of sales (2,500)

Gross profit 1,400
Distribution costs (300)
Administrative expenses (800)

Profit before tax 300
Income tax expense (90)

Profit for the period 210

Statement of financial position at 31 December Year 1
Assets Rs.000 Rs.000
Non-current assets
Property, plant and equipment 1,900
Intangible assets 40

1,940
Current assets
Inventories 350
Trade and other receivables 190
Cash 90

630

Total assets 2,570

Equity and liabilities Rs.000 Rs.000
Equity
Share capital 600
Retained earnings 1,700

2,300
Current liabilities
Trade and other payables 195
Current tax payable 75

270

Total equity and liabilities 2,570

On 30 November Year 1 Saul made the decision to close Division A, which is
located in a different part of the country and covers a separate major line of
business. This decision was immediately announced to the press and to the
workforce and, by the end of Year 1, a buyer had been found.
The directors of Saul have calculated the following.
15% of the entitys income and expenses for the year was attributable to Division
A.

Emile Woolf International 43 The Institute of Chartered Accountants of Pakistan


Advanced accounting and financial reporting

No tax is attributable to Division A.


Property, plant and equipment of Rs. 510,000 and payables of Rs. 10,000 in the
above statement of financial position relate to Division A. The fair value minus
costs to sell of the property, plant and equipment is Rs. 450,000.
Required
Redraft the above financial statements to meet the provisions of IFRS 5:Non-
current assets held for sale and discontinued operations. Work to the nearest
Rs.000.

10.2 SHAHID HOLDINGS


(a) State the definition of both non-current assets held for sale and
discontinued operations and explain the usefulness of information for
discontinued operations.
Shahid Holdings is in the process of preparing its financial statements for
the year ended 31 October 2016. The companys main activity is in the
travel industry mainly selling package holidays (flights and accommodation)
to the general public through the Internet and retail travel agencies. During
the current year the number of holidays sold by travel agencies declined
dramatically and the directors decided at a board meeting on 15 October
2016 to cease marketing holidays through its chain of travel agents and sell
off the related high-street premises. Immediately after the meeting the
travel agencies staff and suppliers were notified of the situation and an
announcement was made in the press. The directors wish to show the
travel agencies results as a discontinued operation in the financial
statements to 31 October 2016. Due to the declining business of the travel
agents, on 1 August 2016 (three months before the year end) Shahid
Holdings expanded its Internet operations to offer car hire facilities to
purchasers of its Internet holidays.
The following are Shahid Holdingss summarised statement of profit or loss
results years ended:
31 October 2016 31 October 2015
Travel
Internet Car hire Total Total
agencies
Rs.000 Rs.000 Rs.000 Rs.000 Rs.000
Revenue 23,000 14,000 2,000 39,000 40,000
Cost of sales (18,000) (16,500) (1,500) (36,000) (32,000)
    
Gross profit/(loss) 5,000 (2,500) 500 3,000 8,000
Operating expenses (1,000) (1,500) (100) (2,600) (2,000)
    
Profit/(loss) before tax 4,000 (4,000) 400 400 6,000
    
The results for the travel agencies for the year ended 31 October 2015
were: revenue Rs. 18 million, cost of sales Rs. 15 million and operating
expenses of Rs. 15 million.
Required
(b) Discuss whether the directors wish to show the travel agencies
results as a discontinued operation is justifiable.

Emile Woolf International 44 The Institute of Chartered Accountants of Pakistan


Questions

(c) Assuming the closure of the travel agencies is a discontinued


operation, prepare the (summarised) statement of profit or loss of
Shahid Holdings for the year ended 31 October 2016 together with its
comparatives.
Note: Shahid Holdings discloses the analysis of its discontinued
operations on the face of its statement of profit or loss.

10.3 PRIMA
Prima is a listed company with a year end of 31 December. It operates two
businesses, the first is the rental of luxury yachts and the second is a chain of
luxury holiday villas in Europe. The directors have requested your advice on the
following matters.

Holiday villas
Primas policy is to carry the holiday villas at their re-valued amount, which,
based on the most recent valuation in 20X0, was Rs. 20m (historical cost was Rs.
10m). Prima is unsure how frequently a revaluation of such properties is required
and so has instructed a surveyor to provide an up-to-date valuation as at 31
December Year 4. This valuation has provided the following information:
Rs. million
Replacement cost 17
Value in use 28
Open market value 25
One of the villas has received very few bookings over the past two years and so a
decision was reached to exclude it from the Year 5 brochure. It is currently up for
sale. The villa has a carrying value of Rs. 1.25m. Its value in use is only
Rs.0.85m and its expected market value is Rs. 1m, before expected agents and
solicitors fees of Rs. 50,000. The directors are unsure as to the accounting
treatment of this villa. A number of potential buyers have expressed an interest in
the property, and it is hoped that a deal will be negotiated in the first few months
of Year 5.
Primas accounting policy is to not charge depreciation on the villas. Its
justification is that the villas are maintained to a high standard and have useful
lives of at least 50 years.

Head Office
Over the past two years, Prima has built its own head office. Construction began
on 1 October Year 2 and finished on 1 June Year 4, although minor modifications
meant that the company did not relocate until 1 September Year 4.
The site cost Rs. 1m and the costs of construction were a further Rs. 8m. Prima
took out a two year loan of Rs. 5m on 1 October Year 2, at an interest rate of 9%
per annum, to help fund the work. In order to encourage businesses to operate in
areas of high unemployment, the government offered a Rs. 1.5m grant towards
the cost of construction. The terms of settlement were that payment would only
be made upon completion of the building once a government inspection had
taken place. This inspection had not taken place by the year end, but Prima is
confident that the grant will be received shortly after the year end.
The company intends to use the head office for the next 50 years and, as for the
villas, does not intend to depreciate the land or buildings.

Emile Woolf International 45 The Institute of Chartered Accountants of Pakistan


Advanced accounting and financial reporting

Yachts
Prima has spent the past year designing a new range of luxury yachts. Work was
completed on 1 April Year 4 at a cost of Rs. 20m. During the construction, the
economy took a downturn and the company now believes that the market value
of the yachts is only Rs. 17m, although the value in use is estimated to be Rs.
18m. The engines of the yachts have a three year life, the interior has a two year
life, and the remainder should have a life of 15 years. The engine cost is believed
to represent 15% of the total cost of manufacture and the interior approximately
25%.

Required
Explain the accounting issues relating to the villas, head office and yachts,
referring to relevant IFRS guidance. Where possible, numerical information
relating to the 31 December Year 4 financial statements should be provided.

Emile Woolf International 46 The Institute of Chartered Accountants of Pakistan


Questions

CHAPTER 11 IFRS 16: LEASES

11.1 X LTD
X Ltd is considering acquiring a machine. It has two options; cash purchase at a
cost of Rs. 11,420,000 or a lease.
The terms of the lease are as follows:
(i) The lease period is for four years from 1 January 2016 with an annual
rental of Rs. 4,000,000 payable on 31 December each year.
(ii) The lessee is required to pay all repairs, maintenance and other incidental
costs.
(iii) The interest rate implicit in the lease is 15% p.a.
Note:
Estimated useful economic life span of the machine is four years.
Required
(a) Prepare a schedule of the allocation of the finance charges in the books of
X Limited for the entire lease period.
(b) Prepare an extract of the Statement of Financial Position of X Limited for
the year ended 31 December 2016.

11.2 PROGRESS LTD


Progress Ltd. acquired a machine from Fine Rentals Ltd. on January 3, 2016
under a lease agreement extending over three years.
The agreement required them to make an initial deposit of Rs. 1,280,000 to be
followed by three annual payments of Rs. 800,000 on 31 December each year
starting from 2016.
The cash price of the machinery was Rs. 3,200,000 and Fine Rentals Ltd. added
12% interest which was duly communicated to Progress Ltd.
The annuity method is used to allocate interest.
Required
(a) Compute the interest element and the capital portion of the annual
repayments; and
(b) Show the journal entries that will record the transaction resulting from the
lease agreement.

11.3 MIRACLE TEXTILE LIMITED


On 1 July 2014, Miracle Textile Limited (MTL) acquired a machine on lease,
from a bank.
Details of the lease are as follows:
(i) Cost of machine is Rs. 20 million.
(ii) The lease term and useful life is 4 years and 10 years respectively.
(iii) Instalment of Rs. 5.80 million is to be paid annually in advance on 1 July.
(iv) The interest rate implicit in the lease is 15.725879%.

Emile Woolf International 47 The Institute of Chartered Accountants of Pakistan


Advanced accounting and financial reporting

(v) At the end of lease term, MTL has an option to purchase the machine
on payment of Rs. 2 million. The fair value of the machine at the end of
lease term is expected to be Rs. 3 million.
MTL depreciates the machine on the straight line method to a nil residual value.
Required
Prepare relevant extracts of the statement of financial position and related notes
to the financial statements for the year ended 30 June 2016 along with
comparative figures. Ignore taxation

11.4 ACACIA LTD


On 1 April 2015 Acacia Ltd entered into the following lease agreements. The
terms of each lease are as follows:
(1) Plant with a fair value of Rs. 275,000 was leased under an agreement
which requires Acacia Ltd to make annual payments of Rs. 78,250 on 1
April each year, commencing on 1 April 2015, for four years. After the four
years Acacia Ltd has the option to continue to lease the plant at a nominal
rent for a further three years and is likely to do so as the asset has an
estimated useful life of six years. The present value of the lease payments
is Rs. 272,850. Acacia Ltd is responsible for insuring and maintaining the
plant during the period of the lease.
(2) Office equipment with a fair value of Rs. 24,000 was leased under a non-
cancellable agreement which requires Acacia Ltd to make annual payments
of Rs. 6,000 on 1 April each year, commencing on 1 April 2015, for three
years. The lessor remains responsible for insuring and maintaining the
equipment during the period of the lease. The equipment has an estimated
useful life of ten years. The present value of the lease payments is Rs.
16,415.
Acacia Ltd allocates finance charges on an actuarial basis. The interest rate
implicit in both of the leases is 10%.
Required
Prepare all relevant extracts from Acacia Ltd's financial statements for the year
ended 31 March 2016 in respect of the above leases. The only notes to the
financial statements required are those in respect of lease liabilities or
commitments.

11.5 SHOAIB LEASING LIMITED


Shoaib Leasing Limited (the lessor) has entered into a three year agreement with
Sarfaraz Limited (the lessee) to lease a machine with an expected useful life of
4 years. The cost of machine is Rs. 2,100,000.
The following information relating to lease transaction is available:
(i) Date of commencement of lease is July 1, 2016.
(ii) The lease contains a purchase bargain option at Rs. 100,000. At the end of
the lease term, the value of the machine will be Rs. 300,000.
(iii) Lease instalments of Rs. 860,000 are payable annually, in arrears, on June
30.
(iv) The implicit interest rate is 12.9972%.

Emile Woolf International 48 The Institute of Chartered Accountants of Pakistan


Questions

Required
(a) Prepare the journal entries for the years ending June 30, 2017, 2018 and
2019 in the books of lessor. Ignore tax.
(b) Produce extracts from the statement of financial position including
relevant notes as at June 30, 2017 to show how the transactions carried
out in 2017 would be reflected in the financial statements of the lessor.
(Disclosure of accounting policy is not required.)

11.6 FLOW
Flow prepares financial statements on 31 March each year. On 1 April Year 4,
Flow sold a freehold property to another company, River, for Rs. 850,000 and
then leased it back under a ten year operating arrangement. Flow had purchased
the property exactly ten years previously for Rs. 500,000 and had charged total
depreciation of Rs. 60,000 on the property up to the date of disposal.
Details of the sale and leaseback arrangement are as follows:
Proceeds from sale Rs. 850,000
Fair value at date of disposal Rs. 550,000
Operating lease rentals (payable in arrears) Rs. 100,000
Normal market rental Rs. 50,000
Required
Explain and illustrate how Flow should reflect in its financial statements:
x The sale of the property to River on 1 April Year 4
x The payment of the first rental to River on 31 March Year 5.

11.7 PINUS LIMITED


Pinus Limited entered into a sale and finance leaseback arrangement with a bank
on 1 April 2015. The arrangement involved the sale at fair value of plant and
machinery to the bank for Rs. 1,440,000.
This amount has been credited to Pinus Limiteds operating income. The carrying
amount of the plant and machinery was Rs. 840,000 and its remaining useful life
was five years at 1 April 2015. No depreciation has been charged in respect of
this plant and machinery for the year ended 31 March 2016.
Under the terms of the lease Pinus Limited is to pay five annual payments, in
arrears on 31 March each year, of Rs. 360,000. The first payment has been
made and has been debited to operating costs. The interest rate implicit in the
lease is 8%.
Required
Explain how the above transaction should be accounted for, with all relevant
calculations, in the financial statements for the year ended 31 March 2016.

Emile Woolf International 49 The Institute of Chartered Accountants of Pakistan


Advanced accounting and financial reporting

11.8 LODHI TEXTILE MILLS LIMITED


Lodhi Textile Mills Limited is facing severe financial difficulties. To improve the
cash flows, the management has decided to sell and lease back three power
generators of the company under three different sale and lease back
arrangements which were signed on August 15, 2016. The company has
assessed that all the leases shall qualify as finance leases.
The related information as on August 15, 2016 is given below:
Cost Book Fair Value Value in Amount of
Value Use Financing
Rs.000 Rs.000 Rs.000 Rs.000 Rs.000
Generator A 10,000 7,500 6,000 6,500 6,000
Generator B 12,000 6,000 4,000 5,000 6,000
Generator C 10,000 7,000 10,000 12,000 8,000
Required
Prepare the accounting entries that should be recorded by the company on
August 15, 2016 in respect of the above transactions.
Note: Ignore tax and deferred tax implications, if any.

11.9 AUTO CONSTRUCTION PAKISTAN LIMITED


Auto Construction Pakistan Limited (ACPL) is engaged in the business of renting
of construction machinery. On March 15, 2016 ACPL negotiated and finalised an
agreement for purchase of used machinery from Malaysia. The price on FOB
basis was agreed at US$ 0.4 million. The machinery was loaded on the ship on
April 1, 2016 and arrived at the company premises on May 31, 2016. According
to the agreement a down payment of 10% was made on the date of loading. The
remaining amount was paid on June 30, 2016. The US$ conversion rates on
April 1, May 31 and June 30 were Rs. 80.90, Rs. 81.60 and Rs. 82.70
respectively. A cost of Rs. 4 million was incurred on freight, taxes and other
charges. Economic life of the machinery is 10 years.
On July 1, 2016, ACPL sold the machinery to Smart Investment Limited for Rs.
40 million and leased it back under the following arrangement:
(i) Lease term of 5 years commencing from July 1, 2016.
(ii) 10 half yearly instalments of Rs. 5.50 million each payable in arrears. (iii)
Interest rate implicit in the lease at 12.506%
On July 1, 2016 ACPL rented the machinery to a customer for three years at a
half yearly rent of Rs. 5 million each, payable in advance with 5% annual
increase.
Required
Prepare notes to the financial statements for the year ended December 31, 2016
in accordance with the requirement of IFRS 16 (Leases).

Emile Woolf International 50 The Institute of Chartered Accountants of Pakistan


Questions

CHAPTER 12 IAS 37: PROVISIONS CONTINGENT LIABILITIES AND


CONTINGENT ASSETS

12.1 ROWSLEY
Rowsley is a diverse group with many subsidiaries. The group is proud of its
reputation as a caring organisation and has adopted various ethical policies
towards its employees and the wider community in which it operates. As part of
its Annual Report, the group publishes details of its environmental policies, which
include setting performance targets for activities such as recycling, controlling
emissions of noxious substances and limiting use of non-renewable resources.
The finance director is reviewing the accounting treatment of various items prior
to finalising the accounts for the year ended 31 March 20X4. All items are
material in the context of the accounts as a whole. The accounts are due to be
approved by the directors on 30 June 20X4.
Closure of factory
On 15 February 20X4, the board of Rowsley decided to close down a large
factory in Derbytown. The board is trying to draw up a plan to manage the effects
of the reorganisation, and it is envisaged that production will be transferred to
other factories. The factory will be closed on 31 August 20X4, but at 31 March
20X4 this decision had not yet been announced to the employees or to any other
interested parties. Costs of the reorganisation have been estimated at Rs. 45
million
Relocation of subsidiary
During December 20X3, one of the subsidiary companies moved from Buckington
to Sundertown in order to take advantage of government development grants. Its
main premises in Buckington are held under an operating lease, which runs until
31 March 20X9. Annual rentals under the lease are Rs. 10 million. The company
is unable to cancel the lease, but it has let some of the premises to a charitable
organisation at a nominal rent. The company is attempting to rent the remainder
of the premises at a commercial rent, but the directors have been advised that
the chances of achieving this are less than 50%.
Legal claim
During the year to 31 March 20X4, a customer started legal proceedings against
the group, claiming that one of the food products that it manufactures had caused
several members of his family to become seriously ill. The groups lawyers have
advised that this action will probably not succeed.
Environmental impact of overseas subsidiary
The group has an overseas subsidiary that is involved in mining precious metals.
These activities cause significant damage to the environment, including
deforestation. The company expects to abandon the mine in eight years time.
The mine is situated in a country where there is no environmental legislation
obliging companies to rectify environmental damage and it is very unlikely that
any such legislation will be enacted within the next eight years. It has been
estimated that the cost of cleaning the site and re-planting the trees will be Rs. 25
million if the re-planting was successful at the first attempt, but it will probably be
necessary to make a further attempt, which will increase the cost by a further Rs.
5 million.
Required
Explain how each of the items above should be treated in the consolidated
financial statements for the year ended 31 March 20X4

Emile Woolf International 51 The Institute of Chartered Accountants of Pakistan


Advanced accounting and financial reporting

12.2 MULTAN PETROCHEM LTD


Multan Petrochem Limited (MPL) operates in the oil extraction and refining
business and is preparing its draft financial statements for the year ended 31
December 2016. The following information has been collected for the preparation
of the provisions and contingencies notes.
(1) A new site was acquired on 1 January 2015 and is being used as the site
for a new oil refinery. Initial preparation work was undertaken at the site at
the start of 2015 and the oil refinery was completed and ready for use on 31
December 2015. The new refinery was expected to have a useful life of 25
years. MPL has a well-publicised policy that it will reinstate any
environmental damage caused by its activities. The present value of the
estimated cost of reinstating the environment is Rs. 1,300,000 for damage
caused during the initial preparation work. This amount is based on a
discount rate of 8%.
(2) An explosion at one of MPLs oil extraction plants on 1 July 2016 has led to
a number of personal injury claims being made by employees who were
injured during the explosion. Five claims have been made to date but if
these claims are successful, it is likely that a further three employees who
were also injured will make a claim. MPLs lawyers estimate that it is
probable that the claims will succeed and that the estimated average cost
of each payout will be Rs. 150,000. The lawyers have recommended that
MPL settles the claims out of court as quickly as possible at their estimated
amount for all eight employees injured to avoid any adverse publicity.
An additional two claims have been made by employees for the stress,
rather than injury, that the explosion has caused them. If these claims were
to succeed the lawyers have estimated that the likely payout would be
around Rs. 10,000 per employee. However, the lawyers have stated that
they believe it to be unlikely that these employees will win such a case.
MPL made an insurance claim to try to recover the personal injury costs
that it is probable that it will incur. The claim is now in its advanced stages
and the insurance company has agreed to meet the cost of the claims in
full. The insurance company will refund MPL once the claims have been
settled.
(3) The future of MPLs business operations is in doubt following the explosion
at the oil extraction plant. The national press criticised MPL for the way that
it handled the problem. To address this, on 1 October 2016 MPL paid Rs.
12,000 to a risk assessment specialist who has recommended introducing a
new disaster recovery plan at an estimated cost of Rs. 500,000.
(4) MPL entered into an operating lease in the previous period for some office
space. However, the companys plans changed and the office space was
no longer required. At 1 January 2016 a correctly calculated provision had
been made for the future outstanding rentals of Rs. 80,000 for the
remaining five years. This was based on a discount rate of 8%. The rent
paid during the period was Rs. 15,000. In addition, MPP has signed a sub-
lease to rent out the space for the first six months of next year for total
rental income of Rs. 6,000. No other tenants are expected to be found for
the office space.
Required
(a) Prepare the provisions and contingencies notes for inclusion in the financial
statements of MPP for the year ended 31 December 2016.
(b) List the amounts that should be recognised in the statement of profit or loss
for the year ended 31 December 2016.

Emile Woolf International 52 The Institute of Chartered Accountants of Pakistan


Questions

12.3 VIOLET POWER LIMITED


Violet Power Limited is running a coal based power project in Pakistan. The
Company has built its plant in an area which contains large reserves of coal. The
company has signed a 20 years agreement for sale of power to the
Government. The period of the agreement covers a significant portion of the
useful life of the plant. The company is liable to restore the site by dismantling
and removing the plant and associated facilities on the expiry of the agreement.
Following relevant information is available:
(i) The plant commenced its production on July 1, 2015. It is the policy of the
company to measure the related assets using the cost model;
(ii) Initial cost of plant was Rs. 6,570 million including erection, installation and
borrowing costs but does not include any decommissioning cost;
(iii) Residual value of the plant is estimated at Rs. 320 million;
(iv) Initial estimate of amount required for dismantling of plant, at the time of
installation of plant was Rs. 780 million. However, such estimate was
reviewed as of June 30, 2016 and was revised to Rs. 1,021 million;
(v) The Company follows straight line method of depreciation; and
(vi) Real risk-free interest rate prevailing in the market was 8% per annum
when initial estimates of decommissioning costs were made. However, at
the end of the year such rate has dropped to 6% per annum.
Required
Work out the carrying value of plant and decommissioning liability as of June 30,
2016.

Emile Woolf International 53 The Institute of Chartered Accountants of Pakistan


Advanced accounting and financial reporting

CHAPTER 13: IAS 19: EMPLOYEE BENEFITS

13.1 LABURNUM LIMITED


Laburnum Limited operates a defined benefit pension plan for its employees. At 1
January 2016 the fair value of the pension plan assets was Rs. 2,600,000 and the
present value of the plan liabilities was Rs. 2,900,000.
The actuary estimates that the current and past service costs for the year ended
31 December 2016 is Rs. 450,000 and Rs. 90,000 respectively. The past service
cost is caused by an increase in pension benefits. The plan liabilities at 1 January
and 31 December 2016 correctly reflect the impact of this increase.
The interest cost on the plan liabilities is estimated at 8% and the expected return
on plan assets at 5%.
The pension plan paid Rs. 240,000 to retired members in the year to 31
December 2016. Laburnum Limited paid Rs. 730,000 in contributions to the
pension plan and this included Rs. 90,000 in respect of past service costs.
At 31 December 2016 the fair value of the pension plan assets is Rs. 3,400,000
and the present value of the plan liabilities is Rs. 3,500,000.
In accordance with the amendment to IAS 19 Employee Benefits, Laburnum
Limited recognises actuarial gains and losses in other comprehensive income in
the period in which they occur.
Required
Calculate the actuarial gains or losses on pension plan assets and liabilities that
will be included in other comprehensive income for the year ended 31 December
2016. (Round all figures to the nearest Rs.000).

13.2 JABEL LIMITED


Jabel Limited operates a defined benefit pension plan for its employees. At 1 July
2015 the fair value of the pension plan assets was Rs. 1,200,000 and the present
value of the plan liabilities was Rs. 1,400,000. The interest cost on the plan
liabilities was estimated at 7% and the expected return on plan assets at 4%.
The actuary estimates that the current service cost for the year ended 30 June
2016 is Rs. 300,000. Jabel Limited made contributions into the pension plan of
Rs. 400,000 in the year. The pension plan paid Rs. 220,000 to retired members in
the year to 30 June 2016.
At 30 June 2016 the fair value of the pension plan assets was Rs. 1,400,000 and
the present value of the plan liabilities was Rs. 1,600,000.
In accordance with the amendment to IAS 19 Employee Benefits, Jabel Limited
recognises actuarial gains and losses in other comprehensive income in the
period in which they occur.
Required
Calculate the net expense that will be included in Jabel Limiteds profit or loss
AND the amounts that would be included in other comprehensive income in
respect of actuarial gains or losses for the year ended 30 June 2016. (Round all
workings to the nearest Rs.000)

Emile Woolf International 54 The Institute of Chartered Accountants of Pakistan


Questions

13.3 KAGHZI LIMITED


Kaghzi Limited operates a defined benefit pension plan for its employees. At 1
January 2016 the fair value of the pension plan assets was Rs. 1,400,000 and the
present value of the pension plan liabilities was Rs. 1,700,000.
The actuary estimates that the service cost for the year to 31 December 2016 is
Rs. 320,000. The interest cost on the plan liabilities is estimated at 7% and the
expected return on plan assets at 4% for the year to 31 December 2016. The
pension plan paid Rs. 170,000 to retired members and Kaghzi Limited paid Rs.
580,000 in contributions to the pension plan for the year to 31 December 2016.
At 31 December 2016 the fair value of the pension plan assets is Rs. 2,100,000
and the present value of the pension plan liabilities is Rs. 2,400,000. In
accordance with the amendment to IAS 19 Employee Benefits, Kaghzi Limited
recognises actuarial gains and losses within other comprehensive income in the
period in which they occur.
Required
Calculate the actuarial gains or losses on pension plan assets and liabilities that
will be included in Kaghzi Limiteds other comprehensive income for the year
ended 31 December 2016. (Round all figures to the nearest Rs.000).

13.4 LASURA LTD


Lasura Ltd operates a defined benefit pension plan for its employees. At 1 July
2015 the fair value of the pension plan assets was Rs. 2,200,000 and the present
value of the pension plan liabilities was Rs. 2,400,000. The interest cost on the
pension plan liabilities was estimated at 8% and the expected return on pension
plan assets at 5%.
The actuary estimates that the current service cost for the year ended 30 June
2016 is Rs. 500,000. Lasura Ltd made contributions into the pension plan of Rs.
300,000 and the pension plan paid Rs. 450,000 to retired members in the year to
30 June 2016. At 30 June 2016 the fair value of the pension plan assets was Rs.
2,300,000 and the present value of the pension plan liabilities was Rs. 2,700,000.
Actuarial gains and losses are included within the other comprehensive income of
Lasura Ltd as incurred.
Required
(i) Calculate the net expense that will be included in Lasura Ltds profit or loss
for the year ended 30 June 2016, in accordance with IAS 19 Employee
benefits.
(ii) Calculate the amount that will be included in Lasura Ltds other
comprehensive income for the year ended 30 June 2016, in accordance
with IAS 19 Employee benefits.

13.5 UNIVERSAL SOLUTIONS


(a) Explain the following as used in IAS 19 Employee Benefits:
(i) The term defined benefit pension plan
(ii) The basis to be adopted in measuring scheme assets
(iii) The basis to be adopted in measuring scheme liabilities
(iv) Actuarial gains and losses.

Emile Woolf International 55 The Institute of Chartered Accountants of Pakistan


Advanced accounting and financial reporting

(b) Universal Solutions operates a defined benefit pension scheme on behalf of


its employees. The company conducts an annual review of funding in
conjunction with their actuaries who have supplied the following
information:
At 31 Dec At 31 Dec
Year 3 Year 4
Rs. Rs.
Present value of pension fund obligations 1,200 1,300
Market value of pension fund assets 1,000 1,100
Information relevant to the actuarial valuation:
Discount rate used to determine pension fund liabilities 5%
Current service cost Rs. 100
Contributions to the pension fund Rs. 140
Benefits paid out amounted to Rs. 95
Required
(i) Show the figures that would appear on the face of the statement of
financial performance as at 31 December Year 3 and Year 4.
(ii) Construct a journal to explain the movement on the defined benefit net
asset (or net liability) during the year ended 31 December Year 4

13.6 DHA INTERIORS LTD


DHA Interiors Ltd operates two pension plans.

Pension Plan 1
The terms of the plan are as follows:
(i) employees contribute 6% of their salaries to the plan
(ii) DHA Interiors Ltd contributes, currently, the same amount to the plan for
the benefit of the employees
(iii) On retirement, employees are guaranteed a pension which is based upon
the number of years service with the company and their final salary.
(iv) This plan was closed to new entrants from 31 October 2015, but which was
open to future service accrual for the employees already in the scheme.
The following details relate to the plan in the year to 31 October 2016:
Rs. m
Present value of obligation at 1 November 2015 200
Present value of obligation at 31 October 2016 240
Fair value of plan assets at 1 November 2015 190
Fair value of plan assets at 31 October 2016 225
Current service cost 20
Pension benefits paid 19
Total contributions paid to the scheme for year to 31 October 2016 17

Emile Woolf International 56 The Institute of Chartered Accountants of Pakistan


Questions

Actuarial gains and losses are recognised in the statement of other


comprehensive income (OCI).
Pension Plan 2
Under the terms of the plan, DHA Interiors Ltd does not guarantee any return on
the contributions paid into the fund. The companys legal and constructive
obligation is limited to the amount that is contributed to the fund. The following
details relate to this scheme:
Rs. m
Fair value of plan assets at 31 October 2016 21
Contributions paid by company for year to 31 October 2016 10
Contributions paid by employees for year to 31 October 2016 10

The discount rates for the two plans are:


1 November 2015 31 October 2016
Discount rate 5% 6%
Required
(a) Explain the nature of and differences between a defined contribution plan
and a defined benefit plan with specific reference to the companys two
schemes.
(b) Show the accounting treatment for the two DHA Interiors Ltd pension plans
for the year ended 31 October 2016 under IAS 19 Employee Benefits.

Emile Woolf International 57 The Institute of Chartered Accountants of Pakistan


Advanced accounting and financial reporting

CHAPTER 14: IFRS 2: SHARE BASED PAYMENTS

14.1 TOSHACK LTD


Toshack Ltd has granted 50 share appreciation rights to each of its 1000
employees on 1 January 2013. The rights are due to vest on 31 December 2016,
with payment being made on 31 December 2017. Assume that 75% of the
awards vest. Shares prices were:
Rs.
01/01/2013 22
31/12/2013 27
31/12/2016 31
31/12/2017 28
Required
In accordance with IFRS 2, Share Based Payment;
(i) What liability would be recorded on 31 December 2016 for the share
appreciation rights?
(ii) How would the settlement of the transaction be accounted for on 31
December 2017.

14.2 IFRS 2
(a) IFRS 2 requires an entity to recognise share-based payment transactions in
its financial statements. These include transactions with the employees or
other parties where they are to be settled in cash, other assets or equity
instruments of the entity.
The IFRS identifies three types of share-based payment transaction and
sets out the measurement principles and specific requirements for each.
Required
(i) Suggest why there was a need for a standard in this area.
(ii) Identify and briefly explain the three types of share based payments
recognised by IFRS 2.
(b) A client of your firm, a listed company with a 31 December year end,
contacts you for advice on a proposed share option scheme for its
employees.
On 1 January Year 5, the client granted 100 options to each of its 500
employees. The grant is conditional upon the employee working for the
client over the next three years. At the grant date, it is estimated that the
fair value of each option is Rs. 15.
Calculate the expense in profit or loss for each year of the vesting period:
(i) assuming that the clients expectations throughout the vesting period
are that all options will vest; and alternatively
(ii) assuming that the clients best estimates of the proportion of options
that will vest are as follows:
Estimate at 31 December Year 5 85%
Estimate at 31 December Year 6 88%
With 44,300 options actually vesting at 31 December Year 7.

Emile Woolf International 58 The Institute of Chartered Accountants of Pakistan


Questions

14.3 SAVAGE LTD


Savage Ltd granted share options to its 300 employees on 1 October 2014. Each
employee will receive 1,000 share options provided they continue to work for
Savage Ltd for the following three years from the grant date. The fair value of the
options at the grant date was Rs. 11 each. In the year ended 30 September 2015,
10 employees left and another 30 were expected to leave over the next two
years. For the year ended 30 September 2016, 20 employees left and another 15
are expected to leave in the year to 30 September 2017.
Required
Discuss the accounting treatment to be adopted for the share options and
calculate the amount to be recognised in the statement of profit or loss in respect
of these options for the year ended 30 September 2016. Prepare appropriate
accounting entries.

14.4 YORATH LTD


Yorath Ltd granted share options to its 600 employees on 1 October 2013. Each
employee will receive 500 share options provided they continue to work for
Yorath Ltd for four years from the grant date. The fair value of each option at the
grant date was Rs. 148.
The actual and expected staff movement over the 4 years to 30 September 2017
is given below:
2014 20 employees left and another 50 were expected to leave over the next
three years.
2015 A further 25 employees left and another 40 were expected to leave over the
next two years.
2016 A further 15 employees left and another 20 were expected to leave the
following year.
2017 No actual figures are available to date.
The sales director of Yorath Ltd has stated in the board minutes that he disagrees
with the treatment of the share options. No cash has been paid out to employees,
therefore he fails to understand why an expense is being charged against profits.
Required
(a) Calculate the charge to the statement of profit or loss for the year ended 30
September 2016 for Yorath Ltd in respect of the share options and prepare
the journal entry to record this.
(b) Explain the principles of recognition and measurement for share-based
payments as set out in IFRS 2 Share-based Payments so as to address the
concerns of the sales director.

14.5 QUALTECH LTD


(a) Qualtech Ltd granted share options to its 300 employees on 1 January
2015. Each employee will receive 1,000 share options provided they
continue to work for Qualtech Ltd for 3 years from the grant date. The fair
value of each option at the grant date was Rs. 122.
The actual and expected staff movement over the 3 years to 31 December
2017 is provided below:

Emile Woolf International 59 The Institute of Chartered Accountants of Pakistan


Advanced accounting and financial reporting

2015: 25 employees left and another 40 were expected to leave over the
next two years.
2016: A further 15 employees left and another 20 were expected to leave
the following year.
Required
(i) Calculate the charge to Qualtech Ltds statement of profit or loss for
the year ended 31 December 2016 in respect of the share options
and prepare the journal entry to record this.
(ii) Explain how the recognition and measurement of a share-based
payment would differ if it was to be settled in cash rather than in
equity, in accordance with IFRS 2 Share-based Payments.

14.6 BRIDGE LTD


Bridge Ltd granted 1,000 share options to each of its 300 employees on 1
January 2015, with the condition that they continue to work for Bridge Ltd for 4
years from the grant date. The fair value of each option at the grant date was Rs.
50
20 employees left in the year to 31 December 2015 and at that date another 65
were expected to leave over the next three years. 23 employees left in the year to
31 December 2016 and at that date another 44 were expected to leave over the
next two years.
Required
(i) Calculate the charge to Bridge Ltds statement of profit or loss for the year
ended 31 December 2016 in respect of the share options and prepare the
journal entry to record this.
(ii) Explain why, in accordance with IFRS 2 Share-based Payment, share
options, such as those granted by Bridge Ltd, generate a charge to the
statement of profit or loss despite no cash transaction having occurred.

14.7 CAPSTAN LTD


Capstan Ltd granted 1,000 share appreciation rights (SARs), to its 300
employees on 1 January 2015. To be eligible, employees must remain employed
for 3 years from the date of issue and the rights must be exercised in January
2014, with settlement due in cash.
In the year to 31 December 2015, 32 staff left and a further 35 were expected to
leave over the following two years.
In the year to 31 December 2016, 28 staff left and a further 10 were expected to
leave in the following year.
The fair value of each SAR was Rs. 80 at 31 December 2015 and Rs. 120 at 31
December 2016.
Required
Prepare the accounting entry to record the expense associated with the SARs, for
the year to 31 December 2016, in accordance with IFRS 2 Share-based
Payments.

Emile Woolf International 60 The Institute of Chartered Accountants of Pakistan


Questions

14.8 NEWTOWN LTD


Newtown Ltd granted 1,000 share appreciation rights (SARs) to each of its 500
employees on 1 July 2014. To be eligible for the rights, employees must remain
employed by Newtown Ltd for 3 years from the date of grant. The rights must be
exercised in July 2017, with settlement due in cash.
In the year to 30 June 2015, 42 employees left and a further 75 were expected to
leave over the following two years.
In the year to 30 June 2016, 28 employees left and a further 25 were expected to
leave in the following year.
The fair value of each SAR was Rs. 90 at 30 June 2015 and Rs. 110 at 30 June
2016.
Required
(i) Prepare the journal entry to record the expense associated with the SARs
for the year ended 30 June 2016, in accordance with IFRS 2 Share-based
payment.
(ii) Explain, in accordance with IFRS 2 Share-based payment, how the
recognition and measurement of a share-based payment would differ, if it
was to be settled in equity rather than cash.

14.9 SINDH TRANSIT LTD


Sindh Transit Ltd granted share options to all of its 400 employees on 1 January
2015. Each employee will receive 1,000 share options provided they continue to
be employed by Sindh Transit Ltd for four years from the grant date. The fair
value of an option at the grant date was Rs. 220.
On the same date Sindh Transit Ltd granted 500 share appreciation rights to
each of its employees. To be eligible, employees again have to be employed by
Sindh Transit Ltd for four years from the grant date.
The rights are exercisable in the two-month period from 1 January 2019 and will
be settled in cash. The fair value of each share appreciation right was Rs. 120 at
31 December 2015 and Rs. 140 at 31 December 2016.
The actual and expected future staff movements as at 31 December 2015 and 31
December 2016 are provided below.
2015: 15 left and another 55 were expected to leave over the next three years.
2016: a further 22 left and another 36 were expected to leave over the next two
years.
Required
(a) Prepare, in accordance with IFRS 2 Share-based Payment, the accounting
entries required in the financial statements of Sindh Transit Ltd for the year
to 31 December 2016 in respect of the two financial instruments identified
above.
(b) Explain the main principle of recognition set out by IFRS 2 Share-based
Payment for share based payments AND why the treatment of the two
financial instruments identified above will differ in the statement of financial
position.

Emile Woolf International 61 The Institute of Chartered Accountants of Pakistan


Advanced accounting and financial reporting

CHAPTER 15: FINANCIAL INSTRUMENTS: RECOGNITION AND


MEASUREMENT

15.1 AJI PANCA LTD


On 1 January Year 1 Aji Panca Ltd has the following capital and reserves.
Equity Rs.
Share capital (Rs. 1 ordinary shares) 1,000,000
Share premium 200,000
Retained earnings 5,670,300

6,870,300

During Year 1 the following transactions took place.
1 January An issue of Rs. 100,000 8% Rs. 1 redeemable preference
shares at a premium of 60%. Issue costs are Rs. 2,237.
Redemption is at 100% premium on 31 December Year 5. The
effective rate of interest is 9.5%.
31 March An issue of 300,000 ordinary shares at a price of Rs. 1.30 per
share. Issue costs, net of tax benefit, were Rs. 20,000
30 June A 1 for 4 bonus issue of ordinary shares.
Profit for the year, before accounting for the above, was Rs. 508,500. The
dividends on the redeemable preference shares have been charged to retained
earnings.
Required
Set out capital and reserves and liabilities resulting from the above on 31
December Year 1.

15.2 PASSILA LTD


On 1 July 2016, Passila Ltd, issued 20,000 8% debentures at Rs. 97.50. The
security is redeemable in five years time. The interest on the debentures is
payable bi-annually on 30 June and 31 December.
On 31 December 2016, the Companys year-end date, the debentures were
quoted on the Karachi Stock Exchange for Rs. 96.00. The company accountant
has suggested each of the following as possible valuation basis for reporting the
debentures liability on the statement of financial position as at 31 December
2016:
(i) Face value of the debentures.
(ii) Face value of the debenture plus interest payment for five years.
(III) Market value on the statement of financial position as at the year end.
Required
(a) Determine the face value of the debentures and the proceeds accruing to
the company.
(b) Determine the amount and explain the nature of the differences between
the face value and the market value of the debentures on 1 July, 2016.
(c) Distinguish between nominal and effective rate of interest.

Emile Woolf International 62 The Institute of Chartered Accountants of Pakistan


Questions

(d) Determine the nominal interest payable on the debentures for the year
ended 31 December 2016.
(e) State arguments for or against each of the suggested alternatives for
reporting the debentures liability on the statement of financial position as at
31 December 2016.

15.3 FINANCIAL INSTRUMENTS


(a) Explain the IFRS 9 rules on the measurement of financial assets and
financial liabilities.
(b) Explain how the following should be accounted for in accordance with IAS
32 and IFRS 9 in the financial statements to 31 December Year 3.
(i) A 3% bond was purchased on 1 January Year 3 for Rs. 250,000. The
nominal value is Rs. 300,000 and redemption will be at par on 31
December Year 6. The coupon is received annually in arrears. The
effective interest rate on the bond is 9.7%. The company intends to
hold the bond until its maturity. The market value of the bond at 31
December Year 3 is Rs. 275,000.
(ii) An investment was made in the equity shares of XYZ. 3,000 shares
were purchased (a 1% stake) at a cost of Rs. 10 per share on 1 April
Year 1. A transaction fee of Rs. 300 was charged on the purchase.
The entity intended to sell the shares within three months but the
market value of the investment continued to rise and so the company
decided not to sell in the near term. The market value of the shares
over the three years has been as follows:
Rs.000
31 December Year 1 32
31 December Year 2 34
31 December Year 3 35
(iii) The company issued a convertible bond at par on 31 December Year
3, raising Rs. 500,000. The coupon on the bond is 4%. The rate on an
equivalent redeemable bond is 7%. The bond can be redeemed at
par on 31 December Year 6 or converted into equity shares at a rate
of five shares per Rs. 100. The bond has not been classed as fair
value through profit and loss.

15.4 ESPANOLA LTD


(a) Espanola Ltd acquired an investment in a debt instrument on 1 January
2016 at its par value of Rs. 3 million. Transaction costs relating to the
acquisition were Rs. 200,000. The investment earns a fixed annual return of
6%, which is received in arrears. The principal amount will be repaid to
Espanola Ltd in 4 years time at a premium of Rs. 400,000. The investment
has been correctly classified as held to maturity. The investment has an
effective interest rate of approximately 7.05%.
Required
(i) Explain how this financial instrument will be initially recorded AND
subsequently measured in the financial statements of Espanola Ltd,
in accordance with IAS 39 Financial Instruments: Recognition and
Measurement.

Emile Woolf International 63 The Institute of Chartered Accountants of Pakistan


Advanced accounting and financial reporting

(ii) Calculate the amounts that would be included in Espanola Ltds


financial statements for the year to31 December 2016 in respect of
this financial instrument.
(b) Espanola Ltd acquired 100,000 shares in X Ltd on 25 October 2016 for Rs.
3 per share. The investment resulted in Espanola Ltd holding 5% of the
equity shares of X Ltd. The related transaction costs were Rs. 12,000. X
Ltds shares were trading at Rs. 3.40 on 31 December 2016. The
investment has been classified as held for trading.
Required
Prepare the journal entries to record the initial AND subsequent
measurement of this financial instrument in the financial statements of
Espanola Ltd for the year to 31 December 2016.

15.5 SANDIA LTD


Sandia Ltd acquired 40,000 shares in another entity, Y Ltd, in March 2016 for Rs.
2.68 per share. The investment was classified as available for sale on initial
recognition. The shares were trading at Rs. 2.96 per share on 31 July 2016.
Commission of 5% of the value of the transaction is payable on all purchases and
disposals of shares.
Required
(i) Prepare the journal entries to record the initial recognition of this financial
asset and its subsequent measurement at 31 July 2016 in accordance with
IAS 39 Financial Instruments: Recognition and Measurement. (3 marks)
The directors of Sandia Ltd are concerned about the value of Sandia Ltds
investment in Y Ltd and in an attempt to hedge against the risk of a fall in its
value, are considering acquiring a derivative contract. The directors wish to
use hedge accounting in accordance with IAS 39.
Required
(ii) Discuss how both the available for sale investment and any associated
derivative contract would be subsequently accounted for, assuming that the
criteria for hedge accounting were met, in accordance with IAS 39.

15.6 GEO ALLOYS LTD


GEO Alloys Ltd made an investment in a debt instrument on 1 July 2014 at its
nominal value of Rs. 4,000,000. The instrument carries a fixed coupon interest
rate of 7%, which is receivable annually in arrears. The instrument will be
redeemed for Rs. 4,530,000 on 30 June 2018. Transaction costs associated with
the investment were Rs. 200,000 and were paid on 1 July 2014. The effective
interest rate applicable to this instrument has been calculated at approximately
8.4%. GEO Alloys Ltd intends to hold this investment until its redemption date.
Required
(a) (i) Explain how this investment should be classified and prepare the
journal entry to initially record it in accordance with relevant
accounting standards.
(ii) Calculate the carrying value of the investment to be included in GEO
Alloys Ltds statement of financial position as at 30 June 2016, in
accordance with IAS 39 Financial instruments: recognition and
measurement.

Emile Woolf International 64 The Institute of Chartered Accountants of Pakistan


Questions

GEO Alloys Ltds main business risk is the price of raw materials. As a
manufacturer of jewellery, its profits can be significantly affected by the price of
precious metals. Therefore, in order to minimise the risk of future price increases
adversely affecting its future profits, GEO Alloys Ltd entered into a forward
contract on 1 May 2016, at nil cost, to purchase 100,000 units of metal A at Rs.
10,500 per unit on 1 August 2016.
At 30 June 2016, the forward rate for purchasing 100,000 units of metal A was
Rs. 10,100 per unit. GEO Alloys Ltd adopts hedge accounting where permitted by
IAS 39 Financial instruments: recognition and measurement.
Required
(b) Explain how this forward contract should be accounted for by GEO Alloys
Ltd in its financial statements for the year ended 30 June 2016, in
accordance with IAS 39 Financial instruments: recognition and
measurement.

15.7 CASCABEL LTD


Cascabel Ltd entered into a forward contract on 31 July 2016 to purchase A$.2
million at a contracted rate of Rs. 1: A$0.64 on 31 October 2016. The contract
cost was nil. Cascabel Ltd prepares its financial statements to 31 August 2016.
At 31 August 2016 an equivalent contract for the purchase of A$2 million could be
acquired at a rate of Rs. 1: A$0.70.
Required
(a) Explain how this financial instrument should be classified and prepare the
journal entry required for its measurement as at 31 August 2016.
(b) Assume now that the instrument described above was designated as a
hedging instrument in a cash flow hedge, and that the hedge was 100%
effective.
Explain how the gain or loss on the instrument for the year ended 31
August 2016 should now be recorded and why different treatment is
necessary.

15.8 Fair value hedge accounting


X Ltd holds an inventory of tin. At 30th September 2015 X Ltd decided to hedge
this position with a 6 month forward sale of 30 tonnes of the same grade of tin.
Relevant information
Tin inventory Forward
(Rs.) contract (Rs.)
Carrying amount of inventory at 30 Sept
1,000,000 N/A
2015, at the lower of cost and NRV
Fair values at 30 Sept 2015 1,300,000 NIL
Fair values at 31 Dec 2015 1,200,000 95,000
Fair values at 31 March 2016, when the tin is
1,150,000 142,000
sold and the contract is closed
Required
(a) Prepare journals for the year ended 31 December 2015
(b) Prepare the journals that are necessary at 31 March 2016

Emile Woolf International 65 The Institute of Chartered Accountants of Pakistan


Advanced accounting and financial reporting

15.9 CASH FLOW HEDGE ACCOUNTING


At 30th November 2015, the company decided to acquire a new drilling machine
from a foreign supplier. The machine is essential, and there is absolutely no
likelihood that the purchase will be delayed.
The price of the machine is A$400,000, payable upon delivery which is
anticipated to be 28 February 2016.
Spot rate a 30th November 2015 is Rs.0.70 = A$1.
In order to hedge the exchange rate risk, the company enters into a forward
foreign exchange contract to buy A$400,000 forward 3 months, at a rate of 0.70
(Rs. = A$1).
At 31 December 2015, the forward rate in the market for 2 month delivery is 0.75.
The machine was duly delivered on 28 February, and the exchange rate ruling on
that day of payment was 0.80. The forward contract was closed out. It is the
companys accounting policy to take any deferred gains/losses on a cash flow
hedge of the acquisition of a non-financial asset, against the cost of that asset (a
basis adjustment).
For both situations, ignore the effect of time value of money and transaction
costs.
Required
(a) Prepare journals for the year ended 31 December 2015
(b) Prepare the journals that are necessary at 28 February 2016)
(Note: A$ is a fictional currency used for the purposes of this example)

15.10 WATERS LTD


Waters Ltd acquired the following financial assets and liabilities in 2016.
1 On 1 September, Waters acquired 2,000 Rs. 100 nominal units of 7%
treasury stock 2022 for Rs. 104.10 per unit. The gross redemption yield at
the date of purchase was 6.30%. Waters does not intend to hold the
treasury stock until maturity, as the cash may be required in the meantime.
Interest is paid annually in arrears.
2 Waters buys and sells goods in Constantia, a country whose currency is the
Prif (PR). On 3 December Waters enters into a futures contract to sell
PR500,000 on 30 April 2017 at an agreed price of PR1.98/Rs. 1. This
contract is not part of a designated hedge. The cost of entering into the
contract was Rs. 750.
3 On 5 February Waters acquired 250,000 ordinary shares in Gilmour Ltd at
Rs. 4.85 per share incurring Rs. 35,000 attributable transaction costs.
4 On 1 July Waters sells goods to Mason for Rs. 500,000 on interest free
credit payable 30 June 2017. The imputed rate of interest is 11%.
5 On 30 April Waters acquired 1,000 Rs. 100 nominal units of 8.5% treasury
stock 2018 at Rs. 107.10 per unit. The gross redemption yield is 5.9%.
Waters intends to hold the investment to maturity. Interest is paid annually
in arrears.
6 On 26 December Waters purchased Rs. 25,000 of quoted company loan
notes. This asset has been designated as being held for short-term trading
purposes.

Emile Woolf International 66 The Institute of Chartered Accountants of Pakistan


Questions

7 On 24 December Waters sold 10,000 shares 'short' in Wright Ltd for Rs.
3.60 each, hoping that the share price would fall so that it could clear its
position by buying the shares in January 2017 at a lower price.
On 31 December 2016, the values are as follows:
1 Rs. 100 nominal units of 7% treasury stock 2022 are trading at Rs. 98.07
per unit at 31 December 2016. The gross redemption yield at that date is
7.3%.
2 The futures rate for a Prif contract with a delivery date of 30 April 2017 is
PR1.99/Rs. 1.
3 The shares in Gilmour are now trading at Rs. 5.20 Rs. 5.25 per share,
having an average of Rs. 5.05 during the year. Disposal costs would be 2%
of the sale proceeds.
4 Amounts receivable from Mason remain outstanding at the reporting date.
The imputed interest rate for current sales is 12%.
5 The 8.5% treasury stock 2018 is now trading at Rs. 101.50 per unit and the
gross redemption yield is currently quoted at 7.48%.
6 The loan notes are now worth Rs. 25,500 due to the market being more
confident that the interest will be paid in full and on time.
7 Shares in Wright Ltd are now trading at Rs. 3.30 each.
Required
Explain and calculate the impact of the above transactions on the financial
statements of Waters Ltd for the year ended 31 December 2016.

15.11 ARIF INDUSTRIES LIMITED


Arif Industries Limited (AIL) owns and operates a textile mill with spinning and
weaving units. Due to recurring losses, AIL disposed of the weaving unit for an
amount of Rs. 100 million on July 1, 2015 and invested the proceeds in Pakistan
Investment Bonds (PIBs).
Details of investment in PIBs are as follows:
(i) The PIBs were purchased through a commercial bank at face value. The
bank initially charged premium and investment handling charges of Rs.
4,641,483. At the time of purchase, AILs intention was to liquidate the
investment after four years and utilize the realized amount for expansion of
its spinning business. The bank has agreed to repurchase the PIBs on June
30, 2019, at their face value.
(ii) The markup on PIBs is 15% for the initial two years and 20% for the
remaining two years. The effective yield on investment at the time of
purchase was 15.50%.
Required
(a) Prepare an amortisation table showing the amortised cost and interest
income over the life of the loan asset.
Following on from the facts in part (a), suppose that on June 30, 2017 AIL
realized that the bank was in financial difficulties. Further investigation led the
company to believe that it would only receive 75% of the expected future cash
flows (both mark-up and capital).

Emile Woolf International 67 The Institute of Chartered Accountants of Pakistan


Advanced accounting and financial reporting

(b) Prepare an amortisation table showing the amortised cost and interest
income over the life of the loan after taking account of any necessary
impairment loss to the loan asset.
Following on from the facts in part (a) and ignoring those on part (b), suppose
that on June 30, 2017 AIL decided to defer the expansion plan by one year. The
bank agreed to extend the holding period accordingly and pay 20% interest in
year 5 but reduced the repurchase price by 2%.
Required
(c) Prepare an amortisation table showing the amortised cost and interest
income over the life of the loan after taking account of any necessary
adjustment to the carrying amount of the loan asset.

15.12 QASMI INVESTMENT LIMITED


On 1 January 2009 Qasmi Investment Limited (QIL) purchased 1 million 12%
Term Finance Certificates (TFCs) issued by Taj Super Stores (TSS), which
operates a chain of five Super Stores. The terms of the issue are as under:
The TFCs have a face value of Rs. 100 each and were issued at a discount of
5%. These are redeemable at a premium of 20% after five years.
Interest on the TFCs is payable annually in arrears on 31 December each year.
Effective interest rate calculated on the above basis is 16.426% per annum.
Due to a property dispute, TSS had to temporarily discontinue operations of two
stores in 2010. Consequently, TSS was unable to pay interest due on 31
December 2010 and 31 December 2011.
At the time of finalization of accounts for the year ended 31 December 2010, QIL
was quite hopeful of recovery of the interest and therefore, no impairment was
recorded. However, in 2011, after a thorough review of the whole situation, QILs
management concluded that it would be able to recover the face value of the
TFCs along with the premium on the due date i.e. 31 December 2013, but the
interest for the years 2010 to 2013 would not be received. Accordingly, QIL
recorded impairment in the value of the TFCs on 31 December 2011.
In 2012, TSS reached an out of court settlement of the property dispute and the
stores became operational. Subsequently, QIL and TSS agreed upon a revised
payment schedule according to which the present value of the agreed future cash
flows on 31 December 2012 is estimated at Rs. 115 million.
Required
Prepare journal entries in the books of QIL for the years ended 31 December
2011 and 2012.
Show all the relevant computations.

Emile Woolf International 68 The Institute of Chartered Accountants of Pakistan


Questions

CHAPTER 16: FINANCIAL INSTRUMENTS: PRESENTATION AND


DISCLOSURE

16.1 SERRANO LTD


On 1 October 2015 Serrano Ltd issued Rs. 10 million 6% convertible loan stock
on the following terms:
The issue price was at par.
The loan stock is convertible into the companys equity shares at the option of the
stockholders four years after the date of its issue (30 September 2019) on the
basis of 20 shares for each Rs. 100 of loan stock. Alternatively it will be
redeemed at par.
Ancho Services had advised that if Serrano Ltd had issued similar loan stock
without the conversion rights, then it would have had to pay an interest (coupon)
rate of 10% on the loan stock. This is because the terms of conversion to equity
shares are favourable.
Ancho Services further advised that because it is almost certain that the loan
stock holders will exercise their right to convert to equity shares, the loan stock
has the substance of equity and can be included as such on the statement of
financial position. This has the added advantage of improving/reducing the
companys gearing (debt/equity) in comparison to what would be the case with
the issue of straight loan stock.
The present value of Rs. 1 receivable at the end of each year, based on discount
rates of 6% and 10% can be taken as:
Required
In relation to the 6% convertible loan stock, calculate the finance cost to be
shown in the statement of profit or loss and the extracts from the statement of
financial position for the year to 30 September 2016; and comment on Ancho
Services advice.

16.2 POBLANO LTD


Poblano Ltd issued Rs. 10 million of 4% convertible loan notes on 1 October
2015, on which interest is paid annually in arrears on 30 September. The loan
notes are convertible into equity shares of Poblano Ltd on 30 September 2018 at
the rate of 20 shares in Poblano Ltd for every Rs. 100 of notes. Alternatively the
notes can be redeemed on that date for cash at par, at the option of the note
holder.
If Poblano Ltd had issued straight loan notes, redeemable at par after 3 years, it
would have had to pay interest at the rate of 7% in order to persuade investors to
subscribe for them.
Required
Show how the convertible loan notes would be accounted for in the financial
statements of Poblano Ltd for the year to 30 September 2016.

Emile Woolf International 69 The Institute of Chartered Accountants of Pakistan


Advanced accounting and financial reporting

16.3 PIQUIN LTD


(a) Piquin Ltd issued 10 million 5% convertible Rs. 1 bonds 2021 on 1 January
2016. The proceeds of Rs. 10 million were credited to non-current liabilities
and debited to bank. The 5% interest paid has been charged to finance
costs in the year to 31 December 2016.
The market rate of interest for a similar bond with a five year term but no
conversion terms is 7%.
Required
Explain AND demonstrate how this convertible instrument would be initially
measured in accordance with IAS 32 Financial Instruments: Presentation
AND subsequently measured in accordance with IAS 39 Financial
Instruments: Recognition and Measurement in the financial statements for
the year ended 31 December 2016.
(b) The directors of Piquin Ltd want to avoid increasing the gearing of the
entity. They plan to issue 5 million 6% cumulative redeemable Rs. 1
preference shares in 2017.
Required
Explain how the preference shares would be classified in accordance with
IAS 32 Financial Instruments: Presentation, AND the impact that this issue
will have on the gearing of Piquin Ltd.

16.4 AJI LTD


Aji Ltd issued Rs. 6 million 7% convertible bonds on 1 January 2016 at par. The
bonds are redeemable at par on 31 December 2019 or convertible at that date on
the basis of two Rs. 1 ordinary shares for every nominal Rs. 10 of bonds.
At the date of issue the prevailing market rate for similar debt without conversion
rights was 9%. The interest due was paid on 31 December 2016 and recorded
within finance costs during the year.
Required
(a) Explain how convertible instruments are initially recognised, in accordance
with IAS 32 Financial Instruments: Presentation, and prepare the journal
entry to record the issue of the bonds by Aji Ltd. (5 marks)

(b) (i) Explain how the bonds will be subsequently measured, in accordance
with IAS 39 Financial Instruments: Recognition and Measurement,
and prepare the journal entry to record the subsequent measurement
of the bonds in the financial statements of Aji Ltd for the year to 31
December 2016.
(ii) Prepare extracts that illustrate how the bonds will be presented in the
statement of financial position of Aji Ltd as at 31 December 2016.

Emile Woolf International 70 The Institute of Chartered Accountants of Pakistan


Questions

16.5 CHILTEPIN LTD


Chiltepin Ltd issued Rs. 4 million 5% convertible bonds on 1 October 2015 for Rs.
3.9 million. The bonds have a four year term and are redeemable at par. At the
time the bonds were issued the prevailing market rate for similar debt without
conversion rights was 7%. The effective interest rate associated with the bonds is
7% and the liability is measured, in accordance with IAS 39 Financial
Instruments: recognition and measurement, at amortised cost. The interest due
was paid and recorded within finance costs during the year.
Required
Prepare the accounting entries to record the issue of the convertible bonds and to
record the adjustment required in respect of the interest expense on the bonds for
the year ended 30 September 2016.

16.6 HABENERO LTD


Habenero Ltd issued 6 million 5% cumulative Rs. 1 preference shares on 1
January 2016 and 2 million Rs. 1 ordinary shares on 1 May 2016.
Required
(a) Explain, with reference to the principles of IAS 32 Financial Instruments:
Presentation how both of these instruments would be classified AND how
their associated dividends would be recorded in the financial statements of
Habenero Ltd for the year ended 31 December 2016.
Habenero Ltd acquired 500,000 shares in X on 1 November 2016 for Rs. 2.80 per
share and classified this investment as held for trading. Habenero Ltd paid 0.5%
commission on the value of the transaction to its broker. Xs shares were trading
at Rs. 3.42 on 31 December 2016.
Required
Prepare the journal entries to record:
(i) the initial measurement of the investment at 1 November 2016; and
(ii) the subsequent measurement of this investment in the financial
statements of Habenero Ltd at 31 December 2016.

Emile Woolf International 71 The Institute of Chartered Accountants of Pakistan


Advanced accounting and financial reporting

CHAPTER 18 IAS 12: INCOME TAXES

18.1 SHAKIR INDUSTRIES


Given below is the statement of profit or loss and other comprehensive income
of Shakir Industries for the year ended December 31, 2016:
2016
Rs. m
Sales 143.00
Cost of goods sold (96.60)
Gross profit 46.40
Operating expenses (28.70)
Operating profit 17.70
Other income 3.40
Profit before interest and tax 21.10
Financial charges (5.30)
Profit before tax 15.80
Following information is available:
(i) Operating expenses include an amount of Rs.0.7 million paid as penalty to
SECP on non-compliance of certain requirements of the Companies
Ordinance, 1984.
(ii) During the year, the company made a provision of Rs. 2.4 million for
gratuity. The actual payment on account of gratuity to outgoing members
was Rs. 1.6 million.
(iii) Lease payments made during the year amounted to Rs.0.65 million
which include financial charges of Rs.0.15 million. As at December 31,
2016, obligations against assets subject to finance lease stood at Rs. 1.2
million. The movement in assets held under finance lease is as follows:
Rs. m
Opening balance 01/01/2016 2.50
Depreciation for the year (0.7)
Closing balance 31/12/2016 1.80
(iv) The details of owned fixed assets are as follows:
Accounting Tax
Rs. m Rs. m
Opening balance 01/01/2016 12.50 10.20
Purchased during the year 5.3 5.3
Depreciation for the year (1.10) (1.65)
Closing balance 31/12/2016 16.70 13.85

(v) Capital work-in-progress as on December 31, 2016 include financial


charges of Rs. 2.3 million which have been capitalised in accordance with
IAS-23 Borrowing Costs. However, the entire financial charges are
admissible, under the Income Tax Ordinance, 2002.

Emile Woolf International 72 The Institute of Chartered Accountants of Pakistan


Questions

(vi) Deferred tax liability and provision for gratuity as at January 1, 2016 was
Rs.0.55 million and Rs.0.7 million respectively.
(vii) Applicable income tax rate is 35%.
Required
Based on the available information, compute the current and deferred tax
expenses for the year ended December 31, 2016.

18.2 DWAYNE LTD (PART 1)


The following information has been obtained in order to allow completion of
Dwayne Ltds deferred tax balances as at 31st December 2015.
Statement of financial position at 31st December 2015 - Extracts
Carrying amount Tax base
Rs. 000 Rs. 000
Assets
Land & buildings 45,500 17,500
Plant and equipment 68,000 26,000
Cost of investment in Larry 750 750
Investments 72,000 65,000
Dividend receivable 150 -

Liabilities
Long-term debt 20,500 21,000
Trade payables 9,500 9,500
Defined benefit liability 1,000
Deferred tax liability
(31st December 2014) 13,500
(i) Dwayne revalues its land and buildings on an annual basis. It has no
investment properties. The fair value of land and buildings was Rs. 60
million at 31st December 2015.The 2015 revaluation has not yet been
accounted for in Dwaynes financial statements. The pre-tax revaluation
surplus as at 31st December 2014 stood at Rs. 24m.
(ii) The balance on the investments line relates to a portfolio of equity holdings.
Some of these are categorised as fair value through profit or loss and the
balance as available-for-sale. The fair value loss on AFS investments was
Rs. 1m during 2015. This loss is considered to be temporary in nature. The
entire portfolio of equity holdings was acquired during 2015.
(iii) Tax relief on the defined benefit expense is given on a cash basis.
(iv) Dividend income is not taxed in the jurisdiction in which Dwayne operates.
(v) Dwayne borrowed Rs. 21m just before the year end and incurred
transaction costs of 500k. Transaction costs are allowable in full in the year
in which a loan is raised.
(vi) The tax rate changed from 30% to 28% in the current year.
Required
(a) Prepare a schedule of temporary differences and resultant deferred tax for
Dwayne.
(b) Prepare a note showing the movement on the consolidated deferred tax
balance for the year ending 31st December 2015.

Emile Woolf International 73 The Institute of Chartered Accountants of Pakistan


Advanced accounting and financial reporting

(c) Prepare a journal showing the movement on the deferred taxation account
showing the entries due to rate changes and temporary differences arising
during the period.

18.3 DWAYNE LTD (PART 2)


The investment in Dwaynes statement of financial position is the cost of 80% of
Larry. The date of this acquisition was 31st December 2015.
The following statement of net assets relates to Larry on 31st December 2015.
Carrying
Fair value Tax base
amount
Rs000 Rs000 Rs000
Buildings 600 400 300
Plant and equipment 56 46 25
Inventory 152 162 144
Trade receivables 120 120 120
Defined benefit liability (100) (150)
Current liabilities (50) (50) (50)
778 528 539
Required
(a) Prepare a schedule of temporary differences and resultant deferred tax for
Larry from the point of view of the group.
(b) Combine the deferred tax figures to obtain the group deferred tax balance.
(c) Prepare a note showing the movement on the consolidated deferred tax
balance for the year ending 31st December 2015.
(d) Calculate the goodwill arising on acquisition of Larry.

18.4 COHORT
Cohort is a private limited company and has two 100% owned subsidiaries,
Legion and Air, both themselves private limited companies. Cohort acquired Air
on 1 January 20X2 for Rs. 5 million when the fair value of the net assets was Rs.
4 million, and the tax base of the net assets was Rs. 3.5 million. The acquisition
of Air and Legion was part of a business strategy whereby Cohort would build up
the value of the group over a three-year period and then list its share capital on
the Stock Exchange.
(a) The following details relate to the acquisition of Air, which manufactures
electronic goods:
(i) Part of the purchase price has been allocated to intangible assets
because it relates to the acquisition of a database of key customers of
Air. The recognition and measurement criteria for an intangible asset
under IFRS 3 Business Combinations and IAS 38 Intangible Assets
do not appear to have been met but the directors feel that the
intangible asset of Rs. 500,000 will be allowed for tax purposes and
have computed the tax provision accordingly. However, the tax
authorities could possibly challenge this opinion.
(ii) Air has sold goods worth Rs. 3 million to Cohort since acquisition and
made a profit of Rs. 1 million on the transaction. The inventory of

Emile Woolf International 74 The Institute of Chartered Accountants of Pakistan


Questions

these goods recorded in Cohorts statement of financial position at the


year ending 31May 20X2 was Rs. 1.8 million.
(iii) The retained earnings of Air at acquisition were Rs. 2 million. The
directors of Cohort have decided that, during the three years leading
up to the date that they intend to list the shares of the company, they
will realise earnings through future dividend payments from the
subsidiary amounting to Rs. 500,000 per year. Tax is payable on any
remittance of dividends and no dividends have been declared for the
current year.
(b) Legion was acquired on 1 June 20X1 and is a company which undertakes
various projects ranging from debt factoring to investing in property and
commodities. The following details relate to Legion for the year ending 31
May 20X2:
(i) Legion has a portfolio of readily marketable government securities
which are held as current assets. These investments are stated at
market value in the statement of financial position with any gain or
loss taken to profit or loss. These gains and losses are taxed when
the investments are sold. Currently the accumulated unrealised gains
are Rs. 4 million.
(ii) Legion has calculated that it requires a general allowance of Rs. 2
million against its total loan portfolio. Tax relief is available when the
specific loan is written off. Management feel that this part of the
business will expand and thus the amount of the general provision will
increase.
(iii) When Cohort acquired Legion it had unused tax losses brought
forward. At 1 June 20X1, it appeared that Legion would have
sufficient taxable profit to realise the deferred tax asset created by
these losses but subsequent events have proven that the future
taxable profit will not be sufficient to realise all of the unused tax loss.
Impairment of goodwill is not allowed as a deduction in determining taxable
profit.
Required
Write a note suitable for presentation to the partner of an accounting firm
setting out the deferred tax implications of the above information for the
Cohort Group of companies.

18.5 MODEL TOWN GROUP


The following statement of financial position relates to Model Town Group, a
public limited company at 30 June 2016:
Rs.000
Assets:
Non-current assets:
Property, plant, and equipment 10,000
Goodwill 6,000
Other intangible assets 5,000
Financial assets (cost) 9,000
30,000

Emile Woolf International 75 The Institute of Chartered Accountants of Pakistan


Advanced accounting and financial reporting

Rs.000
Trade receivables 7,000
Other receivables 4,600
Cash and cash equivalents 6,700
18,300

Total assets 48,300

Equity and liabilities


Share capital 9,000
Other reserves 4,500
Retained earnings 9,130
Total equity 22,630
Non-current liabilities
Long term borrowings 10,000
Deferred tax liability 3,600
Employee benefit liability 4,000
Total non-current liabilities 17,600
Current tax liability 3,070
Trade and other payables 5,000
Total current liabilities 8,070
Total liabilities 25,670

Total equity and liabilities 48,300

The following information is relevant to the above statement of financial position:

(i) The financial assets are investments in equity. Model Town has made an
irrevocable election to recognise gains and losses on these assets in other
comprehensive income. However, they are shown in the above statement
of financial position at their cost on 1 July 2015. The market value of the
assets is Rs. 10.5 million on 30 June 2016. Taxation is payable on the sale
of the assets.
(ii) The stated interest rate for the long term borrowing is 8 per cent. The loan
of Rs. 10 million represents a convertible bond which has a liability
component of Rs. 9.6 million and an equity component of Rs.0.4 million.
The bond was issued on 30 June 2016.
(iii) The tax bases of the assets and liabilities are the same as their carrying
amounts in the statement of financial position at 30 June 2016 except for
the following:
(a) Rs.000
Property, plant, and equipment 2,400
Trade receivables 7,500
Other receivables 5,000
Employee benefits 5,000
(b) Other intangible assets were development costs which were all
allowed for tax purposes when the cost was incurred in 2015.

Emile Woolf International 76 The Institute of Chartered Accountants of Pakistan


Questions

(c) Trade and other payables include an accrual for compensation to be


paid to employees. This amounts to Rs. 1 million and is allowed for
taxation when paid.

(iv) Goodwill is not allowable for tax purposes in this jurisdiction.


(v) Assume taxation is payable at 30%.

Required
Calculate the provision for deferred tax at 30 June 2016 after any necessary
adjustments to the financial statements showing how the provision for deferred
taxation would be dealt with in the financial statements.
(Assume that any adjustments do not affect current tax. You should briefly
discuss the adjustments required to calculate the provision for deferred tax).

Emile Woolf International 77 The Institute of Chartered Accountants of Pakistan


Advanced accounting and financial reporting

CHAPTER 19: BUSINESS COMBINATIONS AND CONSOLIDATION

19.1 HELLO
On 1 January 2015, Hello acquired 60% of the ordinary share capital of Solong
for Rs. 110,000. At that date Solong had a retained earnings balance of Rs.
60,000.
The following statements of financial position have been prepared as at 31
December 2016.
Hello Solong
Rs. Rs.
Assets
Non-current assets
Property, plant and equipment 225,000 175,000
Investments in Solong 110,000

Current assets 271,000 157,000



606,000 332,000

Equity and liabilities


Capital and reserves
Share capital 100,000 100,000
Retained earnings 275,000 90,000

375,000 190,000
Current liabilities 231,000 142,000

606,000 332,000

The fair value of Solongs net assets at the date of acquisition was determined to
be Rs. 170,000.
The difference between the book value and the fair value of the new assets at the
date of acquisition was due to an item of plant which had a useful life of 10 years
from the date of acquisition.
Required
Prepare the consolidated statement of financial position of Hello and its
subsidiary as at 31 December 2016.

Emile Woolf International 78 The Institute of Chartered Accountants of Pakistan


Questions

19.2 HASAN LIMITED


On 1 April 2015, Hasan Limited acquired 90% of the equity shares in Shakeel
Limited. On the same day Hasan Limited accepted a 10% loan note from Shakeel
Limited for Rs. 200,000 which was repayable at Rs. 40,000 per annum (on 31
March each year) over the next five years. Shakeel Limiteds retained earnings at
the date of acquisition were Rs. 2,200,000.
Statements of financial position as at 31 March 2016
Hasan Limited Shakeel Limited
Rs.000 Rs.000 Rs.000 Rs.000
Non-current assets
Property, plant and equipment 2,120 1,990
Intangible software 1,800
Investments equity in Shakeel 4,110
Limited
Investments 10% loan note 200
Shakeel Limited
Investments others 65 210
6,495 4,000
Current assets
Inventories 719 560
Trade receivables 524 328
Shakeel Limited current account 75
Cash 20
1,338 888
Total assets 7,833 4,888

Equity and liabilities:


Capital and reserves
Equity shares of Rs. 1 each 2,000 1,500
Share premium 2,000 500
Retained earnings 2,900 1,955
6,900 3,955
Non-current liabilities
10% Loan note from Hasan Limited 160
Government grant 230 40
230 200
Current liabilities
Trade payables 475 472
Hasan Limited current account 60
Income taxes payable 228 174
Operating overdraft 27
703 733
Total equity and liabilities 7,833 4,888

Emile Woolf International 79 The Institute of Chartered Accountants of Pakistan


Advanced accounting and financial reporting

The following information is relevant:


(i) Included in Shakeel Limiteds property at the date of acquisition was a
leasehold property recorded at its depreciated historical cost of Rs.
400,000. The leasehold had been sub-let for its remaining life of only four
years at an annual rental of Rs. 80,000 payable in advance on 1 April each
year. The directors of Hasan Limited are of the opinion that the fair value of
this leasehold is best reflected by the present value of its future cash flows.
An appropriate cost of capital for the group is 10% per annum.
The present value of a Rs. 1 annuity received at the end of each year
where interest rates are 10% can be taken as:
3 year annuity Rs. 2.50
4 year annuity Rs. 3.20
(ii) The software of Shakeel Limited represents the depreciated cost of the
development of an integrated business accounting package. It was
completed at a capitalised cost of Rs. 2,400,000 and went on sale on 1
April 2014. Shakeel Limiteds directors are depreciating the software on a
straight-line basis over an eight-year life (i.e. Rs. 300,000 per annum).
However, the directors of Hasan Limited are of the opinion that a five-year
life would be more appropriate as sales of business software rarely exceed
this period.
(iii) The inventory of Hasan Limited on 31 March 2016 contains goods at a
transfer price of Rs. 25,000 that were supplied by Shakeel Limited who had
marked them up with a profit of 25% on cost. Unrealised profits are
adjusted for against the profit of the company that made them.
(iv) On 31 March 2016 Shakeel Limited remitted to Hasan Limited a cash
payment of Rs. 55,000. This was not received by Hasan Limited until early
April. It was made up of an annual repayment of the 10% loan note of Rs.
40,000 (the interest had already been paid) and Rs. 15,000 of the current
account balance.
(v) The accounting policy of Hasan Limited for non-controlling interests (NCI) in
a subsidiary is to value NCI at a proportionate share of the net assets.
(v) An impairment test at 31 March 2016 on the consolidated goodwill
concluded that it should be written down by Rs. 120,000. No other assets
were impaired.
Required
Prepare the consolidated statement of financial position of Hasan Limited as at
31 March 2016.

Emile Woolf International 80 The Institute of Chartered Accountants of Pakistan


Questions

19.3 FLAMSTEED LTD AND HALLEY LTD


The draft Statement of Financial Position of Flamsteed Ltd and Halley Ltd on 30
June 2016 were as follows:
Statement of financial position as at 30 June 2016
Flamsteed Halley Ltd
Ltd
Rs.000 Rs.000
Assets:
Non-current Assets:
Property, plant and equipment 100,000 80,000
40,000 ordinary shares in Halley at cost 60,000 -
160,000 80,000
Current assets:
Inventory 6,000 16,000
Owed by Flamsteed Ltd - 20,000
Receivables 32,000 14,000
Cash 4,000 -
42,000 50,000
Total assets 202,000 130,000
Equity and liabilities:
Equity (ordinary shares @ Rs. 1) 90,000 50,000
Revaluation surplus 24,000 10,000
Retained earnings 52,000 56,000
166,000 116,000
Current Liabilities:
Owed to Halley Ltd 16,000 -
Trade payables 20,000 14,000
36,000 14,000
Total equity and liabilities 202,000 130,000
Additional information:
(i) Flamsteed Ltd acquired its investment in Halley Ltd on 1 July 2015, when
the retained earnings of Halley Ltd stood at Rs. 12,000,000.
(ii) The agreed consideration was Rs. 60,000,000 at the date of acquisition and
a further Rs. 20,000,000 on 1 July 2017, Flamsteed Ltds cost of capital is
7%.
(iii) Halley Ltd has an internally developed brand name TOLX which was
valued at Rs. 10,000,000 at the date of acquisition.
(iv) There have been no changes in the capital or revaluation surplus of Halley
Ltd since the date its shares were purchased.
(v) At 30 June 2016, Halley had invoiced Flamsteed Ltd for goods to the value
of Rs. 4,000,000 and Flamsteed Ltd had sent payment in full but this had
not been received by Halley Ltd.

Emile Woolf International 81 The Institute of Chartered Accountants of Pakistan


Advanced accounting and financial reporting

(vi) There is no impairment of goodwill.


(vii) It is the groups policy to value non-controlling interest at full fair value.
(viii) At the acquisition date, the non-controlling interest was valued at Rs.
18,000,000.
Required
(a) Define Impairment loss in accordance with IAS 36 on Impairment of Assets.

(b) Explain any THREE sources of external information which an entity may
consider in assessing whether there is any indication that an asset may be
impaired.
(c) Prepare an extract of consolidated Statement of Financial position of
Flamsteed Ltd for the year ended 30 June 2016, showing the assets side
only.

19.4 BRADLEY LTD


Bradley Ltds purchased 960 million shares in Bliss Ltd a year ago when Bliss
had a credit balance of Rs. 190million in retained earnings. The fair value of the
non-controlling interest at the date of acquisition was Rs. 330million. At the date
of acquisition, the freehold land of Bliss Ltd was valued at Rs. 140million in
excess of its carrying value. The revaluation has not been recorded in the
accounts of Bliss.
The statements of financial position of Bradley Ltd and Bliss Ltd as at 31
December 2016 are as follows:
Bradley Ltd Bliss Ltd
Rs. Rs. Rs. Rs.
million million million million
Non Current Assets
Land and building 630 556
Machinery and equipment 570 440
Investment in Bliss Ltd. 1,320 -
2,520 996
Current Assets
Inventories 714 504
Trade receivables 1,050 252
Cash/bank 316 2,080 60 816
4,600 1,812

Ordinary Shares at Rs. 1 each 3,000 1,200


Retained Earnings 1,160 424
Shareholders fund 4,160 1,624

Current Liabilities
Trade payables 440 188
4,600 1,812

Emile Woolf International 82 The Institute of Chartered Accountants of Pakistan


Questions

Bliss Ltd owes Bradley Ltd Rs. 50million for goods purchased during the year.
Inventory of Bliss Ltd includes goods bought from Bradley Ltd at the price that
includes a profit to Bradley Ltd of Rs. 24million.
The management of Bradley Ltd wants the financial statements to be
consolidated using the acquisition method and wishes to know whether there is
goodwill on acquisition of Bliss Ltd and the amount involved.
Required
Prepare the consolidated statement of financial position as at 31 December 2016.

19.5 X LTD
The statements of financial position for X Ltd and Y Ltd as at 31 December 2016
are provided below:
X Ltd Y Ltd
ASSETS Rs.000 Rs.000
Non-current assets
Property, plant and equipment 12,000 4,000
Available for sale investment (note 1) 4,000 -
Current assets 16,000 4,000
Inventories 2,200 800
Receivables 3,400 900
Cash and cash equivalents 800 300
6,400 2,000
Total assets 22,400 6,000
EQUITY AND LIABILITIES Equity
Share capital (Rs. 1 equity shares) 10,000 1,000
Retained earnings 7,500 4,000
Other reserves 200 -
Total equity 17,700 5,000
Non-current liabilities
Long term borrowings 2,700 -
Current liabilities 2,000 1,000
Total liabilities 4,700 1,000
Total equity and liabilities 22,400 6,000
Additional information:
1. X Ltd acquired a 75% investment in Y Ltd on 1 May 2016 for Rs. 3,800,000.
The investment has been classified as available-for-sale in the books of X
Ltd. The gain on its subsequent measurement as at 31 December 2016 has
been recorded within other reserves in X Ltds individual financial
statements. At the date of acquisition Y Ltd had retained earnings of Rs.
3,200,000.
2. It is the group policy to value non-controlling interest at fair value at the date
of acquisition. The fair value of the non-controlling interest at 1 May 2016
was Rs. 1,600,000.
3. As at 1 May 2016 the fair value of the net assets acquired was the same as
the book value with the following exceptions:

Emile Woolf International 83 The Institute of Chartered Accountants of Pakistan


Advanced accounting and financial reporting

The fair value of property, plant and equipment was Rs. 800,000 higher
than the book value. These assets were assessed to have an estimated
useful life of 16 years from the date of acquisition. A full years depreciation
is charged in the year of acquisition and none in the year of sale.
The fair value of inventories was estimated to be Rs. 200,000 higher than
the book value. All of these inventories were sold by 31 December 2016.
On acquisition X Ltd identified an intangible asset that Y Ltd developed
internally but which met the recognition criteria of IAS 38 Intangible Assets.
This intangible asset is expected to generate economic benefit from the
date of acquisition until 31 December 2017 and was valued at Rs. 150,000
at the date of acquisition.
A contingent liability, which had a fair value of Rs. 210,000 at the date of
acquisition, had a fair value of Rs. 84,000 at 31 December 2016.
4. An impairment review was conducted at 31 December 2016 and it was
decided that the goodwill on the acquisition of Y Ltd was impaired by 20%.
5. X Ltd sold goods to Y Ltd for Rs. 300,000. Half of these goods remained in
inventories at 31 December 2016. X Ltd makes 20% margin on all sales.
6. No dividends were paid by either entity in the year ended 31 December
2016.
Required
Prepare the consolidated statement of financial position as at 31 December 2016
for the X Ltd Group.

19.6 KHAN LIMITED


On January 1, 2010, Khan Limited (KL) acquired 375 million ordinary shares
and 40 million preference shares in Gul Limited (GL) whose general
reserve and retained earnings on the date of acquisition, stood at Rs. 200
million and Rs. 1,000 million respectively.
The following balances were extracted from the records of KL and its
subsidiary on December 31, 2016:
KL GL
Dr Cr Dr Cr
Rs. m Rs. m Rs. m Rs. m
Ordinary share capital (Rs. 10 each) - 6,800 5,000
12% Preference share capital (Rs. 10 each) - - - 1,000
General reserve - 1,750 - 500
Retained earnings - 2,000 - 1,200
Loan from KL at 15% rate of interest - - - 2,000
14% Term Finance Certificates (TFCs) (Rs. - 2,250 - -
100 each)
Accounts payable - 445 - 190
Dividend payable preference shares - - - 60
Dividend payable ordinary shares - 750 - 300
Property, plant and equipment - at cost 16,250 - 25,000 -
Property, plant and equipment - acc. - 9,750 - 17,000
depreciation
Investment in ordinary shares of GL 5,500 - - -

Emile Woolf International 84 The Institute of Chartered Accountants of Pakistan


Questions

KL GL
Dr Cr Dr Cr
Rs. m Rs. m Rs. m Rs. m
Investment in preference shares of GL 400 - - -
Loan to GL at 15% rate of interest 2,000 - - -
Investment in KL's TFCs - - 1,500 -
(purchased at par value)
Profit before tax, interest and dividend - 2,865 - 1,550
Dividend income - 273 - -
Interest income - 300 - 210
Dividend receivable 249 - - -
Current assets 1,069 - 1,316 -
Interest on TFCs 315 - - -
Interest on loan from KL - - 300 -
Taxation 650 - 474 -
Preference dividend - - 120 -
Ordinary dividend interim 750 - 300 -
27,183 27,183 29,010 29,010
Following relevant information is available:
(i) At the date of acquisition, the fair value of buildings, included in property,
plant and equipment of GL was assessed at Rs. 1,000 million above its
carrying value. All other identifiable assets and liabilities were considered
to be fairly valued. GL provides for depreciation on buildings at 10% per
annum on the straight line basis.
(ii) GL purchased the TFCs in KL on January 1, 2016.
(iii) The non-controlling interests are measured at their proportionate share of
the GLs identifiable net assets.
(iv) There is no impairment in the value of goodwill since its acquisition.
(v) There are no components of other comprehensive income.
Required
Prepare the following in accordance with the requirements of International
Financial Reporting Standards:
(a) Consolidated statement of financial position as at December 31, 2016.
(b) Consolidated statement of comprehensive income for the year ended
December 31, 2016.
(c) Consolidated statement of retained earnings for the year ended December
31, 2016.
Note:
Ignore deferred tax and corresponding figures.
Notes to the above statements are not required. However, show workings
wherever it is necessary.

Emile Woolf International 85 The Institute of Chartered Accountants of Pakistan


Advanced accounting and financial reporting

CHAPTER 20: CONSOLIDATED STATEMENTS OF PROFIT OR LOSS AND


OTHER COMPREHENSIVE INCOME

20.1 MILLARD LTD


The profit and loss account of Millard Ltd and its subsidiary Fillmore Limited for
the year ended 31 December 2016 are as follows:
Millard Fillmore
Ltd Ltd
Rs.000 Rs.000
Revenue 312,500 125.000
Cost of Sales (125,000) (50,000)
Gross Profit 187,500 75,000
Distribution Cost (25,000) (10,000)
Administrative Expenses (20,000) (8,000)
Operating Profit 142,500 57,000
Investment Income 7,950 -
Debenture Interest (47,500) (15,000)
Profit on ordinary activities before taxation 102,950 42,000
Taxation on ordinary activities (35,000) (17,500)
Profit on ordinary activities after taxation 67,950 24,500
Dividends:
Preference (13,750) (4,375)
Ordinary (20,000) (5,250)
Retained Profits 34,200 14,875
Retained Profits: 1/1/2016 66,750 19,500
Retained Profits: 31/12/2016 100,950 34,375

Additional information:
(1) Included in the revenue of Fillmore Limited is Rs. 12.5million in respect of
sales to Millard Ltd, giving Fillmore Limited a profit of 25% on cost. These
are sales of components that Fillmore Limited has been supplying to Millard
Ltd on a regular basis for a number of years. The amounts included in the
inventories of Millard Ltd in respect of goods purchased from Fillmore
Limited at the beginning and end of the year were as follows:
Inventories of components in Millard Ltds books
Date Rs.000
31/12/2016 2,000
31/12/2015 1,500
(2) Some years ago, Millard Ltd bought 50 million ordinary shares in Fillmore
Limited at a cost of Rs. 67million. On the same date, Millard Ltd bought
25% of the debentures of Fillmore Limited at par.

Emile Woolf International 86 The Institute of Chartered Accountants of Pakistan


Questions

At the date of Millard Ltds investment in Fillmore Limited, the statement of


financial position of Fillmore limited showed:
Rs.000
Ordinary share capital 62,500
Preference share capital 43,750
Profit and loss account 12,500
118,750
The goodwill acquired by Millard Ltd in Fillmore Limited had been written off fully
in December 2016 as a result of impairment losses.
Required
Prepare the consolidated profit and loss account of Millard Ltd for the year.
Assume that investment income is dealt with by Millard Ltd on an accrual basis.
(15 marks)

20.2 SHERLOCK LTD


The following draft financial statements relate to Sherlock Ltd and its subsidiaries.
Draft statements of profit or loss and other comprehensive income for the year ended
31 December 2016.
Sherlock Mycroft Katie
Ltd Ltd Ltd
Rs. m Rs. m Rs. m
Revenue 400 115 70
Cost of sales (312) (65) (36)
Gross profit 88 50 34
Other income 21 7 2
Administrative costs (15) (9) (12)
Other expenses (35) (19) (8)
Operating profit 59 29 16
Finance costs (5) (6) (4)
Finance income 6 5 8
Profit before tax 60 28 20
Income tax expense (19) (9) (5)
Profit for the year 41 19 15
Other comprehensive income
revaluation surplus 10
Total comprehensive income for year 51 19 15
The following information is relevant to the preparation of the group statement of profit
or loss and other comprehensive income:
1. On 1 January 2015, Sherlock Ltd acquired 60% of the equity interests of
Mycroft Ltd. The purchase consideration comprised cash of Rs. 80 million.
The fair value of the identifiable net assets acquired was Rs. 110 million at
that date. The excess of the fair value of the identifiable net assets at
acquisition is due to non-depreciable land.

Emile Woolf International 87 The Institute of Chartered Accountants of Pakistan


Advanced accounting and financial reporting

Sherlock Ltd measures the non-controlling interest at acquisition at its fair


value. The fair value of the non-controlling interest (NCI) in Mycroft Ltd was
Rs. 45 million on 1 January 2015.
Goodwill has been impairment tested annually and as at 31 December
2016 had reduced in value by 20%. At 31 December 2016, goodwill was
estimated to have a value of Rs. 2 million above its original value.
2. Sherlock Ltd acquired 60% of Katie Ltd on 30 June 2016. There has been
no impairment of goodwill since the date of acquisition.
3. Sherlock Ltd sold inventory to Mycroft Ltd for Rs. 12 million making a loss
of Rs. 2 million on the transaction. The sale was at fair value and Mycroft
Ltd still holds half of the inventory at the year end.
4. The following information relates to Sherlock Ltds pension scheme:
Rs. m
Plan assets at 1 January 2016 48
Defined benefit obligation at 1 January 2016 50
Service cost for year ended 31 December 2016 4
Discount rate at 1 January 2016 10%
Re-measurement loss in year ended 31 December 2016 2
Past service cost 1 January 2016 3
The pension costs have not yet been accounted for.
5. On 1 January 2015, Sherlock Ltd purchased plant for Rs. 12 million and this
is being depreciated using the straight line basis over 10 years with a zero
residual value.
Sherlock Ltd measures plant of this type using the revaluation model. At 31
December 2015, the asset was revalued to Rs. 13 million but at 31
December 2016, the value of the asset had fallen to Rs. 7 million. The
effect of the revaluation at 31 December 2016 had not yet been accounted
for but depreciation for the year has been charged.
6. On 1 January 2015, Sherlock Ltd made an award of 8,000 share Katie Ltds
to each of its seven directors. The condition attached to the award is that
the directors must remain employed by Sherlock Ltd for three years.
The fair value of each Katie Ltd at the grant date was Rs. 100 and the fair
value of each Katie Ltd at 31 December 2016 was Rs. 110. At 31
December 2015, it was estimated that three directors would leave before
the end of four years.
The estimate of directors who were going to leave was revised to one
director at 31 December 2016.
The share Katie Ltd expense for the year has not been included in profit or
loss for the current year and no directors had left by 31 December 2016.
7. A loss on an effective cash flow hedge of Mycroft Ltd of Rs. 3 million has
been included in the subsidiarys finance costs.
8. Any expense adjustments should be made in other expenses.
Required
Prepare a consolidated statement of profit or loss and other comprehensive
income for the year ended 31 December 2016 for the Sherlock Ltd Group. (Ignore
the deferred tax consequences of the above events)

Emile Woolf International 88 The Institute of Chartered Accountants of Pakistan


Questions

20.3 FAISAL LIMITED


Following is the summarised trial balance of Faisal Limited (FL) and its
subsidiaries, Saqib Limited (SL) and Ayaz Industries Limited (AIL) for the year ended
December 31, 2016:
FL SL AIL
Rs. in million

Cash and bank balances 4,920 660 2,700


Accounts receivable 6,240 2,460 6,580
Closing inventory 14,460 4,200 5,680
Investment in SL (at cost) 9,000 - -
Investment in AIL (at cost) 10,500 - -
Other investments 11,100 - -
Property, plant and equipment 22,500 3,480 5,940
Dividend paid 600
Cost of sales 49,200 18,000 21,000
Operating expenses 3,600 2,100 5,400
131,520 30,900 47,900
Accumulated depreciation 5,760 420 1,260
Ordinary share capital (Rs. 10
30,000 12,000 6,000
each)
Retained earnings opening 33,780 - 5,400
Sales 57,600 16,500 33,800
Accounts payable 2,760 1,980 1,440
Gain on sale of non-current
540 - -
assets
Dividend income 1,080 - -
131,520 30,900 47,900

Following additional information is available:


(i) SL was incorporated on February 1, 2016. 75% of the shares were
acquired by FL at par value on the same date.
(ii) FL acquired 80% of AIL on January 1, 2016
(iii) The following inter-company sales were made during the year 2016:

Included in Amount
Sales buyers closing receivable/payable Gross profit
inventories at year end % on sales
Rs. in million
FL to AIL 2,400 900 - 20
SL to AIL 1,800 600 800 10
AIL to FL 3,600 1,200 - 30

Emile Woolf International 89 The Institute of Chartered Accountants of Pakistan


Advanced accounting and financial reporting

(iv) The gain on sale of non-current assets includes a sale of an item of


property, plant and machinery by FL to SL. This transaction occurred on
July 1, 2016. SL. Details of the transaction are as follows:
Rs. in million
Sales value 144
Less: Cost of plant and machineries 150
Accumulated depreciation (60)
Carrying amount at date of sale 90
Gain on sale of plant 54
The plant and machinery was purchased originally by FL on July 1, 2014,
and was being depreciated on the straight line method over a period of five
years. SL computed depreciation thereon using the same method based on
the remaining useful life as at the date of the transfer.
(v) FL billed Rs. 100 million to each subsidiary for management services
provided during the year 2016 and credited it to operating expenses. The
invoices were paid on December 15, 2016.
(iv) Details of cash dividend are as follows:

Dividend
Date of declaration Date of payment %
FL Nov 25, 2016 Jan 5, 2017 20
AIL Oct 15, 2016 Nov 20, 2016 10

Required
Prepare the consolidated statement of financial position and the
consolidated statement of profit and loss of FL and its subsidiaries for the
year ended December 31, 2016. Ignore tax and corresponding figures.

20.4 GOLDEN LIMITED


Golden Limited (GL) is a listed company and has held shares in two
companies, Yellow Limited (YL) and Black Limited (BL), since July 1, 2014.
The details of acquisition of shares in these companies are as follows:
(A) GL acquired 18 million shares in YL at par, when YLs reserves were
Rs. 24 million. The acquisition was made by issuing four shares in GL for
every five shares in YL. The market price of GLs shares at July 1, 2014
was Rs. 20 per share. A fair value exercise was carried out for YLs assets
and liabilities at the time of its acquisition with the following results:
Book Value Fair Value
Rupees in million
Land 170 192
Machines 25 45
Investments 3 6
The remaining life of machine on acquisition was 5 years. The fair values of
the assets have not been accounted for in YLs financial statements.
(B) 6 million shares in BL were acquired for Rs. 12 per share in cash. At the
date of acquisition, the reserves of BL stood at Rs. 40 million.

Emile Woolf International 90 The Institute of Chartered Accountants of Pakistan


Questions

The summarized statements of profit or loss of the three companies for


the year ended June 30, 2016 are as follows:
GL YL BL
Rupees in million
Sales 875 350 200
Cost of sales (567) (206) (244)
Gross profit / (loss) 308 144 (44)
Selling expenses (33) (11) (15)
Administrative expenses (63) (40) (16)
Interest expenses (30) (22) (15)
Other income 65 - -
Profit/(loss) before tax 247 71 (90)
Income tax (73) (15) 8
Profit/(loss) for the period 174 56 (82)
The following relevant information is available:
(i) The share capital and reserves as at July 1, 2015 were as follows:
GL YL BL
Rupees in million
Ordinary share capital of Rs. 10 each 600 200 150
Reserves 652 213 108
The share capitals of all companies have remained unchanged since their
incorporation.
(ii) During the year, GL sold goods amounting to Rs. 40 million to YL. The
sales were made at a mark-up of 25% on cost. 30% of these goods were
still in the inventories of YL at June 30, 2016.
(iii) GL manufactures a component used by BL. During the year, GL sold these
components amounting to Rs. 20 million to BL. Transfers are made at
cost plus 15%. BL held Rs. 11.5 million of these components in inventories
at June 30, 2016.
(iv) All assets are depreciated on straight line method.
(v) Other income includes dividend received from YL on April 15, 2016.
(vi) During the year, YL paid 20% cash dividend to its ordinary shareholders.
(vii) An impairment test was carried out on June 30, 2016 for the goodwill of
YL and investments in BL, appearing in the consolidated financial
statements. The test indicated that:
- goodwill of YL was impaired by 20%;
- due to recent losses, the fair value of investment in BL has been
reduced to Rs. 40 million.
No such impairment was required in previous years.
Required
Prepare, in a format suitable for inclusion in the annual report, a consolidated
statement of profit or loss for the year ended June 30, 2016.

Emile Woolf International 91 The Institute of Chartered Accountants of Pakistan


Advanced accounting and financial reporting

CHAPTER 21: ASSOCIATES AND JOINT VENTURES

21.1 JOINT ARRANGEMENTS


(a) State and explain the TWO types of joint arrangement identified in IFRS 11

(b) A joint operator is expected to recognise and account for certain elements
in relation to the joint operations. State FIVE elements to be recognised.
(c) State TWO characteristics of a joint arrangement.

21.2 HELIUM
The draft statements of financial position as at 31 December 2016 of three
companies are set out below.
Helium Sulphur Arsenic
Rs.000 Rs.000 Rs.000
Assets
Non-current assets
Property, plant and equipment 400 100 160
Investments:
- shares in Sulphur (60%) 75
- shares in Arsenic (30%) 30

Current assets 445 160 80



950 260 240

Equity and liabilities
Share capital 100 30 60
Retained earnings 650 180 100
Non-current loans 200 50 80

950 260 240

The reserves of Sulphur and Arsenic when the investments were acquired were
Rs. 70,000 and Rs. 30,000 respectively
Required
Prepare the consolidated statement of financial position as at 31 December 2016.

21.3 HAMACHI LTD


Hamachi Ltd acquired 90% of Saba Ltds Rs. 1 ordinary shares on 1 April 2014
paying Rs. 3.00 per share. The balance on Saba Ltds retained earnings at this
date was Rs. 800,000. On 1 October 2015, Hamachi Ltd acquired 30% of Anogo
Ltds Rs. 1 ordinary shares for Rs. 3.50 per share. The statements of financial
position of the three companies at 31 March 2016 are shown below:
Hamachi Ltd Saba Ltd Anogo Ltd
Rs.000 Rs.000 Rs.000 Rs.000 Rs.000 Rs.000
Non-current assets
Property, plant and 8,050 3,600 1,650

Emile Woolf International 92 The Institute of Chartered Accountants of Pakistan


Questions

Hamachi Ltd Saba Ltd Anogo Ltd


Rs.000 Rs.000 Rs.000 Rs.000 Rs.000 Rs.000
equipment
Investments 4,000 910 nil
12,050 4,510 1,650
Current assets
Inventory 830 340 250
Accounts receivable 520 290 350
Bank 240 nil 100
1,590 630 700
Total assets 13,640 5,140 2,350

Equity and liabilities


Equity:
Ordinary shares of Rs. 1 5,000 1,200 600
each
Reserves:
Retained earnings b/f 6,000 1,400 800
Profit year to 31 March
2016 1,500 900 600
7,500 2,300 1,400
12,500 3,500 2,000
Non-current liabilities
10% Loan notes 500 240 nil
Current liabilities
Accounts payable 420 960 200
Taxation 220 250 150
Overdraft nil 190 nil
640 1,400 350
Total equity and liabilities 13,640 5,140 2,350

The following information is relevant


(i) Fair value adjustments
On 1 April 2014 Saba Ltd owned an investment property that had a fair
value of Rs. 120,000 in excess of its carrying value (book value). The value
of this property has not changed since acquisition. This property is included
within investments in the balance sheet.
Just prior to its acquisition, Saba Ltd was successful in applying for a six-
year licence to dispose of hazardous waste. The licence was granted by the
government at no cost, however Hamachi Ltd estimated that the licence
was worth Rs. 180,000 at the date of acquisition.
(ii) In January 2016 Hamachi Ltd sold goods to Anogo Ltd for Rs. 65,000.
These were transferred at a mark-up of 30% on cost. Two thirds of these
goods were still in the inventory of Anogo Ltd at 31 March 2016.
(iii) To facilitate the consolidation procedures the group insists that all inter
company current account balances are settled prior to the year-end.
However a cheque for Rs. 40,000 from Saba Ltd to Hamachi Ltd was not
received until early April 2016. Inter company balances are included in
accounts receivable and payable as appropriate.
(iv) Anogo Ltd is to be treated as an associated company of Hamachi Ltd.

Emile Woolf International 93 The Institute of Chartered Accountants of Pakistan


Advanced accounting and financial reporting

(v) An impairment test at 31 March 2016 on the consolidated goodwill of Saba


Ltd concluded that it should be written down by Rs. 468,000. No other
assets were impaired.
Required
(a) Prepare the consolidated statement of financial position of Hamachi Ltd as
at 31 March 2016.
(b) Discuss the matters to consider in determining whether an investment in
another company constitutes associated company status.

21.4 HIDE
Hide holds 80% of the ordinary share capital of Seek (acquired on 1 February
2016) and 30% of the ordinary share capital of Arrive (acquired on 1 July 2015).
Hide had no other investments.
The draft statements of profit or loss for the year ended 30 June 2016, are set out
below.
Hide Seek Arrive
Rs.000 Rs.000 Rs.000
Revenue 12,614 6,160 8,640
Operating expenses (11,318) (5,524) (7,614)
Dividends receivable 150

1,446 636 1,026
Income tax (621) (275) (432)

Profit after taxation 825 361 594

Included in the inventory of Seek at 30 June 2016 was Rs. 50,000 for goods
purchased from Hide in May 2016 which the latter company had invoiced at cost
plus 25%. These were the only goods sold by Hide to Seek but it did make sales
of Rs. 180,000 to Arrive during the year. None of these goods remained in
Arrives inventory at the year end.
Required
Prepare a consolidated statement of profit or loss for Hide for the year ended 30
June 2016.

21.5 HARK, SPARK AND ARK


Hark acquired the following non-current investments on 1 April 2015:
(1) 4 million equity shares in Spark, by means of an exchange of one share in
Handel for every one share in Spark, plus Rs. 6.05 million in cash. The
professional fees associated with the acquisition amounted to Rs. 1 million.
The market price of shares in Hark at the date of the acquisition was Rs. 9
per share. The market price of Spark shares just before the acquisition was
Rs. 7. The cash part of the consideration is deferred and will not be paid
until two years after the acquisition.
(2) 25% of the equity shares in Ark, at a cost of Rs. 6 per share. The money to
make this payment was obtained by issuing one million new shares in Hark
at Rs. 9 per share.

Emile Woolf International 94 The Institute of Chartered Accountants of Pakistan


Questions

None of these transactions has yet been recorded in the summary statements of
financial position that are shown below.
The summarised draft statements of financial position of the three companies at
31 March 2016 are as follows.
Statement of financial position Hark Spark Ark
Rs. Rs. Rs.
million million million
Assets
Non-current assets
Property, plant and equipment 60.0 31.0 16.0
Other equity investments 0.8 nil nil
60.8 31.0 16.0
Current assets 18.2 8.0 9.0
Total assets 79.0 39.0 25.0
Equity and liabilities
Equity shares of Rs. 1 each 16.0 5.0 6.0
Share premium 2.0 4.0 4.0
Retained earnings: at 1 April 2015 36.0 16.0 8.0
- for year ended 31 March 2016 8.0 3.0 2.0
62.0 28.0 20.0
Non-current liabilities
6% loan notes 10.0 - -
7% loan notes - 6.0 3.0
Current liabilities 7.0 5.0 2.0
Total equity and liabilities 79.0 39.0 25.0
The following information is relevant:
(1) Hark has chosen to value the non-controlling interest in Spark using the fair
value method permitted by IFRS 3 (revised). The fair value of the non-
controlling interests at the acquisition date is estimated to be the market
value of the shares before the acquisition.
(2) At the date of acquisition of Spark, the fair values of its assets were equal
to their carrying amounts.
(3) The cost of capital of Hark is 10% per year.
(4) During the year ended 31 March 2016, Spark sold goods to Hark for Rs. 3.6
million, at a mark-up of 50% on cost. Hark had 75% of these goods in its
inventory at 31 March 2016.
(5) There were no intra-group receivables and payables at 31 March 2016.
(6) On 1 April 2015, Hark sold a group of machines to Spark at their agreed fair
value of Rs. 3 million. At the time of the sale, the carrying amount of the
machines was Rs. 2 million. The estimated remaining useful life of the plant
at the date of the sale was four years. Plant and machinery is depreciated
to a residual value of nil using straight-line depreciation and at 1 April 2015
the machines had an estimated remaining life of five years.
(7) Other equity investments are included in the summary statement of
financial position of Hark at their fair value on 1April 2015. Their fair value
at 31 March 2016 is Rs.0.65 million.

Emile Woolf International 95 The Institute of Chartered Accountants of Pakistan


Advanced accounting and financial reporting

(8) Impairment tests were carried out on 31 March 2016. These show that
there is no impairment of the value of the investment in Ark or in the
consolidated goodwill.
(9) No dividends were paid during the year by any of the three companies.
Required
Prepare the consolidated statement of financial position for Hark as at 31 March
2016.

21.6 P, S AND A
The statements of financial position of three entities P, S and A are shown below,
as at 31 December Year 5. However, the statement of financial position of P
records its investment in Entity A incorrectly.
P S A
Rs. Rs. Rs.
Non-current assets
Property, plant and equipment 450,000 240,000 460,000
Investment in S at cost 320,000 - -
Investment in A at cost 140,000 - -

910,000 240,000 460,000
Current assets
Inventory 70,000 90,000 70,000
Current account with P - 60,000 -
Current account with A 20,000 - -
Other current assets 110,000 130,000 40,000

Total assets 1,110,000 520,000 570,000

Equity and reserves
Equity shares of Rs. 1 100,000 200,000 100,000
Share premium 160,000 80,000 120,000
Accumulated profits 650,000 140,000 250,000

910,000 420,000 470,000
Long-term liabilities 40,000 20,000 30,000
Current liabilities
Current account with P - - 20,000
Current account with S 60,000 - -
Other current liabilities 100,000 80,000 50,000

1,110,000 520,000 570,000

Additional information
P bought 150,000 shares in S several years ago when the fair value of the net
assets of S was Rs. 340,000.
P bought 30,000 shares in A several years ago when As accumulated profits
were Rs. 150,000.

Emile Woolf International 96 The Institute of Chartered Accountants of Pakistan


Questions

There has been no change in the issued share capital or share premium of either
S or A since P acquired its shares in them.
There has been impairment of Rs. 20,000 in the goodwill relating to the
investment in S, but no impairment in the value of the investment in A.
At 31 December Year 5, A holds inventory purchased during the year from P
which is valued at Rs. 16,000 and P holds inventory purchased from S which is
valued at Rs. 40,000. Sales from P to A and from S to P are priced at a mark-up
of one-third on cost.
None of the entities has paid a dividend during the year.
P uses the partial goodwill method to account for goodwill and no goodwill is
attributed to the non-controlling interests in S.
Required
Prepare the consolidated statement of financial position of the P group as at 31
December Year 5.

21.7 H Ltd Group


The statements of comprehensive income for H Ltd, S Ltd and A Ltd for the year
ended 31 May 2016 are shown below:
H Ltd S Ltd A Ltd
Rs.000 Rs.000 Rs.000
Revenue 6,000 3,000 1,000
Cost of sales (4,800) (2,400) (800)
Gross profit 1,200 600 200
Distribution costs (64) (32) (10)
Administrative expenses (336) (168) (52)
Finance costs (30) (15) (5)
Profit before tax 770 385 133
Income tax expense (204) (102) (33)
PROFIT FOR THE YEAR 566 283 100
Other comprehensive income:
Revaluation of property 200 100 30
Tax effect of revaluation (42) (21) (6)
Other comprehensive income for the
year, net of tax 158 79 24
TOTAL COMPREHENSIVE INCOME
FOR THE YEAR 724 362 124
Additional information:
1. H Ltd operates a defined benefit pension plan for its employees. At the year
end, there is an actuarial loss of Rs. 52,000 on the pension plan liabilities
and an actuarial gain of Rs. 40,000 on pension plan assets. These amounts
are not reflected in the above statements. In accordance with the
amendment to IAS 19 Employee Benefits, H Ltd recognises actuarial gains
and losses from the defined benefit plan in other comprehensive income in
the period that they occur.

Emile Woolf International 97 The Institute of Chartered Accountants of Pakistan


Advanced accounting and financial reporting

2. H Ltd holds a 15% investment in XY which is designated as available for


sale. The fair value of this investment at 31 May 2016 was Rs. 106,000.
The investment is currently recorded in the financial statements at Rs.
92,000.
3. H Ltd owns 80% of the ordinary share capital of S Ltd and exercises control
over its operating and financial policies. H Ltd owns 30% of the ordinary
share capital of A Ltd and exerts significant influence over its operating and
financial policies.
Required
Prepare the consolidated statement of profit or loss and other comprehensive
income for the H Ltd Group, taking account of the information provided in the
notes above. Ignore any further taxation effects of notes 1 and 2.

Emile Woolf International 98 The Institute of Chartered Accountants of Pakistan


Questions

CHAPTER 22: BUSINESS COMBINATIONS ACHIEVED IN STAGES

22.1 STEP ACQUISITION


On 1 January Year 1, H purchased 25% of the equity of AS for Rs. 80 million. H
then acquired an additional 40% of the equity of AS for Rs. 160 million on 30
June Year 1. At this date it was estimated that the fair value of the original 25%
shareholding in S was Rs. 95 million.
During the year S did not issue any new shares or make any distribution to its
shareholders.
The carrying value of the net assets of AS were as follows:
Rs. million
At 1 January Year 1 260
At 30 June Year 1 300
H measures non-controlling interest at acquisition at fair value. This was
estimated to be Rs. 120m.
The financial year of H ends on 30 June.
Required
For the consolidated financial statements of H for the year to 30 June Year 1,
state:
(i) the total gain or profit attributable to the investment in AS for the year
(ii) total amount of goodwill arising with the acquisition
(iii) the amount of goodwill attributable to the NCI.

22.2 A LTD
The statements of financial position for A Ltd and B Ltd as at 30 September 2016
are provided below:
A Ltd B Ltd
Rs.000 Rs.000
ASSETS
Non-current assets
Property, plant and equipment 22,000 5,000
Available for sale investment (note 1) 4,000 -
Current assets 26,000 5,000
Inventories 6,200 800
Receivables 6,600 1,900
Cash and cash equivalents 1,200 300
14,000 3,000
Total assets 40,000 8,000
EQUITY AND LIABILITIES Equity
Share capital (Rs. 1 equity shares) 20,000 1,000
Retained earnings 7,500 5,000

Emile Woolf International 99 The Institute of Chartered Accountants of Pakistan


Advanced accounting and financial reporting

A Ltd B Ltd
Rs.000 Rs.000
Other components of equity 500
Total equity 28,000 6,000
Non-current liabilities
5% Bonds 2019 (note 2) 3,900 -
Current liabilities 8,100 2,000
Total liabilities 12,000 2,000
Total equity and liabilities 40,000 8,000
Additional information:
1. A Ltd acquired a 15% investment in B Ltd on 1 May 2010 for Rs. 600,000.
The investment was classified as available for sale and the gains earned on
it have been recorded within other reserves in A Ltds individual financial
statements. The fair value of the 15% investment at 1 April 2016 was Rs.
800,000.
On 1 April 2016, A Ltd acquired an additional 60% of the equity share
capital of B Ltd at a cost of Rs. 2,900,000. In its own financial statements, A
Ltd has kept its investment in B Ltd as an available for sale asset recorded
at its fair value of Rs. 4,000,000 as at 30 September 2016.
2. A Ltd issued 4 million Rs. 1 5% redeemable bonds on 1 October 2011 at
par. The associated costs of issue were Rs. 100,000 and the net proceeds
of Rs. 3.9 million have been recorded within non-current liabilities. The
bonds are redeemable at Rs. 4.5 million on 30 September 2019 and the
effective interest rate associated with them is approximately 8.5%. The
interest on the bonds is payable annually in arrears and the amount due
has been paid in the year to 30 September 2016 and charged to the
statement of profit or loss.
3. An impairment review was conducted at the year end and it was decided
that the goodwill on the acquisition of B Ltd was impaired by 10%.
4. It is the group policy to value non-controlling interest at fair value at the date
of acquisition. The fair value of the non-controlling interest at 1 April 2016
was Rs. 1.25 million.
5. The profit for the year of B Ltd was Rs. 3 million, and profits are assumed to
accrue evenly throughout the year.
6. B Ltd sold goods to A Ltd for Rs. 400,000. Half of these goods remained in
inventories at 30 September 2016. B Ltd makes 20% margin on all sales.
7. No dividends were paid by either entity in the year to 30 September 2016.
Required
(a) Explain how the investment in B Ltd should be accounted for in the
consolidated financial statements of A Ltd, following the acquisition of the
additional 60% shareholding.
(b) Prepare the consolidated statement of financial position as at 30
September 2016 for the A Ltd Group.

Emile Woolf International 100 The Institute of Chartered Accountants of Pakistan


Questions

22.3 X LTD GROUP


Extracts from the financial statements of X Ltd, Y Ltd and Z Ltd are presented
below.
Statements of comprehensive income for X Ltd Y Ltd Z Ltd
the year ended 31 December 2016
Rs.000 Rs.000 Rs.000
Revenue 1,200 290 150
Cost of sales (810) (110) (80)
Gross profit 390 180 70
Operating expenses (100) (40) (20)
290 140 50
Investment income 50 -
Finance costs (45) (10) (5)
Profit before tax 295 130 45
Income tax expense (80) (30) (15)
Profit for the year 215 100 30
Other comprehensive income:
Revaluation of property, net of tax 60 20 10
Other comprehensive income for the year,
net of tax 60 20 10
Total comprehensive income 275 120 40

Statements of changes in equity for the X Ltd Y Ltd Z Ltd


year ended 31 December 2016
Rs.000 Rs.000 Rs.000
Equity at 1 January 2016 1,700 840 500
Total comprehensive income for the year 275 120 40
Dividends (100) (50) -
Equity at 31 December 2016 1,875 910 540
Additional information
1 X Ltd acquired 80% of the ordinary share capital of Y Ltd for Rs. 620,000
on 1 January 2010 when the retained reserves of Y Ltd were Rs. 420,000.
Y Ltd has 200,000 Rs. 1 ordinary shares in issue and there have been no
share issues since the acquisition date. The group policy is to measure the
non- controlling interest at fair value at the date of acquisition. The fair
value of the non-controlling interest at 1 January 2010 was Rs. 180,000.
2 On 1 January 2010, the fair value of Y Ltds net assets was the same as
their book value with the exception of depreciable property, the fair value of
which was Rs. 60,000 higher than its book value. The property had a
remaining useful life of 15 years at the date of acquisition. Depreciation on
property is charged to cost of sales.
3 Goodwill on the acquisition of Y Ltd was impaired for the first time by 25%
in the year to 31 December 2015. An impairment review conducted at 31
December 2016 indicated a further impairment of 10% of the remaining
carrying value of goodwill. Impairment losses on goodwill are charged to
group operating expenses.
4 X Ltd acquired 40% of the ordinary share capital of Z Ltd on 1 July 2011,
when the equity was Rs. 435,000.

Emile Woolf International 101 The Institute of Chartered Accountants of Pakistan


Advanced accounting and financial reporting

Required
(a) Prepare for the X Ltd Group for the year ended 31 December 2016:
(i) a consolidated statement of profit or loss and other comprehensive
income; and
(ii) a consolidated statement of changes in equity.
X Ltd purchased a further 10% of the ordinary share capital of Y Ltd on 1 January
2017 for Rs. 120,000.
Required
(b) (i) Explain how the acquisition of this additional investment will be
accounted for in the consolidated financial statements of the X Ltd
group for the year to 31 December 2017.
(ii) Calculate the debit or credit that will be made to the consolidated
retained reserves of the X Ltd group for the year to 31 December
2017 in respect of this additional 10% share purchase.
X Ltd purchased a further 20% of the ordinary share capital of Z Ltd on 1 January
2017.
Required
(c) Explain how the acquisition of the additional investment in Z Ltd will be
accounted for in the consolidated financial statements of the X Ltd group for
the year to 31 December 2017.

22.4 PLAIN LTD


The following statements of financial position are as at 31 March 2016:
Plain Stripes Spots
Rs. m Rs. m Rs. m
Assets
Tangible non-current assets 1,280 440 280
Investment in Stripes 413
Investment in Spots 60
Held to maturity investment 54
Current assets 477 190 130
Total assets 2,284 630 410
Equity and liabilities
Share capital of Rs. 1 800 240 200
Share premium 150 20 30
Revaluation reserve 90
Retained earnings 390 210 94
Total equity 1,430 470 324
Non-current liabilities 640 30 16
Current liabilities 214 130 70
Total equity and liabilities 2,284 630 410

Emile Woolf International 102 The Institute of Chartered Accountants of Pakistan


Questions

Plain acquired the following shareholdings in Stripes and Spots.


Fair value
Holding of Purchase
Date of acquisition
acquired consideration
net assets
Rs. m Rs. m

Stripes 1 April 2013 30% 325 120


1 April 2015 50% 460 260

Spots 1 April 2015 25% 200 60


You are also provided with the following information which will be relevant to the
consolidated financial statements of Plain.
(1) None of the companies have issued any additional share capital since 1
April 2013.
(2) The financial statements of Plain have not yet been adjusted for the gain or
loss arising on gaining control of Stripes.
(3) At 1 April 2013, the carrying value of the net assets of Stripes was the
same as their fair value, Rs. 325 million.
(4) Plain wishes to use the full fair value method of accounting for the
acquisition of Stripes, and at 1 April 2015 the estimated value of goodwill
attributable to non-controlling interests was Rs. 3 million. The estimated fair
value of the initial investment in 30% of the shares of Stripes was Rs. 150
million at 31 March 2015.
(5) Included in the tangible non-current assets of Stripes is land valued at cost
which on 1 April 2015 had a fair value of Rs. 25 million in excess of its
carrying value. There has been no subsequent significant change in that
value.
(6) At 1 April 2014 the fair value of Spotss land was Rs. 16 million in excess of
its carrying value. There has been no subsequent significant change in that
value.
(7) Goodwill arising on acquisition is tested for impairment at each year end. At
31 March 2016 an impairment loss of Rs. 15 million was recognised for
Stripes .
(8) There has been no impairment of the investment in Spots.
(9) During the year the directors of Plain decided to form a defined benefit
pension scheme for its employees. The company contributed cash to it of
Rs. 250 million but the only accounting entry for this has been to include it
in receivables at 31 March 2016.
At 31 March 2016 the following details relate to the pension scheme:
Rs. m
Present value of obligation 317
Fair value of plan assets 302
Current service cost 276
Interest cost on pension scheme liabilities 41
Expected return on pension scheme assets 26
In the consolidated financial statements the directors wish to recognise any
actuarial gain or loss immediately.

Emile Woolf International 103 The Institute of Chartered Accountants of Pakistan


Advanced accounting and financial reporting

(10) The held to maturity investment in Plains financial statements is a zero


coupon loan to an unrelated third party. No interest has yet been
recognised on this amount. The debt is repayable in five years at Rs. 74
million. (Recognise interest on a straight line basis).
Required
Prepare the consolidated statement of financial position of the Plain group as at
31 March 2016.

22.5 MANGO LTD


The draft statements of financial position of Mango Ltd and its subsidiary at 30
November 2016 are as follows:

Mango Ltd Plum Ltd


Rs. m Rs. m
Assets:
Non-current assets
Property, plant and equipment 3,295 2,000
Investments in subsidiary 1,675
4,970 2,000
Current assets 1,685 861
Total assets 6,655 2,861
Equity and liabilities:
Share capital 850 1,020
Retained earnings 3,340 980
Other components of equity 250 80
Total equity 4,440 2,080
Non-current liabilities 1,895 675
Current liabilities 320 106
Total liabilities 2,215 781
Total equity and liabilities 6,655 2,861
The following information is relevant to the preparation of the group financial
statements:
1. On 1 December 2013, Mango Ltd acquired 30% of the ordinary shares of
Plum Ltd for a cash consideration of Rs. 600 million. The fair value of Plum
Ltds identifiable net assets was Rs. 1,840 million at this date. The 30%
holding gave Mango Ltd significant influence over Plum Ltd and Mango Ltd
accounted for the investment as an associate up to 1 December 2015.
Mango Ltds share of Plum Ltds undistributed profit amounted to Rs. 90
million and its share of a revaluation gain amounted to Rs. 10 million.
On 1 December 2015, Mango Ltd acquired a further 40% of the ordinary
shares of Plum Ltd for a cash consideration of Rs. 975 million and gained
control of the company. The cash consideration has been added to the
equity accounted balance for Plum Ltd at 1 December 2015 to give the
carrying amount at 30 November 2016.
At 1 December 2015, the fair value of the equity interest in Plum Ltd held by
Mango Ltd before the business combination was Rs. 705 million.
At 1 December 2015, the fair value of Plum Ltds identifiable net assets was
Rs. 2,250 million.

Emile Woolf International 104 The Institute of Chartered Accountants of Pakistan


Questions

The retained earnings and other components of equity of Plum Ltd at 1


December 2015 were Rs. 900 million and Rs. 70 million respectively. It is
group policy to measure the non-controlling interest at fair value. The fair
value of the non-controlling interest of 30% was assessed as Rs. 620
million
2. At the time of the business combination with Plum Ltd, Mango Ltd has
included in the fair value of Plum Ltds identifiable net assets, an
unrecognised contingent liability of Rs. 6 million in respect of a warranty
claim in progress against Plum Ltd. In March 2016, there was a revision of
the estimate of the liability to Rs. 5 million. The amount has met the criteria
to be recognised as a provision in current liabilities in the financial
statements of Plum Ltd and the revision of the estimate is deemed to be a
measurement period adjustment.
3. The fair value of Plum Ltds identifiable net assets (Rs. 2,250 million)
included an amount of Rs. 200 million being the estimate of the fair value of
buildings with the remainder relating to non-depreciable land.
Mango Ltd had commissioned an independent valuation of the buildings of
Plum Ltd which was not complete at 1 December 2015 and therefore not
considered in the fair value of the identifiable net assets at the acquisition
date. The valuations were received on 1 April 2016 and resulted in a
decrease of Rs. 40 million in the fair value of property, plant and equipment
at the date of acquisition. The buildings have a remaining useful life of 20
years at 1 December 2015. Buildings are depreciated on the straight-line
basis and it is group policy to leave revaluation gains on disposal in equity.
The decrease in the fair value of the buildings does not affect the fair value
of the non-controlling interest at acquisition and has not been entered into
the financial statements of Plum Ltd.
All goodwill arising on acquisitions has been impairment tested with no
impairment being required.
Required
Prepare the group consolidated statement of financial position of Mango Ltd
as at 30 November 2016.

Emile Woolf International 105 The Institute of Chartered Accountants of Pakistan


Advanced accounting and financial reporting

CHAPTER 23: COMPLEX GROUPS

23.1 PARVEZ LTD

Statements of profit or loss for Parvez Ltd, Saad Ltd and Vazir Ltd for the year
ended 31 December 2016 are as follows:
Parvez Ltd Saad Ltd Vazir Ltd
Rs. 000 Rs. 000 Rs. 000
Revenues 45,600 24,700 22,800
Cost of sales (18,050) (5,463) (5,320)
Gross profit 27,550 19,237 17,480
Distribution costs (3,325) (2,137) (1,900)
Administrative expenses (3,475) (950) (1,900)
Operating profit 20,750 16,150 13,680
Interest paid (325)
Profit before tax 20,425 16,150 13,680
Tax (8,300) (5,390) (4,241)
Profit after tax 12,125 10,760 9,439

Statements of financial position as at 31 December 2016 are as follows:


Parvez Ltd Saad Ltd Vazir Ltd
Rs. 000 Rs. 000 Rs. 000
Property, plant and equipment 35,483 24,273 13,063
Investments
Shares in Saad Ltd 6,650
Shares in Vazir Ltd 3,800
Current assets 1,568 9,025 8,883
43,701 37,098 21,946
Equity and liabilities
Ordinary Rs. 1 shares 8,000 3,000 2,000
Retained earnings 22,638 24,075 19,898
30,638 27,075 21,898
Current liabilities 13,063 10,023 48
43,701 37,098 21,946

The following information is available relating to Parvez Ltd, Saad Ltd and Vazir
Ltd:
(1) On 1 January 2010 Parvez Ltd acquired 2,700,000 Rs. 1 ordinary shares in
Saad Ltd for Rs. 6,650,000 at which date there was a credit balance of
retained earnings of Saad Ltd of Rs. 1,425,000. No shares have been
issued by Saad Ltd since Parvez Ltd acquired its interest.

Emile Woolf International 106 The Institute of Chartered Accountants of Pakistan


Questions

(2) On 1 January 2010 Saad Ltd acquired 1,600,000 Rs. 1 ordinary shares in
Vazir Ltd for Rs. 3,800,000 at which date there was a credit balance of
retained earnings of Vazir Ltd of Rs. 950,000. No shares have been issued
by Vazir Ltd since Saad Ltd acquired its interest.
(3) During 2016, Vazir Ltd had made inter-company sales to Saad Ltd of Rs.
480,000 making a profit of 25% on cost and Rs. 75,000 of these goods
were in inventory at 31 December 2016.
(4) During 2016, Saad Ltd had made inter-company sales to Parvez Ltd of Rs.
1
260,000 making a profit of 33 3 % on cost and Rs. 60,000 of these goods
were in inventory at 31 December 2016.
(5) On 1 November 2016 Parvez Ltd sold warehouse equipment to Saad Ltd
for Rs. 240,000 from inventory. Saad Ltd has included this equipment in its
non-current assets. The equipment had been purchased on credit by
Parvez Ltd for Rs. 200,000 in October 2016 and this amount is included in
its current liabilities as at 31 December 2016.
(6) Saad Ltd charges depreciation on its warehouse equipment at 20% on cost.
It is company policy to charge a full years depreciation in the year of
acquisition to be included in the cost of sales.

Required
(a) Prepare a consolidated statement of profit or loss for the Parvez Ltd Group
for the year ended 31 December 2016.
(b) Prepare statement of financial position as at that date.

23.2 HASAN, RIAZ AND SIDDIQ

The summarised balances extracted from the accounting records of Hasan (H)
Ltd, Riaz (R) Ltd and Siddiq (S) Ltd at 31 December 2016 are given below:
H Ltd R Ltd S Ltd
Rs. Rs. Rs.
Property, plant and equipment 1,102,500 271,950 122,550
Investments at cost
75% holding in shares of Riaz Ltd 367,500
40% holding in shares of Siddiq Ltd 49,000
20% holding in shares of Siddiq Ltd 24,500
Inventories 526,610 163,290 85,700
Receivables 241,920 129,680 29,750
Cash and bank balances 88,200 4,725 8,105
2,375,730 594,145 246,105

Share capital 1,750,000 420,000 175,000


Other reserves 350,000 70,000
Accumulated profits/(losses) 180,250 17,500 (17,500)
Payables 95,480 86,645 88,605
2,375,730 594,145 246,105

Emile Woolf International 107 The Institute of Chartered Accountants of Pakistan


Advanced accounting and financial reporting

Further information:
(1) Hasan Ltd purchased its interest in Riaz Ltd and Siddiq Ltd in December
2013 at which date Siddiq Ltd had accumulated losses of Rs. 35,000, and
Riaz Ltd had accumulated profits of Rs. 35,000.
(2) On 30 December 2016 Hasan Ltd despatched and invoiced goods for Rs.
12,500 to Riaz Ltd which were not recorded by the latter until 3 January
2017. A mark-up of 25% is added by Hasan Ltd to arrive at selling price.
Riaz Ltd already had goods in inventories which had been invoiced to them
by Hasan Ltd at Rs. 10,400.
(3) Siddiq Ltd had accumulated losses of Rs. 52,500 when Riaz Ltd purchased
35,000 shares in 2012.
(4) Hasan Ltd received a remittance of Rs. 8,000 on 2 January 2017 which had
been sent by Riaz Ltd on 29 December 2016.
(5) Included in Hasans receivables was a balance of Rs. 25,500 owed by Riaz
Ltd.
(6) Neither Riaz Ltd nor Siddiq Ltd had any other reserves when their shares
were purchased by Hasan Ltd and Riaz Ltd.
(7) Payables of Riaz Ltd included an amount of Rs. 5,000 due to Hasan Ltd.
Required
Prepare the consolidated statement of financial position of Hasan Ltd and its
subsidiaries at 31 December 2016.

Emile Woolf International 108 The Institute of Chartered Accountants of Pakistan


Questions

CHAPTER 24: DISPOSAL OF SUBSIDIARIES

24.1 PATCHE LTD


Patche Ltd owns 85% of the ordinary share capital of Somers Ltd for many years.
The shares were bought for Rs. 765 million and Somers Ltds reserves at the
time of purchase amounted Rs. 60million.
On 31 March 2016, Patche Ltd. sold 120 million of Somers Ltd shares for Rs. 480
million. The only entry made in respect of this transaction was the receipt of the
cash, which was credited to the investment in subsidiary account. No dividend
was paid by either entity during the period.
The summarised financial statements of the companies are as follows:
Statements of profit or loss and other comprehensive income for the year ended
to 30 June 2016.
Patche Somers
Ltd Ltd
Rs.m Rs.m
Profit before tax 390 180
Income tax expenses (120) (60)
Profit for the year 270 120
Other comprehensive income that will not be
reclassified to profit or loss net of tax 60 30
330 150
STATEMENTS OF FINANCIAL POSITION AS AT 30 JUNE 2016
Patche Somers
Ltd Ltd
Non-current assets: Rs.m Rs.m
Property, plant & equipment 1,605 534
Investment in Somers Ltd 285 -
Current assets:
Inventories 960 570
Trade receivables 750 525
Cash and bank 240 267
3840 1896
Equity interest
Share capital: Rs. 1 ordinary shares 1,500 600
Reserves 930 510
2,430 1,110
Current liabilities:
Trade payables 885 513
Income tax 240 180
Provisions 285 93
3,840 1,896
There was no impairment loss in the group during the year.

Emile Woolf International 109 The Institute of Chartered Accountants of Pakistan


Advanced accounting and financial reporting

Assume that the gain as calculated in the parents separate financial statement
will be subject to companies income tax at a rate of 30%, and that profit and
other comprehensive income accrue evenly throughout the year.
Patche Ltd. group policy is to measure non-controlling interest at fair value at the
date of acquisition.
The fair value of the non-controlling interest in Somers Ltd. was Rs. 135million at
the date of acquisition.
Required
Prepare the following
(a) Statement of profit or loss and comprehensive income and statement of
changes in equity of Patche Ltd for the year ended 30 June 2016.
(b) Consolidated statement of profit or loss and comprehensive income of
Patche Ltd. for the same period.
(c) Consolidated statement of financial position as at 30 June 2016.

24.2 DISPOSAL
At 31 December Year 1, Hoo owned 90% of the shares in Spool. At this date the
carrying amount of the net assets of Spool in the consolidated financial
statements of the Hoo Group was Rs. 800 million. None of the assets of Spool
are re-valued.
On 1 January Year 2, Hoo sold 80% of the equity of Spool for Rs. 960 million in
cash.
The remaining shares in Spool held by Hoo are estimated to have a fair value of
Rs. 100 million.
Required
Explain how the disposal of the shares in Spool should be accounted for in the
consolidated financial statements of the Hoo Group.

24.3 PART DISPOSAL


On 1 January Year 2, P acquired 80% of the equity of S for Rs. 620 million in
cash. On 30 June Year 2 it sold 10% of the equity in S for Rs. 94 million. S did
not issue any shares or make any distribution to its shareholders in the year to 31
December Year 2. P uses the partial goodwill method to account for the
acquisition of S and no goodwill is attributed to the non-controlling interest.
The net assets of S were as follows, at carrying value:
Rs. million
At 1 January Year 2 700
At 31 December Year 2 900
At 31 December Year 2, P carries out an impairment review and decides that the
goodwill in its investment in S has been impaired by Rs. 8 million.
Required
Explain how the disposal of the shares in S should be accounted for.

Emile Woolf International 110 The Institute of Chartered Accountants of Pakistan


Questions

24.4 THE A GROUP


The summarised statements of financial position of A and its two subsidiaries B
and C at 31 December Year 3 are shown below:

Summarised statements of financial position at 31 December Year 3


A B C
Rs.000 Rs.000 Rs.000
Investment in subsidiaries:
B 1,164
C 1,120
Other net assets 2,516 1,260 1,400
4,800 1,260 1,400
Ordinary share capital
(Rs. 1 shares) 1,500 500 400
Accumulated profits 3,300 760 1,000
4,800 1,260 1,400
The summarised statement of profit or loss for A and B for the year ended 31
December Year 4 are as follows:
A B
Rs.000 Rs.000
Profit before tax 1,200 250
Taxation (360) (60)

Profit after tax 840 190
Dividends paid (50) (20)

Retained profit for year 790 170
Retained profit at start of year 3,300 760

Retained profits at end of year 4,090 930

Additional information:
(i) A acquired 80% of the ordinary share capital of B on 1 January Year 0
when the reserves of B were Rs. 420,000.
(ii) A acquired 90% of the ordinary share capital of C on 1January Year 1 when
the reserves of C were Rs. 320,000.
(iii) On 1 January Year 4, A disposed of 350,000 shares in C for Rs. 1,925,000.
This transaction has not yet been accounted for by A. The remaining
investment in shares of C at this date had a fair value of Rs. 44,000.
(iv) There were no changes in the issued share capital of the subsidiaries since
acquisition by A.
(v) None of the companies re-value any of their non-current assets.

Emile Woolf International 111 The Institute of Chartered Accountants of Pakistan


Advanced accounting and financial reporting

(vi) The A Group uses the partial goodwill method of accounting for
acquisitions and no goodwill is attributed to non-controlling interests. There
has been no impairment of goodwill.
Required
Prepare As consolidated statement of profit or loss and show the movement on
consolidated equity reserves for the year to 31 December Year 4 and a
consolidated statement of financial position as at that date.

24.5 BARTLETT LTD


Many years ago Bartlett Ltd bought 80% of the ordinary shares of Lymon Ltd for
Rs. 175,000. On 1 July 2016 Bartlett sold all of these shares and used the
proceeds (Rs. 212,000) to purchase 65% of the ordinary shares of Zeigler Ltd on
the same date.
Statements of profit or loss for all three companies for the year ended 31
December 2016 were as follows.
Bartlett Ltd Lymon Ltd Zeigler Ltd
Rs. Rs. Rs.
Revenue 1,926,500 521,600 792,400
Cost of sales 1,207,200) (386,200) (405,900)
Gross profit 719,300 135,400 386,500
Distribution costs (207,500) (79,200) (198,200)
Administrative (192,600) (26,100) (107,100)
expenses
Profit before tax 319,200 30,100 81,200
Taxation (110,000) (9,500) (27,500)
Profit after tax 209,200 20,600 53,700

No entries have been made in Bartlett Ltds statement of profit or loss relating to
the sale of Lymon Ltd. Lymons net assets were Rs. 140,000 at the 1st January
2016.
Goodwill arising on the acquisition of Lymon Ltd was Rs. 25,400.
Required
Prepare the consolidated statement of profit or loss for Bartlett Ltd for the year
ended 31 December 2016.

Emile Woolf International 112 The Institute of Chartered Accountants of Pakistan


Questions

CHAPTER 26: FOREIGN CURRENCY

26.1 DND LIMITED


DND Limited is a listed company, having its operations within Pakistan. During the
year ended December 31, 2016, the company contracted to purchase plants and
machineries from a US Company. The terms and conditions thereof , are given below:
(i) Total cost of contract = US$ 100,000.
(ii) Payment to be made in accordance with the following schedule:

Payment Dates Amount Payable

On signing the contract July 01, 2016 US$ 20,000

On shipment* September 30, 2016 US$ 50,000

After installation and test run January 31, 2017 US$ 30,000

*(risk and rewards of ownership are transferred on shipment)

The contract went through in accordance with the schedule and the company made all
the payments on time. The following exchange rates are available:

Dates Exchange Rates

July 1, 2016 US$ 1 = Rs. 60.50

September 30, 2016 US$ 1 = Rs. 61.00

December 31, 2016 US$ 1 = Rs. 61.20

January 31, 2017 US$ 1 = Rs. 61.50

Required
Prepare journals to show how the above contract should be accounted for under
IAS 21.

26.2 STARLIGHT LIMITED


In December 2014, Lahore Holdings Ltd acquired an 80% interest in a Qatari
investment company, Starlight Limited. Starlight Limited has the Qatari Rial (QR)
as its functional currency.
The acquisition cost was Rs. 2,500,000 and the revenue reserves balance of
Starlight Limited stood at QR 49,300,000 at the acquisition date.
The following financial information was extracted from the books for the year
ended 31 December 2016.

Starlight Limited

Emile Woolf International 113 The Institute of Chartered Accountants of Pakistan


Advanced accounting and financial reporting

QR000
Turnover 344,880
Cost of sales (249,710)
Gross profit 95,170
Expenses (29,490)
Profit before tax 65,680
Taxation (17,325)
Profit after tax 48,355
Interim dividend (16,300)
Retained profit for the year 32,055
Extract of statement of financial position at 31 December, 2016
QR000
Share capital 20,250
Revenue reserve 103,200
Liabilities 34,480
Note
Exchange rate (QR to one rupee)
December 31, 2014 30
December 31, 2015 31
December 31, 2016 33
Average rate for 2016 32
Required
(a) Prepare the translated profit and loss account of Starlight Limited.
(b) Calculate the goodwill on consolidation and the non-controlling interest that
would appear in the consolidated statement of profit or loss.

26.3 PERCEPT LTD


Percept Ltd acquired 70% of the share capital of Trint on 1 January 2016, a
company based in Japan for 6,900,000 Yen. The retained earnings on this date
was 4,500,000 Yen. The fair value of the identifiable net assets of the company
was 12,375,000 Yen and the net asset relates to items of property plant and
equipment. Similarly, the fair value of the non-controlling Interest (NCI) in Trint
Ltd as at 1 January, 2016 was 6,250,000 Yen. It is the policy of Percept Ltd to
use the full goodwill method in the preparation of the groups financial
statements. Trint Ltds profit for the year ended 31 December 2016 was
2,000,000 Yen. The acquisition on 1 January 2016 was done in Japan when the
following exchange rates were in force:
Yen to rupees
01/01/2016 6
31/12/2016 5
The average rate for the year ended 31 December 2016 was 5.5 Yen to Rs. 1.

Trint Ltd: Statement of Financial Position as at 31 December 2016.

Emile Woolf International 114 The Institute of Chartered Accountants of Pakistan


Questions

Yen (000)
Assets:
Non-current assets 9,500
Financial assets 1,250
10,750
Current assets 8,250
Total assets 19,000

Equity and liabilities


Share capital 5,000
Retained earnings 7,500
12,500
Non-current liabilities 4,000
Current liabilities 2,500
6,500
Total equity and liabilities 19,000

Required
(a) Translate the statement of financial position of Trint Ltd. as at 31 December
2016
(b) Calculate the goodwill arising on acquisition of Trint and any gain/loss
arising on retranslation of the goodwill as at 31 December 2016
(c) Calculate the exchange difference arising from the translation of Trint Ltds
net assets.

26.4 ORLANDO
Orlando is an entity whose functional currency is the US dollar. It prepares its
financial statements to 30 June each year. The following transactions take place
on 21 May Year 4 when the spot exchange rate was $1 = 0.8.
Goods were sold to Koln, a customer in Germany, for 96,000.
A specialised piece of machinery was bought from Frankfurt, a German supplier.
The invoice for the machinery is for 1,000,000.
The company receives 96,000 from Koln on 12 June Year 4.
At 31 June Year 4 it still owns the machinery purchased from Frankfurt. No
depreciation has been charged on the asset for the current period to 30 June
Year 4.
The liability for the machine is settled on 31 July Year 4.
Relevant $/ exchange rates are:
12 June Year 4 $1 = 0.9
30 June Year 4 $1 = 0.7
31 July Year 4 $1 = 0.8
Required

Emile Woolf International 115 The Institute of Chartered Accountants of Pakistan


Advanced accounting and financial reporting

Show the effect on profit or loss of these transactions for:


(a) the year to 30 June Year 4
(b) the year to 30 June Year 5.

26.5 MANCASTER AND STOCKPOT


Part A
Required
(1) Define and explain the following terms as used in IAS 21 The effects of
changes in foreign exchange rates.
(a) Functional currency
(b) Presentation currency
(2) Outline the factors to be considered when determining the functional
currency of an overseas subsidiary.
Part B
The statements of financial position of Manchester and its subsidiary Stockpot at
31 March Year 4 and their statement of profit or loss for year ended on that date
are set out below:
Statements of financial position at 31 March Year 4
Mancaster Stockpot
$000 $000 000 000
Non-current assets:
Property, plant and equipment 20,000 30,000
Investments (notes 1 and 2) 5,500 -
25,500 30,000
Current assets:
Inventories 10,000 18,000
Trade receivables 10,000 15,000
20,000 33,000
Total assets 45,500 63,000
Capital and reserves:
Share capital ($1/1 Shares) 9,000 15,000
Accumulated profits 12,500 10,000
21,500 25,000
Non-current liabilities (note 4) 10,000 20,000
Current liabilities
Trade payables 7,900 10,400
Bank overdraft 6,100 7,600
14,000 18,000
Total equity and liabilities 45,500 63,000

Emile Woolf International 116 The Institute of Chartered Accountants of Pakistan


Questions

Statement of profit or loss year ended 31 March Year 4


Mancaster Stockpot
$000 000
Revenue 50,000 60,000
Cost of sales (notes 2 and 5) (25,000) (30,000)
Gross profit 25,000 30,000
Other operating expenses (15,000) (16,000)
Operating profit 10,000 14,000
Investment income (note 3) 1,500 -
Interest payable (1,000) (2,000)
Profit before tax 10,500 12,000
Tax (3,600) (4,200)
Profit after tax 6,900 7,800

Statement of changes in equity year ended 31 March Year 4


Mancaster Stockpot
Share Reserves Share Reserves
capital capital
$000 $000 000 000
Balance at 1 April Year 3 9,000 9,500 15,000 6,600
Profit for the period 6,900 7,800
Dividends paid (3,900) (4,400)
Balance at 31 March Year 4 9,000 12,500 15,000 10,000
You are provided with the following additional information:
(1) Investments represent the acquisition of 11.25 million shares in Stockpot on
31 March Year 0. The retained profits of Stockpot on this date stood at 5
million. Any goodwill arising on the acquisition is to be treated as a foreign
currency asset. Stockpot operates as a reasonably autonomous entity on a
day-to-day basis although Mancaster does control the long-term strategy of
Stockpot.
(2) Exchange rates have been as follows:
Date to $1
31 March Year 0 3.0
31 March Year 3 2.4
31 March Year 4 2.2
Average for Year 4 2.3
(3) Investment income represents dividends received from Stockpot.
(4) The non-current liabilities represent long-term borrowings.
Required
(a) Translate the statement of financial position of Stockpot into the
presentation currency of dollars and prepare the consolidated statement of
financial position of the group at 31 March Year 4.
(b) Translate the statement of profit or loss of Stockpot into dollars and prepare
the consolidated statement of profit or loss of the group for the year ended
31 March Year 4.

Emile Woolf International 117 The Institute of Chartered Accountants of Pakistan


Advanced accounting and financial reporting

26.6 A, B AND C
Extracts from the financial statements of A, its subsidiary, B and its associate, C
for the year to 30 September 2016 are presented below:
Summarised statement of profit or loss and A B C
other comprehensive income
Rs.000 A$000 Rs.000
Revenue 4,600 2,200 1,600
Cost of sales and operating expenses (3,700) (1,600) (1,100)
Profit before tax 900 600 500
Income tax (200) (150) (100)
Profit for the year 700 450 400
Other comprehensive income:
Revaluation of property, plant and equipment 200 120 70
Total other comprehensive income 200 120 70
Total comprehensive income 900 570 470

Statement of financial position A B C


Rs.000 A$000 C$000
Assets
Non-current assets
Property, plant and equipment 7,000 4,000 2,000
Investment in B 5,200
Investment in C 900 _
13,100 4,000 2,000
Current assets 3,000 2,000 1,000
Total assets 16,100 6,000 3,000
Equity and liabilities
Share capital 2,000 1,000 1,000
Reserves 12,100 3,500 1,500
14,100 4,500 2,500
Current liabilities 2,000 1,500 500
Total equity and liabilities 16,100 6,000 3,000

Additional information
1. The functional currency of both A and C is the Rs. and the functional
currency of B is the A$.
2. A acquired 80% of B on 1 October 2013 for Rs. 5,200,000 when the
reserves of B were A$1,800,000. The investment is held at cost in the
individual financial statements of A.

Emile Woolf International 118 The Institute of Chartered Accountants of Pakistan


Questions

3. A acquired 40% of C on 1 October 2011 for Rs. 900,000 when the reserves
of C were Rs. 700,000. The investment is held at cost in the individual
financial statements of A.
4. No impairment to either investment has occurred to date.
5. The group policy is to value the non-controlling interest at fair value at the
date of acquisition. The fair value of the non-controlling interest of B at 1
October 2013 was A$600,000.
6. Relevant exchange rates are as follows:

1 October 2013 Rs./A$0.5000


30 September 2015 Rs./A$0.7100
30 September 2016 Rs./A$0.6300
Average rate for year ended 30 September 2016 Rs./A$0.6500
Required
Prepare the consolidated statement of profit or loss and other comprehensive
income for the A Group for the year ended 30 September 2016 and the
consolidated statement of financial position as at that date.

26.7 OMEGA LIMITED


Omega Limited (OL) is incorporated and listed in Pakistan. On 1 May 2012, it
acquired 20,000 ordinary shares (2% shareholding) in Al-Wadi Limited (AWL), a
Dubai based company at a cost of AED 240,000 which was equivalent to Rs.
6,000,000. The face value of the shares is AED 10 each. OL intends to hold the
shares to avail benefits of regular dividends and capital gains.
On 1 June 2013, AWL was acquired by Hilal Limited (HL), which issued three
shares in HL in exchange for every four shares held in AWL.
Other relevant information is as under:
AWL HL
Final dividend received on 31 March 2013:
Cash 15% -
Bonus shares 10% -
Final cash dividend received on 10 April 2014 - 20%
Fair value per share as at: 31 December 2012 AED 13.00 -
1 June 2013 AED 14.00 AED 18.00
31 December 2013 - AED 19.50
Exchange rates on various dates were as follows:
31-Dec-2012 31-Mar-2013 1-Jun-2013 31-Dec-2013 10-Apr-2014
1 AED Rs. 25.00 Rs. 26.50 Rs. 28.00 Rs. 28.70 Rs. 28.20
Required
Determine the amounts (duly classified under appropriate heads) that would be
included in
OLs statement of comprehensive income for the year ended 31 December 2013
in respect of the above investment.

Emile Woolf International 119 The Institute of Chartered Accountants of Pakistan


Advanced accounting and financial reporting

26.8 PARENT COMPANY LIMITED


Parent Company Limited (PCL) is a listed company and owns 80% and 75%
equity in LS Limited and FS Limited respectively. FS is registered and operates in
a foreign country and its functional currency is CU. Summarised statements of
financial position as at 30 June 2014 and other information relating to the group
companies are as under:
PCL LS FS
Rs. in million CU in million
Assets
Property, plant and
equipment 4,200 3,500 250
Investments in LS and FS 6,500 - -
Current assets 3,500 4,000 450
14,200 7,500 700
Equity and liabilities
Share capital (Rs. 10/CU 10
each 6,000 1,800 120
Retained earnings 3,500 900 280
Current liabilities 4,700 4,800
14,200 7,500 700
Profit after tax for the year
ended 30 June 2014 700 400 30
Final dividend for the year
ended 30 June 2013: 12% - 15%
Cash (paid on 1 January
2014) 10% 20% -
700 400 30
The following information is also available:
i.
At the acquisition
No. of date
Investment
Company shares Cost Fair
date Retained
acquired value of
earnings *
NCI
---- in million ----
Rs.
LS 1-Jan-2012 120 2,000 Rs. 250 Rs. 540
FS 1-Jul-2012 9 CU 300 CU 160 CU 90
*NCI stands for Non-controlling interest
ii. On the date of acquisition, fair value of the net assets of LS and FS were
equal to their book value. However, a contingent liability of Rs. 25 million
was disclosed in the financial statements of LS. PCL's legal adviser had at
that time estimated that LS would be liable to pay Rs. 6 million to settle
the claim and it was finally settled at the same amount in May 2014.

iii. No further shares have been issued by LS and FS since their acquisitions,
except for the bonus issue as mentioned above.

Emile Woolf International 120 The Institute of Chartered Accountants of Pakistan


Questions

iv. An impairment test carried out on 30 June 2014 revealed that goodwill of
FS is impaired by CU 10 million.
v. PCL values non-controlling interest on the date of acquisition at fair value.
vi. The exchange rates in terms of Rs. per CU, were as follows:

1-Jul-2012 30-Jun-2013 1-Jan-2014 30-Jun-2014 Average for


2013-14

Rs. 15.00 Rs. 16.80 Rs. 16.90 Rs. 17.30 Rs. 17.00

vii. The break-up of exchange reserve in the consolidated financial


statements for the year ended 30 June 2013 is as follows:

Relating to goodwill Rs. 148.50 million

Relating to translation of foreign operations Rs. 463.05 million

Required:
In accordance with the requirements of the International Financial Reporting
Standards, prepare:

(a) Consolidated statement of financial position as at 30 June 2014; and


(b) Consolidated statement of other comprehensive income for the year
ended 30 June 2014. (Ignore taxation)

Emile Woolf International 121 The Institute of Chartered Accountants of Pakistan


Advanced accounting and financial reporting

CHAPTER 27: IAS 7: STATEMENTS OF CASH FLOWS

27.1 EVERNEW LTD


The following relates to the financial statements of Evernew Ltd.
(a) Consolidated statement of profit or loss for the year ended 31 December
2016
Rs.000
Operating profit 144,000
Interest expenses (10,080)
Profit after interest 133,920
Profit from disposal of subsidiary 5,040
Profit before taxation 138,960
Taxation (46,800)
Profit after taxation 92,160

Profit for the year attributable to:


Owners of the parent 84,960
Non-controlling interest 7,200
Retained profit c/f 92,160
(b) Consolidated statement of financial position as at 31 December, 2016
2016 2015
Assets Rs.000 Rs.000 Rs.000 Rs.000
Non-current assets 369,720 360,000
Current assets:
Inventory 180,000 165,600
Trade receivables 151,200 136,800
Cash in hand 63,360 14,400
394,560 316,800
Total assets 764,280 676,800
Equity and liabilities
Ordinary share capital 144,000 144,000
Accumulated profits 317,520 232,560
461,520 376,560
Non-controlling interest 36,360 41,400
10% debenture 68,400 90,000

Current liabilities
Trade creditors 108,000 93,600
Taxation 46,800 39,240
Bank overdraft 43,200 36,000
Net current assets 198,000 168,840
Total equity and liabilities 764,280 676,800

Emile Woolf International 122 The Institute of Chartered Accountants of Pakistan


Questions

(c) The following additional information is relevant:


(i) Depreciation for the year in the consolidated profit and loss account
was Rs. 72,720,000. Non-current assets were not disposed by the
group except those made during disposal of the investment in the
shares of Pastit Limited.
(ii) Evernew Ltd sold its investment in Pastit Limited in July 2016. The
entire 80% shareholding in the subsidiary was sold for Rs.
39.6million. Information about the disposal is as follows:
Rs.000 Rs.000
Inventories 14,400
Receivables 18,000
Non-current assets 28,800
Trade creditors (10,800)
Taxation (2,160)
Bank overdraft (1,440)
Debenture stock (3,600) (18,000)
43,200
Non-controlling interest (8,640)
34,560
The investment was acquired many years ago for Rs. 13.68million when
the net assets of Pastit Limited were Rs. 14.4million. Goodwill had been
fully written off before due to impairment.

Required
Prepare the Evernew Ltd group consolidated cash flow statement for the
year ended 31 December, 2016.

27.2 BELLA
The financial statements of Bella include the following:
Statements of financial position as at 31 March Year 6
Year 6 Year 5
Rs.000 Rs.000 Rs.000 Rs.000
Assets
Non-current assets
Property, plant and equipment 12,900 8,000
Intangible assets 800 300
13,700 8,300
Current assets
Inventories 280 100
Trade and other receivables 1,290 1,350
Cash 55 45
1,625 1,495

Emile Woolf International 123 The Institute of Chartered Accountants of Pakistan


Advanced accounting and financial reporting

Year 6 Year 5
Rs.000 Rs.000 Rs.000 Rs.000
Total assets 15,325 9,795
Equity and liabilities
Capital and reserves
Issued capital (Rs. 1 ordinary 1,900 1,100
shares)
Share premium 95 30
Accumulated profits 11,407 7,540
13,402 8,670
Non-current liabilities
Long-term loans 600 500
600 500
Current liabilities
Bank overdraft (repayable on 313 -
demand)
Trade and other payables 430 275
Interest payable 40 25
Current tax payable 540 325
1,323 625
Total equity and liabilities 15,325 9,795

Statement of profit or loss for the year ended 31 March Year 6 (extract).
Rs.000
Operating profit 4,677
Interest payable (60)
Profit before tax 4,617
Tax expense (400)
Profit for the period 4,217
The following occurred during the year.
(1) Dividends of Rs. 350,000 were paid.
(2) New plant was purchased for Rs. 6 million.
(3) Old plant which had a net book value of Rs. 800,000 was sold for Rs.
700,000.
(4) Shares were issued for cash during the period.
Required
Prepare a statement of cash flows for the year ended 31 March Year 6 using the
indirect method.

Emile Woolf International 124 The Institute of Chartered Accountants of Pakistan


Questions

27.3 BISHOP GROUP


You are provided with the information set out below relating to a group of
companies.
Consolidated statement of profit or loss for Bishop Group
for the year ended 31 December 20X2
20X2 20X1
Rs.000 Rs.000
Revenue 19,308 18,173
Cost of sales (4,315) (4,620)

Gross profit 14,993 13,553
Distribution costs (6,439) (6,126)
Administrative expenses (5,705) (6,719)

Profit before tax and finance costs (note 1) 2,849 708
Finance income 90 75
Finance costs (note 2) (350) (230)

Profit before taxation 2,589 553
Income tax expense (note 3) (800) (125)
Profit for the year 1,789 428

Attributable to:
Equity holders of the parent 1,369 318
Non-controlling interest 420 110

1,789 428

Summary statement of changes in equity for the year to
31 December 20X2
Foreign
Issued Share currency Retained
capital premium translation earnings Total NCI Equity
B/fwd 1 Jan 7,500 77 2,100 4,905 14,582 2,500 17,082
Issue of shares 3,500 324 3,824 3,824
Profit for year 1,369 1,369 420 1,789
Dividends (600) (600) (295) (895)
Exchange gain 700 700 175 875

C/fwd 31 Dec 11,000 401 2,800 5,674 19,875 2,800 22,675

Notes: NCI = non-controlling interest
Exchange gain = exchange gain on translation of subsidiary

Emile Woolf International 125 The Institute of Chartered Accountants of Pakistan


Advanced accounting and financial reporting

Consolidated statement of financial position as at 31 December 20X2


20X2 20X1
Rs.000 Rs.000
Non-current assets
Tangible assets 11,720 7,520
Investments 3,000 2,700
Current assets
Inventories 6,135 5,740
Receivables 5,720 4,380
Cash at bank and in hand 820 169
27,395 20,509
Equity and liabilities
Ordinary share capital 11,000 7,500
Share premium 401 77
Foreign currency translation 2,800 2,100
Retained earnings 5,674 4,905
Equity attributable to owners of parent 19,875 14,582
Non-controlling interest 2,800 2,500
Total equity 22,675 17,082
Current liabilities
Payables 1,420 1,760
Tax 700 167
Obligations under finance leases 110 50
Non- current liabilities
Loans 1,200 800
Obligations under finance leases 740 250
Provisions for liabilities and charges
Deferred tax 550 400
27,395 20,509
Notes to the accounts
(1) Operating profit is stated after charging Rs.000 Rs.000
Depreciation: Owned assets 960 840
Assets held under finance leases 240 120
1,200 960

(2) Finance costs Rs.000 Rs.000


Loan interest 120 80
Finance charge on finance leases 205 132
Exchange rate losses on long-term loans 25 18
350 230

(3) Taxation on profits on ordinary activities Rs.000 Rs.000


Tax on income at 30% 600 90
Deferred tax 150 35
Under provision in respect of previous years 50 -
800 125

Emile Woolf International 126 The Institute of Chartered Accountants of Pakistan


Questions

(4) Foreign exchange differences Rs.000 Rs.000


Gains arising on re-translation 700 400
The exchange rate gain relates to the translation of an 80% owned
overseas subsidiary, Louise, under the closing rate method. The gain
comprises:
Non current assets 424
Inventories 117
Receivables 339
Cash 53
Trade payables (58)
875
Attributable to NCI (175)
Attributable to owners of parent company 700
(5) During the year non-current assets additions of Rs. 700,000 were
acquired under finance leases. Payments on finance leases are made in
arrears. The net book value of non-current assets disposed of was Rs.
720,000, with sale proceeds of Rs. 810,000.
Required
(a) Prepare the group statement of cash flows of Bishop in accordance with
IAS 7 together with any required notes for the year ended 31 December
20X2.
(b) Explain why external users of financial statements benefit from receiving a
statement of cash flows.

27.4 THE GRAPE GROUP


The draft statements of financial position and statement of profit or loss of the
Grape Group at 31 March Year 4 and 31 March Year 3 are as follows:
Notes Year 4 Year 3
Rs.000 Rs.000
Non-current assets
Intangible assets 24 -
Property, plant and equipment (1) 13,515 12,990
Investments associated undertakings 1,966 1,920
15,505 14,910
Current assets
Inventory 11,657 10,530
Receivables 7,209 6,936
Cash at bank and in hand 5,190 1,728
24,056 19,194
39,561 34,104

Emile Woolf International 127 The Institute of Chartered Accountants of Pakistan


Advanced accounting and financial reporting

Capital and reserves


Share capital 8,100 7,425
Share premium 1,989 1,470
Accumulated profits 13,200 8,700
23,289 17,595
Non-current liabilities (2) 6,900 7,890
Current liabilities (3) 9,372 8,619
39,561 34,104

Statement of profit or loss: Year to 31 March Year 4 Year 3


Rs.000 Rs.000
Sales revenue 74,100 59,400
Cost of sales (54,138) (42,240)
Gross profit 19,962 17,160
Distribution costs (5,889) (4,869)
Administrative expenses (4,092) (3,384)
Operating profit 9,981 8,907
Income from interests in associates 139 144
Loss on sale of tangible non-current assets (18) -
Interest expense (552) (651)
Profit before tax 9,550 8,400
Income tax expense (2,950) (2,400)
Profit after tax 6,600 6,000

Notes
(1) Property, plant and equipment
Year 4 Year 3
Rs.000 Rs.000
Cost
At 1 April 20,598 19,416
Additions 1,875 2,022
Disposals (429) (840)
At 31 March 22,044 20,598
Depreciation
At 1 April 7,608 6,984
Charge for year 1,176 936
Disposals (255) (312)
At 31 March 8,529 7,608
Net book value 13,515 12,990

Emile Woolf International 128 The Institute of Chartered Accountants of Pakistan


Questions

(2) Non-current liabilities


Year 4 Year 3
Rs.000 Rs.000
8% Loan notes 6,900 6,900
10% Unsecured loan notes - 990
6,900 7,890
(3) Current liabilities
Year 4 Year 3
Rs.000 Rs.000
Accounts payable 6,422 6,219
Income tax 2,950 2,400
9,372 8,619
You are provided with the following additional information:
Interest on the 8% loan note is payable half-yearly on 30 September and 31
March.
Income tax is payable nine months after the year-end.
Dividends of Rs. 2.1m and Rs. 2.4m were proposed for Year 3 and Year 4
respectively.
During the year, the Grape Group acquired 100% of the ordinary share
capital of Pip. The purchase was financed by Rs. 346,000 in cash and the
issue of 54,000 ordinary shares of Rs. 1 each. The ordinary shares had a
market value of Rs. 152,000. The following figures related to Pip at the date
of acquisition:
Rs.000
Property, plant and equipment 315
Inventory 139
Receivables 85
Cash at bank and in hand 3
Payables (68)
474
Share capital 180
Reserves 294
474
(4) The share capital consists of Rs. 1 ordinary shares.
Required
Prepare a statement of cash flows and related notes for the Grape Group for the
year ended 31 March Year 4.

Emile Woolf International 129 The Institute of Chartered Accountants of Pakistan


Advanced accounting and financial reporting

CHAPTER 28: IAS 33: EARNINGS PER SHARE

28.1 AIRCON LTD


Mr Hamad, currently owns 20 million shares in Aircon Ltd. He recently received
the published financial statements of Aircon Ltd for the year ended 31 March
2016. Mr Hamad is not sure how the performance of the company during the
year will affect the market value of the entitys shares but he is aware that the
earnings per share statistics are often used by analysts in assessing the
performance of listed companies.
Extracts from these published financial statements and other relevant information
are given below.
Statement of profit or loss for the period ended 31 March 2016
2016 2015
Rs.m Rs.m
Revenue 18,000 15,300
Cost of sales (11,340) (9,180)
Gross profit 6,660 6,120
Operating expenses (3,420) (3,240)
Operating profit 3,240 2,880
Interest payable (540) (576)
Profit before tax 2,700 2,304
Taxation (846) (720)
Profit after tax 1,854 1,584
Statement of financial position as at 31 March 2016
2016 2015
Rs.m Rs.m Rs.m Rs.m
Intangible assets 5,400 -
Tangible assets 7,200 6,660

12,600 6,660
Current Assets
Inventory 2,340 1,800
Receivables 2,700 2,160
Cash in bank 180 5,220 162 4,122

17,820 10,782

Capital and Reserves
Share Capital 2,700 900
Share Premium 4,860 900
Retained Earnings 1,620 1,206

9,180 3,006
Current Liabilities
Trade Payables 3,060 2,160
Taxation 900 756
Bank Overdraft 1,080 1,260

5,040 4,176

14,220 7,182
15% Loan Note 3,600 3,600

17,820 10,782

Emile Woolf International 130 The Institute of Chartered Accountants of Pakistan


Questions

The following information is also relevant:


(i) The share capital of the company comprises Rs. 1 equity shares only.
(ii) On 1 October 2015, the company made a rights issue to existing
shareholders of two new shares for every one share held at a price of Rs.
5.94 per share and paid issue cost of Rs. 180,000.
(iii) The market price of shares immediately before the rights issue was Rs.
6.30 per share.
(iv) No other changes took place in the equity capital of Aircon Ltd in the year
ended 31 March 2016.
Required
(a) Compute EPS for the year and the comparative figures that will be included
in the published financial statements of Aircon Ltd for the year ended 31
March 2016.
(b) Using the extracts you have been provided with, write a report to Mr Hamad
identifying the key factors which led to the change in the EPS of Aircon Ltd
since the year ended 31 March 2016.
(c) Comment on the relevance of the EPS statistics to shareholders.

28.2 CACHET LTD


The statement of profit or loss for the year ended 31 December 2016 relates to
Cachet Ltd.
Rs. Rs.
Profit Before Tax 121,900
Less: Taxation 52,900
69,000
Less: Transfer to general reserve 5,750
Dividends:
Preference shares 1,380
Ordinary shares 2,070
(92,00)
Retained profit 59,800
1 January 2016, the issued share capital of Cachet Ltd was 23,000 6%
preference shares of Rs. 1 each and 20,700 ordinary shares of Rs. 1 each.
Required
Calculate the basic and diluted earnings per share for the year ended 31
December, 2016 under the following circumstances:
(i) No change in the issued share capital.
(ii) The company made a bonus issue of one ordinary share for every four
shares in issue at 30 September, 2016.
(iii) The company made a rights issue of shares on 1 October 2016 in the
proportion of 1 for every 5 shares held at a price of Rs. 1.20. The middle
market price for the shares on the last day of quotation cum rights was Rs.
1.80 per share.

Emile Woolf International 131 The Institute of Chartered Accountants of Pakistan


Advanced accounting and financial reporting

28.3 MARY
On 1 January Year 5, Mary had 5 million ordinary shares in issue. The following
transactions in shares took place during the next year.
1 February A 1 for 5 bonus issue
1 April A 1 for 2 rights issue at Rs. 1 per share. The market price of the
shares prior to the rights issue was Rs. 4.
1 June An issue at full market price of 800,000 shares.
In Year 5 Mary made a profit before tax of Rs. 3,362,000. It paid ordinary
dividends of Rs. 1,200,000 and preference dividends of Rs. 800,000. Tax was
Rs. 600,500. The reported EPS for Year 4 was Rs.0.32.
Required
Calculate the EPS for Year 5, and the adjusted EPS for Year 4 for comparative
purposes.

28.4 MANDY
Mandy has had 5 million shares in issue for many years. Earnings for the year
ended 31 December Year 4 were Rs. 2,579,000. Earnings for the year ended 31
December Year 3 were Rs. 1,979,000. Tax is at the rate of 30%.
Outstanding share options on 500,000 shares have also existed for a number of
years. These can be exercised at a future date at a price of Rs. 3 per share. The
average market price of shares in Year 3 was Rs. 4 and in Year 4 was Rs. 5.
On 1 April Year 3 Mandy issued Rs. 1,000,000 convertible 7% bonds. These are
convertible into ordinary shares at the following rates.
On 31 December Year 6 30 shares for every Rs. 100 of bonds
On 31 December Year 7 25 shares for every Rs. 100 of bonds
On 31 December Year 8 20 shares for every Rs. 100 of bonds
Required
Calculate the diluted EPS for Year 4 and the comparative diluted EPS for Year 3.

28.5 AAZ LIMITED


The profit after tax earned by AAZ Limited during the year ended December 31,
2016 amounted to Rs. 127.83 million. The weighted average number of shares
outstanding during the year were 85.22 million.
Details of potential ordinary shares as at December 31, 2016 are as follows:
The company had issued debentures which are convertible into 3 million
ordinary shares. The debenture holders can exercise the option on
December 31, 2018. If the debentures are not converted into ordinary
shares they shall be redeemed on December 31, 2018. The interest on
debentures for the year 2016 amounted to Rs. 7.5 million.
Preference shares issued in 2013 are convertible into 4 million ordinary
shares at the option of the preference shareholders. The conversion option
is exercisable on December 31, 2020. The dividend paid on preference
shares during the year 2016 amounted to Rs. 2.45 million.

Emile Woolf International 132 The Institute of Chartered Accountants of Pakistan


Questions

The company has issued options carrying the right to acquire 1.5 million
ordinary shares of the company on or after December 31, 2016 at a strike
price of Rs. 9.90 per share. During the year 2016, the average market price
of the shares was Rs. 11 per share.
The company is subject to income tax at the rate of 30%.
Required
(a) Compute basic and diluted earnings per share.
(b) Prepare a note for inclusion in the companys financial statements for
the year ended December 31, 2016 in accordance with the requirements
of International Accounting Standards.

28.6 ABC LIMITED


The following information pertains to ABC Limited, in respect of year ended
March 31, 2016.
Rs. in 000
Consolidated profit for the year (including non-controlling interest) 15,000
Profit attributable to non-controlling interest 2,000
Dividend paid during the year to ordinary shareholders 4,000
Dividend paid on 10% Cumulative preference shares for the year
2015 2,000
Dividend paid on 10% Cumulative preference shares for the year
2016 2,000
Dividend declared on 12% Non-cumulative preference shares for the
year 2016 2,400
(i) The company had 10 million ordinary shares at March 31, 2015.
(ii) The cumulative preference shares were issued at the time of inception of
the company.
(iii) The 12% non-cumulative preference shares are convertible into ordinary
shares, on or before December 31, 2017 at a premium of Rs. 2 per share.
The conversion rights are not adjusted for subsequent bonus issues.
(iv) 0.50 million non-cumulative preference shares were converted into ordinary
shares on July 1, 2015.
(v) The dividend declared on the non-cumulative preference shares, as
referred above, was paid in April 2016.
(vi) 1.20 million right shares of Rs. 10 each were issued at a premium of Rs.
1.50 per share on October 1, 2015. The market price on the date of issue
was Rs. 12.50 per share.
(vii) 20% bonus shares were issued on January 1, 2016.
(viii) Due to insufficient profit no dividend was declared during the year ended
March 31, 2015.
(ix) The average market price for the year ended March 31, 2016 was Rs. 15
per share.
Required
Compute the basic and diluted earnings per share and prepare a note for
inclusion in the consolidated financial statements for the year ended March 31,
2016.

Emile Woolf International 133 The Institute of Chartered Accountants of Pakistan


Advanced accounting and financial reporting

28.7 ALPHA LIMITED


Alpha Limited (AL), a listed company, acquired 80% equity in Zee Limited (ZL) on
1 July 2010. The following information has been extracted from their draft
financial statements:
AL ZL
----- Rs. in '000 -----
Balance as at 1 January 2013:
Share capital (Rs. 100 each) 80,000 35,000
12% Convertible bonds (Rs. 100 each) 30,000 -
Profit for the year ended 31 December 2013 (after tax) 60,000 25,000
Following information is also available:
(i) The bonds were issued at par on 1 January 2011 and are convertible
at any time before the redemption date of 31 December 2015, at the
rate of five ordinary shares for every four bonds.
(ii) Cost and fair value information of ZLs investment property is as
under:
31-Dec-2013 31-Dec-2012
-------- Rs. in '000 --------
Cost 65,000 60,000
Fair value 67,000 59,000
ZL uses cost model while the group policy is to use the fair value
model to account for investment property.
(iii) AL operates a defined benefit gratuity scheme for its employees. The
actuarys report has been received after the preparation of draft
financial statements and provides the following information pertaining
to the year ended 31 December 2013:
Rs. in '000
Actuarial losses 150
Current service costs 8,000
Net interest income 3,000
(iv) On 1 August 2013, under employees share option scheme, 60,000
shares were issued by AL to its employees at Rs. 150 per share
against the average market price of Rs. 250 per share.
(v) Dividend details are as under:
AL ZL
2013 (Interim) 2012 (Final) 2013 (Interim) 2012 (Final)
Cash 18% 10% 12% 15%
Bonus
shares - 20% - 16%
At the time of payment of dividend, income tax at 10% was deducted
by AL and ZL.
(vi) Applicable tax rate for business income is 35%.

Required:
Extracts from the consolidated profit and loss account of Alpha Limited (including
earnings per share) for the year ended 31 December 2013 in accordance with the
International Financial Reporting Standards.
(Note: Comparative figures and information for notes to the financial statements
are not required)

Emile Woolf International 134 The Institute of Chartered Accountants of Pakistan


Questions

CHAPTER 29: ANALYSIS AND INTERPRETATION OF FINANCIAL


STATEMENTS

29.1 ALPHA LIMITED AND OMEGA LIMITED


Alpha Limited and Omega Limited are in the same trade, but operate in different
areas. Their accounts for the year ended 31 December, 2016 are as follows:
Profit and loss account Alpha Limited Omega Limited
Rs.000 Rs.000 Rs.000 Rs.000
Sales 1,440 1,720
Less: Cost of sales 1,120 1,342

Gross profit 320 378
Less: Overheads 220 300

Profit before tax 100 78
Taxation 40 30
Dividends 20 24

60 54

Retained earnings 40 24

Statement of financial position

Share capital of Rs. 1 each 600 200


Reserves 240 104

840 304
8% Debentures - 120
840 424

Represented by:
Non-current assets at cost 660 520
Less: Depreciation 200 160

460 360
Current assets:
Inventory 280 172
Receivables 310 300
Cash 30 32

620 504

Current liabilities:
Taxation 40 30
Creditors 180 344
Bank overdraft - 42
Dividends 20 24

240 440

Net Current assets 380 64

840 424

Emile Woolf International 135 The Institute of Chartered Accountants of Pakistan


Advanced accounting and financial reporting

Required
(a) Compute the following ratios for each of the companies:
(i) Current ratio
(ii) Acid test
(iii) Creditors ratio
(iv) Collection period or Receivables Ratio
(iv) Earnings per share
(b) Carry out comparative analysis of the companies based on the computed
ratios in (a) above.

29.2 COOK LIMITED


The following are the financial statements of Cook Limited for the years ended 31
December, 2015 and 2016.
Profit and loss accounts for the year ended 31 December 2016
2016 2015
Rs.m Rs.m
Sales 5,600 1,800
Cost of sales (3,360) (720)
Gross profit 2,240 1,080
Selling expenses (540) (300)
Bad debts (280) (36)
Depreciation (416) (116)
Interest (384) (24)
Net profit 620 604
Balance b/fwd 1,258 654
Balances c/fwd 1,878 1,258
Statements of financial position
2016 2015
Rs.m Rs.m Rs.m Rs.m
Non-current assets
Factory 882 900
Machinery 3,582 980
4,464 1,880
Current assets
Inventory 476 60
Receivables 1,166 166
Bank 24
1,642 250
Current liabilities
Creditors (350) (72)
Bank (22) --
(372) (72)

Emile Woolf International 136 The Institute of Chartered Accountants of Pakistan


Questions

2016 2015
Rs.m Rs.m Rs.m Rs.m
Net current assets 1,270 178
5,734 2,058
Loans (3,200) (200)
2,534 1,858

Share capital 656 600


Profit and loss 1,878 1,258
2,534 1,858
The Directors of Cook Limited appointed a new sales manager towards the end
of 2016. This manager devised a plan to increase sales and profit by means of a
reduction in selling price and extended credit terms to customers.
This involved considerable investment in new machinery early in 2016 to meet
the increased sales. All sales are on credit.
Required
(a) Explain whether the performance for the year ended 31 December 2016
and the financial position at the date have improved as a result of the new
policies adopted by the company. You should support your answer with
appropriate profitability and liquidity ratios.
(b) Calculate the amount of cash which would be released if the company
could impose a collection period of 45 days.

29.3 FITZROY LIMITED


The following ratios and an un-completed statement of financial position of Fitzroy
Limited were extracted from the working sheet of a financial consultant to the
company.
RATIOS
1. Cash flow ratio = 0.6711:1
2. Current ratio = 2.5:1
3. Cash ratio = 0.2:1
4. Average collection period = 46.596 days
5. Gearing assets = 0.34545:1
6. Non-current assets = 70% of total assets
7. Inventory turnover = 5.74 times
1
8. Margin and mark-up = /5 and respectively
9. Creditors payment period = 27.06 days
Fitzroy Limited

Emile Woolf International 137 The Institute of Chartered Accountants of Pakistan


Advanced accounting and financial reporting

Uncompleted statement of financial position as at 31 December, 2016


Capital Rs.000 Non-current assets Rs.000
Ordinary shares X Land & building 5,000
71/2 debenture X Furniture & fitting 1,450
RESERVES Motor vehicles 2,300
Profit & Loss X 8,750

Current liabilities Current assets


Creditors X Inventories 1,800
Dividend X Receivables X
Cash X
X X
You are given the following additional information relating to the financial
statements.
1. Depreciation on all non-current assets for the year is Rs. 972,000
2. Dividend payable is 11.76% of ordinary shares
3. Company tax is estimated to be Rs. 400,000
4. Retained profit brought forward from the previous year is Rs. 800,000
Required
From the above information, you are required to complete the statement of
financial position of Fitzroy Limited as at 31 December, 2016 and prepare the
statement of profit and loss for the period then ended. (15 marks)

29.4 TRAVELWELL LTD


The recently produced financial statements of Travelwell Ltd for the year to 30
September 2016 are shown below, together with comparative figures for the
previous year.
Statement of financial position as at 30 September 2016 2015
Rs.000 Rs.000
Non-current assets
Property, plant and equipment 252,000 105,000
Goodwill 12,000 nil
264,000 105,000
Current assets
Inventory 29,000 17,000
Trade receivables 17,000 9,000
Bank Nil 28,000
46,000 54,000
Total assets 310,000 159,000
Equity and liabilities
Equity shares of Rs. 1 each 100,000 100,000
Retained earnings 44,000 41,000
144,000 141,000

Emile Woolf International 138 The Institute of Chartered Accountants of Pakistan


Questions

Statement of financial position as at 30 September 2016 2015


Rs.000 Rs.000
Non-current liabilities
7.5% loan notes 130,000 nil
Current liabilities
Trade payables 28,000 15,000
Bank overdraft 2,000 nil
Current tax payable 6,000 3,000
36,000 18,000
Total equity and liabilities 310,000 159,000

Statement of profit or loss for the year ended 2016 2015


30 September 2016
Rs.000 Rs.000
Revenue 300,000 210,000
Cost of sales (240,000) (175,000)
Gross profit 60,000 35,000
Operating expenses (26,250) (23,000)
Finance costs (9,750) nil
Profit before tax 24,000 12,000
Income tax expenses (at 25%) (6,000) (3,000)
Profit for the year 18,000 9,000
The following are extracts from the report of the Chief Executive Officer.
Highlights of the performance of Travelwell Ltd for the year ended 30 September
2016 are:
(1) an increase of 43% in sales revenue
(2) an increase in the gross profit margin from 16.7% to 20%
(3) a 100% increase in profit for the year.
In response to the improvement in financial performance the board of directors
paid a dividend of Rs. 15m in September 2016, an increase of 25% on the
previous year.
The following information is also available.
On 1 October 2015, Travelwell Ltd purchased 100% of the net assets of Rondel
Ltd for Rs. 130 million. Rondel Ltd was previously a privately-owned business
entity. The contribution of this purchase to the financial results of Travelwell Ltd
for the year to 30 September 2016 was as follows:
Rs.000
Revenue 90,000
Cost of sales (50,000)
Gross profit 40,000
Operating expenses (11,000)
Profit before tax 29,000

Emile Woolf International 139 The Institute of Chartered Accountants of Pakistan


Advanced accounting and financial reporting

There were no disposals of non-current assets during the year.


The following ratios were calculated for the year ended 30 September 2015:
Return on year-end capital employed 8.5%
profit before interest and tax
(= /total assets less current liabilities)
Net asset turnover (net assets = capital employed) 1.5
Net profit margin (before tax) 5.7%
Current ratio 3.0
Closing inventory holding period (in days) 35
Trade receivables collection period (in days) 16
Trade payables payment period (in days and based on cost of sales) 31
Gearing ratio (debt/debt plus equity) nil

Required
(a) Calculate ratios for Travelwell Ltd for the year ended 30 September 2016
equivalent to those shown above for the year to 30 September 2015. Show
your workings.
(b) Assess the financial performance and financial position of Travelwell Ltd for
the year ended 30 September 2016, in comparison with the previous year
referring to the comments in the report of the Chief Executive Officer and
you should also assess the effect of the purchase of the net assets of
Rondel Ltd.

29.5 SACHAL LIMITED


Sachal Limited (SL) is planning to acquire 100% shareholdings in Waris
Limited (WL).
Before submission of financial proposal, SL is carrying out an analysis of WLs
financial and operating performance. The CFO of SL has gathered the following
information which is based on the financial statements for the year ended
December 31, 2016:

WLs Industry Ratios


Description
Ratios High Low Average
Operating performance
Ratios
Gross profit 29% 30% 20% 25%
Operating profit 11% 15% 10% 13%
Return on shareholders 9% 13% 7% 10%
equity
Working capital ratios
Current ratio 1.54 : 1 2:1 1:1 1.5 : 1
Inventory turnover days 83 days 114 days 81 days 91 days
Receivables collection 93 days 95 days 60 days 74 days

Emile Woolf International 140 The Institute of Chartered Accountants of Pakistan


Questions

WLs Industry Ratios


Description
Ratios High Low Average
Gearing ratios
Debt equity ratio 55 : 45 60 : 40 40 : 60 50 : 50
Interest cover 1.3 times 3 times 1.2 times 2 times
Investors ratios
Earnings per share Re. 0.9 Rs. 1.8 Re. 0.75 Rs. 1.2
Dividend per share Re. 0.2 Re. 0.9 Re. 0.25 Re. 0.6

Required
(a) Draft a report to the board of directors, on behalf of the CFO, analyzing
the financial performance of Waris Limited by evaluating each category of
ratios in comparison with the industry.
(b) List any four types of additional information which would have helped you
in a better analysis.

29.6 OPAL INDUSTRIES LIMITED


Opal Industries Limited (OIL) is a listed company. As at 30 June 2014 OIL has
various investments as detailed under:
At the acquisition
date
Investment Equity Share capital Retained
Company date held Cost (Rs. 100 each) earnings
--------------- Rs. in million ---------------
AL 1-Jul-2012 30% 50 80 60
BL 31-Dec-2011 10% 8 70 40
GL 1-Jan-2014 65% 195 150 95
Information pertaining to profit and dividend of the investee companies is as
follows:
Profit/(loss) for the year Final cash dividend for year
ended ended
Company 2014 2013
Rs. in million 2014 2013
AL 30 28 20% 16%
BL (10) 14 - 18%
GL 55 50 30% 15%
BL is a listed company and fair value of its shares as at 30 June 2014 was Rs.
110 per share (2013: Rs. 160). OIL classifies investment in BL as available for
sale.
AL and GL are private companies and market value of their shares is not
available.

Emile Woolf International 141 The Institute of Chartered Accountants of Pakistan


Advanced accounting and financial reporting

GL is the first subsidiary of OIL, since its incorporation. Following information


pertains to OIL:
2013 2012
Rs. in million
Share capital (Rs. 100 each) 2,875 2,500
Profit for the year 1,260 1,100
Closing retained earnings balance 850 465
Final dividend - cash 25% 20%
- bonus issue - 15%
OILs profit for the year ended 30 June 2014 prior to taking effects of the
transactions of its investee companies was Rs. 1,450 million and it has
announced a final cash dividend of 30%.
Required:
Prepare following for inclusion in the first separate financial statements of OIL for
the year ended 30 June 2014 as required by the International Financial Reporting
Standards.

(a) Movement in retained earnings for inclusion in the statement of changes in


equity; and
(b) Note on investments.
(Show comparative figures and ignore taxation)

Emile Woolf International 142 The Institute of Chartered Accountants of Pakistan


Questions

CHAPTER 30: SUNDRY STANDARDS AND INTERPRETATIONS

30.1 GUJRANWALA FOODS LIMITED


Gujranwala Foods Limited has the following information in its records for the year
ended 31 January 2017.
Chickpea Apricot Dates Onion
Rs.000 Rs.000 Rs.000 Rs.000
Sales 200 300 750 250
Purchases 75 150 300 80
Subsidies 40 80 60 60
Own consumption 45 40 75 20
Opening Inventory 40 40 70 -
Closing Inventory 60 110 300 60
The farm expenses for the period 1 February 2016 to 30 April 2017 are as
follows:
Rs.
Casual labour 20,000
Regular workers 30,000
Land preparation and clearing costs 80,000
Hire of tractors 60,000

The farms non- current assets for the year ended 31 January 2017 were as
follows:
Rs.
Farms irrigation at cost 800,000
Farms implement and equipment 400,000

Additional Information:
(i) Farms irrigation costs are to be written off over 10 years
(ii) Farms implement and equipment were purchased on 31 April 2016 and
these are to be depreciated at 20% per annum.
Required
(a) Prepare Gujranwala Foods Limiteds gross output and statement of profit or
loss for the year ended 31 January 2017.
(b) In accordance with IAS 41 on Agriculture, you are required to define the
following terms:
(i) Biological assets
(ii) Biological transformation
(iii) Harvest

Emile Woolf International 143 The Institute of Chartered Accountants of Pakistan


Advanced accounting and financial reporting

30.2 WAH AGRIPROD LTD


The following trial balance was extracted from Wah Agriprod Ltds books at 31
December 2016:
Rs.000 Rs.000
Revenue 855,000
Purchases 503,600
Distribution costs 31,950
Administrative expenses 104,400
Finance costs 1,020
Investment income 18,250
Leased property at cost 125,000
Plant and equipment at cost 98,800
Accumulated amortisation/depreciation at 1/1/2016:
Leased property 35,000
Plant and equipment 28,800
Financial instruments 40,500
Inventory at 1/1/2016 85,075
Trade receivables 72,400
Trade payables 62,550
Bank 5,175
Share premium 8,400
Equity shares (Rs. 1 per share) 115,000
Retained earnings at 1/1/2016 55,600
Investment in subsidiary 131,030
6% Convertible Loan Notes 10,000
1,193,775 1,193,775
The following notes are relevant:
(i) Revenue includes goods sold and dispatched from 15 December 2016, on
a 30-day right of return basis. Their selling price was Rs. 3.6million and
they were sold at a gross profit margin of 20%. Wah Agriprod Ltd is
uncertain as to whether any of these goods will be returned within the 30-
day period.
(ii) The directors decided to revalue the leased property in line with recent
increases in market values. On 1 September 2016, an independent
surveyor valued the leased property at Rs. 99million, which the directors
have accepted. The leased property was amortised over an original life of
25 years which has not changed. Wah Agriprod Ltd does not make a
transfer to retained earnings in respect of excess amortisation.
(iii) The plant and equipment is depreciated at 15% per annum using the
reducing balance method and all depreciation and amortisation are charged
to cost of sales. No depreciation or amortisation has yet been charged on
any non-current asset for the year ended 31 December 2016.
(iv) The financial instruments are investments in equities of public companies
and had a fair value of N39.7million on 31 December 2016. There were no
purchases or disposals of any of these investments during the year. Wah
Agriprod Ltd has not made the election in accordance with IFRS 9 on
Financial Instruments. The company adopts this standard when accounting
for its financial assets.

Emile Woolf International 144 The Institute of Chartered Accountants of Pakistan


Questions

(v) On 20 November 2016, Wah Agriprod Ltds share price stood at Rs. 2.20
per share. On this date, a dividend that was calculated to give a dividend
yield of 5% was paid by the company. The dividend paid was included as
part of administrative expenses figure shown in the trial balance.
(vi) The inventory on Wah Agriprod Ltds premises at 31 December 2016, after
stock taking was valued at cost of Rs. 106million and a provision for income
tax for the year then ended of Rs. 86.75million is required.
(vii) During the year, the company issued ten million shares at a premium of
20%. The conversion rate for the loan note is Rs. 100 loan notes for three
ordinary shares. The current market price per share is Rs. 2.54.
Required
(a) Prepare the statement of profit or loss and other comprehensive income for
the year ended 31 December 2016.
(b) Prepare the statement of changes in equity for the same period.
(c) Explain the term financial assets and state the FOUR classes of financial
assets identified in IAS 39 and how each is measured.
(d) What are biological assets? State any THREE conditions to be met before
a biological asset or agricultural produce can be recognised in the books of
accounts.

30.3 HELIOS GROUP


Helios Ltd is an agricultural production company.
Helios Ltd acquired 70% of the ordinary shares of Sol Ltd, an agricultural based
company for Rs. 600 million on 1 January 2015, when the reserves of Sol Ltd
were Rs. 300 million. At the date of acquisition the fair value of the non-
controlling interest in Sol Ltd was Rs. 160 million.
The fair values of the net assets of Sol Ltd are the same as their carrying values
with the exception of a plot of agricultural land. This land was carried by Sol Ltd
at its cost of Rs. 300 million. It was estimated at a fair value of Rs. 360 million.
Statement of financial position of Helios Ltd and Sol Ltd as at 31December
2016
Helios Ltd Sol Ltd
Rs.000 Rs.000
Assets
Non-current assets:
Property, plant and equipment 600,000 450,000
Investments 800,000 -

Current assets:
Inventories 160,000 150,000
Trade & other receivables 120,000 280,000
Cash and cash equivalent 20,000 50,000
Total assets 1,700,000 930,000
Equity and liabilities:
Ordinary share capital 160,000 120,000
Share premium 40,000 20,000
Reserves 590,000 500,000

Emile Woolf International 145 The Institute of Chartered Accountants of Pakistan


Advanced accounting and financial reporting

Helios Ltd Sol Ltd


Rs.000 Rs.000
Non-current liabilities:
Loan notes 600,000 170,000
Current liabilities
Trade & other payables 310,000 120,000
Total equity & liabilities 1,700,000 930,000

Additional information:
Immediately after acquisition, the following agricultural products were procured
and included in property, plant and equipment and inventories of Sol Ltd as at 31
December 2016:
(i) Included in property, plant and equipment of Sol Ltd are: Rs.000
Dairy livestock immature 40,000
Dairy livestock mature 50,000

(ii) Included in inventories of Sol Ltd is:


Cotton plants 20,000
Required
(a) Prepare the consolidated statement of financial position for Helios Ltd
group as at 31 December 2016 as expected for an agricultural business.
(b) State how to measure agricultural products harvested by an entity in line
with the requirements of IAS 41 on Agriculture.

30.4 FASHION BLUE ENTERPRISES


Fashion Blue Enterprises carries out business of readymade garments through
large shops in the major cities of Pakistan.
Its inventory ledger account balance at December 31, 2017 under the perpetual
inventory system was Rs. 73,410,000. The physical count revealed that the cost
of inventory on hand was Rs. 71,400,000 only. Its owner Mr. Kaizer expected a
small inventory shortfall due to damage and petty theft, but considered this
shortfall to be excessive.
On January 5, 2018, Kaizer carried out an investigation and discovered the
following:
(i) Goods costing Rs. 300,000 were invoiced to Ebrahim Limited for Rs.
425,000 on December 29, 2017 on FOB basis. The goods were actually
dispatched to the customer on January 2, 2018.
(ii) Included in the physical count were goods worth Rs. 200,000 which were
held on behalf of a third party.
(iii) Goods costing Rs. 410,000 purchased on credit from Mustafa & Co. were
received on December 28, 2017 and included in the physical count.
However, the purchase had not been recorded.
(iv) On December 23, 2017 goods costing Rs. 400,000 were purchased on
credit from Mubina Supplies, Faisalabad. The purchase was recorded on
December 27, 2017 i.e. when the goods were lifted by the transport
company appointed by Mr. Kaizar, from the warehouse of Mubina Supplies.
The goods arrived on January 3, 2018.

Emile Woolf International 146 The Institute of Chartered Accountants of Pakistan


Questions

(v) List of inventory at a shop situated in Sialkot had been under cast by Rs.
90,000.
(vi) On December 25, 2017 goods costing Rs. 310,000 were sold on credit to
Skims Industries for Rs. 500,000. The goods were shipped on December
28, 2017 and were received by the customer on January 2, 2018.
(vii) Goods costing Rs. 2,500,000 had been returned to Ali Garments on
December 30, 2017. A credit note was received from the supplier on
January 5, 2018 and entered in the books in January 2018. No payment
had been made for the goods prior to their return.
(viii) Goods sold to a customer Mr. Saleem were recorded in inventory ledger
account at the sale price of Rs. 780,000. The goods were sold at cost plus
30%.
Required
(a) Reconcile the ledger balance with the physical record to determine the
shortage (if any).
(b) Determine the value of inventory that should be recorded in the statements
of financial position.
(c) Prepare the adjusting entries that should be recorded in the books of
Fashion Blue Enterprises, in December 2017.

30.5 KHAN LIMITED


Khan Limited closes its accounts on June 30 each year. The company was
unable to take inventory of physical inventory until July 14, 2017 on which date
the physical inventory was valued at Rs. 185,000. The following details are
available in respect of the period July 1 to July 14, 2017:
(i) Payments against purchases amounted to Rs. 48,000 and included:
Rs. 5,000 in respect of goods received on June 28, 2017;
Rs. 6,000 in respect of goods received on July 18, 2017;
Rs. 2,000 in respect of goods received and returned to supplier on the
same date i.e. July 7, 2017.
(ii) Collection against sales amounted to Rs. 60,000 and included:
Rs. 1,500 in respect of goods which left the warehouse on June 29,
2017;
Rs. 2,800 in respect of goods which were not dispatched until July 15,
2017;
Rs. 760 in respect of goods invoiced and dispatched on July 10, 2017
but returned by the customers on July 12. These were included in the
inventory taken on July 14, 2017.
(iii) The rate of gross profit is 25% of selling price.
(iv) Goods costing Rs. 6,000 were purchased on June 28 but remained unpaid
till July 24, 2017.
(v) An invoice amounting to Rs. 10,000 was raised on July 9, 2017 but
remained uncollected till July 14, 2017.
(vi) An item costing Rs. 9,000 which had been purchased on June 25, 2017
was damaged on July 4, 2017. It can be repaired at a cost of Rs. 1,000 and

Emile Woolf International 147 The Institute of Chartered Accountants of Pakistan


Advanced accounting and financial reporting

sold for Rs. 7,000 and has been taken in inventory at its net realizable
value.
(vii) An inspection of the inventory count sheets prepared on July 14, 2017
showed that a page total of Rs. 5,000 had been carried to the summary as
Rs. 6,000. In addition, the total of another page had been undercast by Rs.
200.
(viii) Included in the physical count were goods costing Rs. 2,200 which were
held on behalf of a supplier.
Required
Determine the amount of inventory required to be disclosed in the financial
statements as at June 30, 2017.

30.6 AFRIDI
Afridi does not keep perpetual records of inventory. At the end of each quarter,
the value of inventory is determined through physical inventory. However, the
record of inventory taken on 31 March 2017 was destroyed in an accident and
Afridi has extracted the following information for the purpose of inventory
valuation:
(i) Invoices entered in the purchase day book, during the quarter, totalled Rs.
138,560 of which Rs. 28,000 related to the goods received on or before 31
December 2016. Invoices entered in April 2017 relating to goods received
in March 2017 amount to Rs. 37,000.
(ii) Sales invoiced to customers amounted to Rs. 151,073 of which Rs. 38,240
related to goods dispatched on or before 31 December 2016. Goods
dispatched to customers before 31 March 2017 but invoiced in April 2017
amounted to Rs. 25,421.
(iii) Credit notes of Rs. 12,800 had been issued to customers in respect of
goods returned during the period.
(iv) Purchases included Rs. 2,200 spent on acquisition of a ceiling fan for the
shop.
(v) A sale invoice of Rs. 5,760 had been recorded twice in the sales day book.
(vi) Goods having sale value of Rs. 2,100 were given by way of charity.
(vii) Afridi normally sells goods at a margin of 20% on cost. However, he had
allowed a special discount of 10% on goods costing Rs. 6,000 which were
sold on 15 February 2017.
(viii) On 31 December 2016, the inventory was valued at Rs. 140,525. However,
while reviewing these inventory sheets on 31 March 2017 the following
discrepancies were found:
(a) A page total of Rs. 15,059 had been carried to the summary as Rs.
25,059.
(b) 1,000 items costing Rs. 10 each had been valued at Rs. 0.50 each.
Required
Calculate the amount of inventory in hand as on 31 March 2017.

Emile Woolf International 148 The Institute of Chartered Accountants of Pakistan


Questions

CHAPTER 31: IFRS 1: FIRST TIME ADOPTION OF IFRS

31.1 IFRS 1
A company which has always prepared its Financial Statements to 31 December
each year, prepared its first IFRS Financial Statements for the year ended 31
December 2016. These statements show comparative figures for the year ended
31 December 2015.
Required
(a) Identify the first IFRS reporting period and state the date of transition to
IFRS.
(b) Present the procedures which must be followed in order to prepare the
financial statements for the year ended 31 December 2016.
(c) Identify the reconciliations which the company must include in its financial
statements for the year ended 31 December 2016.
(d) State the contents of a typical statement of changes in equity.

Emile Woolf International 149 The Institute of Chartered Accountants of Pakistan


Advanced accounting and financial reporting

CHAPTER 32 SPECIALISED FINANCIAL STATEMENTS

32.1 IFRS FOR SMES


International Financial Reporting Standards (IFRS) are unnecessary impositions
on developing countries with preponderance of small and medium entities (SME).
Some IFRS are unnecessarily demanding and might be difficult for these SMEs
to meet. Some of the information produced by adopting IFRS are not necessary
for users of SME Financial Statements.
Required
Discuss the following:
(a) The need to develop a set of IFRS specifically for SMEs.
(b) The need for the modification of existing IFRS to meet the needs of SMEs.
(c) How items not dealt with by an IFRS for SMEs should be treated.

32.2 Akmal General Insurance Limited


Akmal General Insurance Limited (AGIL) is engaged in general insurance
business. The following information is available for the year ended December 31,
2016:
(i) During the year, AGIL earned direct and facultative premiums of Rs.
5,586,382 thousand against which it incurred reinsurance expense
amounting to Rs. 2,076,499 thousand. Details of premium earned and
reinsurance expenses are as follows:

Fire & Marine, Motor Misc.


Property Aviation
Damage &Transport

Rupees in thousand

Premiums 1,905,027 883,942 2,495,120 302,293

Reinsurance expense 1,520,962 300,605 4,671 250,261

(ii) The outstanding balance of unearned premium reserve and prepaid


reinsurance premium ceded were as follows:

Fire & Marine,


Property Aviation &
Damage Transport Motor Misc.

Rupees in thousand

Balances as of
December 31, 2016

Unearned premium reserve 1,014,552 174,780 1,053,094 152,911

Prepaid reinsurance
premium ceded 741,934 93,702 311 122,866

Emile Woolf International 150 The Institute of Chartered Accountants of Pakistan


Questions

Fire & Marine,


Property Aviation &
Damage Transport Motor Misc.

Rupees in thousand

Balances as of
December 31, 2015

Unearned premium
reserve 844,425 159,844 1,191,933 133,424

Prepaid reinsurance
premium ceded 726,800 59,098 - 114,190

(iii) Premium received under the treaty arrangements (proportional) amounted


to Rs. 167,108 thousand. The outstanding balance of unearned premiums
reserve relating to treaty arrangement as of December 31, 2016 was Rs.
56,128 thousand (2015: Rs. 61,303 thousand).
Required
Prepare the statement of premiums for the year ended December 31, 2016.
Ignore the corresponding figures.

32.3 MAHFOOZ GENERAL INSURANCE LIMITED


Mahfooz General Insurance Limited (MGIL) is a listed company. The
information pertaining to the business underwritten inside Pakistan for the year
ended June 30, 2016 is as under:
Direct and
facultative Treaty
Fire & Marine
property aviation & Accident
damage transport Motor & health Proportional
Rupees in million
Claims:
Total claims paid 900 450 1,150 250 13
Outstanding -
Opening 600 400 900 300 10
Outstanding -
Closing 500 450 750 150 12
Reinsurance and
other recoveries:
Total received 600 300 850 160 -
Outstanding -
Opening 500 300 700 150 -
Outstanding -
Closing 350 400 550 80 -
Required
Prepare a statement of claims for the year ended June 30, 2016 in accordance
with the Insurance Ordinance, 2000. Ignore the comparative figures.

Emile Woolf International 151 The Institute of Chartered Accountants of Pakistan


Advanced accounting and financial reporting

32.4 Dee General Insurance Limited


Dee General Insurance Limited is a listed company. The following information
relates to the year ended 31 December 2016:
Direct and facultative Treaty
Marine
Fire and aviation
property and Miscellane Proportion
damage transport Motor ous al
(Rs. In million)
Commissions:
Paid / payable 321.41 126.87 215.00 90.94 0.30
Deferred:
opening 148.79 11.31 128.50 38.59 -
Deferred: closing 160.43 5.68 114.23 35.17 -
Receipts from
reinsurers 270.44 5.70 12.72 82.40 -
Net premium
earned 907.75 768.70 2,745.64 948.48 0.70
During the year, management expenses (other than commission) amounted to
Rs. 978 million. These expenses are allocated on the basis of net premium
earned.
Required
Prepare a statement of expenses for inclusion in the financial statements for the
year ended 31 December 2016. (Ignore comparative figures)

32.5 Bank Lateef Bank Limited


Lateef Bank Limited (LBL) is listed on Karachi and Lahore Stock Exchanges
and has 150 branches including 10 overseas branches. The LBLs lending to
financial institutions as of September 30, 2016 comprised of the following:
(i) Call money lending at year end amounted to Rs. 850 million (2015: Rs.
1,200 million).
The markup on these unsecured lendings ranged between 15% to 17%
(2015: 10% to 12%) and they matured on various dates, in October 2016.
(ii) Short term lending on account of repurchase agreement (reverse repo)
amounted to Rs. 2,100 million (2015: Rs. 2,850 million). These carried
markup ranging from 9.5% to 13.2% (2015: 8% to 10.5%) and matured
on various dates, in October 2016. These were secured against Market
Treasury Bills of Rs. 1,650 million (2015: Rs. 1,850 million) and Pakistan
Investment Bonds of Rs. 450 million (2015: Rs. 1,000 million). The market
value of these securities held as collateral, on September 30, 2016,
amounted to Rs. 2,250 million (2015: Rs. 2,930 million).
The above amounts include lendings in foreign currencies amounting to
Rs. 110 million (2015: Rs. 150 million).
Required
Prepare a note on lendings to financial institutions for inclusion in LBLs
financial statements for the year ended September 30, 2016 giving appropriate
disclosures in accordance with the guidelines issued by State Bank of Pakistan.

Emile Woolf International 152 The Institute of Chartered Accountants of Pakistan


Questions

32.6 SECURED BANK LIMITED


Secured Bank Limited (SBL) is listed on all the Stock Exchanges in Pakistan.
The cost of various types of Investments held by the bank as of December 31,
2016 are as follows:
2016 2015
Rupees in million
Market treasury bills 366 309
Pakistan investment bonds 69 61
Government of Pakistan bonds (USD/Euro) 26 30
Investments in associates 9 8
Fully paid-up ordinary shares listed 6 5
Fully paid-up ordinary shares unlisted 2 3
Corporate debt instruments listed 19 30
Corporate debt instruments unlisted 260 210
Investments of mutual funds 32 28
Overseas government securities 60 52
Other investments 19 29
Provision for diminution / impairment in the value of investments as at
January 1, 2015 amounted to Rs. 28 million. Other information relevant to the
provision is as under:
Impairment (reversal) / loss for the year (6) 2
Charge for the year 17 12
Amounts written off during the year 5 3
Required
Prepare a note on investments by segments for inclusion in SBLs financial
statements for the year ended December 31, 2016 giving appropriate
disclosures in accordance with the guidelines issued by the State Bank of
Pakistan.

32.7 AL-AMIN BANK LIMITED


Al-Amin Bank Limited is listed on all the stock exchanges in Pakistan. At
year end, the total advances amounted to Rs 75,000 million which include
non-performing advances of Rs. 5,000 million. The break-up of the non-
performing advances and the provisions there-against is as under:
Other
assets
especially Sub-
mentioned Standard Doubtful Loss Total
Rs. in million
Advances 100 660 840 3,400 5,000
Provisions required and held 5 120 530 3,345 4,000
The sub-standard category includes advances of Rs. 260 million pertaining to
overseas operations of the bank. The required provision of Rs. 50 million has
been made against such advances.

Emile Woolf International 153 The Institute of Chartered Accountants of Pakistan


Advanced accounting and financial reporting

During the year the movement in the specific provision was as under:
Rs. in
million
Opening balance 3,320
Charge for the year 802
Reversals (90)
Amounts written off (50)
Exchange rate adjustment 18
Total 4,000
In addition to the above specific provisions, it is the banks policy to make
additional general provision based on the judgment of the bank. The opening
balance for general provision was Rs. 65 million. During the year, the bank made
provisions of Rs. 25 million and Rs. 15 million against consumer and agriculture
advances respectively.
Required
Prepare relevant notes on non-performing advances and provisions for
inclusion in the financial statements of Al-Amin Bank Limited giving appropriate
disclosure in accordance with the guidelines issued by the State Bank of
Pakistan.

32.8 BLUE-CHIP ASSET MANAGEMENT LIMITED


Blue-chip Asset Management Limited is in the process of finalizing the financial
statements of one of its open ended mutual funds. The following information is
available from the funds records;
Rs. in
000
Net assets - opening balance 350,050
Net income for the year 65,325
765,900 units issued during the year against 85,015
717,480 units redeemed during the year against 77,488
The par value of each unit is Rs. 100.
Required
Prepare the statement of movement in unit holders Fund for the year ended June
30, 2016.

32.9 A-ONE ASSET MANAGEMENT FUND LIMITED


Following information has been extracted from the records of A-One Asset
Management Fund Limited for the year ended March 31, 2016.

Rs. in
million
Net assets at the beginning of the year (900 million units) 27,000
100 million units issued during the year 3,500
95 million units redeemed during the year 3,277

Emile Woolf International 154 The Institute of Chartered Accountants of Pakistan


Questions

Investments classified as available for sale


- Fair value at year end 1,800
- Carrying value at year end 1,200
- Net unrealized appreciation in fair value of investments at the
beginning of the year 480

Investments classified as at fair value through profit or loss - held


for trading
- Fair value at year end 2,500
- Carrying value at year end 2,200

Element of income and capital gains included in prices of units


issued/redeemed and transferred to income statement 173
- Capital gains 400
- Other net income for the year 3,000
Final distribution for the year ended March 31, 2016 of Rs. 5.00 per unit (2015:
Rs. 4.00 per unit) was announced on April 16, 2016.
Required
Prepare a statement of movement in unit holders' fund for the year ended March
31, 2016.

32.10 IAS 26
IAS 26: Accounting and Reporting by Retirement Benefit Plans and IAS 19:
Employee Benefits deal with employee benefits but there are differences between
the two standards.
(a) Highlight the main differences between IAS 26 and IAS 19.
(b) What is a Defined Benefit Plan?
(c) What is a Defined Contribution Plan?
(d) Explain the meaning of the actuarial present value of promised retirement
benefit.

32.11 SOGO LIMITED


SOGO Limited operates an approved funded gratuity scheme for all its
employees.
Benefits under the scheme become vested after 5 years of service. No benefit is
payable to an employee if he leaves before 5 years of service. A total of 752
employees were eligible for the benefits under the fund as of December 31, 2016.
Following is the trial balance of the Fund as of June 30, 2016:
Debit Credit
Amounts in Rupees
Cash at bank - current account 17,930,120
Receivable from SOGO Limited 1,147,150
Defence Savings Certificate 102,133,664
Term Finance Certificates 11,832,089
Term Deposits 6,414,058
Investment SUN Limited 17,594,893

Emile Woolf International 155 The Institute of Chartered Accountants of Pakistan


Advanced accounting and financial reporting

Debit Credit
Amounts in Rupees
Investment PEACE Company Limited 587,169
Investment - NIT Units 16,911,510
Due to outgoing members 4,301,017
Accrued expenses 3,822
Withholding tax payable 61,251
Members Fund 142,472,122
Profit on investments 23,389,251
Dividend income 2,696,399
Contribution for the year 10,623,106
Transferred / paid to outgoing members 12,432,973
Bank charges 3,342
Audit fee 10,000
Liabilities no more payable 3,450,000
186,996,968 186,996,968
Following are the details of investments and income thereon:
Balance as During the year 2016
at Profit / Profit /
Principal
July 01, Addition interest interest
realized
2015 accrued realized
Government
Securities
Defence Savings 21,376,80 (1,600,00 (5,456,00
-
Certificate 87,812,855 9 0) 0)
Unlisted Securities
and deposits
Term Finance (12,873,0 (1,893,72
19,943,656 5,000,000 1,655,223
Certificates 68) 2)
(5,300,00
Term Deposits 11,584,631 - 357,219 (227,792)
0)
Listed Securities
SUN Limited 8,220,957 9,373,936 - - -
PEACE Limited 587,169 - - - -
NIT Units 16,911,510 - - - -
The following gains/(losses) on restatement of investments at their fair values,
have not been accounted for:
Rupees
SUN Limited (784,518)
PEACE Limited 317,728
NIT Units 4,026,551
Required
Prepare the following in accordance with the requirements of International
Accounting Standards:
(a) Statement of net assets available for benefits along with the note on
investments.
(b) Statement of changes in net assets available for benefits.

Emile Woolf International 156 The Institute of Chartered Accountants of Pakistan


Questions

CHAPTER 36: ETHICAL ISSUES IN FINANCIAL REPORTING

36.1 ETHICAL ISSUES


Waheed is a chartered accountant, recently employed by AA Limited as deputy to
the finance director, Arif (also a chartered accountant). AA Limited is listed on the
Lahore stock exchange.
On Waheeds first day on the job he met with Arif who said Look, keep it to
yourself but Im having a second interview next week for a new job. The first thing
that I need you to do is to review the financial statements before the auditors
arrive. I qualified a few years ago and am not up to date on all of the little
technicalities in IFRS. You should know these better than me and youll know
more about what the auditors might focus on. We must do our best to present the
financial statements in the most favourable light as the bonus paid to employees
(including me) depends on profit being more than 10% higher than last years
and remember that you qualify for this too. Keep this in mind when you carry out
the review as we do not really want to find anything. Do well at this and I might
put in a good word for you when I leave as Im sure youll be a great replacement
for me.
Required
Explain the ethical issues inherent in the above conversation and what Waheed
should do about them.

36.2 SINDH INDUSTRIES LTD


Jafar has recently been appointed as financial controller to Sindh Industries Ltd.
Until a month ago, Sindh Industries had a finance director, who resigned
suddenly, due to ill health. Since Jafar joined the company, he has learned that
his resignation was related to stress caused by a series of disagreements with
the managing director about the performance of the business. The directors have
not yet appointed a replacement.
It is now March 2017 and you have been asked to finalise the financial
statements for the year ended 31 December 2016. The draft statement of profit or
loss extract and statement of financial position are shown below:
Draft statement of profit or loss for the year ended 31 December 2016
Rs.000
Profit before tax 2,500
Draft statement of financial position at 31 December 2016
Rs.000
Property, plant and equipment 12,000
Current assets 3,500
Total assets 15,500

Share capital 2,000


Retained earnings 6,000
Equity 8,000
Non-current liabilities 5,000
Current liabilities 2,500
Total equity and liabilities 15,500

Emile Woolf International 157 The Institute of Chartered Accountants of Pakistan


Advanced accounting and financial reporting

During the year ended 31 December 2016 Sindh Industries entered into the
following transactions.
(1) Just before the year end Sindh Industries signed a contract to deliver
consultancy services for a period of 2 years at a fee of Rs. 500,000 per
annum. The full amount of this fee has been paid in advance and is non-
refundable.
(2) Sindh Industries has constructed a new factory. The construction has been
financed from the pool of existing borrowings. Land at a cost of Rs. 1.8
million was acquired on 1 February 2016 and construction began on 1 June
2016. Construction was completed on 30 September 2016 at an additional
cost of Rs. 2.7 million. Although the factory was usable from that date, full
production did not commence until 1 December 2016. Throughout the year
the companys average borrowings were as follows:
Annual
interest
Amount rate
Rs. %
Bank overdraft 1,000,000 9.75
Bank loan 1,750,000 10
Debenture 2,500,000 8
An amount of Rs. 450,000 has been included in property, plant and
equipment in respect of borrowing costs relating to the construction of the
factory. The useful life of the factory has been estimated at 20 years. No
depreciation has been charged for the year. The reason for this is that the
factory has only been in use for one month and that the depreciation charge
would be immaterial.
(3) A blast furnace with a carrying amount at 1 January 2016 of Rs. 3.5 million
has been depreciated in the draft financial statements on the basis of a
remaining life of 20 years. In December 2016 the directors carried out a
review of the useful lives of various significant items of plant and
machinery, including the blast furnace and came to the conclusion that the
useful life of the furnace was 20 years at 31 December 2016. The
reasoning behind this judgement was that the lining of the furnace had
been replaced in the last week of December 20X6 at a cost of Rs. 1.4
million. Provided that the lining is replaced every five years, the life of the
furnace can be extended accordingly. You have found a report,
commissioned by the previous finance director and prepared by a firm of
asset valuation specialists, which assesses the remaining useful life of the
main structure of the furnace at 1 January 2016 at 15 years and the lining
of the furnace at 5 years. You have also found evidence that the managing
director has seen this report.
Jafar has had a conversation with the managing director who told him, We
need to make the figures look as good as possible so I hope youre not
going to start being difficult. The consultancy fee is non-refundable so
theres no reason why we cant include it in full. I think we should look at our
depreciation policies. Were writing off our assets over far too short a
period. As you know, were planning to go for a stock market listing in the
near future and being prudent and playing safe wont help us do that. It
wont help your future with this company either.

Emile Woolf International 158 The Institute of Chartered Accountants of Pakistan


Questions

Required
(a) Explain the required IFRS accounting treatment of these issues, preparing
relevant calculations where appropriate.
(b) Prepare a revised draft of the statement of profit or loss extract for the year
ended 31 December 2016 and the statement of financial position at that
date.
(c) Discuss the ethical issues arising from your review of the draft financial
statements and the actions that you should consider.

36.3 SOHAIB AND OMAR


Sohaib is student member of ICAP who is will sit his finals in the next few months.
One of his friends, Omar, has asked him for advice as he is planning to start up a
business that provides plant care services to firms occupying upmarket office
accommodation.
Omar wants to assess the profitability and financial position of two similar
businesses in order to prepare his business plan.
These two businesses are called, Eden Ltd (Eden) and Gethsemene Ltd
(Gethsemene), both unlisted companies. Omar has asked Sohaib to write a
report analysing the two companies' most recent financial information.
Omar is short of money and because of their friendship Sohaib has agreed to
undertake the work for no fee. The Eden financial statements have been filed with
the regulatory authorities. However, the Gethsemene statements have not yet
been filed, and the documents provided to Sohaib by Omar are stamped 'Draft:
strictly private and confidential'. Omar recently completed a work placement at
Gethsemene. When Sohaib asks him for further information, Omar replied I
obtained these financial statements quite legitimately, and in any case, I am sure
they will become filed very soon. Anyway you are only a student member and I
am not paying you so what is the fuss'.
Required
Advise Sohaib about the ethical issues arising in respect of his work for Omar,
referring, where appropriate, to the ICAP's Code of Ethics, and explaining any
action he should take.

36.4 ABBAS AND BASHIR


Nourish Limited is an unlisted company manufactures and transports food
products to its customers (supermarkets and stores) throughout Pakistan.
In June 2018 Nourish Limited was awarded a five-year contract to supply
BigShop Limited, one of the Pakistan's leading supermarket groups. The
successful bid for the new contract has involved Nourish Limited in substantial
amounts of capital investment, funded principally by bank borrowing.
Insight Ltd is a firm of multi-disciplinary consultants. Nourish Limited has engaged
Insight Ltd to provide advice on business planning and logistics. Bashir, a
logistics specialist employed by Insight Ltd, has been assigned as project leader.
Bashir has asked Abbas, a member of his team, to produce a preliminary report
on Nourish Limited's most recent financial statements, for the year ended 31
December 2016. Abbas is a recently qualified chartered accountant.

Emile Woolf International 159 The Institute of Chartered Accountants of Pakistan


Advanced accounting and financial reporting

BigShop Limited also uses the consulting services of Insight Ltd. Both Bashir and
Abbas have worked on consulting projects commissioned by BigShop Limited,
and are likely to do so again in the future.
Abbas is the only chartered accountant on his team (in fact, Insight Ltd employs
few other chartered accountants) and is concerned that there may be a potential
conflict of interest between his work on Nourish Limited and BigShop Limited
projects. He is not sure whether such conflicts are covered by the ICAP's ethical
guidance.
Required
Advise Abbas about his concerns over the potential conflict of interest arising
from his work for Nourish Limited and BigShop Limited, referring where
appropriate to the ICAP's Code of Ethics, and explain any action he should take.

Emile Woolf International 160 The Institute of Chartered Accountants of Pakistan


Certified Finance and Accounting Professional
Advanced accounting and financial reporting

SECTION
B
Answers
CHAPTER 1 REGULATORY FRAMEWORK

1.1 GENERAL PURPOSE FINANCIAL STATEMENTS


(a) A general purpose financial statement is a statement that is intended to
meet the need of users who are not in a position to demand information
that are tailored to their needs. Such information is useful to existing and
potential investors.
(b) Reasons for the prevalence of IFRS in many countries
(i) Production of high quality financial statements
It involves the preparation of financial statements that have qualitative
features, that is, faithful representation, error free, neutrality,
understandability etc.
(ii) Preparation of user-friendly financial statements
Production of financial statements that contains necessary
information that will assist users of financial statements to make
crucial economic decisions.
(iii) Uniformity of financial statements prepared
Financial accounting is a language of business. This language must
not be different across countries of the world for it to assist
international investors.
(iv) Access to international finances
With a General Purpose Financial Statements, it would be easier for
multi-national entities to have more access to international funds.
(v) Enhancement of major economic decisions
High quality financial statements will assist users of the statements to
make informed and important financial decisions.
(vi) Comparability of financial statements
General purpose financial statements enhance comparability of
financial information among similar industries.

Emile Woolf International 161 The Institute of Chartered Accountants of Pakistan


Advanced accounting and financial reporting

(vii) Globalisation and integration


With IFRS, Chartered Accountants become more mobile, since the
standards are the same across the countries.
(viii) Job Creation
Jobs are created across countries and a Chartered Accountant in
Pakistan can practice in other countries with convenience.
(ix) Others
Saving of time and money
Promotion of Regional trade
Easier accounting and auditing practices
(c) Arguments for and against financial reporting standards
Arguments for:
(i) They guide preparers and users of financial statements
(ii) Their use enhances objectivity and comparability of financial
statement which would in turn engender reliability
(iii) Their use helps to curtail or significantly narrow the divergence in the
principles adopted by preparers of financial statements.
(iv) Standards improve the uniformity of treatment of transactions in the
financial statements among companies thereby increasing the
credibility and comparability of financial statement.
(v) Standards compel organisations to disclose information which they
may not want to disclose had the standards not been in existence.
(vi) Standards reduce the number of choices in the methods used to
prepare financial statement thereby reducing the risk of creative
accounting.
(vii) Foreign companies which are targets for takeovers or mergers can be
more easily evaluated.
Arguments against:
(i) The cost of setting up and maintaining a standard-setting apparatus is
quite significant and not all countries can afford it.
(ii) The standards cannot address all issues or transactions. There are
some which are unique and so rare/unusual that global standards are
not and cannot be available for them.
(iii) Low level of details or explanations.

Emile Woolf International 162 The Institute of Chartered Accountants of Pakistan


Answers

CHAPTER 2 ACCOUNTING AND REPORTING CONCEPTS

2.1 DEFINITIONS
An asset is:
a resource controlled by the entity
as a result of past events
from which economic benefits are expected to flow to the entity.
A liability is:
a present obligation of the entity
arising from past events
the settlement of which is expected to result in an outflow of economic
benefits.
Income includes both revenue and gains.
Revenue is income arising in the course of the ordinary activities of the entity
such as sales revenue and income from investments.
Gains include, for example, the gain on disposal of a non-current asset. They
might arise in the normal course of business activities. They might be realised or
unrealised. Unrealised gains occur whenever an asset is revalued upwards, such
as the upward revaluation of marketable securities.
Expenses include:
expenses arising in the normal course of activities, such as the cost of
sales and other operating costs, including depreciation of non-current
assets.
losses, including, for example, the loss on disposal of a non-current asset,
and losses arising from damage due to fire or flooding.
Appraisal of statement of financial position as all that is required
The statement of financial position does show the position of a business at a
point in time (like a snapshot), but by itself is insufficient to give a comprehensive
view of performance and/or adaptability.
The IASB Framework states that information on financial performance is provided
by the statement of profit or loss and other comprehensive income. This is
because the statement of financial position fails to give any account of
transactions leading up to the statement of position.
It is the statement of profit or loss and other comprehensive income and the
SOCIE that show the performance of a business in a given period and reconcile
the opening and closing statements of financial position.
Information on financial adaptability is given primarily by the statement of cash
flows. This is because financial adaptability is the ability to take effective action to
alter the amount and timing of cash flows. Some information comes from the
statement of financial position (e.g. the note about future finance lease
commitments) but the statement of financial position is by no means all that is
required.

Emile Woolf International 163 The Institute of Chartered Accountants of Pakistan


Advanced accounting and financial reporting

Statement of profit or loss is a superfluous statement


Although income (revenue and gains) and expenses (and losses) would be
reflected in an increase/decrease in the assets and liabilities in the statement of
financial position, the volume and type of income generated would give a better
indication of company performance. It is the statement of profit or loss and other
comprehensive income that provides such detailed information; without it the
performance of the business cannot be properly evaluated.

2.2 CONCEPTUAL FRAMEWORK


(a) (i) Conceptual framework is a constitution, a coherent system of
interrelated objectives and fundamentals that can lead to consistent
standards and prescribes the nature, functions and limits of financial
accounting and financial statements.
(ii) It enables certain critical issues to be addressed.
(iii) It also facilitates the development of accounting standards and
Generally Accepted Accounting Practice (GAAP.),in accordance with
the principles and underlying assumptions of the concepts.
(iv) It also promotes consistency in the application of accounting
principles and policies. Without conceptual framework there will not
be a common definition of the elements of financial statements
(asset, liabilities, income and expenses). For example, in line with
the framework for preparation and presentation of financial
statements gives a precise definition of assets and liabilities and no
expenditure on these could be recognized unless such expenditure
meets the definition of the framework.
(b) Assets
Assets are resources controlled by an entity as a result of past events and
from which future economic benefits are expected to flow to the entity
Liabilities
Liabilities are present obligations of the entity arising from past events, the
settlement of which, are expected to result in an outflow from the entity of
resources embodying economic benefit.

(c) (i) Substance over form


Substance over form refers to the impact which a transaction or event
has on the reporting entity, and this determines the accounting
treatment. Substance over form indicates that transactions and
events should be reported in the financial statement in accordance
with their economic reality or their commercial intent. In applying the
substance over form, IFRS framework requires the entity to adopt the
statement of financial position liability method.
This is done by looking at the impact of the event or transaction on
assets or liabilities on the reporting date.
(ii) Materiality
An item is considered material if its inclusion, omission or
misstatement will have a fundamental effect on the financial
statements as a whole and affect the economic decision of the user.
The requirement of the framework, is that items which are material in

Emile Woolf International 164 The Institute of Chartered Accountants of Pakistan


Answers

nature to an entity should be accorded separate recognition,


presentation and disclosure, while those that are immaterial (small
and separate unimportant) should be aggregated or added up.
(iii) Comparability
This implies that the financial statements of a given year should have
relative figure of the past period or periods. This helps to evaluate
the performance of the entity and trend analysis over time. It also
helps to assess the comparability on consistent application of
accounting policies over time.
(iv) Going Concern
The going concern concept assumes that the entity will be in
operational existence in the foreseeable future time, and that it has no
intention to scale down its operation significantly. Except otherwise
stated, financial statements are usually prepared on the going
concern concept.
If, however, the entity can no longer continue as a going concern,
then, subsequent financial statement should be prepared on a break-
up basis that is, the assets should be stated at fair value less cost to
point of sales (net realizable value) rather than at cost less
accumulated depreciation.
(d) (i) Lenders
The lenders are concerned with the ability of the company to pay the
finance cost on the borrowed fund and pay the loan when due. They
are also interested in the availability of assets to secure loans.
(ii) Suppliers
The suppliers are interested in information that indicates that their
debts can be paid by the entity and that the entity will continue as a
going concern in order to ensure continued patronage.
(iii) Customers
The customers are interested in information relating to the entitys
continued existence, especially for those that depend on the entity to
meet their daily needs.
(iv) Employees
The employees are concerned with their job security and the
companys ability to be profitable, in order to guarantee the payment
of their salaries in the future.
(v) Government and their agencies
Government and their agencies are interested in information relating
to taxes, regulations, resource allocation and evaluation of
government policies on businesses.

Emile Woolf International 165 The Institute of Chartered Accountants of Pakistan


Advanced accounting and financial reporting

2.3 CARRIE
(a) Physical (b) Financial capital maintenance
capital
maintenance
(i) Historical (ii) Constant
cost purchasing
accounting power
Profit for the year accounting
Rs. Rs. Rs.
Sales 1,400 1,400 1,400
Cost of sales (1,000) (1,000) (1,000)
Inflation adjustment
- Specific
(1,100 1,000) (100) - -
- General
(1,000 u 7%) - - (70)

Profit 300 400 330

Balance sheet as at
31 December Year 1
Cash at bank 1,400 1,400 1,400

Share capital
(1,000 + 100)
(1,000 + 70) 1,100* 1,000 1,070*
Reserves 300 400 330

1,400 1,400 1,400

Tutorial note
Share capital at the year end is restated under the physical capital maintenance
concept for an increase in specific price changes and under CPP accounting for
general price changes. This is the other side of the entry to the inflation
adjustments in the statement of profit or loss

Emile Woolf International 166 The Institute of Chartered Accountants of Pakistan


Answers

CHAPTER 3 PRESENTATION OF FINANCIAL STATEMENTS

3.1 CLIFTON PHARMA LIMITED


(a) Clifton Pharma Limited
Statement of profit or loss for the year ended 30 September 2016
Rs.in 000
Revenue 338,300
Cost of sales: see working (1) (180,000)
Gross profit 158,300
Operating expenses: see working (2) (36,600)
Investment income 2,000
Finance costs: Loan notes see working (3) 3,000
Finance lease see working (2) 1,700
(4,700)
Profit before tax 119,000
Income tax expense: see working (4) (21,000)
Profit for the period 98,000

(b) Clifton Pharma Limited


Statement of financial position as at 30 September 2016
Non-current assets Rs.in 000
Property, plant and equipment: see working (5) 358,000
Investments 92,400
450,400
Current assets
Inventory 23,700
Trade receivables 76,400
Bank 12,100
112,200
Total assets 562,600
Equity and liabilities
Capital and reserves
Share capital 280,000
Share premium 20,000
Retained earnings: see working (6) 117,300
417,300
Revaluation surplus 20,000
Non-current liabilities
3% loan notes: see working (3) 51,500
Deferred tax: see working (4) 23,000
Finance lease obligation: see working (2) 11,700
86,200
Current liabilities
Trade payables 14,100
Accrued lease finance costs: see working (2) 1,700
Finance lease obligation: see working (2) 5,300
Income tax payable 18,000
39,100
Total equity and liabilities 562,600

Emile Woolf International 167 The Institute of Chartered Accountants of Pakistan


Advanced accounting and financial reporting

Workings
(1) Cost of sales Rs.in 000
As given in the trial balance 134,000
Depreciation of plant and equipment: 20% u (197,000 30,000
47,000)
Depreciation of leased vehicles: 24,000/4 years 6,000
Amortisation of leasehold property: 250,000/25 years 10,000
180,000

(2) Vehicle rentals and finance lease. Operating expenses Rs.in 000
Rental costs given in the trial balance 8,600
Relating to finance lease (7,000)
Balance: relating to operating lease operating expense 1,600
Other operating expenses (trial balance in question) 35,000
Total operating expenses 36,600

Finance lease Rs.in 000


Fair value of leased assets 24,000
Less: First rental payment, paid in advance (7,000)
1 October 2015
Remaining obligation, 1 October 2015 17,000
Interest at 10% to 30 September 2016 (current liability) 1,700
Lease payment due 1 October 2016 7,000
Capital repayment due (= balance, current liability) (5,300)
Remaining lease obligation = non-current liability 11,700

(3) Loan notes


The effective interest rate is 6%. Actual interest paid was Rs. 1,500,000 (in
trial balance); therefore the balancing Rs. 1,500,000 should be added to the
loan notes obligation, to make the total loan notes liability Rs. 50 million +
Rs. 1,500,000 = Rs. 51.5 million.

(4) Taxation
Rs.in 000
Deferred tax liability b/f 20,000
Deferred tax: debit in the statement of profit or loss 3,000
Deferred tax liability c/f (92,000 u 25%) 23,000

Tax expense Rs.000


Income tax on profits for the year 18,000
Deferred tax movement 3,000
Tax charge in the statement of profit or loss 21,000

Emile Woolf International 168 The Institute of Chartered Accountants of Pakistan


Answers

(5) Non-current assets and depreciation


Leasehold property Rs.in 000
Carrying value in the trial balance (250,000 40,000) 210,000
Amortisation charge for the year to 30 September 2016 (10,000)
200,000
Re-valued amount 220,000
Transfer to revaluation reserve 20,000
The annual depreciation charges for plant and equipment and the leased
vehicles are shown in workings (1)
Cost or Accumulated Carrying
valuation depreciation amount
Rs.in 000 Rs.in 000 Rs.in 000
Leasehold property 220,000 0 220,000
Plant and equipment 197,000 77,000 120,000
(non-leased)
Leased vehicles 24,000 6,000 18,000
441,000 83,000 358,000

(6) Retained profits Rs.in 000


At 1 October 2015 (trial balance) 19,300
Profit for the year 98,000
Retained profits at 30 September 2016 117,300

3.2 BSZ LIMITED


BSZ Limited
Statement of financial position as at June 30, 2016
Note Rs.in
million
ASSETS
Fixed Assets
Property, plant & equipment 1 576
Intangible assets 2 8
584
Long term advances considered good 4

Current assets
Stocks in trade 90
Accounts receivable 3 57
Advances, deposits, prepayments and other
receivables 4 45
Cash at banks 5 29
221
809

Emile Woolf International 169 The Institute of Chartered Accountants of Pakistan


Advanced accounting and financial reporting

Rs.in
million
EQUITY AND LIABILITIES
Share capital and reserves
Authorized share capital
50,000,000 shares of Rs. 10 each 500

Issued, subscribed and paid up capital


40,000,000 shares of Rs. 10 each 400
Unappropriated profit 65
465

Surplus on revaluation of fixed assets 120

Non-current liabilities
Deferred taxation 40

Current liabilities
Short term loan 85
Account and other payables 6 82
Provision for taxation 17
184
809

Rs.in
Notes million
1. Property, plant and equipment
Operating assets 556
Capital work in progress building 20
576

1.1 Operating assets Rs.in million


Cost/revalued Freehold
amount land Building Machines Fixtures Total
As of July 01 2010 375.0 130.0 100.0 19.0 624.0
Additions - - - 8.0 8.0
Disposals - - (15.0) - (15.0)
As at June 30
85.0 27.0 617.0
2016 375.0 130.0

Accumulated
depreciation
As of July 01 2010 - 19.5 22.5 5.9 47.9
For the year - 6.5 18.1
(105 85) + 10% 9.5
15 8/12)
(105 19) + 10% 2.1
8 3/12)
Disposals - - (5.0) - (5.0)
As at June 30
27.0 8.0 61.0
2016 - 26.0
Carrying amount 375.0 104.0 58.0 19.0 556.0
Depreciation rate - 5% 10% 10%

Emile Woolf International 170 The Institute of Chartered Accountants of Pakistan


Answers

1.2 Revaluation
During the year 2012, the first revaluation of freehold land was carried out. The
valuation was carried out under market value basis by an independent valuer, Mr.
Dee, Chartered Civil Engineer of M/s SSS Consultants (Pvt.) Ltd., Islamabad. It
resulted in a surplus of Rs. 120 million over book values which was credited to
surplus on revaluation of fixed assets. Had there been no revaluation, the value of
freehold land would be Rs. 255 million.

1.3 Disposal of machine


Rs.in
million
Proceeds 13.0
Cost 15.0
Accumulated depreciation (5.0)
Carrying amount (10.0)

Profit on disposal 3.0

Note 2016
Rs.in
million
2. Intangible Assets
Cost of computer software/license 10.0
Accumulated Amortization as of July 1, 2010 1.0
Amortization for the year 1.0
Accumulated Amortization as of June 30, 2016 2.0

Carrying value as at June 30, 2016 8.0

Amortization rate 10%

3 - Accounts Receivable
Considered good
- Secured 30
- Unsecured 27
57
Considered doubtful 3
60
Less: Provision for bad debts 3.1 3
57

3.1 - Provision for bad debts


Balance as at July 1, 2010 3.4
Provision made during the year 1.0
Amount written off during the year (1.4)
Balance as at June 30, 2016 3.0

Emile Woolf International 171 The Institute of Chartered Accountants of Pakistan


Advanced accounting and financial reporting

Note 2016
Rs.in
million
4 - Advances, Deposits, Prepayments and Other Receivables
Advances
- suppliers - considered good 12
- staffs 6
18
Deposits 11
Prepayments 4
Sales tax receivable 12

45
5 - Cash at banks
Cash at banks - current accounts 7
saving accounts 5.1 22

29
5.1: It carries interest / mark-up ranging from 3% to 7% per annum.
6 - Accounts and other payables
Accounts payable 75
Accrued liabilities 7

82

3.3 YASIR INDUSTRIES LIMITED


(a) Yasir Industries Limited:
Statement of financial position as at June 30, 2016
Rs.in
million
Assets
Non-current assets
Property, plant and equipment (W2) 351.00
Intangible assets (20 12) 8.00
359.00
Current assets
Inventories (W6) 64.50
Trade receivables (W5) 39.00
103.50
462.50
Equity and Liabilities
Equity
Issued, subscribed and paid up capital 120.00
Retained earnings (W4) 87.10
207.10

Revaluation surplus 41.25

Emile Woolf International 172 The Institute of Chartered Accountants of Pakistan


Answers

(a) Yasir Industries Limited:


Statement of financial position as at June 30, 2016
Rs.in
million
Non-current liabilities
Redeemable preference shares 40.00
Debentures 80.00
Deferred taxation (W 10) 9.00
129.00
Current liabilities
Trade payables 30.40
Accrued expenses (W3) 25.00
Taxation 16.50
Bank overdraft 13.25
85.15
Total equity and liabilities 462.50

(b) Yasir Industries Limited


Statement of profit or loss and other comprehensive income for the
year ended June 30, 2016
Rs.in
million
Sales revenue (W5) 445.40
Cost of sales (W7) (250.72)
Gross profit 194.68
Distribution costs (W8) (20.05)
Administrative expenses (W8) (40.38)
Financial charges (W9) (9.10)
125.15
Loss due to fraud (30.00)
Profit before tax 95.15
Income tax expense (W10) (19.50)
Profit for the year 75.65
Workings
(W1) Leasehold property
Annual depreciation before the revaluation (230 40 years) = Rs. 5.75
million per annum.
Depreciation this year has been charged incorrectly on cost (whereas it
should have been on the revalued amount).
This years charge must be added back
Dr Cr
Accumulated depreciation 5.75
Cost of sales (50%) 2.88
Administrative expenses (30%) 1.72
Distribution costs (20%) 1.15

Rs.in million
Carrying amount at the 30 June (as per trial
balance)(230.00 40.25) 189.75
Add back depreciation incorrectly charged (see
above) 5.75
Carrying amount of property at the start of the year 195.5

Emile Woolf International 173 The Institute of Chartered Accountants of Pakistan


Advanced accounting and financial reporting

Revaluation surplus Rs.in million


Revalued amount of leasehold property 238.00
Less: WDV of leasehold property at revaluation 195.50
Revaluation surplus arising in the year 42.50
Transfer to retained earnings in respect of incremental
depreciation (Rs. 7 million Rs. 5.75 million) (1.25)
41.25

Depreciation of revalued property


Number of years depreciation by the year end: (40.25 5.75) = 7 years.
Therefore, remaining useful life as at the year-end = 33 years
Revaluation was at the start of the year
Remaining useful life at the start of the year = 34 years
Depreciation charge based on the revalued amount (238/34 years) = Rs. 7 million

Dr Cr
Cost of sales (50%) 3.5
Administrative expenses (30%) 2.1
Distribution costs (20%) 1.4
Accumulated depreciation 7.00

(W2) Property, plant and equipment


Rs.in million
Leasehold property (Rs. 238m 7) 231
Machines (Rs. 168.6 Rs. 48.6m) 120
351
(W3) Accrued expenses
Rs.in million
As per trial balance 16.20
Accrued interest on debentures (Rs. 80m 12% 6/12) 4.80
Dividend on preference shares (Rs. 40m 10%) 4.00
25.00
(W4) Retained earnings
Rs.in million
Balance as per trial balance 10.20
Profit for the year 75.65
Transfer from revaluation surplus 1.25
87.10
(W5) Sales and receivables
Sales. Rec.
Rs.in million
Given in the trial balance 472.40 66.00
Deduct revenue incorrectly recognised (sale or return) (27.00) (27.00)
Adjusted sales and receivables 451.40 39.00
(W6) Closing inventory
Rs.in million
Given in the question 42.00
Add back inventory held by customer on sale or return
(100/120 u 27) 22.50
Adjusted closing inventory 64.50

Emile Woolf International 174 The Institute of Chartered Accountants of Pakistan


Answers

(W7) Cost of sales


Rs.in million
Opening inventory as of July 1, 2015 38.90
Purchases 175.70
Direct labour 61.00
Manufacturing overheads excluding incremental depreciation 39.00
Less: Closing inventory (adjusted (W6)) (64.50)
Deduct depreciation incorrectly charged on cost (2.88)
Add depreciation charged on revalued amount 3.50
Cost of sales 250.72
(W8) Administrative expenses and distribution costs
Admin. Dist/
Rs.in million
Given in the trial balance 40.00 19.80
Deduct depreciation incorrectly charged on cost (1.72) (1.15)
Add depreciation charged on revalued amount 2.10 1.40
Adjusted expenses 40.38 20.05
(W9) Financial charges
Rs.in million
Balance as per trial balance 0.30
Accrued interest on debentures (Rs. 80m 12% 6/12) 4.80
Preference dividend for the year (Rs. 40m 10%) 4.00
9.10
(W10) Taxation
Deferred taxation Rs.in million
Balance b/f 6.00
Charge for the year (balancing figure) 3.00
Balance c/f (30% u Rs. 30 million temporary difference) 9.00

Tax expense Rs.in million


Current tax 16.50
Deferred tax (see above) 3.00
19.50

3.4 FIGS PAKISTAN LIMITED


Figs Pakistan Limited
Statement of profit or loss and other comprehensive income
For the year ended 31 December 2016
Note Rs.in
million
Sales 1 44,758
Cost of sales 2 (26,203)
Gross profit 18,555
Distribution costs 3 (6,431)
Administrative expenses 4 (752)
Other operating expenses 5 (399)
Other operating income 6 30
Profit from operations 11,003

Emile Woolf International 175 The Institute of Chartered Accountants of Pakistan


Advanced accounting and financial reporting

Finance costs 7 (166)


Profit before tax 10,837
Taxation 8 (2,532)
Profit after tax 8,305
Other comprehensive income -
Total comprehensive income for the year 8,305

Figs Pakistan Limited


Notes to the financial statements
For the year ended 31 December 2016

1 Sales Note Rs.in


million
Manufactured goods
Gross sales 56,528
Sales tax (10,201)
46,327
Imported goods
Gross sales 1,078
Sales tax (53)
1,025
Sales discounts (2,594)
44,758
2 Cost of sales
Rs.in
million
Raw material consumed (1,751 + 22,603 - 2,125) 22,229
Stores and spares consumed 180
Salaries, wages and benefits (2,367 55%) 2.1 1,302
Utilities (734 85%) 624
Depreciation and amortizations (1,287 70%) 901
Stationery and office expenses (230 25%) 58
Repairs and maintenance (315 85%) 268
25,562
Opening work in process 73
Closing work in process (125)
25,510
Opening finished goods (manufactured) 1,210
Closing finished goods (manufactured) (1,153)
25,567
Finished goods (imported)
Opening stock 44
Purchases 658
702
Closing stock (66)
636
26,203

2.1 Salaries, wages and benefits include Rs. 30 million (54 55%) and Rs. 24
million (44 55%) in respect of defined contribution plan and defined
benefit plan respectively.

Emile Woolf International 176 The Institute of Chartered Accountants of Pakistan


Answers

3 Distribution costs Rs.in


million
Advertisement and sales promotion 4,040
Outward freight and handling 1,279
Salaries, wages and benefits (2,367 30%) 3.2 710
Utilities (734 5%) 37
Depreciation and amortization (1,287 20%) 257
Stationery and office expenses (230 40%) 92
Repairs and maintenance (315 5%) 16
6,431

3.1 Salaries, wages and benefits include Rs. 16 million (54 30%) and Rs. 13
million (4430%) in respect of defined contribution plan and defined benefit
plan respectively.
Rs.in
4 Administrative expenses million
Salaries, wages and benefits (2,367 15%) 4.1 355
Utilities (734 10%) 73
Depreciation and amortization (1,287 10%) 129
Stationery and office expenses (230 35%) 80
Repairs and maintenance (315 10%) 31
Legal and professional charges 71
Auditor's remuneration 4.2 13
752

4.1 Salaries, wages and benefits include Rs. 8 million (54 15%) and Rs. 7
million (4415%) in respect of defined contribution plan and defined benefit
plan respectively.
4.2 Auditor's remuneration Rs.in
million
Audit fees 8
Taxation services 4
Out of pocket expenses 1
13
5 Other operating expenses
Donation 5.1 34
Worker's Profit Participation Fund 257
Worker Welfare Fund 98
Loss on disposal of property, plant and equipment 10
399
5.1 Donations
Donations include Rs. 5 million given to Dates Cancer Foundation (DCF).
One of the companys directors, Mr. Peanut is a trustee of DCH.
Donations other than that mentioned above were not made to any donee in
which a director or his spouse had any interest at any time during the year.
6 Other operating income Rs.in
million
Income from financial assets
Dividend income 12
Return on savings account 2
Income from non-financial assets
Scrap sales 16

Emile Woolf International 177 The Institute of Chartered Accountants of Pakistan


Advanced accounting and financial reporting

30
7 Finance costs
Finance charges on short term borrowings 133
Exchange loss 22
Finance charges on lease 11
166
8 Taxation
Current - for the year 1,440
Deferred (3,120 35%) 1,092
2,532

3.5 FAZAL LIMITED


The related parties comprise a subsidiary, an associated undertaking/an entity
having significant influence, director and key management personnel.
Aggregate transactions with related parties are as follows:
Entity Key
having
Subsidiary Associate Director Managem
significant ent
influence Personnel
Rupees Rupees Rupees Rupees
Transactions
Sales 500,000,000
Sales
25,000,000
discount
Sales return 5,500,000
Purchase of
5,000,000
raw material
Purchase of
3,000,000
equipment
Purchase of 14,000,00
machinery 0

Balances
Advances
At beginning 1,400,00
of the year 0
Repaid during
300,000
the year
At the end of 1,100,00
the year 0

(i) Sales discount represents a special discount which is not usually allowed to
other customers.
(ii) All transactions with related parties have been carried out on commercial
terms and conditions.

Emile Woolf International 178 The Institute of Chartered Accountants of Pakistan


Answers

3.6 BABER LIMITED


(i) AK Associates will not be treated as related party merely on the ground
that both entities have a director in common.
However, if it can be proved that an entity has some influence on other
entity; they will be treated as related parties.
(ii) Provider of finance is not necessarily a related party. However, SS Bank
has power to appoint its nominee director in the Board and therefore
enjoys significant influence; it will be treated as related party.
(iii) Mr. Zee will not be treated as related party unless it can be proved that
he has significant influence over the CEO. Further, IAS-24 does not
explicitly include Brother in the definition of close family member of an
individual.
(iv) A distributor with whom an entity transacts a significant volume of
business will not be treated as related party, merely by virtue of the
resulting economic dependence.
(v) Since Mr. Tee is the key management personnel of the company, he will
be treated as related party.
(vi) A post-employment benefit plan for the benefits of the employee is
treated as related party.
(vii) A supplier with whom an entity transacts a significant volume of
business cannot be termed a related party, merely by virtue of the
resulting economic dependence.
(viii) Ms. Vee will be treated as related party as she is a close family member
of CEO, and hence in a position to influence her husband.

3.7 GOLDEN LIMITED


Golden Limited
Notes to the Financial Statements
For the year ended December 31, 2011

Platinum Limited is the parent company which holds majority shares of the
company.

Related party transactions


The transaction with related parties are carried out in the ordinary course of
business at commercial rates except stated otherwise.

Associated Key Major


Parent
Under- Management Share-
Company
takings Personnel holders
Rupees in '000
Transactions:
Sales 18,000
Sales discount 1,500
Sale of property 10,000
Reimbursement of expenses on sale
of property 500
Interest free loans granted 2,000
Short term borrowings acquired 25,000
Interest on short term borrowings 1,500

Emile Woolf International 179 The Institute of Chartered Accountants of Pakistan


Advanced accounting and financial reporting

Associated Key Major


Parent
Under- Management Share-
Company
takings Personnel holders
Rupees in '000
Balances:
Accounts receivable 6,500 5,000
Loans to staff 1,800
Loans payable 25,000
Interest payable on the loan at
12% 1,500

20.1 Sales to related parties have been made at 20% mark-up as against
GL's policy to sell at a markup of 30%.
20.2 Administrative services are provided by the parent company free of cost
as per the agreement. Market value of these services is Rs. 350,000.
20.3 In respect of sale of property, a buyer is required to bear all costs
incurred on transfer. But in this case the company has reimbursed the
costs to SL
20.4 The interest free loan has been granted to the executive director as per
the terms of employment.

3.8 METAL LIMITED


IN THE BOOKS OF METAL LIMITED
Transactions with Related Parties
Related parties comprises of the companys subsidiaries. Transactions with
related parties are as follows:
2013 2012
Rupees
Subsidiaries
Sale of machine (at carrying amount plus 20%) - 19,200,000
Management fees income (Note 23.1) 12,000,000 -
Management fee receivable 1,000,000
Other receivables - Sale of machine - 19,200,000

No management fee was charged during the year ended 30 June 2012. Except
for this, all transactions have been carried out on arms length basis, as
approved by the board of directors of the company.

IN THE BOOKS OF COPPER LIMITED


Transactions with Related Parties
Related parties comprise of Metal Limited (parent company) and its subsidiaries.
Transaction with related parties can be summarized as follows:
2013 2012
Rupees
Parent Company
Purchase of machine - 19,200,000
Management fees (Note 23.1) 6,000,000 -
Management fee payable 500,000 -
Other payables - Sale of machine 19,200,000

2 No management fee was charged for the year ended 30 June 2012. Except
3. for this, all transactions have been carried out on arms length basis, as
1 approved by the board of directors of the company.

Emile Woolf International 180 The Institute of Chartered Accountants of Pakistan


Answers

IN THE BOOKS OF ZINC LIMITED


23 Transactions with Related Parties
Related parties comprise of Metal Limited (parent company) and its subsidiaries.
Transaction with related parties can be summarized as follows:
2013 2012
Rupees
Parent Company
Contract for factory extension project (Note 23.1) 15,000,000 -
Management fees (Note 23.2) 6,000,000 -
Management fee payable 500,000 -

23. The contract has been awarded to Iron Builders and Developers in which
1 one of the directors of the parent company is a partner.
23. No management fee was charged for the year ended 30 June 2012.
2 Except for this, all transactions have been carried out on arms length
basis, as approved by the board of directors of the company.

IN THE BOOKS OF STEEL LIMITED


Related parties comprise of Metal Limited (parent company) and its subsidiaries.
However, there was no related party transaction during the year.

3.9 ENGINA
Report to: The Board of Directors of Engina
From: XXXXXXXX
Date:
Subject: Related party transactions
Related party transactions
This report addresses the disclosure requirements of IAS 24 Related Party
Disclosures with regard to Engina. IAS 24 requires that all entities, listed or
otherwise, provide disclosure of such transactions as they may affect the
assessments made by users of an entitys operations, risks and opportunities.
It is understood that Engina is reluctant to disclose related party transactions
because they are believed to be both politically and culturally sensitive, however
the following advice must be followed in order to secure a listing/stock exchange
registration.
IAS 24: Scope and purpose
IAS 24 does not provide any exclusion from its scope, and so disclosure must be
made. Related party transactions are a normal feature of business, but an entitys
ability to succeed in business is often affected by the strength of its relationship
with other entities and individuals. The results of the entity may be affected if
these relationships were to be terminated. For example, the ability of an entity to
trade in a particular country may only be possible because of the presence of its
subsidiary in that local market. Similarly, prices and terms of trade may be
preferential because of the strength of the relationship. Therefore IAS 24 requires
knowledge of these transactions to be provided to the reader of the financial
statements.
The results of an entity may be affected even if the related party transactions do
not occur. A parent may cease trading with a business partner upon acquisition
of a subsidiary that can supply similar products.

Emile Woolf International 181 The Institute of Chartered Accountants of Pakistan


Advanced accounting and financial reporting

Disclosure must be given irrespective of whether the transactions took place at


an arms length value, as such transactions may still be lost if the relationship is
terminated. Hence the knowledge of such transactions provides valuable
information to investors and regulators.
Disclosure requirements
IAS 24 requires that, at a minimum, the following disclosures must be given:
The amount of the transaction
The amount of any outstanding balance and the terms, conditions and
guarantees attached
Allowance for any irrecoverable debts or amounts written off in the period
Disclosure that transactions were at an arms length value can only be
given if this information can be substantiated.
Disclosures relevant to Engina
The following outlines the related party disclosure requirements for the three
transactions you have specifically requested comment on. It is your responsibility
to bring any further related party transactions to our attention in order that they
can also be incorporated into your financial statement disclosures.
(a) Sale of goods to directors
The sale of goods and a company car to Mr Satay both constitute related
party transactions, due to Mr Satays position as a director of Engina. IAS
24 requires disclosure of all related party transactions with key
management personnel. However, accounting standards only apply to
material transactions. An item is considered material where knowledge of
that transaction might influence the decisions of a user of the financial
statements. Materiality is not just a matter of size, as small transactions
with a director may still be of relevance to an investor if the transaction is
material to the director, despite not being material to the entity.
In the situation described, the transactions amount to Rs. 600,000 of sales
and the sale of a company car for Rs. 45,000 (market value Rs. 80,000). In
terms of value these transactions appear not to be material to Engina and
neither do they appear to be material in value to Mr Satay. However, given
the sensitive nature of transactions with directors, and especially senior
directors like Mr Satay, the transactions should be disclosed in the financial
statements in accordance with good corporate governance practice.
Significant contracts with directors, such as these with Mr Satay, may also
require disclosure by the local Stock Exchange.
(b) Hotel property
The sale of the hotel to the brother of Mr Soy, constitutes a related party
transaction because of Mr Soys status as Managing Director. The property
seems to have been sold at below market price and IAS 24 requires
disclosure of any information surrounding a transaction which will allow the
reader to understand its impact on the financial statements.
The hotel had a carrying value of Rs. 5m, but given the fall in market values
it should have been written down to its recoverable amount in accordance
with IAS 36 Impairment. Recoverable amount is measured at the higher of
value in use (Rs. 3.6m) and fair value minus costs of sale (Rs. 4.3 - 0.2m).
Hence the property should have been recorded in the statement of financial
position at Rs. 4.1m.

Emile Woolf International 182 The Institute of Chartered Accountants of Pakistan


Answers

As the property was sold at Rs. 100,000 less than this impaired value,
disclosure of this fact should be made, together with any other information
relevant to the reader, such as the reason for the sale in light of the
expected decline in prices in the future.
(c) Mr Satay
Mr Satay has investments in 100% of the equity of Car and 80% of the
equity of Wheel. In turn, Wheel owns 100% of Engina. Engina and Wheel
are related because of their parent-subsidiary relationship. In addition,
because all three entities are under the common control of Mr Satay, IAS
24 also considers Engina and Car to be related. Therefore, the transactions
between Engina and both Wheel and Car are related party transactions.
The transactions will need to be disclosed in the individual financial
statements of all three entities. In the group accounts, all intra-group
transactions are cancelled on consolidation, and so disclosure need not be
made at this level.
Further disclosure requirements of directors interests in the equity of
Engina may be necessary under local Companies Acts requirements and
Stock Exchange rules.

3.10 SHAZAD INDUSTRIES LTD


(a) The purposes of segmental information are:
(i) to provide users of financial statements with sufficient details for them
to be able to appreciate the different rates of profitability, different
opportunities for growth and different degrees of risk that apply to an
entitys classes of business and various geographical locations.
(ii) to appreciate more thoroughly the results and financial position of the
entity by permitting a better understanding of the entitys past
performance and thus a better assessment of its future prospects.
(iii) to create awareness of the impact that changes in significant
components of a business may have on the business as a whole.
b) IFRS 8 defines an operating segment as a component of an entity:
that engages in business activities from which it may earn revenues
and incur expenses (including revenues and expenses relating to
transactions with other components of the same entity).
whose operating results are regularly reviewed by the entitys chief
operating decision-maker to make decisions about resources to be
allocated to the segment and assess its performance.
for which discrete financial information is available.
In order to identify the separate reportable segments, the following
criteria should be adopted:
(i) The reported revenue of the segment in Shazad Industries Ltd,
including both sales to external customers and inter-segment
sales, is ten percent or more of the combined revenue of its four
operating segments.
(ii) The Assets of the segment in Shazad Industries Ltd are ten
percent or more of the combined assets of its four operating
segments.

Emile Woolf International 183 The Institute of Chartered Accountants of Pakistan


Advanced accounting and financial reporting

(iii) The reported profit or loss of the segment in Shazad Industries


Ltd should be ten percent or more of the greater, in absolute
amount, of:
the combined reported profit of all its operating segments that did not
report a loss and
the combined reported loss of all operating segments that reported
losses.
(c) IFRS 8 lays down some very broad and inclusive criteria for reporting
segments. Unlike earlier attempts to define segments in more quantitative
terms, segments are defined largely in terms of the breakdown and
analysis used by management. This is, potentially, a very powerful method
of ensuring that preparers provide useful segmental information.
There will still be problems in deciding which segments to report, if only
because management may still attempt to reduce the amount of
commercially sensitive information that they produce.
The growing use of executive information systems and data management
within businesses makes it easier to generate reports on an ad hoc basis.
It would be relatively easy to provide management with a very basic set of
internal reports and analyses and leave the individual managers to prepare
their own more detailed information using the interrogation software
provided by the system.
If such analyses become routine then they would be reportable under IFRS
8, but that would be very difficult to check and audit.
There are problems in the measurement of segmental performance if the
segments trade with each other. Disclosure of details of inter-segment
pricing policy is often considered to be detrimental to the good of a
company. There is little guidance on the policy for transfer pricing.
Different internal reporting structures could lead to inconsistent and
incompatible segmental reports, even from companies in the same
industry.

3.11 AZ
(a) (i) Usefulness of segmental data
Many entities carry out several classes of business and operate in a
number of countries across the world. Each of these businesses and
geographical segments carries with it different opportunities for
growth, different rates of profit and varying degrees of risk. Some
business segments may be strongly influenced by the health of the
economy whereas other segments may be unaffected by recession.
One country may be experiencing growth; another country may be
less stable because of political events. Awareness of these cultural
and environmental differences is important to investors in order to
allow them to fully understand the performance and position of the
entity over the past, its prospects for the future and the risks that it
faces.
IFRS 8 requires that segmental information should be provided to
enable investors to understand the impact that the different segments
of a business may have on the business as a whole. If the user of
financial statements is only provided with figures for the entity as a
whole, this might hide the risks and problems or profits and

Emile Woolf International 184 The Institute of Chartered Accountants of Pakistan


Answers

opportunities of the underlying business segments. The


disaggregated financial information provided by segmental reporting
allows for analytical review on a segment by segment basis which will
provide greater understanding of the entitys position and
performance and allow a better assessment of its future.
(ii) Analysing segments
IFRS 8 defines an operating segment as a component of an entity
that engages in business activities from which it may earn revenues
and incur expenses, whose operating results are reviewed regularly
by the chief operating decision maker in the entity and for which
discrete financial information is available.
Not every part of a business is necessarily an operating segment or
part of an operating segment. Head office is an example, since head
office does not usually earn revenues. Generally an operating
segment has a segment manager who is directly accountable to and
maintains regular contact with the chief operating decision-maker, to
discuss the performance of the segment.
IFRS 8 requires that entities should report information about each
operating segment that is identified and that exceeds certain
quantitative thresholds for size of revenue, operating profit or loss or
assets. Financial information about operating segments with similar
characteristics can be aggregated.
IFRS 8 sets out the information about each reportable operating
segment that should be disclosed, including total assets, profit or
loss, revenue from external customers, revenue from sales to other
segments, interest income and expense, depreciation, material items
of income or expense and tax. The amount reported for each item
should be the same measure that is reported for the segment to the
chief operating decision maker of the entity.
IFRS8 applies to quoted companies only.

(b) AZ segmental reporting issues


(i) Fleet of aircraft
The answer to (a) explains how operating segments should be
identified. It seems probable that information is provided separately
for aircraft seat sales and for holiday homes to the chief operating
decision maker of AZ; therefore it seems probable that these should
be treated as reportable operating segments for the purpose of IFRS
8, provided that they exceed the minimum threshold limits for size (at
least 10% of revenues, or profit or loss, or assets of the reporting
entity).
(ii) Aircraft manufacturing plant
IFRS 8 requires that in measuring and reporting segment revenue
from transactions by an operating segment with other segments,
inter-segment transfers should be measured on the basis of the
information provided to the chief operating decision maker. In this
case, the revenues from inter-segment sales should be based on the
prices actually charged.
IFRS 8 also requires that AZ must disclose the basis of accounting for
transactions between reportable segments.

Emile Woolf International 185 The Institute of Chartered Accountants of Pakistan


Advanced accounting and financial reporting

(iii) Exceptional loss


Items which are unusual in size, nature or incidence and hence where
knowledge of their nature and amount is relevant to a readers
understanding of performance are required to be disclosed in the
notes or on the face of the statement of profit or loss in accordance
with IAS 1.
In addition, IFRS 8 requires disclosure for each reportable segment of
any material items of income or expense that is disclosed in
accordance with IAS1. If the activities of the airline manufacturing
plant are a reportable segment or part of a reportable segment, the
loss should therefore be reported within the segmental information in
the financial statements.
(iv) Discontinued operations
IFRS 8 does not deal with discontinued operations as this is the
subject of IFRS 5 Non-current assets held for sale and
discontinued operations. IFRS 5 requires the disclosure of the
impact of a discontinued operation, and has strict conditions that must
be met when identifying a discontinued operation.
To be classified as a discontinued operation, IFRS 5 requires that the
component of the entity has been sold in the year or is classified as
held for sale. A decision by the board and an announcement to the
press alone is insufficient to be classed as held for sale. The criteria
that must be met by the end of the reporting period are as follows:
The asset must be available for immediate sale in its present
condition; and
The sale must be highly probable. For this to be so the following
sub-conditions must also be met:
The appropriate level of management must be committed to the
plan to sell
An active programme to locate a buyer must have been initiated
The asset is being marketed at a reasonable price, and
The sale should be expected to be completed within one year
from the date of classification.
Hence, unless marketing of the holiday business is in progress at a
reasonable price, and sale in the next 12 months is expected, the
operation need not be classified as discontinued. If however, all of the
above criteria are met, then the results of this operation must be
disclosed as a single line on the face of the statement of profit or loss
together with supporting disclosures in the notes to the accounts.

(v) 40% investment in Eurocat


IFRS 8 requires that information should be disclosed for each
reportable segment of the entitys interest in the profit or loss of
associates and joint ventures accounted for by the equity method. In
addition, if the information is provided to the chief operating decision
maker of the entity, for each reportable segment there should be
disclosure of the amount of investment in associates or joint ventures
accounted for under the equity method.
Therefore, AZ must disclose the results of the associate in its
segmental disclosures.

Emile Woolf International 186 The Institute of Chartered Accountants of Pakistan


Answers

3.12 J-MART LIMITED


(a) Adjusting events:
Adjusting events are events that provide further evidence of conditions
that existed at the reporting date.
Examples of adjusting events include:
(i) The subsequent determination of the purchase price or of the
proceeds of sale of non-current assets purchased or sold before the
year end.
(ii) The renegotiation of amounts owing by customers or the insolvency
of a customer
(iii) Amounts received or receivable in respect of insurance or the
insolvency of a customer.
(iv) The settlement after the reporting date of a court case that confirms
that the entity had a present obligation at the reporting date.
(v) The receipt of the information after the reporting date indicating that
an asset was impaired at the reporting date.
(vi) The discovery of fraud or errors that show that the financial
statements are incorrect.
Non-adjusting events:
Non-adjusting events are indicative of conditions that arose subsequent to
the reporting date.
Examples of non-adjusting events might be:
(i) Losses of non-current assets or inventories as a result of a
catastrophe such as fire or flood
(ii) Closing a significant part of the trading activities if this was not
begun before the year end
(iii) The value of an investment falls between the reporting date and the
accounts are authorised
(iv) Announcement of dividend after year end.
(b) (i) The conditions attached to the sale give rise to a constructive
obligation on the reporting date.
A provision for the sales return should be recognised for 5% of
June 2016 sales. The related cost should also be reversed.
(ii) Since the law suit was already in progress at year-end and the
amount of compensation can also be estimated, it is an adjusting
event.
A provision of Rs. 400,000 should be made.
(iii) There is no obligating event at the year-end either for the costs of
fitting the smoke detectors or for fines under the legislation.
No provision should be recognised in this regard.
(iv) The obligating event is the communication of decision to the
customers and employees, which gives rise to a constructive
obligation from that date, because it creates a valid expectation

Emile Woolf International 187 The Institute of Chartered Accountants of Pakistan


Advanced accounting and financial reporting

that the division will be closed.


Since no communication has yet been made, no provision is
required in this regard.
(v) The obligating event is the signing of the lease contract, which
gives rise to a legal obligation.
A provision is required for the unavoidable rent payments.
(vi) Since the declaration was announced after year-end, there is no
past event and no obligation at year-end and is therefore non-
adjusting event.
Details of the dividend declaration must, however, be disclosed.

3.13 QALLAT INDUSTRIES LIMITED


(i) Provision must be made for estimated future claims by customers for
goods already sold.
The expected value i.e. Rs. 10 million ([Rs. 150m x 2%] + [Rs. 70m x
10%]) is the best estimate of the provision.
(ii) Warehouse A: It is an onerous contract as the warehouse has been
sublet at a loss of Rs. 200,000 per month. QIT should therefore create a
provision for the onerous contract that arises on vacating the warehouse.
This is calculated as the excess of unavoidable costs of the contract over
the economic benefits to be received from it. Therefore, QIL should
immediately provide for the amount of Rs. 13.2 million. [5.5 years x 12
month x Rs. 200,000] in its financial statements i.e. for the year ended
June 30, 2016.
Warehouse B: It is not an onerous contract because the warehouse has
been sublet at profit. Hence this would require no adjustment.
(iii) A provision is to be made by QIL.
(i) There is a present obligation (legal or constructive) as a result of a
past event; i.e. accident occurred on June 15, 2016.
(ii) It is probable that outflow of resources will be required to settle the
obligation; and
(iii) A reliable estimate can be made of the amount of the obligation.
The amount of provision shall be Rs. 2.0 million i.e. the most probable
amount as determined by the lawyer.
(iv) A provision of Rs.0.4 million is required in relation to penalty for March 1
to June 30, 2016 because at the reporting date there is a present
obligation in respect of a past event.
The reimbursement of penalty amount from the vendor shall be
recognised when and only when it is virtually certain that reimbursement
will be received if the entity settles the obligation. The reimbursement
should be treated as a separate asset in the statement of financial
position. However, in profit and loss statement, the expense relating to a
provision may be netted off with the amount recognised as recoverable, if
any.

Emile Woolf International 188 The Institute of Chartered Accountants of Pakistan


Answers

3.14 SKYLINE LIMITED


(i) Although the debt owing by the customer existed at the reporting date, the
customers inability to pay did not exist at that point. This condition only
arose in January 2017 after the fire.
Thus, this is a non-adjusting event. However, if it is material for the
financial statements, the following disclosure should be made.
Nature of the event
An estimate of its financial effect
(ii) The amount withdrawn before year end i.e. Rs. 1.5 million is an adjusting
event as although it was discovered after year end it existed at the year
end. However, since 60% has been recovered subsequently, Rs.0.6
million would be provided.
The further withdrawal of Rs. 6.0 million is a non-adjusting event as it
occurred after year end. However, if the events are considered material
the following disclosures should be made:

Nature of the event


The gross amount of contingency
The amount recovered subsequently

(iii) SL should not recognise the contingent gain until it is realised. However, if
recovery of damages is probable and material to the financial statements,
SL should disclose the following facts in the financial statements:

Brief description of the nature of the contingent asset


An estimate of the financial effect.

(iv) SL should make a provision of the expected amount i.e. Rs. 1.2 million
(Rs. 1.0 million x 60% + Rs. 1.5 million x 40%) because

it is a present obligation as a result of past event;


it is probable that an outflow of resources embodying economic
benefits will be required to settle the obligations; and
a reliable estimate can be made of the amount.

In addition, SL should disclose the following in the notes to the financial


statements:

Brief nature of the contingent liability


The amount of contingency
An indication of the uncertainties relating to the amount or timing of
any outflow.

Emile Woolf International 189 The Institute of Chartered Accountants of Pakistan


Advanced accounting and financial reporting

3.15 WALNUT LIMITED


(i) This is an adjusting post reporting event as it provides evidence of
conditions that existed at the end of the reporting period. The reasons for
the competitors price reduction will not have arisen overnight, but will
normally have occurred over a period of time, may be due to superior
investment in technology.
An inventory write down of Rs. 2.5 million should be recognised and the
amount included as inventory on the Statement of Financial Position
reduced to Rs. 12.5 million.
(ii) The provision should be recognised because the obligating event is the
communication of event to the public which creates a valid expectation
that the division will be closed.
However, the provision should only be recognised to the extent of
redundancy costs. IAS prohibits the recognition of future operating
losses, staff training and profits on sale of assets.
(iii) This is a non-adjusting event because the burglary and theft of
consumable stores occurred after reporting date. However, if the event is
material, it should be disclosed in the financial statements unless the loss
is recoverable from the insurance company.
(iv) The drop in value of investment in shares is a non-adjusting event. Since
the legislation was announced after the reporting date, the event is not a
past event. However, if the amount is material, it should be disclosed in
the financial statements.
(v) This is an adjusting event as it provides evidence of conditions that
existed at the end of the reporting period. The insolvency of a debtor and
the inability to pay usually builds up over a period of time and it can
therefore be assumed that it was facing financial difficulty at year-end.
A bad debts expense of Rs. 1.5 million should be recognised in SOCI.
(vi) It is a non-adjusting event because the declaration was announced after
the year-end and there was no obligation at year end. Details of the
bonus shares declaration must, however, be disclosed.

Emile Woolf International 190 The Institute of Chartered Accountants of Pakistan


Answers

CHAPTER 4: IAS 8: ACCOUNTING POLICIES, CHANGES IN ACCOUNTING


ESTIMATES AND ERRORS

4.1 WONDER LIMITED


(a) Wonder Limited
Statement of financial position as at 30 June 2016 (extracts)
2015
2016 (Restated)
Rs. m Rs. m
Property, plant and equipment 178.50 111.50
Retained earnings 158.65 95.05
Deferred tax liability 41.85 21.45

Workings: Property, plant and equipment 2016 2015


Given in question 189.0 130.0
Deduct repair cost debited in error (20.0) (20.0)
Add back depreciation charged on this
(20 u 10% u 21/12) 3.5
(20 u 10% u 9/12) 1.5
(16.5) (18.5)
Incorporate review of useful life
Charged already (70/5 years) 14.0 -
Correct charge ((70 u 4/5) = 56/7 years 8.0 -
Add back 6.0 -
178.5 111.5

Workings: Deferred tax liability 2016 2015


Given in question 45.00 27.00
Adjustments to PP&E
Correction of error (16.50) (18.50)
Change of useful life 6.00 -
Net adjustment (Cr to PP&E) 10.50 18.50
Tax thereon (at 30%) (Dr DT liability) (3.15) (5.55)
41.85 21.45

Workings: Retained earnings 2016 2015


Given in question 166.00 108.00
Net adjustment
(Cr to PP&E therefore Dr retained earnings)) (10.50) (18.50)
Tax thereon (at 30%)
(Dr DT liability therefore Cr Retained earnings) 3.15 5.55
158.65 95.05

(b) Wonder Limited


Extracts of statement of profit or loss for the year ended 30 June 2016
2015
2016 (Restated)
Rs. m Rs. m
Profit before taxation 98.00 101.50
Taxation (34.40) (36.45)
Profit after taxation 63.60 65.05

Emile Woolf International 191 The Institute of Chartered Accountants of Pakistan


Advanced accounting and financial reporting

Workings: Profit before tax 2016 2015


Given in question 90.0 120.0
Deduct repair cost capitalised in error (20.0)
Add back depreciation on this
(20 u 10% u 12/12) 2.0
(20 u 10% u 9/12) 1.5
2.0 (18.5)
Adjustment for re-estimate of useful life
Charged already (70/5 years) 14.0 -
Correct charge ((70 u 4/5) = 56/7 years ) 8.0 -
Add back 6.0 -
98.0 101.5

(c) Wonder Limited


Extracts of statement of changes in equity for the year ended 30 June
2016
Retained
earnings
Rs. m
Balance as on 1 July 2014 (108-78) 30.00
Profit for the year ended 30 June 2015 (78 - 12.95
(restated) 65.05
Balance as at 30 June 2015 (restated) 95.05
Profit for the year ended 30 June 2016 63.60
Balance as at 30 June 2016 158.65

(d) Wonder Limited


Notes to the financial statements for the year ended 30 June 2016
Correction of error: During the year ended 30 June 2014, a repair was
erroneously debited to machinery account. The effect of this error is as
follows:
2015
Effect on the statement of profit or loss Rs. m
(Increase) / decrease in expenses or losses
Repairs and maintenance (20.00)
Depreciation (20 u 10% u 9/12) 1.50

Tax expense (30% u (20  1.5)) 5.55


Decrease in profit for the year 1.4 (12.95)

Effect on the statement of financial position


Increase / (decrease) in assets
Property, plant and equipment (20 1.5) (18.50)

(Increase) / decrease in liabilities


Deferred tax liability (Rs. 18.5 30%) 5.55
Retained earnings (18.50 - 5.55) (12.95)

Emile Woolf International 192 The Institute of Chartered Accountants of Pakistan


Answers

4.2 DUNCAN
Statement of changes in equity (extract) for the year ended December 31,
2016
Retained Retained
earnings earnings
2016 2015
Rs.000 Rs.000
Opening balance as reported 23,950 22,500
Change in accounting policy (W2) 450 400

Re-stated balance 24,400 22,900
Profit after tax for the period (W1) 4,442 3,250
Dividends paid (2,500) (1,750)

Closing balance 26,342 24,400

Workings
(1) Revised profit
2016 2015
Rs.000 Rs.000
Per question 4,712 3,200
Add back: Expenditure for the year 600 500
Minus: Depreciation (870) (450)

Revised profit 4,442 3,250

(2) Prior period adjustment
The prior period adjustment is the reinstatement of the Rs. 400,000 asset
on 1 January 2015 and the Rs. 450,000 asset at 1 January 2016. On 31
December 2016 the closing balance above of Rs. 26,342,000 can be
reconciled as the original Rs. 26,162,000 plus the reinstatement of the
remaining asset of Rs. 180,000.

4.3 MOHANI MANUFACTURING LIMITED


Mohani Manufacturing (Private) Limited
Statement of changes in equity
For the year ended December 31, 2016
Retained
Earnings
Rs.in million
Balance at December 31, 2014 as previously reported
(Rs. 89m Rs. 21m) 68.00
Effect of change in accounting policy (Rs. 37m - Rs. 35.5m) (1.50)
Balance at December 31, 2014 restated 66.50
Profit for the year ended December 31, 2015 - restated (W1) 39.70
Balance at December 31, 2015 restated 106.20
Profit for the year ended December 31, 2016 (W2) 8.80

Balance at December 31, 2016 115.00

Emile Woolf International 193 The Institute of Chartered Accountants of Pakistan


Advanced accounting and financial reporting

W1: Profit for the year ended December 31, 2015 (as
Rs.in million
restated)
Profit as previously reported 21.00
Incorrect recording of depreciation (Rs. 25 million Rs. 10
million) 15.00

Reversal of FIFO method


Opening inventory 37.00
Closing inventory (42.30)
(5.30)
Application of weighted average method
Opening inventory (35.50)
Closing inventory 44.50
9.00
39.70

W2: Adjusted profit for year ended June 30, 2016 Rs.in million
Profit as per draft financial statements 15.00
Adjustment in Opening Inventory
FIFO 42.30
Weighted average (44.50)
(2.20)
Adjustment in Closing Inventory
FIFO (58.40)
Weighted average 54.40
(4.00)
Adjusted profit 8.80

Emile Woolf International 194 The Institute of Chartered Accountants of Pakistan


Answers

CHAPTER 5: IFRS 15: REVENUE FROM CONTRACT WITH CUSTOMER

5.1 PARVEZ LIMITED


(1) Sale and repurchase agreement
The transaction is in the nature of sale and repurchase agreement
therefore the economic phenomenon of the transaction is that of a loan for
which the goods have been given as security. Therefore no contract of sale
of goods or services is identified.
The difference between the sale price of Rs.18m and the repurchase price
of Rs.19m represents the interest on the loan for a period of four months.
To account for the transaction in accordance with its substance:
The goods should remain in inventories of PL at the lower of cost and
net realisable value.
No sale should be recorded.
The amount once received from the bank should be treated as a
current loan liability of Rs.18m.
Interest should be charged applying implicit rate to profit or loss for
each reporting period.
(2) Consignment inventories
There is a contract for sale of cars between Pervez Limited (PL) and dealer
containing confirmation of respective right and obligation, payment term,
commercial substance and probability of collection of price.
There is only one performance obligation, namely, the transfer of cars to
the dealer.
As per contract, the transaction price would be list price on the date of sale
to third parties during the six month period. Thereafter, though not
specifically mentioned, after the lapse of fifteen days the list price
applicable on sixteenth day would be the transaction price of the unsold
cars not returned.
Since there is only one performance obligation, the question of allocation of
transaction price does not arise till the time of sale to third parties.
PL will recognize revenue upon satisfaction of performance obligation.
Performance obligation would be satisfied once the dealer has sold any
cars to third parties during the six month period. Thereafter, if the dealer
does not return the unsold cars within fifteen days, the performance
obligation would be considered as satisfied on sixteenth day.
On 31 March 2017 the vehicles should remain in inventories in PL books of
accounts.

Emile Woolf International 195 The Institute of Chartered Accountants of Pakistan


Advanced accounting and financial reporting

5.2 SACHAL LIMITED


International Financial Reporting Standard (IFRS 15) provides that the revenue is
recognized:
(a) when the performance obligation is satisfied by the entity by transferring a
promised good or service (ie an asset) to the customer; and
(b) the asset is transferred when the customer obtains the control of that
asset.
Based on this principle, the following is the considerations to be taken into
account in determining accounting for revenue:
a) Restaurant management software
There exists a contract for sale of Restaurant management software
between SL and customers containing confirmation of respective right
and obligation, payment term, commercial substance and price is
collected in advance.
There are two performance obligations, namely:
x Explicit: delivery of software and
x Implicit: six month on-site support
As per contract, the transaction price is Rs.1.5 million for both
performance obligations.
Based on stand-alone selling price approach, software will be priced as
Rs.1.35 million (i-e. 1.50 m 0.15) and six month on-site support services
will be priced as Rs.0.15 million (i-e. 0.30 million x 6/12).
PL will recognize revenue from sale of software upon delivery if SL can
objectively conclude that the software meets the requirements of the
customer. The term of full payment of transaction price in advance is a
reasonable evidence of clarity of specification between SL and customer.
The agreed thirty days trial time will be considered as a formality of the
contract.
PL will recognize revenue from on-site support services over six months
period on straight-line basis.
b) Maintenance support for the standard software package
Such service is provided under a written contract that contains
confirmation of respective right and obligation, payment term, commercial
substance. SL will assess the collectability of the price if not received in
advance.
The performance obligation is to provide maintenance and support
services.
The price of the service is Rs.0.30 million for one year term.

Emile Woolf International 196 The Institute of Chartered Accountants of Pakistan


Answers

Since there is only one performance obligation, the question of allocation


of transaction price does not arise.
PL will recognize revenue over one year period on straight-line basis, as
in this case input method is appropriate. The pattern of resources
consumed by SL is evenly spread over the period of contract.
c) Customized software
Such service is provided under a written contract that contains
confirmation of respective right and obligation, payment term, commercial
substance. SL will assess the collectability of the price.
The performance obligations are:
x Designing and development of customized software, and
x Maintenance and support services of the said software
The price of the service will be determined on the basis of terms of
contract.
The price will be allocated between the two performance obligations. Price
of maintenance services for the first year is included in the total contract
price. The allocated price would be 10% of the contract price, which is the
stand-alone price of the said services.
Satisfaction of performance obligation:
Revenue from design and development - PL will recognize revenue from
design and development over time, because the software at every stage
is expected to be customer specific and would have no alternative use for
SL. The terms of payment at different stages of project also confirms that
SL would have an enforceable right to receive payment if the contract is
terminated before completion. In this case output method would be
appropriate, as the resources applied on different stages vary. Therefore,
the amount of recognized revenue would correspond to the development
stage of the software at the end of reporting period.
Revenue from Maintenance and support services - PL will recognize
revenue over one year period on straight-line basis, as in this case, input
method is appropriate. The pattern of resources consumed by SL is
evenly spread over the period of contract.

5.3 BRILLIANT LIMITED


Identification of performance obligations
There are three performance obligations:
1. Transfer of 15 Plastic card printing machines and its software
2. Transfer of 8 Laminators
3. Transfer of 100,000 plastic cards

Emile Woolf International 197 The Institute of Chartered Accountants of Pakistan


Advanced accounting and financial reporting

Although the software is distinct from printing machine, but both are highly
dependable to each other and inter-related. In the context of this contract, these
are providing a combined output to PL. Therefore, software is not a separate
performance obligation.
The total transaction price as per the contract is Rs.9.2 million.
On the basis of available information the stand-alone prices of each item will be
estimated using the following approaches:
Plastic card printing machines and its software:
In the absence of observable stand-alone price, we may use adjusted market
assessment approach. The competitors machine is sold at Rs.750,000 which is
similar (not identical) to BLs machine. As per given information, we may use
customers rating for adjustment of competitors price that worked out as follows:
Rupees
Competitors price 750,000
Adjusted price of BL machine (7/9*750,000) 583,000
Total price (15*583,000) 8,745,000

Laminators:
There is neither observable stand-alone price nor any comparable competitors
product available in the market in which BL operates. In this case, we may use
expected cost plus a margin approach. The estimated stand-alone price is
worked out as follows:
Rupees
Expected cost to BL 200,000
Margin estimated (800,000 - 600,000)/600,000 = 33% 66,000
266,000
Total price (8*266,000) 2,128,000
Plastic cards:
Observable stand-alone price is available
Total price (100,000*12) 1,200,000

Total of stand-alone prices is:


Plastic card printing machines and its software 8,745,000
Laminators 2,128,000
Plastic cards 1,200,000
Total 12,073,000

Emile Woolf International 198 The Institute of Chartered Accountants of Pakistan


Answers

Allocation of Rs.9.2 million (transaction price) will be based on relative stand-


alone prices, as the difference of Rs.2.873 million between stand-alone price and
transaction price is not specific to any performance obligation.
Rupees
Plastic card printing machines and its software 6,663,961
(9,200,000*8,745,000/12,073,000)
Laminators 1,621,602
(9,200,000*2,128,000/12,073,000)
Plastic cards 914, 437
(9,200,000*1,200,000/12,073,000)
Total 9,200,000

5.4 WAQAS LIMITED


The following is the available data of the original project:
Transaction price Rs.20 million
Cost of the project Rs.12 million
At the signing of the contract only one performance obligation is identified.
Therefore, the question of allocation the transaction price of Rs.20 million would
not arise.
The revenue would be recognized over time because the installation and
construction will be done on the land of ACL and control of asset will be
transferred progressively and will create right of payment for WL. Amount of
revenue recognized would correspond to the progress of the project. The
progress will be measured using input method, that is, cost incurred plus margin.

At the end of seventh month:


Additional Reservoir:
a) is distinct from original RO plant project
b) increased the price of the contract by Rs.2.5 million which reflected WLs
stand-alone price of similar construction work. The following working
explains it further:
Cost estimated Rs.1.8 million
Usual margin (8/12*100=67%) Rs.1.2 million
Normal price Rs.3.0 million
Agreed consideration Rs.2.5 million

The reduced price is reasonable due to less administrative resources is to be


applied for additional work.

Emile Woolf International 199 The Institute of Chartered Accountants of Pakistan


Advanced accounting and financial reporting

The contract of additional reservoir will be treated as separate contract and its
revenue will be recognized separate from original contract. The revenue from this
contract will be recognized over time, as construction of reservoir will be done on
the land of ACL and control of asset will be transferred progressively and will
create right of payment for WL.
At this stage the revenue from RO plant project will be recognized as follows:
Percentage of work completed
(4.2/12.0*100) 35%
Revenue to be recognized
(35%*20) Rs.7.0 million

At the end of tenth month:


Increasing the size of reservoir will increase the scope of the contract, but it
cannot be considered as a distinct work already agreed. Increased contract price
also does not reflect WLs stand-alone price of similar work because it is equal to
the cost of work. Therefore, WL should account for this modification as part of
single performance obligation that is partially satisfied on the date of modification.
A cumulative catch-up adjustment will be done, which is worked out as follows:
Original contract Modified
Contract price 20.0 21.0
Total contract cost 12.0 13.0
Cost incurred so far 7.2 7.2
% of completion 60% 55%
Cumulative Revenue recognition 12.0 11.55

Difference between the two amounts of cumulative revenue will be the


adjustment to the revenue account.

Revenue from additional reservoir


% of completion (0.72/1.8 * 100) 40%
Revenue to be recognized (40% * 2.5) Rs.1.0 million

At the end of sixteenth month:


Additional work of pumping and piping facility increased the scope of the
contract. It is also distinct from the RO plant project. However, the increased
price of the contract does not reflect WLs stand-alone price of similar work
because it provides nominal margin to WL. Therefore, this contract cannot be
accounted for as separate contract. This contract will terminate the existing
contract and create a new contract. There will be two performance obligations (a)
Transfer of RO plant; and (b) transfer of pumping and piping facility.

Emile Woolf International 200 The Institute of Chartered Accountants of Pakistan


Answers

The price of new contract is worked out as follows:


% of completion of existing contract (11.70/13.0*100) 90%
Rupees
Revenue recognized (21.0 * 90%) 18.9m
Remaining promised consideration (21.0 18.9) 2.1m
Consideration of modification 3.0m
New contract price 5.1m

Allocation of new contract price on the basis of cost plus margin approach
Total estimated cost of new modified contract (13.0+2.8) 15.8m
Less: Already incurred cost 11.7m
Cost to be incurred 4.1m

Allocation
RO plant project (1.3/4.1*5.1) 1.62m
Pumping and piping facility (2.8/4.1*5.1) 3.48m

The revenue will be recognized over time.

Revenue from additional reservoir


% of completion (1.35/1.8 * 100) 75%
Revenue to be recognized (75% * Rs. 2.5m) 1.875m

5.5 ANABELLE
Included in statement of profit or loss
Rs.000
Revenue (860 + 619 + 387 + 120) (W3) 1,986
Costs (balancing figure) (1,732)

Profit (170 + 129 45) (W3) 254

Included in statement of financial position
Rs.000
Current assets
Due from customers on contracts (29 + 20) (W4) 49

Receivables: Amounts recoverable on contracts ((690 600) + 140


(650 600)

Non-current liabilities
Due to customers on contracts (W4) 290

Emile Woolf International 201 The Institute of Chartered Accountants of Pakistan


Advanced accounting and financial reporting

Workings
(1) Total profit expected on the contracts
Contract
A B C D
Rs.000 Rs.000 Rs.000 Rs.000
Contract price 1,850 750 960 800
Minus costs to date (1,490) (590) (405) (120)
Estimated future costs - (25) (600) (480)

Total expected profit/(foreseeable loss) 360 135 (45) 200

(2) Proportion of work completed to date


Contract
A B C D
Costs to date 1,490 590 405 Too early to assess
Total costs 1,490 590 + 25 405 + 600 with reasonable
certainty therefore
take nil profit (i.e.
= 100% 95.9% 40.3% revenue = costs)

(3) Statement of profit or loss figures


Revenue:
Contract
A B C D
Rs.000 Rs.000 Rs.000 Rs.000
Cumulative to year end 120
(750 u 95.9% (W2)) (960 (= costs
u 40.3%(W2)) 1,850 719 387 to date)
Minus taken to statement
of profit or loss in
previous years (990) (100) - -
This year (balancing 860 619 387 120
figure)

Profit:
Contract
A B C D
Rs.000 Rs.000 Rs.000 Rs.000
Cumulative to year end 360 129 (45) nil
(135 (W1) x 95.9% (W2)) (W1) (Take
whole of
loss)
Minus taken to statement of
profit or loss in previous years (190) - - -
This year (balancing figure) 170 129 (45) -

Emile Woolf International 202 The Institute of Chartered Accountants of Pakistan


Answers

(4) Figures in statement of financial position


Contract
A B C D
Rs.000 Rs.000 Rs.000 Rs.000
Costs incurred 1,490 590 405 120
Recognised profits/(losses) (W2) 360 129 (45) nil
Minus progress billings (1,850) (690) (650) (100)
Due from customers on
contracts/ (Due to customers on
contracts) - 29 (290) 20

5.6 REAL CONSTRUCTION COMPANY LTD


(a) Accounting for construction contracts in accordance with IAS 11 is
dependent upon whether the outcome of a contract can be measured
reliably. A contract can be measured reliably when:
(i) It is probable that the economic benefits associated with the contract
will flow to the entity; and
(ii) The contract costs can be identified and measured reliably.
In the case of a fixed price contract, the following must also be measured
reliably:
(i) Total contract revenue;
(ii) Contract costs to completion; and
(iii) Stage of contract completion as at reporting date.
Where a contract can be measured reliably, revenue is recognized by
reference to the stage of completion. This is an application of the concept
of accruals. Where a contract cannot be measured reliably, revenue is
recognized only to the extent that contract costs incurred are recoverable.
This is an application of the concept of prudence.
(b) Statement of profit or loss for Real Construction Company Ltd. for the
year ended 31 March 2016

Rs. m

Revenue (W3) 70

Cost of sales (W3) (81)

Gross loss (W3) (11)

Statement of Financial Position of Real Construction Company Ltd. for the


year ended 31 March 2016.
Current Assets: Amounts recoverable on construction contract (W4) Rs. 59m
Workings
The outcome of this contract can be estimated reliably, and therefore
incomes are recognised by reference to the stage of completion.

Emile Woolf International 203 The Institute of Chartered Accountants of Pakistan


Advanced accounting and financial reporting

(W1) Expected Profit

Rs.m

Contract price 300

Costs to date (195)

Estimated costs to completion (45)

Expected Profit 60

Note that rectification costs are not taken into account at this stage
because they are expensed in the period in which they are incurred
rather than spread across duration of the contract

(W2) Stage of completion

Sales value to date Rs. 220*


 100= 73.3%
Contract price Rs. 300

*Sales value to date Rs.m

(100180)
Work certified to 29 February 2016 = 200
90

Work completed in March 2016 20

220

W3 Statement of profit or loss

Cumulative Taken Current


Previously Year

Rs.m Rs.m Rs.m

Revenue (work certified) 220 (150) 70

Cost of sales 176 (112) (64)

Profit (73.3% x 60 (W1) 44 (38) 6

Rectification costs must be recognized in the year incurred and


therefore the current year figures calculated are amended as follows:

Rs.m

Revenue 70

Cost of sales (64 + 17) (81)

Loss (11)

Emile Woolf International 204 The Institute of Chartered Accountants of Pakistan


Answers

(W4) Gross amount due to or from customers


Rs.m
Costs to date 212
Profit to date (W3) 44
Progress billing (180)
Rectification costs (17)
Due from customer 59

5.7 GLADSTONE LTD


(a) Statement of profit or loss (extracts)
For the year ended 31 December
2013 2014 2015 2016
Rs.000 Rs.000 Rs.000 Rs.000
Revenue (W3) 3,143 1,968 5,272 2,117
Cost of sales (W3) (2,750) (3,861) (3,339) (1,150)
Gross profit /(loss) 393 (1,893) 1,933 967

(b) Statement of financial position (extracts)


As at 31 December
2013 2014 2015 2016
Rs.000 Rs.000 Rs.000 Rs.000
Contract revenue recognised
as revenue in the period: 3,143 1,968 5,272 2,117
Contract costs incurred and
recognised profits (less
recognised losses ) to date 3,143 4,250 10,383 12,500
Gross amounts due from
customers for contract work 143 Nil Nil Nil
Gross amounts due to
customers for contract work Nil 750 617 Nil

Workings
(1) Percentage completion
2013 2014 2015 2016
Rs.000 Rs.000 Rs.000 Rs.000
Costs to date 2,750
2,750+3,000 5,750
5,750+4,200 9,950
9,950+1,150 11,100
Estimated future costs 7,750 7,750 1,550
Estimated total costs 10,500 13,500 11,500 11,100

Emile Woolf International 205 The Institute of Chartered Accountants of Pakistan


Advanced accounting and financial reporting

Percentage complete (Costs to


date Estimated total costs) 26% 43% 86% 100%
393 433 1,400

Loss in full (1,500)


Less prior periods (393) 1,500 (433)
393 (1,893) 1,933 967

(2) Contract profitability (check)


2013 2014 2015 2016
Rs.000 Rs.000 Rs.000 Rs.000
Contract revenue 12,000 12,000 12,000 12,500
Estimated total costs (10,500) (13,500) (11,500) (11,100)
Estimated outcome
(actual for 2016) 1,500 (1,500) 500 1,400

Loss expected based on information in 2014 must be recognised in full.


Note that this will also need to take into account any profit previously
recognised. Therefore the loss recognised at the end of year 2 must be
1,500 + any previously recognised profit.
This is achieved by recognising sufficient cost to interact with the revenue
to date to produce the required loss.
(3) Revenue recognition
2013 2014 2015 2016
Contract revenue 12,000 12,000 12,000 12,500
Percentage complete u 26% u 43% u 86% u 100%
Revenue to date 3,143 5,111 10,383 12,500
Less recognised in prior
periods (3,143) (5,111) (10,383)
Revenue recognised in
year 3,143 1,968 5,272 2,117
Contract costs:
Costs to date 2,750 5,750 9,950 11,100
Less costs recognised in
previous period (2,750) (5,750) (9,950)
Costs incurred in period 2,750 3,000 4,200 1,150
Extra cost (loss
provision) 861 (861)
(2,750) (3,861) (3,339) 1,150
393 (1,893) 1,933 967

Emile Woolf International 206 The Institute of Chartered Accountants of Pakistan


Answers

Tutorial note:
If revenue is recognised on a cost based percentage of completion, the
costs recognised will normally be those incurred in the period.
The company recognised profit of Rs. 393 in 2013 but by the end of 2014 it
must recognise a loss of Rs. 1,500. Therefore, the total loss recognised in
2014 needs to be Rs. 1,893 in order to reach the overall loss of Rs. 1,500.
This is done by recognising an extra cost above and beyond what would
normally be recognised.
In year 3 the situation has changed. With hindsight the company
recognised too much expense previously. So instead of recognising the
costs incurred in the period it recognises that amount less the extra cost
previously recognised.
(4) Disclosure workings
2013 2014 2015 2016
Contract costs incurred 2,750 5,750 9,950 11,100
Profits /losses 393 (1500) 433 1,400
3,143 4,250 10,383 12,500
Billings (3,000) (5,000) (11,000) (12,500)

143 (750) (617) nil

Positive = a receivable; Negative = a payable.

5.8 SILVER CONSTRUCTION LIMITED

(a) Silver Construction Limited


Extracts from statement of profit or loss for the year ended June
30, 2016
Rs. in
million

Contract revenue recognized 2,318.18


Contract costs recognized (2,108.00)

Silver Construction Limited


Extracts from statement of financial position as at June 30, 2016
Rs. in
million
ASSETS
Due from customers 106.75

LIABILITIES
Due to customers 21.76

Emile Woolf International 207 The Institute of Chartered Accountants of Pakistan


Advanced accounting and financial reporting

Working Schedule

I II III IV V VI Total
Rupees in million
Contract price 300 375 280 400 270 1,200 2,825.00
Incentive
payments - - - 40 - - 40.00
Total
contract
price (A) 300 375 280 440 270 1,200 2,865.00
Contract cost
incurred to
date (B) 248 68 186 246 185 1,175 2,108.00
Estimated
further costs 67 221 - 164 15 - 467.00
Total
estimated
costs to
complete (C) 315 289 186 410 200 1,175 2,575.00
Completion %
B/C x 100 (D) 78.73% 23.53% 100% 60% 92.50% 100%
Revenue to
be recognized
AxD (E) 236.19 88.24 280.00 264.00 249.75 1,200 2,318.18
Expected
losses from
contracts (A-C) (15.00) - - - - - (15.00)
Amount
recoverable
from customer (E) *233.00 88.24 280.00 264.00 249.75 1,200
Progress
billings 200.00 110.00 280.00 235.00 205.00 1,200
Due from
customers 33.00 - - 29.00 44.75 - 106.75
Due to
customers - (21.76) - - - (21.76)
* Cost to be recognized expected losses = 248 15 = 233

(b) Comments on additional information


(i) Incentive payments are included in contract revenue when:
The contract is sufficiently advanced that it is probable that the
specified performance standards will be met or exceeded; and
The amount of the incentive payment can be measured reliably.
Since the Contract IV is in advance stage and the probability to
achieve the target is very high, the company should recognize the
incentive payment to be received, on this contract.

Emile Woolf International 208 The Institute of Chartered Accountants of Pakistan


Answers

(ii) Claims are recorded in contract revenue only when:


Negotiations have reached an advanced stage such that it is
probable that the customer will accept the claim; and
The amount that it is probable will be accepted by the customer
can be measured reliably.
Since the claim amount cannot be measured reliably, the claim should
not be recognized as contract revenue.

5.9 XYZ LTD


The changes required for compliance are not considered to be a distinct
performance obligation. The contractor measures performance based on costs
incurred. The modification will be accounted for as part of the existing contract
and a cumulative catch-up adjustment will be required to revenue as follows:

Adjustment
XYZ Ltd. Original Modified
Required
Contract value 25,000,000 30,000,000 -
Costs incurred to date 9,000,000 9,000,000 -
Total expected cost 20,000,000 24,500,000 -
% Estimate completed 45% 37% -
Gross Margin % 20% 18% -
Revenue recognized 11,250,000 11,020,408 (229,592)

5.10 ABC LTD


Case A - Additional products for a price that reflects the stand-alone selling price
When the contract is modified, the price of the contract modification for the
additional 30 products is an additional Rs.2,850 or Rs.95 per product. The pricing
for the additional products reflects the stand-alone selling price of the products at
the time of the contract modification and the additional products are distinct from
the original products.
The contract modification for the additional 30 products is, in effect, a new and
separate contract for future products that does not affect the accounting for the
existing contract. The entity recognizes revenue of Rs.100 per product for the
120 products in the original contract and Rs.95 per product for the 30 products in
the new contract.
Case B - Additional products for a price that does not reflect the standalone
selling price
During the process of negotiating the purchase of an additional 30 products, the
parties initially agree on a price of Rs.80 per product. However, the customer
discovers that the initial 60 products transferred to the customer contained minor
defects that were unique to those delivered products. The entity promises a
partial credit of CU15 per product to compensate the customer for the poor
quality of those products.
The entity and the customer agree to incorporate the credit of Rs.900 (Rs.15
credit 60 products) into the price that the entity charges for the additional 30

Emile Woolf International 209 The Institute of Chartered Accountants of Pakistan


Advanced accounting and financial reporting

products. Consequently, the contract modification specifies that the price of the
additional 30 products is Rs.1,500 or Rs.50 per product. That price comprises the
agreed-upon price for the additional 30 products of Rs.2,400, or Rs.80 per
product, less the credit of Rs.900.
At the time of modification, the entity recognizes the Rs.900 as a reduction of the
transaction price and, therefore, as a reduction of revenue for the initial 60
products transferred. In accounting for the sale of the additional 30 products, the
entity determines that the negotiated price of Rs.80 per product does not reflect
the stand-alone selling price of the additional products.
Consequently, the contract modification does not account for as a separate
contract. Because the remaining products to be delivered are distinct from those
already transferred. The entity accounts for the modification as a termination of
the original contract and the creation of a new contract.
Consequently, the amount recognized as revenue for each of the remaining
products is a blended price of Rs.93.33 {[(Rs.100 60 products not yet
transferred under the original contract) + (Rs.80 30 products to be transferred
under the contract modification)] 90 remaining products}.

5.11 DX LTD
The legal fees and commission will be considered an incremental cost of
obtaining a contract because these costs were only incurred as a result of
obtaining the contract. The company expects to recover these costs through the
future advertising revenue that will be earned under the contract. Had the
contract not been obtained, these costs would not have been incurred. If the legal
fees had been incurred prior to obtaining the contract, they would not be
capitalized.
The meals and entertainment costs are not eligible to be capitalized because
they would have been incurred regardless of whether the contract had been
obtained.
The costs related to the Directors time and the costs associated with hiring
actors are direct labour costs associated with providing the advertising services
and are considered to be costs directly related to the contract and are not
covered by any other standard. These costs also meet the required criteria for
capitalization, because they:

x are specifically identified through time-sheet allocations and invoices (i.e.,


relate directly to the contract)
x provide a resource (e.g., Director and Actor) used to satisfy the performance
obligation
x will be recovered through contract revenue earned. Once these costs are
capitalized, they will be amortized and assessed for impairment in
accordance with IFRS 15.

5.12 PL LTD
Initially, PL Ltd will record a contract liability for the Rs.100 gift card. The entity
will also have to account for the 20% breakage. However, since this amount
cannot be recognized upfront, it will initially remain as part of the Rs.100 liability.
As the customer exercises its rights by redeeming the card, a proportionate
amount of breakage can be recognized. In year one, the customer makes one

Emile Woolf International 210 The Institute of Chartered Accountants of Pakistan


Answers

purchase of Rs.50 using the gift card. Therefore, when this purchase is made,
rather than recognizing Rs.50 of revenue, Rs.62.50 of revenue will be recognized
to incorporate expected breakage (i.e., Rs.50 *(100/80)).

5.13 FX LTD
There are two warranties in this contract:
1. assurance-type warranty for first year after purchase
2. service-type warranty for two years after expiry of the initial standard
warranty.
Given that the customer has the option of purchasing the additional warranty
separately, this is a service-type warranty and is accounted for as a separate
performance obligation. Deferred revenue of Rs.200,000 is recognized (as
opposed to revenue being recorded) until the performance obligation is satisfied.
The assurance-type warranty is accounted for in accordance with IAS 37
Provisions, Contingent Liabilities and Contingent Assets.

Emile Woolf International 211 The Institute of Chartered Accountants of Pakistan


Advanced accounting and financial reporting

CHAPTER 6 IAS 16: PROPERTY, PLANT AND EQUIPMENT

6.1 FAM
Accounting policies
(a) Property, plant and equipment is stated at historical cost less depreciation,
or at valuation.
(b) Depreciation is provided on all assets, except land, and is calculated to
write down the cost or valuation over the estimated useful life of the asset.
The principal rates are as follows.
Buildings 2% pa straight line
Plant and machinery 20% pa straight line
Fixtures and fittings 25% pa reducing balance

tools and equipment


Fixed asset

account and assets


Land and buildings

movements

Fixtures, fittings,

in the course of
Payments on

construction
machinery
Plant and

Total
Cost/valuation Rs.000 Rs.000 Rs.000 Rs.000 Rs.000
Cost at 1 January 900 1,613 390 91 2,994
2016
Revaluation 600 600
adjustment
Additions 154 40 73 (W1) 267
Reclassifications 100 (100)
Disposals (277) (41) (318)

As at 31 December
2016 1,600 1,490 389 64 3,543

Depreciation
At 1 January 2016 80 458 140 678
Revaluation (80) (80)
adjustment
Provisions for year 17 298 70 385
(W2)
Disposals (195) (31) (226)

At 31 December 2016 17 561 179 757

Written down value 1,583 929 210 64 2,786

Land and buildings have been revalued during the year by Messrs Jackson
& Co on the basis of an existing use value on the open market.

Emile Woolf International 212 The Institute of Chartered Accountants of Pakistan


Answers

The corresponding historical cost information is as follows.


Land and
buildings
Rs.000
Cost
Brought forward 900
Reclassification 100

Carried forward 1,000

Depreciation
Brought forward 80
Provided in year 10

Carried forward 90

Net book value 910

Workings
Rs.000
(1) Additions to assets under construction 53
Deposit on computer 20

73

Rs.000
600
(2) Depreciation on buildings 40 + (100 u 2%) 17
2% straight line depreciation is equivalent to a 50 year life.
The buildings are ten years old at valuation and therefore
have 40 years remaining.
Depreciation on plant (1,613 + 154 277) u 20% 298
Depreciation on fixtures (390 + 40 41 140 + 31) u 25% 70

6.2 GUJRAT CONSTRUCTION LIMITED


Property, plant and equipment note (extract) for the year ended 31 December
2016
constructio
equipment
machinery
Plant and
Land and
buildings

Assets
Office

under

Total
n

Cost or valuation Rs Rs Rs Rs Rs
At 1 January
2016 1,500,000 1,276,500 356,400 - 3,132,900
Additions 135,000 36,500 29,200 200,700
Classified as held
for sale (50,000) (50,000)

Emile Woolf International 213 The Institute of Chartered Accountants of Pakistan


Advanced accounting and financial reporting

constructio
equipment
machinery
Plant and
Land and
buildings

Assets
Office

under

Total
n
Disposals (104,000) (104,000)
At 31 December
2016 1,500,000 1,257,500 392,900 29,200 3,179,600
Depreciation
At 1 January
2016 315,000 879,300 210,400 1,404,700
Held for sale
(W3) (40,500) (40,500)
Disposals (W4) (65,000) (65,000)
Impairment
losses (W1 and
W3) 101,875 9,250 111,125
Charge for year
(W2) 13,175 320,917 74,930 409,022
At 31 December
2016 430,050 1,103,967 285,330 1,819,347
Carrying amount
At 31 December
2016 1,069,950 153,533 107,570 29,200 1,360,253
At 1 January
2016 1,185,000 397,200 146,000 1,728,200

Workings
(W1) Impaired workshop
Rs.
Valuation on 31 December 2013 210,000
Depreciation to 31 December 2015 (210,000 48 u 2) (8,750)
Depreciation to 31 December 2016 (210,000 48) (4,375)
Carrying amount at 31 December 2016 196,875
Recoverable amount (100,000 5,000) (95,000)
Impairment 101,875
(W2) Depreciation charges for year
Land and buildings
Rs.
Impaired workshop (W1) 4,375
Other ((1,500,000 850,000 210,000 (W1)) 50) 8,800
13,175

Emile Woolf International 214 The Institute of Chartered Accountants of Pakistan


Answers

Plant and machinery


Rs.
Depreciation on assets held for whole year
((1,276,500-50,000-104,000) 25%) 280,625
Depreciation on asset disposed of (104,000 25% 6/12) 13,000
Depreciation on additions (135,000 25% 6/12) 16,875
Depreciation on asset classified as held for sale but not
sold (W3) 10,417
320,917
Office equipment
Rs.
Held for whole year (356,400 20%) 71,280
Additions (36,500 20% 6/12) 3,650
74,930
(W3) Asset classified as held for sale
Rs.
Cost on 1 May 2014 50,000
Depreciation to 31 December 2015 (50,000 25% (1
and 8/12)) (20,833)
Depreciation from 1 January 2016 to 1 November 2016
(50,000 25% 10/12) (10,417)
Carrying amount at 1 November 2016 18,750
Fair value less costs to sell (11,000 1,500) (9,500)
Impairment loss on classification as held for sale (9,250)
Total to be eliminated on classification as held for sale
(20,833 + 10,417 + 9,250) 40,500
(W4) Accumulated depreciation on disposal
Rs.
To 31 December 2015 (104,000 25% 2) 52,000
For current year (W2) 13,000
65,000

Emile Woolf International 215 The Institute of Chartered Accountants of Pakistan


Advanced accounting and financial reporting

CHAPTER 7 NON-CURRENT ASSETS: SUNDRY STANDARDS

7.1 SPIN INDUSTRIES LIMITED


Rs.
Commitment fee 125,000
Actual borrowing costs of specific loan (W1) 2,050,000
General borrowing costs (W1) 1,175,283
Less: Investment income (W2) (137,500)
Interest costs to be capitalised 3,212,783

W1
Rate Borrowing
Outstanding Outstanding
of cost to be
amount Months outstanding month up to
interes capitalised
completion
Rs. t Rs.
Specific loan
Utilised till first
repayment 25,000,000 1-Sep-15 31-Jan-16 5 12% 1,250,000
Utilised after
the first
repayment 20,000,000 1-Feb-16 31-May-16 4 12% 800,000
2,050,000
General
Borrowings (W4)
Utilised after
specific loan
exhausted on
nd
2 payment to
contractor 12.08
(W3) 8,125,000 1-Dec-15 31-May-16 6 % 490,750
Principal
payment of 12.08
specific loan* 5,000,000 1-Feb-16 31-May-16 4 % 201,333
rd
3 payment to 12.08
contractor 12,000,000 1-Feb-16 31-May-16 4 % 483,200
rd
4 payment to 1-Jun-16 12.08
contractor 9,000,000 31-May-16 0 % -
1,175,283

*Note: Para 10 of IAS 23 says that the borrowing costs that should be capitalised
are those which "would have been avoided" if the expenditure had not been
made. The repayment of the loan on the specific borrowing has been made out
of general borrowing. In effect, another specific amount of Rs. 5,000,000 has
been borrowed to fund the project out of general borrowing. This is somewhat
confusing but, in substance borrowing of Rs. 5,000,000 was repaid and replaced
by a specific loan of Rs. 5,000,000 at 12.08%.

W2: Investment income Rs.


Surplus fund available from 1-Sep-15 to 30-Nov-15
(Rs. 25m Rs.0.125m Rs. 8m Rs. 10m) 8% 3/12 137,500

W3: Specific loan utilization


Commitment fee 125,000
Payment for obtaining permit 8,000,000
1st payment to contractor 10,000,000
2nd payment to contractor (balancing) 6,875,000

Emile Woolf International 216 The Institute of Chartered Accountants of Pakistan


Answers

25,000,000

2nd payment to contractor (total) 15,000,000


Less: paid out of specific loan (as worked out above) 6,875,000
Paid from general borrowing 8,125,000

W4: Weighted average rate of borrowing

Weighted average
amount of loan Interest Rs.
Rs.
From Bank A 25,000,000 Rs. 25,000,000 13% 9/12 = 2,437,500
From Bank B 20,000,000 3,000,000
45,000,000 5,437,500

Weighted average rate of borrowing (Rs. 5,437,500 / 45,000,000) 12.08%

7.2 QURESHI STEEL LIMITED


Capital work in progress Factory building Rs.000
Progress invoices received from the contractor
(30,000+20,000+10,000+15,000) 75,000.00
(Rain damages paid would be chargeable to profit and loss
account/ insurance claim)

Borrowing costs to be capitalised:


Loan processing charges 500.00
Interest on bank loan W1 1,841.67
Interest on running finance W2 2,730.00
Interest income from surplus loan amount W4 (395.00)
Capital work in progress June 30, 2016 79,676.67

W1: Interest on bank loan:


Rs.000
Interest amount Outstanding Interest
From To Months loan amount at 13%
01-12-2015 31-05-2016 6 25,000 1,625.00
01-06-2016 30-06 -2016 1 20,000 216.67
1,841.67

W2: Interest on running finance


Rs.000
Payment Payments from Interest
s Months at 15%
Runnin outstandin per
Right Bank
Payments Invoice net of g g up to annum
issue loan
date Description amount deductions finance 30-6-10 (W3)

01-07-15 Advanced
payment 10,000 10,000 10,000 12.00 1,500
15-10-15 1st
progress
bill 30,000 25,500 15,000 10,500 8.50 1,116

Emile Woolf International 217 The Institute of Chartered Accountants of Pakistan


Advanced accounting and financial reporting

W2: Interest on running finance


Rs.000
Payment Payments from Interest
s Months at 15%
Runnin outstandin per
Right Bank
Payments Invoice net of g g up to annum
issue loan
date Description amount deductions finance 30-6-10 (W3)

15-01-16 2nd
progress
bill 20,000 17,000 17,000 - - -
15-04-16 3rd
progress
bill 10,000 8,500 7,500 1,000 2.50 31
31-05-16 Loan
interest 1,625 1,625 1.00 20
31-05-16 Loan
instalment 5,000 5,000 1.00 63
15,000 *24,500 28,125 2,730

*Loan amount of Rs. 25,000,000 less processing charges of Rs. 500,000


W3: Average rate of interest for running finance facility
(9,000/60,000) 15%
W4: Interest income from surplus loan amounts:
Rs.000
Interest income Surplus loan Interest
From To Months amounts income at 8%
01-12-15 15-01-13 1.5 24,500 (245)
16-01-16 15-04-13 3.0 7,500 (150)
(395)

7.3 IMRAN LIMITED


(a) Specific borrowings
Rs.
Borrowing costs incurred:
13% bank loan outstanding for 10 months
(Rs. 32 million x 306/365 x 13%) 3,487,562
11% bank loan outstanding for 5 months
(Rs. 10 million x 153/365 x 11%) 461,096
Borrowing costs 3,948,658
Less: Interest that relates to suspension
13% bank loan: (Rs. 32 million x 61/365 x 13%) 695,233
11% bank loan (Rs. 10 million x 61/365 x 11%) 183,836
(879,068)
3,069,590
Less: Investment income on temporary investment of the
borrowings (500,000)
2,569,590

Emile Woolf International 218 The Institute of Chartered Accountants of Pakistan


Answers

(b) General borrowings


Phase 1 Phase 2 Phase 3
Building cost capitalised 20,000,000 18,000,000 16,000,000
Financed out of rights issue (15,000,000)
Financed from borrowing 5,000,000 18,000,000 16,000,000
Period to the year end
March 1 to December 31 306
April 1 to December 31 275
October 1 to December 31 92
Period of suspension (61) (61) 
Number of days for which
borrowing should be
capitalised 245 214 92
Weighted average
borrowing rate (W3) u 12.73% u 12.73% u 12.73%
Fraction of the year for
which the rate should be
applied to costs incurred u 245/365 u 214/365 u 92/365
Capitalised borrowing 427,240 1,343,451 513,385
Total 2,284,076
Workings
W1: Average borrowings Rs. m
13% bank loan outstanding for 10 months
(Rs. 32 million x 306/365 days) 26,827,397
11% bank loan outstanding for 5 months
(Rs. 10 million x 153/365 days) 4,191,781
Average outstanding for the year 31,019,178
W2: Borrowing costs incurred (or from part a) Rs. m
13% bank loan outstanding for 10 months
(Rs. 32 million x 306/365 x 13%) 3,487,562
11% bank loan outstanding for 5 months
(Rs. 10 million x 153/365 x 11%) 461,096
Borrowing costs 3,948,658
W3: Weighted average rate
Borrowing costs
/ Average outstanding for the year =
3,948,658 (W2)
/31,019,178 (W1) = 12.73%
7.4 KATIE
Option 1 Net grants off related expenditure
Statement of financial position as at 30 June Year 2 (extracts)
Rs.
Non-current assets
Property, plant and equipment 223,333
Current liabilities
Other current liabilities 100,000

Emile Woolf International 219 The Institute of Chartered Accountants of Pakistan


Advanced accounting and financial reporting

Notes to the financial statements for the year ended 30 June Year 2
(extracts)
Property, plant and equipment Rs.
Cost (350,000 100,000) 250,000
Accumulated depreciation ((250,000 50,000) 5 u 8/12) (26,667)

Carrying amount 223,333

Included in statement of profit or loss for the year ended 30 June Year 2
Rs.
Depreciation charge 26,667
Training costs (70,000 40,000) 30,000
Option 2 Show grants separately from related expenditure
Statement of financial position as at 30 June Year 2 (extracts)
Rs.
Non-current assets
Property, plant and equipment 310,000

Current liabilities
Other current liabilities 186,667

Notes to the financial statements for the year ended 30 June Year 2
(extracts)
Rs.
Property, plant and equipment
Cost 350,000
Accumulated depreciation ((350,000 50,000) 5 u 8/12) (40,000)

Carrying amount 310,000

Other current liabilities
Deferred income relating to government grants 86,667
(100,000 - (100,000 5 u 8/12))
Government grant repayable 100,000

186,667

Included in statement of profit or loss for the year ended 30 June Year 2
Rs.
Depreciation charge 40,000
Training costs 70,000
Government grant received (40,000)
Release of deferred government grant (13,333)

Tutorial note
The Rs. 100,000 grant in (3) has conditions attached to it. In such a situation, IAS
20 states that grants should not be recognised until there is reasonable
assurance that the entity will comply with any conditions attaching to the grant.
Since Katie is struggling to recruit, and there is only one month left for
recruitment to meet these conditions, then it does not seem that there is
reasonable assurance. Hence the grant should not be recognised as such, but
should be held in current liabilities, pending repayment.

Emile Woolf International 220 The Institute of Chartered Accountants of Pakistan


Answers

7.5 ALNUS LTD AND BUTEA LIMITED


Government grant
The current accounting treatment is not correct. IAS 20 Accounting for
Government Grants and Disclosure of Government Assistance requires
government grants to be recognised in profit or loss over the periods in which the
entity recognises as expenses the costs which the grants are intended to
compensate.
Government grants provided to assist in the acquisition of an asset should be
presented in the statement of financial position either:
1. By setting up the grant as deferred income in the statement of financial
position, and recognising it in profit or loss on a systematic basis over the
useful life of the asset, normally corresponding to the method of
depreciation of the asset.
2. By deducting the grant in arriving at the carrying amount of the asset (i.e.
netting off), thereby reducing the depreciation charge. This method will
make the entity less comparable with a similar entity without government
assistance.
For Alnus Ltd, therefore, the two methods will result in the following presentation
in the statement of financial position and statement of profit or loss:
Statement of financial position
Method 1 Method 2
Rs. Rs.
Asset (400 200) 400,000 200,000
less depreciation ((400/200)/4yrs 9/12) (75,000) (37,500)
325,000 162,500
Liabilities
Current deferred income (12/48 200) 50,000
Non-current deferred income (27/48 200) 112,500
162,500
Statement of profit or loss
Depreciation (75,000) (37,500)
Deferred income (9/48 200) 37,500
The net effect on income and net assets will be the same under either method:
grant income is matched with the use of the asset. The grant recognised in the
year is Rs. 37,500 (Rs. 200,000/4 years 9/12
Capitalised borrowing costs
Under IAS 23 Borrowing Costs certain borrowing costs form part of the cost of a
qualifying asset. A qualifying asset is an asset that takes a substantial period of
time to get ready for its intended use or sale. This includes property, plant and
equipment provided it is not ready for use. The development of the holiday park
is therefore a qualifying asset, as it was in disrepair and not in use.
Borrowing costs are defined as interest and other costs that an entity incurs in
connection with the borrowing of funds. Only borrowing costs that are directly
attributable to the acquisition, construction or production of the qualifying asset
should be capitalised which are those borrowing costs that would have been
avoided if the expenditure on the qualifying asset had not been made. If the
construction is financed out of the general borrowing of the entity, then:

Emile Woolf International 221 The Institute of Chartered Accountants of Pakistan


Advanced accounting and financial reporting

The borrowing costs capitalised should be calculated by reference to the


weighted average cost of the borrowings.
The weighted average calculation should exclude borrowings to finance a
specific purpose or building. In the case of Butea Ltd, although there are
two loans, neither is for a specific purpose, and so the weighted average
cost of the loans should be used to determine the borrowing cost rate for
capitalisation purposes.
The correct calculation of the weighted average borrowing rate is therefore:
((Rs. 1.8m 5%) + (Rs. 1.2m 8.5%))/Rs. 3m = 6.4%
Capitalisation of borrowing costs should commence when the entity meets all
three of the following conditions:
1. It incurs expenditures for the asset (1 January 2017).
2. It incurs borrowing costs (already being incurred).
3. It undertakes activities that are necessary to prepare the asset for its
intended use or sale.
Such activities include obtaining planning permission. (1 November 2016)
January 2017 is the date at which the last of the three conditions was met, so the
period for capitalisation is six months (1 January to 30 June 2017) rather than
nine months.
The amount to be capitalised is therefore: Rs. 800,000 6.4% 6/12 = Rs.
25,600, and an adjustment to correct finance costs capitalised in error is needed
of Rs. 25,400 (51,000 25,600).
As this is a correction in the financial statements of Butea Ltd, the Alnus Ltd
group will need to apportion this between the group (80%) and NCI (20%), so
that group profit will fall by Rs. 20,320 (25,400 80%) and NCI by Rs. 5,080
(25,400 20%).

7.6 VICTORIA
(a) Treatment in the financial statements for the year ended 31 December Year
8 (IAS16)
Property 1
This is used by Victoria as its head office and therefore cannot be treated
as an investment property. It will be stated at cost minus accumulated
depreciation in the statement of financial position. The depreciation for the
year will be charged in the statement of profit or loss.
Property 2
This is held for its investment potential and should be treated as an
investment property. It will be carried at fair value, Victorias policy of choice
for investment properties. It will be revalued to fair value at each year end
and any resultant gain or loss taken to the statement of profit or loss (Rs.
400,000 gain in Year 8).
Property 3
This is held for its investment potential and should be treated as an
investment property. However, since its fair value cannot be arrived at
reliably it will be held at cost minus accumulated depreciation in the

Emile Woolf International 222 The Institute of Chartered Accountants of Pakistan


Answers

statement of financial position. The depreciation for the year will be an


expense in the statement of profit or loss.
This situation provides the exception to the rule whereby all investment
properties must be held under either the fair value model, or the cost
model.
(b) Analysis of property, plant and equipment for the year ended 31 December
Year 8
Other land Investment Investment
and property property
buildings held held at Total
(W1) at fair cost (W2)
value
Rs. Rs. Rs. Rs.
Cost/valuation
On 1 January Year 8 1,000,000 2,300,000 2,000,000 5,300,000
Revaluation - 400,000 - 400,000

On 31 December Year 8 1,000,000 2,700,000 2,000,000 5,700,000

Accumulated
depreciation
On 1 January Year 8 87,500 - 220,000 307,500
Charge for the year (W1) 12,500 - 40,000 52,500

On 31 December Year 8 100,000 - 260,000 360,000

Carrying amount
On 31 December Year 7 912,500 2,300,000 1,780,000 4,992,500

On 31 December Year 8 900,000 2,700,000 1,740,000 5,340,000

Tutorial note
In practice, with a more complex property, plant and equipment table the
investment properties would be included within the land and buildings
column with the required disclosures being given separately in a note to the
table.

Workings
(1) Depreciation on Property 1
Rs.
Brought forward (500,000 40 u 7) 87,500
Year 8 (500,000 40) 12,500
(2) Depreciation on Property 3
Rs.
Brought forward (2,000,000 50 u 5.5) 220,000
Year 8 (2,000,000 50) 40,000

Emile Woolf International 223 The Institute of Chartered Accountants of Pakistan


Advanced accounting and financial reporting

CHAPTER 8 IAS 38: INTANGIBLE ASSETS

8.1 BROOKLYN
1 Development expenditure
IAS 38 on intangibles requires that research and development be
considered separately:
research which must be expensed as incurred
development which must be capitalised where certain criteria are
met.
It must first be clarified how much of the Rs. 3 million incurred to date (10
months at Rs. 300,000) is simply research and how much is development.
The development element will only be capitalised where the IAS 38 criteria
are met. The criteria are listed below together with the extent to which they
appear to be met.
The project must be believed to be technically feasible. This appears
to be so as the feasibility has been acknowledged.
There must be an intention to complete and use/sell the intangible.
Completion is scheduled for June 2017
The entity must be able to use or sell the intangible. Interest has been
expressed in purchasing the knowhow on completion
It must be considered that the asset will generate probable future
benefits. Confirmation is required from Brooklyn as to the extent of
interest shown by the pharmaceutical companies and whether this is
of a sufficient level to generate orders and to cover the deferred
costs.
Availability of adequate financial and technical resources must exist
to complete the project. The financial position of Brooklyn must be
investigated. A grant is being obtained to fund further work and the
terms of the grant, together with any conditions, must be discussed
further.
Able to identify and measure the expenditure incurred. A separate
nominal ledger account has been set up to track the expenditure.
If all of the above criteria are met, then the development element of the Rs.
3m incurred to date must be capitalised as an intangible asset.
Amortisation will not begin until commercial production commences.
2 Provision
Although the claim was made after the reporting period, IAS 10 considers
this to be an adjusting event after the reporting period. The employment of
the individual dates back to 20X2 and so the lawsuit constitutes a current
obligation for the payment of damages as a result of this past event (the
employment).
The amount and the timing are not precisely known but the likelihood of
payment of damages by Brooklyn is probable and so a provision should be
made for the estimated amount of the liability, as advised by the lawyer.
Disclosure, rather than provision, would only be appropriate if the expected
settlement was possible or remote, and the lawyers view is that a payment
is more likely than not.

Emile Woolf International 224 The Institute of Chartered Accountants of Pakistan


Answers

It is not appropriate to calculate an expected value where there is only one


event, instead a provision should be made for the most likely outcome. The
lawyer has various views on the possible, but the most likely payout is Rs.
500,000 as this has a 50% probability. As settlement of the provision is not
anticipated until 2019, the provision should be discounted back at 8% to
give a liability of Rs. 476,280.
Provided that the payment from the insurance company is virtually certain,
this should be shown as an asset, also at its discounted value of Rs.
47,628, being 10% of the provision.
In both cases the discounting should be unwound over the coming three
years through profit or loss.
3 Revaluation
IAS 16 on Property, Plant and Equipment does not impose a frequency for
updating revaluations. It simply requires a revaluation where it is believed
that the fair value of the asset has materially changed. Hence, if in the past
there have been material differences between the carrying amount and fair
value at the 5 yearly review then Brooklyn should consider having more
frequent valuations following on from this years valuation.
Revaluations should be regular and not timed simply when property prices
are at a peak. It is not acceptable for Brooklyn to defer its next revaluation
while values are low. If property prices do fall in 2017, then it may be
necessary to perform an impairment test in accordance with IAS 36
Impairment of assets.
If it is believed that an asset value has moved materially, then all assets in
that class must be revalued. Hence it is not sufficient for Brooklyn to just
revalue the London property.
IAS 16 does not require the valuation to be performed by an external party,
and so the use of the property manager to conduct the valuations is
acceptable. Notes to the financial statements will disclose that he is not
independent of the company.

8.2 RAISIN INTERNATIONAL


(a) Following are the criteria that should be used while recognizing intangible
assets from research and development work.
(i) No intangible asset arising from research shall be recognised.
(ii) An intangible arising from development shall be recognised if, and
only if , an entity can demonstrate all of the following:
the technical feasibility of completing the intangible asset so that
it will be available for use or sale.
its intention to complete the intangible asset and use or sell it.
its ability to use or sell the intangible asset.
how the intangible asset will generate probable future economic
benefits. Among other things, the entity can demonstrate the
existence of a market for the output of the intangible asset or the
intangible asset itself or, if it is to be used internally, the
usefulness of the intangible asset.

Emile Woolf International 225 The Institute of Chartered Accountants of Pakistan


Advanced accounting and financial reporting

the availability of adequate technical, financial and other


resources to complete the development and to use or sell the
intangible asset.
its ability to measure reliably the expenditure attributable to the
intangible asset during its development.
(b) (i) Since the product met all the criteria for the development of the
product, it should be recognised as an intangible in the statement of
financial position (SOFP) of the company. However, RI should
capitalise only the development work (i.e. Rs. 9 million) as intangible
asset. IAS-38 does not allow capitalization of cost relating to the
research work, training of staff and cost of trial run.
Since the product has a useful life of 7 years, the amortization
expense amounting to Rs.0.32 million (Rs. 9 million 3/12 7
years) should be recorded in the statement of profit or loss and
other comprehensive income (SOCI).
(ii) This purchasing of right to manufacture should be recognised as an
intangible in the SOFP because:
it is for an established product which would generate future
economic benefits.
cost of the patent can be measured reliably.
Since there is a finite life, the patent must be amortised over its
useful life. The useful life will be shorter of its actual life (i.e. 10
years) and its legal life (i.e. 5 years. The amortization to be recorded
in SOCI is Rs. 2.83 million (Rs. 17 million 10/12 5).
(iii) The acquired brand should be recognised as an intangible in the
SOFP because acquisition price is a reliable measure of its value.
The amortization to be recorded in SOCI is Rs.0.12 million (Rs. 2
million 10 years x 7/12).
(iv) The carrying value of the intangible asset should be increased to
Rs. 10 million in the SOFP. Since there is an indefinite useful life of
the intangible assets, it should not be amortised. Instead, RI should
test the intangible asset for impairment by comparing its recoverable
amount with its carrying amount. Impairment testing should be done
at least annually

8.3 OXTAIL LTD


(a) Explanation of required IFRS accounting treatment
(1) Research and development costs
In accordance with IAS 38 Intangible Assets the evolution of internally
generated assets (other than goodwill) should be split into the
research phase and the development phase. All expenditure that
arises in the research phase should be recognised as an expense
when it is incurred. Hence, the initial Rs. 70,000 spent on research
should be written off as an expense in 2016 to profit or loss.
Development costs qualify for recognition as an intangible asset
provided that the entity can demonstrate that the strict criteria of IAS
38 are met:

Emile Woolf International 226 The Institute of Chartered Accountants of Pakistan


Answers

It is technically feasible to complete the asset.


The entity intends (and is able to) to complete the asset and
use or sell it.
The asset will generate probable future economic benefits.
Adequate technical, financial and other resources are available
to allow completion/use/sale of the asset.
The development expenditure can be reliably measured.
Once these criteria are met the development expenditure should be
capitalised. The phrase in the question 'would be economically viable'
and the fact that Oxtail Ltd goes on to complete the asset and start
production, are taken to mean that these criteria were met. Hence,
the further costs of Rs. 120,000 incurred after the project review
should be capitalised as an intangible asset, within non-current
assets.
In general, under IAS 38, an intangible asset should only be
recognised in the financial statements if it can be measured reliably
and future economic benefits can be attributed to it. Hence, the
promotional advertising costs of Rs. 15,000 should not be capitalised
as it is difficult to attribute specific future economic benefits to it as
Oxtail Ltd cannot know how successful the advertising per se has
been. Accordingly, IAS 38 states that expenditure on training,
advertising and promotional activities and relocation/reorganisation
should be recognised as an expense when it is incurred. Therefore
only Rs. 105,000 of the Rs. 120,000 spent should be capitalised.
Amortisation of the Rs. 105,000 will commence on 1 January 2017,
when production of the new product commences.
(2) Technical know-how
According to IAS 38 an item is recognised as an intangible asset if it
is identifiable, its cost can be measured reliably, and it is probable
that future economic befits can be attributed to it.
The technical know-how is identifiable as it has been purchased.
Costs appear to have been reliably measured. Given that this know-
how will 'completely change the way (the entitys) manufacturing
process operates' it also appears to be probable that future economic
benefits can be attributed to it.
The costs which should be capitalised are those which can be directly
attributed to creating, producing or preparing the asset for its intended
use. Therefore the purchase cost of Rs. 180,000, legal costs of Rs.
4,000, supervisors time of Rs. 3,200 and testing costs of Rs. 4,800 (a
total of Rs. 192,000) can all be capitalised. The costs of staff training
of Rs. 13,000 (see above) must be expensed as incurred.
Once an intangible asset has been recognised, it should be carried
under the cost model or the revaluation model. Oxtail Ltd use the cost
model so this intangible will be carried at cost less any accumulated
amortisation and any accumulated impairment losses.
An intangible asset with a finite useful life should be amortised over
its expected useful life. The know-how of Rs. 192,000 should
therefore be amortised over the four year 'life' of the new
manufacturing process. Amortisation for the six months to 31

Emile Woolf International 227 The Institute of Chartered Accountants of Pakistan


Advanced accounting and financial reporting

December 2016, will therefore be calculated as Rs. 24,000 (Rs.


192,000/4 x 6/12). This gives the know-how a carrying amount at the
point that an impairment is identified of Rs. 168,000 (Rs. 192,000
Rs. 24,000).
According to IAS 36: Impairment of Assets, if there is an indication of
impairment, the assets carrying amount of Rs. 168,000 should be
compared to its recoverable amount. If the recoverable amount is
lower than the carrying amount then an impairment loss should be
recognised in the statement of profit or loss for the period.
The recoverable amount of an asset is defined by IAS 36 as the
higher of the assets fair value less costs to sell (here Rs. 152,000)
and its value in use (here Rs. 157,000). So the recoverable amount of
the know-how is Rs. 157,000 and the Rs. 168,000 should be written
down to that amount ie by Rs. 11,000 (Rs. 168,000 Rs. 157,000).
The Rs. 11,000 should be recognised as an expense in profit or loss
for the current year.
(b) Summary of costs included in profit or loss for the year ended 31
December 2016
Rs.
Amortisation (see a) 24,000
Impairment of know-how (see a) 11,000
Research costs 70,000
Promotional advertising costs 15,000
Staff training costs 13,000
Statement of financial position as at 31 December 2016 (extract)
Rs.
Non-current assets:
Intangible assets (120,000 15,000 + 157,000 (OF)) 262,000

8.4 SKY LIMITED


(i) Cost incurred in the planning stage should be expensed out as research.
(ii) (a) Cost incurred on development of internal website should be
charged off because the benefits (if any) cannot be estimated
reliably.
(b) Cost of External Website
Cost incurred on development of external website including
the cost of linking it to credit card facilities should be
capitalized because it can be established that external
revenue is generated directly with the use of such website
through external orders.
However, a reasonable estimate of future revenues should
be made for impairment testing.
(iii) (a) Cost of purchase of servers plus cost of their operating software
should be capitalized as tangible assets in line with the
requirements of IAS 16 and depreciated according to their
expected useful economic life.
(b) Cost of purchase of software licenses other than operating
software should be capitalized as intangible assets because

Emile Woolf International 228 The Institute of Chartered Accountants of Pakistan


Answers

economic benefit is accruing to the company.


(iv) Cost of maintenance of websites is a recurring expenditure and should
be expensed out.
(v) IAS-38 does not allow the capitalization of training costs. Therefore,
these must be expensed.
(vi) Cost of advertising should be expensed as and when incurred.

8.5 COMFORT SHOES LIMITED


(a) In accordance with the IAS transactions related to the trademark as given
in the question should be accounted for as explained below:
(i) As the costs and benefits of the trade mark cannot be measured
reliably, and it was not even decided at that time to buy the trademark,
the cost of Rs. 1 million incurred in 2010 to carry out the market
survey should have been expensed in the year 2010.
(ii) In 2011, the rights to use the trademark for the companys products
have been obtained and costs and benefits of the trademark were
measured reliably. Therefore, initially the trademark should have been
accounted for as an intangible asset at a cost of Rs. 5 million.
At that time the trademark was estimated to have an indefinite useful
life as there was an expectation that it will contribute to net cash
inflows indefinitely. Therefore, the trademark should not have been
amortised.
However, the trademark should have been tested for impairment and
the cost should have been reduced, if required.
A trademark fee payable at 1% of annual sales should have been
treated as a periodical cost and charged to expense in the year of
sales.
(b) Comfort Shoes Limited
Notes to the Financial Statements
For the year ended December 31, 2015
1 Intangible Assets Trademark
2016 2015
Rupees in 000
Cost January 1 4,500 5,000
For the year - impairment - (500)
December 31 4,500 4,500
Amortization January 1 - -
For the year 2,250 -
December 31 2,250 -
Carrying
amount December 31 2,250 4,500
% / useful life 50% / 2
years -
1.1 The amortisation expense for the year has been allocated to cost of
sales.

Emile Woolf International 229 The Institute of Chartered Accountants of Pakistan


Advanced accounting and financial reporting

CHAPTER 9 IAS 36: IMPAIRMENT OF ASSETS

9.1 CHARLOTTE
Effect on Year 7 profit or loss
Rs.
Impairment loss
Machine 1 (W1) 122,300
Machine 2 (W2) 41,000
163,300
Depreciation charge
Machine 1: (100,000 5) 20,000

Gain on disposal
Machine 2: (W2) 10,000
Machine 3: (210,000 - 195,000 (W2)) 245,000
255,000
Workings
(1) Machine 1
Rs.
Cost on 1 January Year 1 420,000
Depreciation to 1 January Year 6
5 years u ((420,000 50,000)/10 years)) (185,000)
Carrying amount on 1 January Year 6 235,000
Revalued to: 275,000
Revaluation gain before tax 40,000
In the year to 31 December Year 6 (on 1 January), the asset is revalued
upwards by Rs. 40,000. Of this, Rs. 28,000 is taken to the revaluation
reserve and Rs. 12,000 (Rs. 40,000 u 30%) to deferred tax as a liability.
Dr (Rs.) Cr (Rs.)
Property, plant and equipment 145,000
Accumulated depreciation 185,000
Net effect on non-current assets 40,000
Revaluation surplus 28,000
Deferred tax liability 12,000
The total useful life of the asset was assessed as 15 years on 1 January
Year 6. The asset has already been owned for 5 years and depreciation in
year 6 is based on the remaining useful life of 10 years.
The company must also recognise incremental depreciation in accordance
with section 235 of the Companies Ordinance 1984. An amount equal to
the incremental depreciation net of deferred taxation must be transferred to
retained earnings through the statement of changes in equity.
Dr (Rs.) Cr (Rs.)
Depreciation charge for the year
(275,000/10 years) 27,500
Accumulated depreciation 27,500
Revaluation surplus
(Rs. 28,000/10 years) 2,800
Retained earnings 2,800

Emile Woolf International 230 The Institute of Chartered Accountants of Pakistan


Answers

Impairment loss:
Rs.
Carrying amount on 1 January Year 6 275,000
Depreciation to 1 January Year 7 (275,000 (15 5)) (27,500)
Carrying amount at 1 January Year 7 247,500
Recoverable amount (100,000)
Impairment loss 147,500

In the year to 31 December Year 7, the impairment loss is Rs. 147,500. Of


this, Rs. 40,000 reverses the gain in the previous year. The revaluation
reserve is reduced by Rs. 25,200 (Rs. 28,000 Rs. 2,800). The remaining
impairment loss of Rs. 122,300 is written off as a loss in Year 7.
Also in the year to 31 December Year 7 the asset would be depreciated
based on the estimate of its remaining useful life of 5 years giving a charge
of Rs. 20,000 (Rs. 100,000/ 5 years).
(2) Machine 2
Rs.
Cost on 1 January Year 1 500,000
Depreciation to 1 January Year 7
6 years u ((500,000 60,000)/10 years)) (264,000)
Carrying amount on 1 January Year 7 236,000
Fair value minus cost to sell (200,000 5,000) (195,000)
Impairment loss 41,000

On 31 March Year 7 the machine is sold for Rs. 210,000 giving a gain on
sale as follows:
Rs.
Proceeds 210,000
Selling costs (assumed to be as forecast) (5,000)
205,000
Carrying amount (195,000)
10,000
(3) Machine 3
Rs.
1 January Year 1 Cost 600,000
Depreciation to 1 January Year 2 (30,000)
Carrying amount on 1 January Year 2 570,000
Revalued to 800,000
Taken to revaluation reserve/deferred tax 230,000
The revaluation would have been accounted for as follows at 1 January
Year 2
Dr (Rs.) Cr (Rs.)
Property, plant and equipment 200,000
Accumulated depreciation 30,000
Net effect on non-current assets 230,000
Revaluation surplus 161,000
Deferred tax liability 69,000
Depreciation and incremental depreciation would have been recognised in
Year 2 to Year 6 inclusive as follows:

Emile Woolf International 231 The Institute of Chartered Accountants of Pakistan


Advanced accounting and financial reporting

Dr (Rs.) Cr (Rs.)
Depreciation charge for the year
(800,000/8 years) 100,000
Accumulated depreciation 100,000

Revaluation surplus
(Rs. 161,000/8 years) 20,125
Retained earnings 20,125
This would result in balances for machine 3 and the revaluation surplus in
respect of machine 3 as follows:
Revaluation
Machine 3 surplus
Rs. Rs.
Carrying amount on1 January Year 2 800,000 230,000
Depreciation (5 years) (500,000)
Incremental depreciation (5 years) (100,625)
Balance at 1 January Year 7 300,000 129,375

Fair value on classification as held for sale 550,000


Costs to sell (5,000)
Fair value less costs to sell 545,000
Value at lower of carrying amount and fair
value less costs to sell: 300,000
On 31 March Year 7 the machine is sold for Rs. 550,000 giving a gain on
sale as follows:
Rs.
Proceeds 550,000
Selling costs (assumed to be as forecast) (5,000)
545,000
Carrying amount (300,000)
245,000
The balance on the revalution reserve is transferred to retained earnings on
the disposal of the asset.
Dr (Rs.) Cr (Rs.)
Revaluation surplus 129,375
Retained earnings 129,375

9.2 ABA LIMITED


Aba Limited statement of profit or loss (extracts) year to 31 March 2016
Note: workings in brackets are in Rs.000 Rs. Rs.
Depreciation: head office 6 months to 1 October
2015
(1,200/25 u 6/12) 24,000
6 months to 31 March
2016
(1,350/22.5 (W1) u 6/12) 30,000

54,000

Emile Woolf International 232 The Institute of Chartered Accountants of Pakistan


Answers

Depreciation: training premises


6 months to 1 October
2016
(900/25 u 6/12) 18,000
6 months to 31 March
2016
(600/10 u 6/12) 30,000

48,000

Impairment loss (W2) 210,000

258,000

Statement of financial position (extracts) as at
31 March 2016 Rs. Rs.
Non-current assets
Land and buildings head office (700 + 1,350 30) 2,020,000
training premises (350 + 600
30) 920,000

2,940,000

Revaluation reserve
Head office land (700 500) 200,000
Building (1,350 1,080 (W1)) 270,000
Training premises land (350 300) 50,000

520,000
Transfer to realised profit (270/22.5 (W1) u 6/12
re depreciation of buildings) (6,000)

514,000

Workings
(W1) The date of the revaluation is two and a half years after acquisition. This
means the remaining life of the head office would be 22.5 years. The
carrying value of the head office building at the date of revaluation is Rs.
1,080,000 i.e. its cost less two and a half years at Rs. 48,000 per annum
(Rs. 1,200,000 Rs. 120,000).
(W2) Impairment loss: the carrying value of training premises at date of
revaluation is Rs. 810,000 i.e. its cost less two and a half years at Rs.
36,000 per annum (Rs. 900,000 Rs. 90,000). It is revalued down to Rs.
600,000 giving a loss of Rs. 210,000. As the land and the buildings are
treated as separate assets the gain on the land cannot be used to offset the
loss on the buildings.

Emile Woolf International 233 The Institute of Chartered Accountants of Pakistan


Advanced accounting and financial reporting

9.3 HUSSAIN ASSOCIATES LTD


(a) Impairment of plant
The plant had a carrying amount of Rs. 240,000 on 1 October 2015. The
accident that may have caused impairment occurred on 1 April 2016 and
an impairment test would be done at this date. The depreciation on the
plant from 1 October 2015 to 1 April 2016 would be Rs. 40,000 (640,000 x
121/2% x 6/12) giving a carrying amount of Rs. 200,000 at the date of
impairment. An impairment test requires the plants carrying amount to be
compared with its recoverable amount. The recoverable amount of the
plant is the higher of its value in use of Rs. 150,000 or its fair value less
costs to sell. If Hussain Associates Ltd trades in the plant it would receive
Rs. 180,000 by way of a part exchange, but this is conditional on buying
new plant which Hussain Associates Ltd is reluctant to do. A more realistic
amount of the fair value of the plant is its current disposal value of only Rs.
20,000. Thus the recoverable amount would be its value in use of Rs.
150,000 giving an impairment loss of Rs. 50,000 (Rs. 200,000 Rs.
150,000).
The remaining effect on income would be that a depreciation charge for the
last six months of the year would be required. As the damage has reduced
the remaining life to only two years (from the date of the impairment) the
remaining depreciation would be Rs. 37,500 (Rs. 150,000/ 2 years u
6/12).Thus extracts from the financial statements for the year ended 30
September 2016 would be:
Statement of financial position
Non-current assets Rs.
Plant (150,000 37,500) 112,500

Statement of profit or loss


Plant depreciation (40,000 + 37,500) 77,500
Plant impairment loss 50,000

(b) Purchase of Sparkle

There are a number of issues relating to the carrying amount of the assets
of Sparkle Limited that have to be considered. It appears the value of the
brand is based on the original purchase of the Sparkle Spring brand. The
company no longer uses this brand name; it has been renamed Refresh.
Thus it would appear the purchased brand of Sparkle Spring is now
worthless. Sparkle Limited cannot transfer the value of the old brand to the
new brand, because this would be the recognition of an internally
developed intangible asset and the brand of Refresh does not appear to
meet the recognition criteria in IAS 38. Thus prior to the allocation of the
impairment loss the value of the brand should be written off as it no longer
exists.
The inventories are valued at cost and contain Rs. 2 million worth of old
bottled water (Sparkle Spring) that can be sold, but will have to be
relabelled at a cost of Rs. 250,000. However, as the expected selling price
of these bottles will be Rs. 3 million (Rs. 2 million u 150%), their net
realisable value is Rs. 2,750,000. Thus it is correct to carry them at cost i.e.
they are not impaired. The future expenditure on the plant is a matter for
the following years financial statements.

Emile Woolf International 234 The Institute of Chartered Accountants of Pakistan


Answers

Applying this, the revised carrying amount of the net assets of Sparkle
Limiteds cash-generating unit (CGU) would be Rs. 25 million (Rs. 32
million Rs. 7 million re the brand). The CGU has a recoverable amount of
Rs. 20 million, thus there is an impairment loss of Rs. 5 million. This would
be applied first to goodwill (of which there is none) then to the remaining
assets pro rata. However under IAS2 the inventories should not be reduced
as their net realisable value is in excess of their cost. This would give
revised carrying amounts at 30 September 2016 of:
Rs.000
Brand nil
Land containing spa: 12,000 [(12,000/20,000) u 5,000] 9,000
Purifying and bottling plant:
8,000 [(8,000/20,000) u 5,000] 6,000
Inventories 5,000
20,000

9.4 IMPS
(a) Impairment loss
Rs. m
Carrying value 500
Recoverable amount (385)
Impairment loss 115
Recoverable amount is value in use (Working 1) as this is higher than the
fair value less costs of disposal (Working 2).
Workings
(1) Value in use:
Forecast cash flows discounted at 12%:
Rs. m
Year 1 (185 0.893) 165.2
Year 2 (160 0.797) 127.5
Year 3 (130 0.712) 92.6
Total 385.3
(2) The fair value less costs of disposal:
Rs. m
Goodwill 0
Freehold 270
Freehold land and buildings 50
320

Emile Woolf International 235 The Institute of Chartered Accountants of Pakistan


Advanced accounting and financial reporting

(b) Treatment of impairment loss


IAS 36 requires the impairment loss to be allocated to the various non-
current assets in the following order: firstly, goodwill, secondly, to other
assets, either pro-rata or on another more appropriate basis.
Before Impairment After
impairment loss (W1) impairment
Rs. m Rs. m Rs. m
Goodwill 70 (70) -
Land and buildings 320 (33) 287
Plant and machinery 110 (12) 98
500 (115) 385
Because the land and buildings have been re-valued, the impairment is
treated as a revaluation decrease until the carrying amount of the asset
reaches its depreciated historical cost. The revaluation reserve relating to
the asset is Rs. 65 million and so is adequate to cover the full impairment of
Rs. 33m. The impairment must be separately disclosed and the notes to
the accounts must specify by class of asset the impairment recognised
directly to equity.
The impairment loss on the goodwill and plant (Rs. 82 million) must be
recognised in profit or loss for the year. The notes to the accounts must
specify the line item in which the impairment loss has been included.
Where the impairment write-down is material, information must also be
provided as to the events and circumstances that led to the loss, the nature
of the assets affected, the segment to which the asset belongs, that
recoverable amount was based on value in use and the discount rate used
to calculate this.
Workings
Loss on the various non-current assets
After the impairment loss has been recognised on the goodwill there is still
115 - 70 = 45 loss to be allocated to the other noncurrent assets, on a pro-
rata basis.

Loss on land and buildings:


320
x 45 33
320110

Loss on plant and machinery:


110
x 45 12
320110

Emile Woolf International 236 The Institute of Chartered Accountants of Pakistan


Answers

CHAPTER 10 IFRS 5: NON-CURRENT ASSETS HELD FOR SALE AND


DISCONTINUED OPERATIONS

10.1 SAUL
Statement of profit or loss for the year ended 31 December Year 1
Rs.000
Continuing operations
Revenue 3,315
Cost of sales (2,125)

Gross profit 1,190
Distribution costs (255)
Administrative expenses (680)

Profit before tax 255
Income tax expense (90)

Profit for the period from continuing operations 165
Discontinued operations
Loss for the period from discontinued operations (W) (15)

Profit for the period 150

Statement of financial position as at 31 December Year 1
Rs.000 Rs.000
Assets
Non-current assets
Property, plant and equipment (1,900 510) 1,390
Intangible assets 40

1,430
Current assets
Inventories 350
Trade and other receivables 190
Cash 90

630

2,060
Non-current assets classified as held for sale 450

Total assets 2,510

Equity and liabilities
Equity
Share capital 600
Retained earnings (1,700 60) 1,640

2,240
Current liabilities
Trade and other payables (195 10) 185
Current tax payable 75
Liabilities classified as held for sale 10

270

Total equity and liabilities 2,510

Emile Woolf International 237 The Institute of Chartered Accountants of Pakistan


Advanced accounting and financial reporting

Tutorial note
Division A is classified as discontinued in Year 1 because, although it has not
been sold during the period it meets the IFRS 5 criteria for classification as held
for sale.
Working: Discontinued operation
Continuing Discontinued
operations operations Total
Rs.000 Rs.000 Rs.000
Revenue 3,315 585 3,900
Cost of sales (2,125) (375) (2,500)

Gross profit 1,190 210 1,400
Distribution costs (255) (45) (300)
Administrative expenses (680) (120) (800)
Impairment loss (510 450) - (60) (60)

Profit before tax 255 (15) 240
Income tax expense (90) - (90)

Profit/(loss) for the period 165 (15) 150

10.2 SHAHID HOLDINGS


(a) IFRS 5 Non-current assets held for sale and discontinued operations
defines non-current assets held for sale as those assets (or a group of
assets) whose carrying amounts will be recovered principally through a sale
transaction rather than through continuing use. A discontinued operation is
a component of an entity that has either been disposed of, or is classified
as held for sale and:
(i) represents a separate major line of business or geographical area of
operations
(ii) is part of a single co-ordinated plan to dispose of such, or
(iii) is a subsidiary acquired exclusively for sale.
IFRS 5 says that a component of an entity must have operations and cash
flows that can be clearly distinguished from the rest of the entity and will in
all probability have been a cash-generating unit (or group of such units)
whilst held for use. This definition also means that a discontinued operation
will also fall to be treated as a disposal group as defined in IFRS 5. A
disposal group is a group of assets (possibly with associated liabilities) that
it is intended will be disposed of in a single transaction by sale or otherwise
(closure or abandonment). Assets held for disposal (but not those being
abandoned) must be presented separately (at the lower of cost or fair value
less costs to sell) from other assets and included as current assets (rather
than as non-current assets) and any associated liabilities must be
separately presented under liabilities. The results of a discontinued
operation should be disclosed separately as a single figure (as a minimum)
on the face of the statement of profit or loss with more detailed figures
disclosed either also on the face of the statement of profit or loss or in the
notes.

Emile Woolf International 238 The Institute of Chartered Accountants of Pakistan


Answers

The intention of this requirement is to improve the usefulness of the


financial statements by improving the predictive value of the (historical)
statement of profit or loss. Clearly the results from discontinued operations
should have little impact on future operating results. Thus users can focus
on the continuing activities in any assessment of future income and profit.
(b) The timing of the board meeting and consequent actions and notifications is
within the accounting period ended 31 October 2016. The notification of
staff, suppliers and the press seems to indicate that the sale will be highly
probable and the directors are committed to a plan to sell the assets and
are actively locating a buyer. From the financial and other information given
in the question it appears that the travel agencies operations and cash
flows can be clearly distinguished from its other operations. The assets of
the travel agencies appear to meet the definition of non-current assets held
for sale; however the main difficulty is whether their sale and closure also
represent a discontinued operation. The main issue is with the wording of a
separate major line of business in part (i) of the above definition of a
discontinued operation. The company is still operating in the holiday
business, but only through Internet selling. The selling of holidays through
the Internet compared with through high-street travel agencies requires
very different assets, staff knowledge and training and has a different cost
structure. It could therefore be argued that although the company is still
selling holidays the travel agencies do represent a separate line of
business. If this is the case, it seems the announced closure of the travel
agencies appears to meet the definition of a discontinued operation.
(c) Shahid Holdings statement of profit or loss year ended 31 October:
2016 2015
Rs.000 Rs.000
Continuing operations
Revenue 25,000 22,000
Cost of sales (19,500) (17,000)
 
Gross profit 5,500 5,000
Operating expenses (1,100) (500)
 
Profit/(loss) from continuing operations 4,400 4,500
Discontinued operations
Profit/(loss) from discontinued operations (4,000) 1,500
 
Profit for the period 400 6,000
 
Analysis of discontinued operations
Revenue 14,000 18,000
Cost of sales (16,500) (15,000)
 
Gross profit/(loss) (2,500) 3,000
Operating expenses (1,500) (1,500)
 
Profit/(loss) from discontinued operations (4,000) 1,500
 
Note: other presentations may be acceptable.

Emile Woolf International 239 The Institute of Chartered Accountants of Pakistan


Advanced accounting and financial reporting

10.3 PRIMA
Holiday villas
IAS 16 allows property, plant and equipment to be re-valued or left at historical
cost. Revaluation should be based on the fair value (the open market value in an
arms length transaction). Revaluation is not required every year, but must be
conducted when it is believed that the fair value differs materially from the
carrying value.
The method of accounting for the villa that is to be sold is covered by IFRS 5
which requires that where, at the end of a reporting period, an asset is held for
sale it should be reclassified, re-measured and no longer depreciated. An asset
is only classified as held for sale where the following conditions are all met:
The asset is available for sale in its present condition.
The sale is believed to be highly probable:
Appropriate level of management is committed to the sale;
x There is an active programme underway to find a buyer;
x The asset is marketed at a realistic price.
x Completion of sale expected within 12 months of classification.
From the limited information provided it appears that these conditions have been
met and therefore, under the rules of IFRS 5, the villa should be re-measured to
the lower of its carrying value and its fair value minus costs to sell.
Therefore, the villas should be valued at 31 December Year 4 as follows:
Fair Carrying
value value
Rs. Rs.
All villas 25.00 20.00
Property held for sale (1.00) (1.25)
Properties to be retained 24.00 18.75
The villas to be retained should be re-valued to Rs. 24m, resulting in an increase
in the revaluation reserve of Rs. 5.25m (24-18.75).
The villa to be sold should be written down from its carrying value to its fair value
minus costs to sell of Rs.0.95m (Rs. 1m 50,000). This impairment of Rs.
300,000 (1.25m 0.95m) will be charged against the revaluation reserve for this
asset. If there is insufficient revaluation reserve, then the write down must be
charged to profit or loss.
The villa held for sale must be re-classified from Non-current assets to Current
assets as a separate line item.
Depreciation should not be charged when an asset has been classified as held
for sale. However, the other villas should be depreciated. IAS 16 states that
expenditure on repairs and maintenance does not remove the need to depreciate
an asset. The villas have a finite useful life and therefore must be depreciated. If
the residual value of these assets is greater than the carrying value then the
depreciation charge will be zero. It is not acceptable therefore to have a policy of
non-depreciation on such assets, and a prior year adjustment should be made to
correct the error if the error is material.

Emile Woolf International 240 The Institute of Chartered Accountants of Pakistan


Answers

Head office
The head office should be recorded under property, plant and equipment at cost.
IAS 23 (revised 2009) requires that borrowing costs should be capitalised as part
of the cost of an asset if they are directly attributable to the acquisition,
construction or production of a qualifying asset. A qualifying asset is an asset
that necessarily takes a long period of time to get ready for its intended use or
sale.
In this situation the company is therefore required to capitalise the borrowing
costs as part of the asset cost. Capitalisation must cease when the asset is
substantially complete. Construction finished on 31 May Year 4 and, although
minor modifications continued for a further three months, the standard states that
minor modifications indicate that the asset is substantially complete.
Cost at 30 June Year 4: Rs.000 Rs.000
Land 1,000
Building: Construction cost 8,000
Interest 9% 5million (20/12) years
(1 October Year 2 to 1 June Year 4) 750
8,750
Total 9,750
Prima is to receive a government grant. IAS 20 requires that the grant be
recognised when there is reasonable assurance that the entity will meet any
conditions and receive the grant. As the grant has not been received, a
receivable will be recorded under current assets. The credit can be treated in one
of two ways:
Option 1: Record as deferred income and release to profit or loss over the useful
life of the asset
Option 2: Deduct the grant from the carrying amount of the asset.
If the second option is taken, the asset will be carried at Rs. 8.25m rather than at
Rs. 9.75m. The effect on profit or loss will be the same in both cases.
Land should not normally be depreciated, because land has an indefinite useful
life in most situations. However, as buildings have a limited useful life, a residual
value must be allocated to the building and the depreciable amount must then be
written off over the 50 year useful life. Depreciation will be charged in Year 4 for
the four months from 1 September to 31 December.
The estimates of residual value and useful life must be revised each year and the
depreciation amended prospectively.
Yachts
It is important to note that the yachts are held for rental purposes, so they are
non-current assets, not inventory.
The yachts cost Rs. 20m to build, but the recoverable amount on completion
(higher of value in use and net selling price) is only Rs. 18m, and so the assets
must be initially recognised at their recoverable amount. The impairment write
down of Rs. 2m will be charged to profit or loss in Year 4 in accordance with IAS
36.

Emile Woolf International 241 The Institute of Chartered Accountants of Pakistan


Advanced accounting and financial reporting

Cost Recoverable amount


Rs. m Rs. m
Engines (15%) 3 2.7
Interior (25%) 5 4.5
Remainder (60%) 12 10.8
20 18
IAS 16 requires that each part of the asset that has a cost that is significant in
relation to the total cost must be depreciated separately. Therefore, in the first
year the depreciation charge will be as follows:
Rs. m
Engines Rs. 2.7m 1/3 9/12 = 0.675
Interior Rs. 4.5m 1/2 9/12 = 1.688
Remainder Rs. 10.8m 1/5 9/12 = 1.620
Charge to profit or loss in Year 4 3.983

Emile Woolf International 242 The Institute of Chartered Accountants of Pakistan


Answers

CHAPTER 11 IFRS 16: LEASES

11.1 X LTD
(b)
A B C D E
Period Opening Fin. Charge Rentals Closing
Balance at 15% of B Balance
(B (D - C)
Rs.000 Rs.000 Rs.000 Rs.000
2016 11,420 1,713 4,000 9,133
2017 9,133 1,370 4,000 6,503
2018 6,503 975 4,000 3,478
2019 3,478 522 4,000

4,580 16,000

(c)
Statement of Financial Position (Extract) as at 31 December 2016
Rs.000
Non-Current assets
(Rs. 11,420,000 Rs. 2,855,00) 8,565

Non-Current Liabilities
(Obligation under finance lease) 6,503

Current Liabilities
Obligation under finance lease
(Rs. 9,133,000 Rs. 6,503,000) 2,630

Note: Annual Depreciation


11,420,000
= = Rs. 2,855,000
4

Emile Woolf International 243 The Institute of Chartered Accountants of Pakistan


Advanced accounting and financial reporting

11.2 PROGRESS LTD


(a) Annuity method
Year 1 Year 2 Year 3
Rs. Rs. Rs.
Cash flow 3,200,000 -- --
Outstanding -- 1,920,000 1,350,400
Capital repayment 1,280,000 569,600 637,952
Balance 1,920,000 1,350,400 712,448
Interest @ 12% of balance 230,400 162,048 85,494
Capital repayment 569,600 637,952 714,506
800,000 800,000 800,000
(b) Journal entries
Dr Cr
Rs. Rs.
2016
Jan. 3 Plant & Machinery 3,200,000
Fine Rentals Limited 3,200,000
Initial recognition of machine
Jan. 3 Fine Rentals Limited 1,280,000
Bank 1,280,000
Payment of initial deposit under
lease
Dec. 31 Fine Rentals Limited 569,600
Finance lease interest 230,400
Bank 800,000
Apportionment of annual
installment between Principal
repayment and interest
Dec. 31 Profit and Loss Account 230,400
FL Interest Expense Account 230,400
Write-off of FL interest expense to
Profit and loss account
2017
Dec. 31 Fine Rentals Ltd 637,952
FL interest expense 162,048
Bank 800,000
Apportionment of annual
installment for the year between
Principal repayment and interest
Dec. 31 Profit and Loss Account 162,048
FL Interest Expense Account 162,048
Write-off of FL interest expense to
Profit and loss account

Emile Woolf International 244 The Institute of Chartered Accountants of Pakistan


Answers

Dr Cr
Rs. Rs.
2018
Dec. 31 Fine Rentals Limited 714,506
lease interest 85,494
Bank 800,000
Apportionment of annual
installment for the year between
Principal repayment and interest

Dec. 31 Profit and Loss Account 85,494


FL Interest Expense Account 85,494
Write-off of FL interest expense to
Profit and loss account

11.3 MIRACLETEXTILELIMITED
Miracle Textile Limited
Statement of financial position (extracts) a at 30 June 2016
Note 2016 2015
ASSETS Rs. Rs.
Non-current assets
Property, plant and equipment 4 16,000,000 18,000,000

LIABILITIES
Non-current liabilities
Obligation under lease 9 6,505,219 10,633,074

Current liabilities
Current portion of obligation 9 4,127,856 3,566,925

Miracle Textile Limited


Notes to the financial statements (extracts) for the year ended 30 June 2016
4- Property, plant and equipment 2016 2015
Rs.000 Rs.000
Leased assets
Cost
Opening balance 20,000,000 -
Addition during the year - 20,000,000
20,000,000 20,000,000
Accumulated depreciation
Opening balance (2,000,000) -
Depreciation for the year (2,000,000) (2,000,000)
(4,000,000) (2,000,000)
Balance as at 30 June 16,000,000 18,000,000

Emile Woolf International 245 The Institute of Chartered Accountants of Pakistan


Advanced accounting and financial reporting

9- Obligations under lease (W1)


30-Jun-16 30-Jun-15
Financial Financial
lease charges Principal lease charges Principal
payment for future outstanding payment for future outstanding
periods periods
Rs. Rs. Rs. Rs. Rs. Rs.
Not later
than one
year 5,800,000 1,672,144 4,127,856 5,800,000 2,233,075 3,566,925
Later than
one year but
not later than
five years 7,800,000 1,294,781 6,505,219 13,600,000 2,966,926 10,633,074
Later than
five years - - - - - -
13,600,000 2,966,926 10,633,074 19,400,000 5,200,000 14,200,000

9.1 The Company has entered into a lease agreement with a bank in respect of a
machine. The finance lease liability bears interest at the rate of 15.725879%
per annum. The company has the option to purchase the machine by paying
an amount of Rs. 2 million at the end of the lease term. The lease rentals are
payable in annual instalments ending in June 2016. There are no financial
restrictions in the lease agreement.
W1: Lease Schedule
Payment Opening Principal Interest @ Closing
Instalment
date principal repayment 15.725879% principal
01-Jul-14 20,000,000 5,800,000 5,800,000 - 14,200,000
01-Jul-15 14,200,000 5,800,000 3,566,925 2,233,075 10,633,075
01-Jul-16 10,633,075 5,800,000 4,127,856 1,672,144 6,505,219
01-Jul-17 6,505,219 5,800,000 4,776,997 1,023,003 1,728,222
30-Jun-18 1,728,222 2,000,000 1,728,222 271,778 -
20,000,000 5,200,000

11.4 ACACIA LTD


(a) Relevant extracts
Statements of profit or loss for the year ended 31 March 2016 (extracts)
Rs.
Depreciation (272,850 6) 45,475
lease payments 6,000
Finance costs (W) 19,460
Statement of financial position as at 31 March 2016 (extracts)
Rs.
Non-current assets
Property, plant and equipment (272,850 45,475) 227,375
Non-current liabilities
lease liabilities (Note 1) 135,810
Current liabilities
lease liabilities (Note 1) 78,250

Emile Woolf International 246 The Institute of Chartered Accountants of Pakistan


Answers

Statement of cash flows for the year ended 31 March 2016 (extracts)
Cash flows from financing activities
Payment of lease liabilities (78,250)
Notes to the financial statements (extracts)
(1) Analysis of lease liabilities
Gross basis
Rs.
lease liabilities include the following:
Gross payments due within
One year 78,250
Two to five years (2 78,250) 156,500
234,750
Less: Finance charges allocated to future periods
((78,250 4) 272,850 19,460) (20,690)
214,060

(Alternatively) Net basis


Rs.
lease liabilities include the following:
Amounts due within
One year 78,250
Two to five years 135,810
214,060
(2) Commitments under operating leases
The lease payments under non-cancellable operating leases are as
follows:
Rs.
Within one year 6,000
Within two to five years () 6,000
12,000
WORKING:
Lease of plant
Year to Interest
31 March B/f Payment Capital @ 10% C/f
Rs. Rs. Rs. Rs. Rs.
2016 272,850 (78,250) 194,600 19,460 214,060
2017 214,060 (78,250) 135,810

Emile Woolf International 247 The Institute of Chartered Accountants of Pakistan


Advanced accounting and financial reporting

11.5 SHOAIBLEASINGLIMITED
(a) Entries in the books of Lessor

Date Particulars Dr. Cr.


1-Jul-13 Lease payments receivable (W1) 2,680,000
Machine 2,100,000
Unearned finance income (W1) 580,000

30-Jun-14 Bank 860,000


Lease payments receivable 860,000

30-Jun-14 Unearned finance lease 272,941


Finance income (W2) 272,941

30-Jun-15 Bank 860,000


Lease payments receivable 860,000

30-Jun-15 Unearned finance lease 196,640


Finance income (W2) 196,640

30-Jun-16 Bank 960,000


Lease payments receivable 960,000

30-Jun-16 Unearned finance lease 110,419


Finance income (W2) 110,422

Emile Woolf International 248 The Institute of Chartered Accountants of Pakistan


Answers

W1: Total finance income Rs.


Total future lease payments (Rs. 860,000 x 3) 2,580,000
Add: Purchase bargain option 100,000
Gross investment in finance lease 2,680,000
Less: Cost of assets 2,100,000
Total finance income 580,000

W2: Amortization schedule


Principal Principal
Instalment Interest Principal
Date Opening Closing
Rs.
30-Jun-14 2,100,000 860,000 272,941 587,059 1,512,941
30-Jun-15 1,512,941 860,000 196,640 663,360 849,581
30-Jun-16 849,581 960,000 110,419 849,578 nil
580,000 2,099,997

(b) Shoaib Leasing Limited


Extracts from the statement of financial position as at June 30, 2017
Note 2017
Rs.
Non-current Assets
Net investment in leases 10 849,578

Current Assets
Current portion of net Investment in leases 663,360
10 Net investment in leases

lease payments receivables 10.1 1,720,000


Add: Residual value of leased assets
(part of MLP) 100,000
Gross Investments in leases 1,820,000
Less: Unearned lease income (307,062)
Net investment in leases 10.2 1,512,938
Less: Current portion of net investment in
leases (663,360)
849,578

10.1 lease payments


Less than one year 860,000
More than one year and less than 5
years 960,000
1,820,000

10.2 Net investment in leases


Less than one year 663,360
More than one year and less than 5
years 849,578
1,512,938

Emile Woolf International 249 The Institute of Chartered Accountants of Pakistan


Advanced accounting and financial reporting

11.6 FLOW
Sale and operating leaseback
The nature of the leaseback arrangement affects the treatment of the sale
proceeds. As the arrangement is an operating leaseback, it means that Flow
permanently transfers the risks and rewards in the asset when it makes the sale.
Therefore, on the sale, Flow will de-recognise the asset and the leaseback
payments will be charged as rentals to profit or loss.
If the sale is made at fair value, then the profit on disposal is recognised in profit
or loss immediately. However, the transaction here is significantly above fair
value resulting in an abnormally high profit which is then compensated by higher
operating leaseback rentals. In this situation, IAS 17 requires the excess of
proceeds over fair value (the abnormal profit) to be deferred and amortised over
the period during which the asset is expected to be used. This will serve to offset
and normalise the high future rental charges.
On this basis, the accounting treatment becomes:
Dr Cr
Rs. Rs.
Sale of property
Cash 850,000
Non-current assets 440,000
Profit or loss 110,000
normal profit (550,000 440,000)
Deferred income (liability)
abnormal profit (850,000 550,000) 300,000
Payment of rental
Cash 100,000
Profit or loss rental 100,000
Profit or loss amortised deferred income
(300,000/10 years) 30,000
Deferred income 30,000

11.7 PINUS LIMITED


The length of the lease is the same as the plant and machinerys useful life;
therefore the transaction represents a sale and finance leaseback as it is
essentially a financing arrangement.
The asset should be derecognised, however no profit should be recognised
immediately on the sale but it should be deferred and recognised over the lease
term.
A profit of Rs. 120,000 ((Rs. 1,440,000 840,000) / 5 years) should be released
in the financial year and the remaining Rs. 480,000 should be shown as deferred
income.
The sale proceeds have been incorrectly credited to operating income, and the
operating costs have been incorrectly debited with the lease payment. Both
amounts should be reversed.
A non-current asset of Rs. 1,440,000 should be recognised with a corresponding
liability (see lease working).

Emile Woolf International 250 The Institute of Chartered Accountants of Pakistan


Answers

Depreciation of Rs. 288,000 (Rs. 1,440,000 / 5 years) should be recognised and


the year-end carrying amount should be Rs. 1,152,000.
The release of the deferred income of Rs. 120,000 against the depreciation
results in a net charge of Rs. 168,000 in profit and loss. This is equivalent to the
depreciation that would have been recognised on the original carrying amount of
Rs. 840,000 (ieRs. 840,000 / 5yrs = Rs. 168,000).
Finance interest of Rs. 115,200 (Rs. 1,440,000 8%) should be recognised in
respect of the lease.
The remaining liability of Rs. 1,195,000 (rounded) should be shown as Rs.
931,000 non-current and Rs. 264,000 as current.
The Rs. 360,000 payment should be shown in the statement of cash flows.
Lease working
B/fwd Finance cost at 8% Payment C/fwd
31/3/2016 1,440,000 115,200 (360,000) 1,195,200
31/3/2017 1,195,200 95,616 (360,000) 930,816

11.8 LODHITEXTILEMILLSLIMITED
Particulars Debit Credit
Generator A
(i) Cash / Bank 6,000,000
Accumulated depreciation Generator 2,500,000
Loss on sale/ Impairment loss (this should be
deferred as well just like income) *1,500,000
Property, plant and equipment - Generator 10,000,000
*(This amount comprises of impairment loss
amounted to Rs. 1 million and loss on disposal
amounted to Rs.0.5 million.)
(ii) Assets subject to lease - Generator 6,000,000
Liabilities against assets subject to lease 6,000,000
Generator B
(i) Cash / Bank 6,000,000
Accumulated depreciation Generator 6,000,000
Property, plant and equipment - Generator 12,000,000
(ii) Assets subject to lease - Generator 6,000,000
Liabilities against assets subject to finance
lease 6,000,000
(iii) Impairment loss 1,000,000
Accumulated impairment (ASFL)
Generator 1,000,000
Generator C
(i) Cash / Bank 8,000,000
Accumulated depreciation Generator 3,000,000
Property, plant and equipment - Generator 10,000,000
Deferred income OR Surplus on
revaluation of fixed assets 1,000,000
(ii) Assets subject to lease - Generator 8,000,000
Liabilities against assets subject to finance
lease 8,000,000

Emile Woolf International 251 The Institute of Chartered Accountants of Pakistan


Advanced accounting and financial reporting

11.9 AUTO CONSTRUCTIONPAKISTAN LIMITED


Auto Construction Limited
Notes to the Financial Statements
For the year ended December 31, 2016
1. Liabilities against assets subject to finance lease

Rs. in
'000

Present value of lease payment W1 37,001

Less: Current portion shown under current liabilities W1 (6,572)

30,429

PV of
Future
lease lease
finance
payments liability
cost
2016

Rs. in '000

Not later than one year W2 11,000 952 10,048

Later than one year and not later than


five years W2 38,500 11,547 26,953

49,500 12,499 37,001

The lease payments have been discounted at interest rate of 12.506% per
annum to arrive at the present value.

2. Operating lease rental receivable

Later than
Not later
one year and
than Total
not later than
one year
five years

Rs. in '000

Future lease payment W3 10,250 16,276 26,526

For the construction machinery the company has entered into an operating
lease agreement on July 1, 2016 for 3 years at a half yearly rent of Rs. 5
million, payable in advance with 5% annual increase.

Emile Woolf International 252 The Institute of Chartered Accountants of Pakistan


Answers

W1: Finance lease interest and payment schedule


Finance
Instalment payment Instalment expense Principal Closing
date amount @ recovery balance
12.506%
July 1, 2016
(Total lease amount) 40,000
31-Dec-2016 (Paid) 5,500 2,501 2,999 37,001
30-Jun-2010 5,500 2,314 3,186 33,815
31-Dec-2010 5,500 2,114 3,386 30,429
11,000 4,428 6,572
30-Jun-2011 5,500 1,903 3,597 26,832
31-Dec-2011 5,500 1,678 3,822 23,010
30-Jun-2012 5,500 1,439 4,061 18,949
31-Dec-2012 5,500 1,185 4,315 14,634
30-Jun-2013 5,500 915 4,585 10,049
31-Dec-2013 5,500 627 4,873 5,176
30-Jun-2014 5,500 324 5,176 0
Payable later than
one year and not 38,500 8,071 30,429
later than five years
55,000 15,000 40,000
W2: Present value
Present value Instalment Present Financial
Due date
at 12.506% amount value charges
30-Jun-2010 0.9411 5,500 5,176 324
31-Dec-2010 0.8858 5,500 4,872 628
Not later than 1 year 11,000 10,048 952
31-Dec-2010 0.8336 5,500 4,585 915
30-Jun-2011 0.7846 5,500 4,315 1,185
31-Dec-2011 0.7384 5,500 4,061 1,439
30-Jun-2012 0.6949 5,500 3,822 1,678
31-Dec-2012 0.6541 5,500 3,598 1,902
30-Jun-2013 0.6156 5,500 3,386 2,114
31-Dec-2013 0.5793 5,500 3,186 2,314
Later than 1 year and not later than 5 years 38,500 26,953 11,547
49,500 37,001 12,499
W3: Operating lease payments
Not later than 1 year 01-Jan-10 5,000
01-Jul-10 5,250 10,250
Later than one year and not later than
five years 01-Jan-11 5,250
01-Jul-11 5,513
01-Jan-12 5,513 16,276
26,526

Emile Woolf International 253 The Institute of Chartered Accountants of Pakistan


Advanced accounting and financial reporting

CHAPTER 12 IAS 37: PROVISIONS CONTINGENT LIABILITIES AND


CONTINGENT ASSETS

12.1 ROWSLEY

Introduction
All four scenarios relate to the rules of IAS 37 Provisions, contingent liabilities
and contingent assets. In each scenario, the key issue is whether or not a
provision should be recognised.
Under IAS 37, a provision should only be recognised when three conditions are
met:
there is a present obligation as a result of a past event; and
it is probable that a transfer of economic benefits will be required to settle
the obligation; and
a reliable estimate can be made of the amount of the obligation.
Factory closure
As the factory closure changes the way in which the business is conducted (it
involves the relocation of business activities from one part of the country to
another) it appears to fall within the IAS 37 definition of a restructuring.
The key issue here is whether the group has an obligation at the end of the
reporting period to incur expenditure in connection with the restructuring. There is
clearly no legal obligation, but there may be a constructive obligation. IAS 37
states that a constructive obligation only exists if the group has created valid
expectations in other parties such as employees, customers and suppliers that
the restructuring will actually be carried out. As the group is still in the process of
drawing up a formal plan for the restructuring and no announcements have been
made to any of the parties affected, there cannot be an obligation to restructure.
A board decision alone is not sufficient. Therefore no provision should be made.
If the group starts to implement the restructuring or makes announcements to
those affected after the end of the reporting period but before the accounts are
approved by the directors it may be necessary to disclose the details in the
financial statements as a non-adjusting post event after the reporting period in
accordance with IAS 10. This will be the case if the restructuring is of such
importance that non-disclosure would affect the ability of the users of the financial
statements to reach a proper understanding of the groups financial position.
Operating lease
The lease contract appears to be an onerous contract as defined by IAS 37 as
the unavoidable costs of meeting the obligations under it exceed the economic
benefits expected to be received from it.
Because the enterprise has signed the lease contract there is a clear legal
obligation and the enterprise will have to transfer economic benefits (pay the
lease rentals) in settlement. Therefore, the group should recognise a provision for
the net present value of the remaining lease payments.
In principle, a corresponding asset may be recognised in relation to the future
rentals expected to be received, if these receipts are virtually certain. The current
arrangement with the charity generates only nominal rental income and so the
asset is unlikely to be material enough to warrant recognition. The chances of

Emile Woolf International 254 The Institute of Chartered Accountants of Pakistan


Answers

renting the premises at a commercial rent are less than 50% and so no further
potential rent receivable may be taken into account as the outcome is not virtually
certain and so recognition would not be prudent.
The financial statements should disclose the carrying amount of the onerous
lease provision at the end of the reporting period, a description of the nature of
the obligation and the expected timing of the lease payments. Disclosure should
also be made of the contingent assets where the amount of any expected rentals
receivable from sub-letting are material and the likelihood is believed probable.
Legal proceedings
It is unlikely that the group has a present obligation to compensate the customer;
therefore no provision should be recognised. However, there is a contingent
liability. Unless the possibility of a transfer of economic benefits is remote, the
financial statements should disclose a brief description of the nature of the
contingent liability, an estimate of its financial effect and an indication of the
uncertainties relating to the amount or timing of any outflow.
Environmental damage
It is clear that there is no legal obligation to rectify the damage. However, through
its published policies, the group has created expectations on the part of those
affected that it will take action to do so. There is, therefore, a constructive
obligation to rectify the damage and a transfer of economic benefits is probable.
The group must recognise a provision for the best estimate of the cost. As the
most likely outcome is that more than one attempt at re-planting will be needed,
the full amount of Rs. 30 million should be provided. The expenditure will take
place sometime in the future, and so the provision should be discounted at a pre-
tax rate that reflects current market assessments of the time value of money and
the risks specific to the liability.
The financial statements should disclose the carrying amount at the end of the
reporting period, a description of the nature of the obligation and the expected
timing of the expenditure. The financial statements should also give an indication
of the uncertainties about the amount and timing of the expenditure.

12.2 MULTAN PETROCHEM LTD


(a) Provisions and contingencies
Environmental Legal Onerous Total
damage claims lease
At 1 Jan 2016 1,300,000 80,000 1,380,000
Unwinding of the
discount (8%) 104,000 6,400 110,400
Utilised in the year (15,000) (15,000)
Charge/(credit) to
statement of profit or
loss 1,200,000 (6,000) 1,194,000
At 31 Dec 2016 (W) 1,404,000 1,200,000 65,400 2,669,400
Environmental damage
The provision in respect of the environmental damage relates to restoration
of land following the initial ground work undertaken to set up a new oil
refinery. The company has an advertised policy that it will restore all
environmental damage caused by its business operations. The provision is

Emile Woolf International 255 The Institute of Chartered Accountants of Pakistan


Advanced accounting and financial reporting

based on the estimated cost of reinstating the environmental damage


caused and is not likely to be paid until 2040.
Legal claims
During the year an explosion at one of the companys oil extraction plants
caused a number of employees to suffer injury. This provision is to cover
personal injury claims made by the individuals concerned. The provision is
based on lawyers best estimate of the likely amount at which the claims
can reasonably be settled. It is hoped that the claims will be settled in the
next financial year. It is expected that the full amount of these claims will be
reimbursed by an insurance company following their payment.
Onerous lease
The company has an ongoing lease obligation in respect of office space
that is not being utilised by the company. The outstanding lease liability at
the year-end was Rs. 65,000 and the lease has another four years to run.
MPP has found a tenant for the office space on a six-month short lease and
this will reduce the outstanding obligation by Rs. 6,000 in 2017.
Contingent liability
Following the explosion at the oil extraction plant a number of employees
have made claims against the company for undue stress. Based on
lawyers advice the company do not believe that it is probable that a court
case against the company will be brought. If such a case was to be heard
the estimated payout in total is Rs. 20,000.
Workings
Personal injury claims: 8 150,000 = 1,200,000
Onerous lease: (80,000 15,000) 6,000 = 59,000
(b) Summary of amounts included in income statement for year ended 31
December 2016
Operating costs: Rs.
Movement in provision 394,000
Consultancy fees 12,000
Depreciation on oil refinery environmental damage 52,000
(1,300,000 25yrs)
Borrowing costs
Unwinding of the discount 110,400
Other operating income:
Insurance reimbursement 400,000

12.3 VIOLET POWER LIMITED


Rs. in
million
Assets carrying value as at June 30, 2016 (Asset)
Cost (Given) 6,570
Decommissioning liability on July 1, 2015
(780 / (1+0.08)20) 167
Depreciation for the year (321) Working 1
Adjustment for revision in provision for decommissioning
cost 157 Working 2
6,573

Emile Woolf International 256 The Institute of Chartered Accountants of Pakistan


Answers

Rs. in
million
Decommissioning liability on June 30, 2016 (1,021 /
(1+0.06)19) 337

Working 1: Depreciation for the year (P&L)


Cost 6,570
Decommissioning liability on July 1, 2015 167
Residual value (320)
6,417
Depreciation (6,417 / 20) 321

Working 2: Increase in decommissioning liability


during the year ended June 30, 2016
Decommissioning liability on June 30, 2016 337
Less: Decommissioning liability on July 1, 2015 (167)
Less: Unwinding of interest for the year (167 x 8%) (13)
157

Emile Woolf International 257 The Institute of Chartered Accountants of Pakistan


Advanced accounting and financial reporting

CHAPTER 13: IAS 19: EMPLOYEE BENEFITS

13.1 LABURNUM LIMITED


Calculation of the actuarial gain/losses in year to 31 December 2016
FV of PV of
plan plan
assets liabilities
Rs.000 Rs.000
Opening balance 2,600 2,900
Service cost 450
Interest cost (8% x Rs. 2,900,000) 232
Expected return (8% x Rs. 2,600,000) 208
Past service cost 90
Benefits paid (240) (240)
Contributions 730
3,298 3,432
Actuarial gain on assets 102 -
Actuarial loss on liabilities - 68
Closing balance 3,400 3,500

13.2 JABEL LIMITED


Statement of profit or loss expense Rs.000
Service cost 300
Interest cost (7% x Rs. 1,400,000) 98
Expected return (7% x Rs. 1,200,000) (84)
Net expense 314
The net expense in profit or loss will be Rs. 314,000.
Actuarial gains and losses FV of plan PV of plan
assets liabilities
Rs.000 Rs.000
Opening balance 1,200 1,400
Service cost 300
Interest cost (7% x Rs. 1,400,000) 98
Expected return (7% x Rs. 1,200,000) 84
Benefits paid (220) (220)
Contributions 400
1,464 1,578
Actuarial loss on plan assets (64) -
Actuarial loss on plan liabilities - 22
Closing balance 1,400 1,600
Within other comprehensive income there will be an actuarial loss on plan assets
of Rs. 64,000 and an actuarial loss on plan liabilities of Rs. 22,000.

Emile Woolf International 258 The Institute of Chartered Accountants of Pakistan


Answers

13.3 KAGHZI LIMITED


Calculation of the actuarial gains/losses in the year to 31 December 2016
FV of plan PV of plan
assets liabilities
Rs.000 Rs.000
Opening balance 1,400 1,700
Service cost 320
Interest cost (7% x Rs. 1,700,000) 119
Expected return (7% x Rs. 1,400,000) 98
Benefits paid (170) (170)
Contributions 580
1,908 1,969
Actuarial gain on plan assets 192 -
Actuarial loss on plan liabilities - 431
Closing balance 2,100 2,400

13.4 LASURA LTD


(i) Statement of profit or loss expense
Rs.000
Service cost 500
Interest cost (8% x Rs. 2,400,000) 192
Expected return (8% x Rs. 2,200,000) (176)
Net expense in profit or loss 516
(ii) Other comprehensive income
Rs.000
Actuarial gain on plan assets (W1) (74)
Actuarial loss on plan liabilities (W1) 58
Net actuarial gain in OCI (16)

Working 1 FV of assets PV of
liabilities
Rs.000 Rs.000
Opening balance 2,200 2,400
Service cost 500
Interest cost (8% x Rs. 2,400,000) 192
Expected return (8% x Rs. 2,200,000) 176
Contributions paid in 300
Paid to retired members (450) (450)
2,226 2,642
Actuarial gain on plan assets 74
Actuarial loss on plan liabilities 58
Closing balance 2,300 2,700

Emile Woolf International 259 The Institute of Chartered Accountants of Pakistan


Advanced accounting and financial reporting

13.5 UNIVERSAL SOLUTIONS


(a) (i) Defined benefit pension scheme
The employees of a defined benefit scheme will be guaranteed a
pension based on their final salary and their number of years of
service. Accordingly, the higher paid the employee is on retirement
and the longer the length of service:
the greater the employees pension entitlement and
the greater the liability of the pension fund.
An actuary will advise the company of the cash contributions to be
paid into the plan each year in order to provide the promised
pensions. This is a complicated calculation involving many estimates
such as employee mortality, future increases in salary and expected
future investment returns.
The employer has an open-ended liability to make additional
contributions should there be a deficit in the defined benefit pension
fund. A deficit may arise, for example, if salary levels rise more than
expected or staff turnover reduces, increasing service years.
It will be necessary for the actuary to regularly re-value the pension
funds assets and liabilities to assess the surplus or deficit position
and revise the companys contributions.
(ii) The basis to be adopted in measuring scheme assets
Assets should be measured at their fair value. For quoted securities,
for example, this means their market price.
(iii) The basis to be adopted in measuring scheme liabilities
Liabilities should be measured on an actuarial basis (i.e. discounted
cash flow), using the projected unit method.
The projected unit method is an accrued benefits valuation method in
which the schemes liabilities reflect projected future earnings. To
derive the scheme liabilities, the expected future pension payments
should be discounted at a rate that reflects the time value of money,
for example, using an AA (high quality) corporate bond rate.
(iv) Actuarial gains and losses
Actuarial gains and losses are deficits or surpluses that arise
because:
events have not coincided with the actuarial assumptions made
at the last valuation (experience gains and losses) or
the actuarial assumptions have changed.
For example, if the actuary forecast that investment returns were
going to be 7% in a year, but in fact the return actually achieved was
only 5%, this would give rise to an actuarial deficit.
(b)
(i) Statement of financial position - extract
Year 3 Year 4
Defined benefit net liability 200 300

Emile Woolf International 260 The Institute of Chartered Accountants of Pakistan


Answers

(ii) Journal
Dr Cr
P&L
Interest cost 10
Current service cost 100
110
OCI 130
Cash 140
Defined benefit net liability 100
Workings
Company
Pension fund position
Statement
of financial
Liabilities Assets position
Rs. Rs. Rs.
Opening balance 1 January Year 4 (1,200) 1,000 (200)
Interest cost (5%) (60) 50 (10)
Current service cost (100) (100)
Contributions to the pension fund 140 140
Benefits paid out 95 (95)
Amounts recorded by company (1,265) 1,095 (170)
Actuarial difference (balance) (135) 5 (130)
Closing balance 31 Dec Year 4 (1,400) 1,100 (300)

13.6 DHA INTERIORS LTD


Report to the Directors of DHA Interiors Ltd
Terms of Reference
This report sets out the differences between a defined contribution and defined
benefit plan, and the accounting treatment of the companys pension plans. It
also discusses the principles involved in accounting for warranty claims, and the
accounting treatment of those claims.
(a) Pension plans IAS 19
A defined contribution plan is a pension plan whereby an employer pays
fixed contributions into a separate fund and has no legal or constructive
obligation to pay further contributions (IAS 19 paragraph 7). Payments or
benefits provided to employees may be a simple distribution of total fund
assets, or a third party (an insurance company) may, for example, agree to
provide an agreed level of payments or benefits. Any actuarial and
investment risks of defined contribution plans are assumed by the
employee or the third party. The employer is not required to make up any
shortfall in assets and all plans that are not defined contribution plans are
deemed to be defined benefit plans.

Emile Woolf International 261 The Institute of Chartered Accountants of Pakistan


Advanced accounting and financial reporting

A defined benefit plan is any plan other than a defined contribution plan. It
is the residual category. An employers obligation under a defined benefit
plan is to provide the agreed amount of benefits to current and former
employees. The differentiating factor between defined benefit and defined
contribution schemes is in determining where the risks lie. If an employer
cannot demonstrate that all actuarial and investment risk has been shifted
to another party and that its obligations are limited to contributions made
during the period, then the plan is a defined benefit plan. Any benefit
formula that is not solely based on the amount of contributions, or that
includes a guarantee from the entity or a specified return, means that
elements of risk remain with the employer and must be accounted for as a
defined benefit plan.
For defined contribution plans, the cost recognised in the period is the
contribution payable in exchange for service rendered by employees during
the period. Accounting for a defined contribution plan is straightforward
because the employers obligation for each period is determined by the
amount to be contributed for that period. Often, contributions are a
percentage of employee salary in the period as its base. No actuarial
assumptions are required to measure the obligation or the expense.
The employer should account for the contribution payable at the end of
each period based on employee services rendered during that period,
reduced by any payments made during the period. If the employer has
made payments in excess of those required, the excess is a prepaid
expense to the extent that the excess will lead to a reduction in future
contributions or a cash refund.
For defined benefit plans, the amount recognised in the statement of
financial position is the present value of the defined benefit obligation (that
is, the present value of expected future payments required to settle the
obligation resulting from employee service in the current and prior periods),
as reduced by the fair value of plan assets at the reporting date. If the
balance is an asset, the amount recognised may be limited under IAS 19
Pension Plan 1 is a defined benefit plan as the employer has the
investment risk as the company is guaranteeing a pension based on the
service lives of the employees in the scheme. The employers liability is not
limited to the amount of the contributions. There is a risk that if the
investment returns fall short the employer will have to make good the
shortfall in the scheme. Pension Plan 2 is a defined contribution scheme
because the employers liability is limited to the contributions paid.
(b) Accounting for the two plans
Pension Plan 1
The accounting for the defined benefit plan results in a liability of Rs. 20.5
million as at 31 October 2016, an expense in the statement of profit or loss
of Rs. 20.5 million and a charge in other comprehensive income of Rs. 1.5
million for the year (see Appendix 1).
Pension Plan 2
The company does not recognise any assets or liabilities for the defined
contribution scheme but charges the contributions payable for the period
(Rs. 10 million) to operating profit. The contributions paid by the employees
will be part of the wages and salaries cost and when paid will reduce cash.

Emile Woolf International 262 The Institute of Chartered Accountants of Pakistan


Answers

Appendix 1
The accounting for the defined benefit plan is as follows:
31 October 2016 1 November 2015
Rs. m Rs. m
Present value of obligation 240 200
Fair value of plan assets (225) (190)

Liability recognised 15 10

Expense in Statement of profit or loss year ended 31 October 2016:
Rs. m
Current service cost 20.0
Net interest expense 0.5

Expense 205

Analysis of amount in statement of other comprehensive income (OCI):
Rs. m
Actuarial loss on obligation (w2) 29
Actuarial gain on plan assets (w2) (275)

Actuarial loss on obligation (net) 15

Working 1
Movement in net liability in statement of financial position at
31 October 2016:
Rs. m
Opening liability 10.0
Expense 205
Contributions (17.0)
Actuarial loss 1.5

Closing liability 15.0

Working 2 Change in present value of the obligation and fair value
of plan assets
Fair
value of
PV of plan Net
obligation assets liability
Rs.000 Rs.000 Rs.000
At start of year (200.0) 190.0 (10.0)
Interest expense (5% 200) (10.0) (10.0)
Interest earned (5% 270) 9.5 9.5
Net interest (0.5)
Current service cost (20.0) (20.0)
Contributions paid 17.0 17.0
Benefits paid out (given) 19.0 (19.0) 0
Expected year end position (211.0) 197.5 (13.5)
Remeasurement (balancing figure) (29.0) 27.5 (1.5)
Actual year end position (240.0) 225.0 (15.0)

Emile Woolf International 263 The Institute of Chartered Accountants of Pakistan


Advanced accounting and financial reporting

CHAPTER 14: IFRS 2: SHARE BASED PAYMENTS

14.1 TOSHACK LTD


(i) Liability as at 31 December 2016:
Appreciation in price between 1/1/2013 31/12/2016
Rs. 31 Rs. 22 = Rs. 9
No of employees on which rights vest = 75% u 1000 = 750
No of rights granted per employee = 50
Liability Rs. 9 u 750 u 50 = Rs. 337,500
(ii) Gain between 1/1/2016 31/12/2017
(Rs. 28 Rs. 22) u 750 u 50 = Rs. 225,000
Payment to employees Rs. 112,500
Comment: The transaction would be accounted for as a cash settled share
base payment if the entity has incurred a liability to settle in cash or other
asset.

14.2 IFRS 2
(a) (i) The need for accounting standard regulation
Share options are often granted to employees at an exercise price
that is higher than the market price of the shares. Therefore, the
options have no intrinsic value to the company and, prior to the issue
of IFRS 2, these transactions were not generally recognised until
such time as the shares were issued. This approach could be seen as
resulting in a distortion of reported results between accounting
periods and leaving liabilities unrecorded.
In addition, the subject of accounting for share-based payments
contains a number of other contentious issues, notably relating to the
measurement principles to be applied in recognising the transactions.
If employees agree to stay until their options vest, the organisation
must recognise the service they will provide in return, but how should
this be valued?
IFRS 2 was therefore issued in February 2004 to provide
comprehensive guidance on these matters.
(ii) The three types of share based payments
These can be summarised as follows:
Category Features
Equity-settled share- The entity pays for goods or services by
based payment issuing equity instruments in the form of
transactions shares or share options.
Cash-settled share- The entity incurs a liability for goods or
based payment services and the settlement amount is
transactions based on the price (or value) of the
entitys shares or other equity
instruments.

Emile Woolf International 264 The Institute of Chartered Accountants of Pakistan


Answers

Category Features
Share based payments Transactions where an entity acquires
with cash alternatives goods or receives services and either
the entity or the supplier can choose
payment to be a cash amount based on
the price (or value) of the entitys
shares or other equity instruments, or
equity instruments of the entity.
(b) (i) Assuming all options vest
31 December Year 5 Profit and Equity
loss
Expected outcome
(at grant date value)
500 employees 100 options 750,000
Rs. 15 fair value
3 years to vest 1/3
Year 1 charge 250,000
Balance carried forward 250,000

31 December Year 6 Rs.


Expected outcome (at grant date
value)
500 100 Rs. 15 750,000
2/3
Recognised by the year end 500,000
Minus expense previously recognised (250,000)
Year 2 charge 250,000
Balance carried forward 500,000
31 December Year 7 Rs.
Actual outcome (at grant date value)
500 100 Rs. 15 750,000
Minus expense previously recognised (500,000)
Year 3 charge 250,000
Balance at end of year 3 750,000
(ii) Reflecting revised vesting assumptions
Profit Equity
or loss
31 December Year 5 Rs.
Expected outcome (at grant date
value)
85% 500 100 Rs. 15 637,500
1/3
Year 1 charge 212,500
Balance carried forward 212,500

Emile Woolf International 265 The Institute of Chartered Accountants of Pakistan


Advanced accounting and financial reporting

Profit Equity
or loss
31 December Year 6 Rs.
Expected outcome (at grant date
value)
88% 500 100 Rs. 15 660,000
2/3
440,000
Minus expense previously recognised (212,500)
Year 2 charge 227,500
Balance carried forward 440,000

31 December Year 7 Rs.


Actual outcome (at grant date value)
44,300 Rs. 15 664,500
Minus expense previously recognised (440,000)
Year 3 charge 224,500
Balance at the end 664,500

14.3 SAVAGE LTD


This is an equity-settled share-based transaction and in accordance with IFRS 2
the fair value of the share options is used to estimate the fair value of the
services provided by the employees. The total fair value is allocated over the
three year vesting period and is based on the fair value at the grant date.
2015
Total expected expense:
1,000 options x Rs. 11 x (300 10 30) Rs. 2,860,000
Fraction of vesting period by the year end 1/3
Expense in 2015 Rs. 953,333
Dr Statement of profit or loss Rs. 953,333
Cr Equity Rs. 953,333
2016
Total expected expense:
1,000 options x Rs. 11 x (300 -10 20 15) Rs. 2,805,000
Fraction of vesting period by the year end 2/3
Expense recognised by the year end Rs. 1,870,000
Recognised in 2015 Rs. 953,333
To be recognised in 2016 Rs. 916,667
Dr Statement of profit or loss Rs. 916,667
Cr Equity Rs. 916,667

Emile Woolf International 266 The Institute of Chartered Accountants of Pakistan


Answers

14.4 YORATH LTD


(a) 2016 equity balance required:
Total expected expense (at end of 2016)
500 options x Rs. 148 x 520 (600 20 25 15 20) Rs. 38,480,000
Fraction of vesting period by the year end
Expense recognised by the year end Rs. 28,860,000

Total expected expense (at end of 2015)


500 options x Rs. 148 x 515 (600 20 25 40) Rs. 38,110,000
2
Fraction of vesting period by the year end /4
Expense recognised by the last year end Rs. 19,055,000
To be recognised in 2016 Rs. 9,805,000

Recorded in 2016 financial statements:


Dr Statement of profit or loss staff costs Rs. 9,805,000
Cr Equity other reserves Rs. 9,805,000
(b) The sales director is incorrect, despite no cash changing hands, the share
options are issued in exchange for employees providing services to Yorath
Ltd. Possibly the options have been given as a reward for service provided
or in lieu of a pay rise or bonus which would otherwise have been paid in
cash. As there is no direct wage cost, we instead must calculate an
equivalent cost of receiving staff services and match this with the revenue
that the staff helps to generate. We do this by estimating the value inherent
in the options and allocating that over the period in which employees must
remain with Yorath Ltd, in this case 4 years.
The amount chargeable to the statement of profit or loss is based on the
fair value of the share options at the grant date. This is not subsequently
remeasured as these share options represent an equity-settled share-
based payment. The equivalent cost will be updated each year for those
employees that are still eligible or expected to be eligible at the year end to
ensure that the amount charged reflects the amount that is expected to
vest.

14.5 QUALTECH LTD


(i) Share-based payment
Total expected expense (at end of 2016)
1,000 options x Rs. 122 x 240 (300 25 15 20) Rs. 29,280,000
2
Fraction of vesting period by the year end /3
Expense recognised by the year end Rs. 19,520,000

Total expected expense (at end of 2015)


1,000 options x Rs. 122 x 235 (300 25 40) Rs. 28,670,000
1
Fraction of vesting period by the year end /3
Expense recognised by the last year end Rs. 9,556,667
To be recognised in 2016 Rs. 9,963,333

Emile Woolf International 267 The Institute of Chartered Accountants of Pakistan


Advanced accounting and financial reporting

Double entry in 2016:


Dr Statement of profit or loss staff costs Rs. 9,963,333
Cr Other reserves (equity) Rs. 9,963,333
Being the charge for share-based payment for the year ended 31
December 2016
(ii) Share-based payments that are to be settled in cash would be credited
instead to liabilities in the statement of financial position and the liability
would be remeasured using the fair value of the shares at each year-end
date until the end of the vesting period.

14.6 BRIDGE LTD


(i) Statement of profit or loss charge
Total expected expense (at end of 2016)
1,000 options x Rs. 50 x 213 (300 20 23 44) Rs. 10,650,000
2
Fraction of vesting period by the year end /4
Expense recognised by the year end Rs. 5,325,000

Total expected expense (at end of 2015)


1,000 options x Rs. 50 x 215 (300 20 65) Rs. 10,750,000

Fraction of vesting period by the year end
Expense recognised by the last year end Rs. 2,687,500
To be recognised in 2016 Rs. 2,637,500
Double entry in 2016:
Dr Statement of profit or loss staff costs Rs. 2,637,500
Cr Other reserves (equity) Rs. 2,637,500
Being the charge for share-based payment for the year ended 31
December 2016
(ii) Share options, such as those granted by Bridge Ltd, are given by an entity
in return for services provided by its employees. In effect the share options
are given to the employees as a form of bonus or reward for these services
and are therefore part of the employees remuneration package. The
value of these options (or relevant part thereof) must then be reflected in
the staff costs included within the statement of profit or loss.

14.7 CAPSTAN LTD


SARs are an example of a cash-settled share-based transaction and, in
accordance with IFRS 2 Share-based payments, are initially measured at fair
value at the grant date and subsequently remeasured to fair value at each year-
end. The liability is remeasured and any difference is charged to the statement
of profit or loss as an expense.
2015
Total expected expense (at end of 2015)
1,000 SARs x Rs. 80 x 233 (300 32 35) Rs. 18,640,000
1
Fraction of vesting period by the year end /3
Liability to be recognised by the year end Rs. 6,213,333

Emile Woolf International 268 The Institute of Chartered Accountants of Pakistan


Answers

Double entry in 2015:


Dr Statement of profit or loss staff costs Rs. 6,213,333
Cr Liability Rs. 6,213,333
2016
Total expected expense (at end of 2016)
1,000 SARs x Rs. 120 x 230 (300 32 28 10) Rs. 27,600,000
2
Fraction of vesting period by the year end /3
Liability to be recognised by the year end Rs. 18,400,000
Less opening liability (see above) (Rs. 6,213,333)
To be recognised in 2016 Rs. 12,186,667

Double entry in 2016:


Dr Statement of profit or loss staff costs Rs. 12,186,667
Cr Liability Rs. 12,186,667

14.8 Newtown Ltd


(i) 2015
Total expected expense (at end of 2016)
1,000 SARs x Rs. 110 x 405 (500 42 28 25) Rs. 44,550,000
2
Fraction of vesting period by the year end /3
Liability to be recognised by the year end Rs. 29,700,000

Less opening liability:


Total expected expense (at end of 2015)
1,000 SARs x Rs. 90 x 383 (500 42 75) Rs. 34,470,000
1
Fraction of vesting period by the year end /3
Liability recognised by the end of 2015 Rs. 11,490,000
To be recognised in 2016 Rs. 18,210,000
Double entry in 2016:
Dr Statement of profit or loss staff costs Rs. 18,210,000
Cr Liability Rs. 18,210,000

(ii) If a share-based payment was settled in equity rather than cash the
implications would be:
Recognition:
There would be a credit to other reserves within equity in the statement of
financial position, rather than a liability. However the debit would still be to
staff costs.
Measurement:
The amount would be initially and subsequently measured using the fair
value of the rights at the grant date rather than re-measured at each year
end.

Emile Woolf International 269 The Institute of Chartered Accountants of Pakistan


Advanced accounting and financial reporting

14.9 SINDH TRANSIT LTD


(a) Accounting entries
Accounting entries for year ended 31 December 2016:
Share options
Dr Staff costs (statement of profit or loss) (W1)Rs. 17,820,000
Cr Equity Rs. 17,820,000
Share appreciation rights (SARs)
Dr Staff costs (statement of profit or loss) (W2)Rs. 6,495,000
Cr Liabilities (non-current) Rs. 6,495,000
Working 1: Options
Total expected expense (at end of 2016)
1,000 options x Rs. 220 x 327 (400 15 22 36) Rs. 71,940,000
2
Fraction of vesting period by the year end /4
Expense recognised by the year end Rs. 35,970,000
Total expected expense (at end of 2015)
1,000 options x Rs. 220 x 330 (400 15 55) Rs. 72,600,000
1
Fraction of vesting period by the year end /4
Expense recognised by the last year end Rs. 18,150,000
To be recognised in 2016 Rs. 17,820,000
Working 2: SARs
Total expected expense (at end of 2016)
500 SARs x Rs. 140 x 327 (400 15 22 36) Rs. 22,890,000
2
Fraction of vesting period by the year end /4
Liability to be recognised by the year end Rs. 11,445,000
Less opening liability:
Total expected expense (at end of 2015)
500 SARs x Rs. 120 x 330 (400 15 55) Rs. 19,800,000
1
Fraction of vesting period by the year end /4
Liability recognised by the end of 2015 Rs. 4,950,000
To be recognised in 2016 Rs. 6,495,000
(b) In accordance with IFRS 2, the share options and the share appreciation
rights are recognised as an expense in the statement of profit or loss as
they are awarded in return for employee service.
The treatment of each however is different in the statement of financial
position. The share appreciation rights will result in a future outflow of cash
and therefore represent an obligation and are presented as a liability. The
liability should reflect the most reliable measurement at each balance sheet
date and so the total amount payable that is estimated at each year-end
date is estimated using the updated fair values.
The options represent an equity-settled share-based payment and do not
meet the definition of obligation, and so instead the entry is to equity. The
equity element is measured initially and subsequently at the fair value at
the grant date.

Emile Woolf International 270 The Institute of Chartered Accountants of Pakistan


Answers

CHAPTER 15: FINANCIAL INSTRUMENTS: RECOGNITION AND


MEASUREMENT

15.1 AJI PANCA LTD


Capital and reserves Rs.
Share capital (Rs. 1 ordinary shares) (W2) 1,625,000
Share premium (W3) -
Retained earnings 6,116,812
7,741,812
Liabilities (W5) 164,751
Workings
(1) Profit for the year
Rs.
Original 508,500
Minus: Finance charges (W5) (14,988)
493,512
(2) Ordinary share capital
Rs.
At 1 January 1,000,000
Issue at full price on 31 March 300,000
1,300,000
Bonus issue on 30 June (1,300,000 4) 325,000
1,625,000
(3) Share premium
Rs.
At 1 January 200,000
Issue at full price on 31 March ((300,000 u 0.30) 20,000) 70,000
270,000
Bonus issue on 30 June (270,000)
NIL
(4) Retained earnings
Rs.
At 1 January 5,670,300
Minus: Bonus issue on 30 June (325,000 (W2) 270,000
(W3) (55,000)
Add: Profit for the year (W1) 493,512
Add back: Preference dividends charged to retained
earnings (W5) 8,000
6,116,812

Emile Woolf International 271 The Institute of Chartered Accountants of Pakistan


Advanced accounting and financial reporting

(5) Redeemable preference shares


Rs.
Liability at beginning of year
Year 1 ((100,000 u Rs. 1.60) 2,237)) 157,763
Finance charge at 9.5% 14,988
Interest paid at 4% (8,000)
Liability at end of year 164,751

15.2 PASSILA LTD


(a) The face value of the debentures
Rs. 100 X 20,000 = Rs. 2,000,000
The amount accrued to the company as proceeds =
Rs. 97.5 X 20,000 = Rs. 1,950,000
(b) The difference between the face value and the market value of the
debentures is Rs. 50,000. This is as a result of discount allowed on the
issue on the debentures. Discount on debentures attracts investors.
(c) Nominal interest rate is the rate based specifically on the face value of the
loan capital. In case of Passila Ltd., the nominal interest rate on the
debentures is 8% per annum on Rs. 2,000,000.
The effective interest is the rate based on the market value. This is the
actual value collected on issue which can be at par, discount or premium.
For Passila Ltd., the effective interest rate will be 8% of Rs. 1,950,000
(d) The nominal interest payable
Rs. 2,000,000 X 8% X 6 months 12 months
= Rs. 80,000
(e) (i) The face value of Rs. 2,000,000 will be the most appropriate
valuation
to be disclosed in the Statement of financial position. The
management may be interested in the quoted market value or the
proceeds, but for the sake of outside investors who would only be
interested in the company having good reputations devoid of trading
losses, it is advisable that the face value be adopted.
(ii) Disclosing the debentures liability at face value plus interest payment
for five years may seem proper in the eyes of external investors and
credit institutions, but principally, it would be wrong to credit
debentures account with both the face value and the interest
payments. An interest payment on debentures is a revenue item
which is debited to the Profit and Loss Account.
(iii) Disclosing debentures liability at market value on the Statement of
financial position will amount to disclosure at replacement value. The
market value should be disclosed.

Emile Woolf International 272 The Institute of Chartered Accountants of Pakistan


Answers

15.3 FINANCIAL INSTRUMENTS


(a) IFRS 9 requires that all financial assets and financial liabilities are
recognised at fair value at initial recognition. Separate guidance is given for
the treatment of transaction costs.
When a financial instrument is acquired or issued it must be classified into
one of two asset categories or one of two liability categories. This
classification then determines the subsequent accounting treatment for the
item. The two categories are at amortised cost and at fair value through
profit or loss.
Amortised cost
Amortised cost is calculated as:
Initial cost recognised
Plus: Interest at the effective rate
Minus: Cash received/paid
Fair value
Fair value is defined as the price that would be received to sell an asset or
paid to transfer a liability in an orderly transaction between market
participants at the measurement date.
Financial assets
A financial asset must be measured at amortised cost if both of the
following conditions are met:
the asset is held within a business model whose objective is to hold
assets in order to collect contractual cash flows; and
the contractual terms of the financial asset give rise on specified
dates to cash flows that are solely payments of principal and interest
on the principal amount outstanding.
Any asset which is not measured at amortised cost must be measured at
fair value. Any fair value movement on these financial assets is reported in
profit or loss.
In addition an entity is allowed to designate a financial asset that otherwise
meets the amortised cost criteria as FVTPL if it meets certain conditions.
Furthermore an entity is allowed to make an irrevocable decision on initial
recognition of equity that any fair value difference on that equity be
recognised in OCI rather than P&L
Financial liabilities
A financial liability must be measured at amortised cost with specific
exceptions including:
Derivatives that are liabilities at the reporting date; and
Financial liabilities that might arise when a financial asset is
transferred but this transfer does not satisfy the derecognition criteria.
.
A company is allowed to designate a financial liability as measured at fair
value through profit or loss. This designation can only be made if:
it eliminates or significantly reduces a measurement or recognition
inconsistency; or

Emile Woolf International 273 The Institute of Chartered Accountants of Pakistan


Advanced accounting and financial reporting

this would allow the company to reflect a documented risk


management strategy.
Any such designation is irrevocable.
(b) (i) 3% Bond
The bond must initially be recorded at its purchase price of Rs.
250,000. The bond seems to satisfy the amortised cost criteria. The
company seem to operate a business model whose objective is to
hold financial assets in order to collect contractual cash flows and it
seems that the cash it will collect will be solely payment of interest
and principle. The market value is not relevant.
Interest will be credited to profit or loss using the effective interest
rate, resulting in finance income of Rs. 24,250 (9.7% 250,000). The
effective rate reflects the total return received by the investor over the
duration of the bond being the coupon + Rs. 50,000 premium on
redemption. The coupon recorded in the statement of cash flows is
Rs. 9,000 (3% 300,000).
The difference between the effective interest and the actual coupon is
added to the investment to give an amortised cost at the end of Year
3 of Rs. 265,250 (250,000 + 24,250 9,000).

(ii) Equity shares in XYZ


The shares must be classified as FVTPL. They will initially be
recorded at their cost of Rs. 30,000. As they have been classed as
fair value through profit or loss the transaction costs must be
expensed to profit or loss immediately. At the end of each reporting
period, the shares must be re-measured to their market value, with
the resulting gain or loss being taken to profit or loss.
At 1 January Year 3, the investment has a carrying value of Rs.
34,000. By the 31 December Year 3 this value is now Rs. 35,000. A
Rs. 1,000 gain will therefore be recognised in profit or loss for the
year.

(iii) Convertible bond


A convertible bond is a compound instrument. In essence, issuing a
convertible bond is equivalent to issuing a non-convertible bond plus
a call option on the entitys shares. Therefore, the bond should be
divided into a liability portion and an equity portion in accordance with
the rules in IAS 32.
Note that the investor would have to do something similar
To establish the liability (debt) element, the future cash flows from the
bond are discounted at the normal market rate to establish the value
of an equivalent but redeemable bond.
Using a rate of 7% this gives a net present value of:
20,000 20,000 520,000
  $460,635
1.07 1.072 1.073
As the bond was issued for Rs. 500,000, it implies that the call option
embedded within the bond was sold for Rs. 39,365 (Rs. 500,000
460,635).

Emile Woolf International 274 The Institute of Chartered Accountants of Pakistan


Answers

The liability component is measured at amortised cost after initial


recognition. Interest will be recognised at the effective rate of 7%. The
difference between the cash interest paid (4%) and the interest
expense recognised will increase the amortised cost of the liability
year on year until the bond is redeemed.

15.4 ESPANOLA LTD


(a) (i) The held to maturity investment will be initially recorded at fair value
plus transaction costs. It will be subsequently measured at each year-
end at amortised cost using the effective interest rate.
(ii) Held to maturity investment -amortised cost using effective interest
rate of 7.05%.
Year Opening Effective interest Interest Closing
end balance 7.05% received balance
Rs. Rs. Rs. Rs.
2016 3,200,000 225,600 (180,000) 3,245,600
Investment income - Income from HTM investment Rs. 225,600
Non-current assets - Held to maturity investment Rs. 3,245,600
(b) Held for trading investment
Initial recording:
Dr Current asset investment Rs. 300,000
Cr Bank Rs. 300,000
Being the purchase of shares
Dr Statement of profit or loss Rs. 12,000
Cr Bank Rs. 12,000
Being the write off of the transaction costs to the statement of profit or loss
as the investment is an asset held at fair value through profit or loss
Subsequent measurement
Dr Current asset investment Rs. 40,000
Cr Statement of profit or loss gain Rs. 40,000
Being the uplift in value and the recording of the gain in the statement of
profit or loss

15.5 SANDIA LTD


(i) Available for sale (AFS) Investment initially recorded at fair value plus
transactions costs:
Dr AFS Investment (40,000 shares x Rs. 2.68) Rs. 107,200
Cr Bank Rs. 107,200
Being initial recognition of AFS asset
Dr AFS Investment Rs. 5,360
Cr Bank Rs. 5,360
Being 5% commission paid on purchase

Emile Woolf International 275 The Institute of Chartered Accountants of Pakistan


Advanced accounting and financial reporting

The investment is subsequently measured at the fair value of the shares


with the gain or loss calculated as fair value of the investment less its
carrying amount.
This is a valuation exercise, not a transaction, so there is no need to
account for commission when calculating the year end valuation [(40,000 x
Rs. 2.96) - Rs. 112,560].
Dr AFS Investment Rs. 5,840
Cr Equity other reserves Rs. 5,840
Being subsequent measurement of AFS asset
(ii) In accordance with IAS 39, all derivative contracts are classified as fair
value through profit and loss, therefore any gain or loss in the value of the
derivative contract is taken directly to the statement of profit or loss. Gains
or losses on available for sale investments are normally recorded through
other comprehensive income. However, as hedge accounting can be
applied (because it has been designated as a hedge) then the gain/loss on
both the investment (hedged item) and the derivative contract (hedging
instrument) can be offset within the statement of profit or loss. Hedge
accounting (for this fair value hedge) ensures that the gain/loss on the AFS
investment is taken to profit or loss and matched against the gain/loss on
the hedging instrument.

15.6 GEO ALLOYS LTD


(a) (i) The investment should be classified as held to maturity investment
because GEO Alloys Ltd intends to hold the investment until its
redemption date. Initially the investment will be measured at its fair
value (which in this case is its cost), plus any associated issue costs.
The initial journal entry required is therefore:
DR: Investment in HTM investment Rs. 4,200,000
CR: Cash Rs. 4,200,000
(ii) Subsequent measurement
Year end Opening Effective interest Interest Closing
30 June balance 8.4% received balance
7% x Rs.
4m
Rs.000 Rs.000 Rs.000 Rs.000
2015 4,200 353 (280) 4,273
2016 4,273 359 (280) 4,352
The investment will be held at Rs. 4,352,000 in the statement of
financial position at 30 June 2016.
(b) This forward contract is an example of a derivative and in accordance with
IAS 39 such derivative contracts are classified as an asset or liability held
at fair value through profit and loss. This would mean that at each year end
the contract would need to be re-valued to its fair value (being the
difference between the derivative price and the market price of the
underlying asset under the contract). Any gains or losses would usually
then be recorded in profit or loss.

Emile Woolf International 276 The Institute of Chartered Accountants of Pakistan


Answers

However this contract is specifically intended to mitigate the risk of future


adverse cash flows as a result of potential increases in raw material prices.
This contract is therefore being used as a cash flow hedge (because its
being used to fix the price of material to be acquired in the future on 1
August). In such circumstances IAS 39 has some special hedge accounting
rules. Hedge accounting allows the gains or losses on a derivative contract
being used in a cash flow hedge to be taken to reserves until the cash flow
it is designed to hedge against is recognised in the financial statements. In
this case, the gains or losses will be held in reserves until the year ended
30 June 2017 which is the year in which the cash flow is actually incurred,
and then released to the profit or loss.
In this case, a loss on the derivative of Rs. 40,000,000 (100,000 x
Rs.(10,500  10,100)) will be included in reserves at 30 June 2016.

15.7 CASCABEL LTD


(a) At 31 July 2016 this instrument meets the definition of a derivative:
Small or no initial investment.
Its value is dependent on an underlying economic item; exchange rate.
Its settlement will take place at some future date.
As a derivative it should be accounted for as an asset or liability held at fair
value through profit or loss. The value of the derivative instrument will be
the difference between the value of the contract when settled compared
with the cost of A$2m being purchased at the spot rate at the year-end
date.
Cost of A$2m at a contracted rate of A$0.64 = Rs. 3,125,000
Cost of A$2m at the forward rate of A$0.70 = Rs. 2,857,143
The derivative results in a liability at the year-end date of Rs. 267,857 (Rs.
3,125,000 - Rs. 2,857,143) as the contract has unfavourable terms when
compared to the spot rate. The loss on derivative would be charged to the
statement of profit or loss in the year to 31 August 2016.
Recorded as:
Dr Statement of profit or loss (loss on derivative) Rs. 267,857
Cr Liabilities derivatives Rs. 267,857
(b) If the derivative was designated as a hedging instrument in a cash flow
hedge then the loss of Rs. 267,857 would be recognised in other
comprehensive income until the related cash flow (hedged item) occurred,
and shown as a loss in other comprehensive income in the year ended 31
August 2016. This ensures that the movements in the hedged item and the
hedging item can be offset in the same accounting period.

Emile Woolf International 277 The Institute of Chartered Accountants of Pakistan


Advanced accounting and financial reporting

15.8 FAIR VALUE HEDGE ACCOUNTING


(a) Journals at 31 December 2015
Debit Credit
Forward contract asset 95,000
P&L account fair value gain 95,000

Being mark-to-market for the derivative

Debit Credit
P&L account fair value loss on inventory 100,000
Inventory 100,000

Being fair value loss on inventory, attributable to the risk being hedged

(b) Journals at 31 March 2016


Debit Credit
Forward contract asset (142000 95000) 47,000
P&L account fair value gain 47,000

Being mark-to-market for the derivative

Debit Credit
P&L account further fair value loss on inventory 50,000
Inventory 50,000

Being fair value loss on inventory, attributable to the risk being hedged

Debit Credit
Bank 1,150,000
P&L account 1,150,000

Being sales proceeds

Debit Credit
Bank 142,000
Forward contract 142,000

Being forward contract closed out for cash

Debit Credit
P&L account cost of sales 850,000
Inventory 850,000

Being inventory carrying value now derecognised upon sale

Emile Woolf International 278 The Institute of Chartered Accountants of Pakistan


Answers

The results of these journals can be summarised as:


Debit/(Credit)
Cash Derivative Inventory P&L
Inventory brought
forward 1,000,000
December:
Change in fair value
(FV) of forward 95,000 (95,000)
Change in FV of
inventory (100,000) 100,000
March:
Change in FV of
forward 47,000 (47,000)
Change in FV of
inventory (50,000) 50,000
Revenue 1,150,000 (1,150,000)
Close out derivative 142,000 (142,000)
Cost of sale (850,000) 850,000
TOTALS 1,292,000 NIL NIL 292,000

15.9 CASH FLOW HEDGE ACCOUNTING


Fair value of the derivative contract at 31 December:
31 Dec 28 Feb
The cash flow under the contract will be Rs. Rs.
400,000 * 0.7 = 280,000 280,000
The cash flow available in the market is Rs.
400,000 * 0.75 = 300,000
The cash flow available in the market is Rs.
400,000 * 0.80 = 320,000
Therefore the fair value of the derivative (an asset)
is Rs. 20,000 Rs. 40,000

(a) Journals at 31 December


Debit Credit
Forward contract 20,000
Equity cash flow hedge reserve 20,000
Being fair value change, deferred to equity as an effective cash flow hedge

(b) Journals at 28 February


Debit Credit
Forward contract 20,000
Equity cash flow hedge reserve 20,000
Being fair value change January and February 2016

Emile Woolf International 279 The Institute of Chartered Accountants of Pakistan


Advanced accounting and financial reporting

Debit Credit
Bank 40,000
Forward contract 40,000
Contract closed with payment from the FX dealer of 400,000 * (0.70-0.80) = Rs.
40,000

Debit Credit
Property, Plant and Equipment 320,000
Bank 320,000
Being initial recognition of purchase price of machine: 400,000 * 0.80 = Rs.
320,000

Debit Credit
Equity cash flow hedge reserve 40,000
Property, Plant and Equipment 40,000
Being transfer of deferred gains/losses on closure of a cash flow hedge
The machine will therefore be recorded at 320,000 40,000 = Rs. 280,000

The results of these journals can be summarised as:


Debit/(Credit)
Cash Derivative Equity PP&E
December:
Change in (FV) of forward 20,000 (20,000)
February:
Change in FV of forward 20,000 (20,000)
Purchase of drilling rig (320,000) 320,000
Basis adjustment 40,000 (40,000)
Close out of derivative 40,000 (40,000)
TOTALS (280,000) - - 280,000

15.10 WATERS LTD


1 Investment in 7% treasury stock 2022
As there is not an intention to hold the investment to maturity, the
investment should be classified as at fair value (with gains and losses
recognised in profit or loss).
At initial recognition it will be measured at fair value which is the
consideration given of Rs. 208,200.
There is no interest received up to year end (first payment will be received
on 31 October 2017)
The market value of the stocks at the reporting date is Rs. 196,140 and the
revaluation loss of Rs. 12,060 will be recognised in profit or loss.

Emile Woolf International 280 The Institute of Chartered Accountants of Pakistan


Answers

Alternative:
Waters Ltd could choose to recognise finance income in the statement of
profit or loss at 6.3% u 208,200 u 4/12 = Rs. 4,372.
The financial asset will then have a carrying amount of Rs. 212,572
(208,200 + 4,372) prior to remeasuring to the fair value. The market value
of the stocks at the reporting date is Rs. 196,140 and the revaluation loss of
Rs. 16,432 will be recognised in profit or loss.
Notice that the overall profit impact is the same in each case as *4,372 +
(16,432) = 12,060.
2 Futures Prif contract
The derivative will be classified as at fair value through profit or loss.
Initial transaction costs cannot be included as part of the carrying amount
and therefore the fee of Rs. 750 will be immediately charged to profit or
loss.
At the reporting date the contract is valued at the fair value of PR1.99/Rs. 1
so the loss is Rs. 1,269 to be included in profit or loss and as a liability on
the statement of financial position
3 Investment in Gilmour Ltd
This would normally be classified as at fair value (with gains and losses
recognised in profit or loss).
On initial recognition it would be valued at fair value which would be the
cost of Rs. 1,212,500. The directly attributable transaction costs (Rs.
35,000) would be expensed to profit or loss.
At the reporting date the shares will be valued at fair value (Rs. 5.20 per
share) ignoring selling costs = Rs. 1,300,000.
The revaluation gain of Rs. 87,500 will be recognised in profit or loss.
Alternative:
Waters Ltd could have made an irrevocable election at initial recognition to
recognise gains and losses in other comprehensive income. If this election
had been made the shares would have been measured on initial
recognition at the cost of Rs. 1,212,500 plus directly attributable transaction
costs Rs. 35,000 = Rs. 1,247,500.
At the reporting date the shares would then have been be valued at fair
value with the revaluation gain of Rs. 52,500 recognised in other
comprehensive income.
4 Amount receivable from Mason
On recording the sale, the revenue needs to be discounted at the imputed
rate of interest of 11%. Revenue recognised on 1 July is therefore Rs.
450,450 (500,000 y 1.11).
The receivable on the statement of financial position will include the
accrued interest element of Rs. 24,775 (Rs. 450,450 x 0.11 x 6/12) and so
will be Rs. 475,225 (Rs. 450,450 + Rs. 24,775) in total. The accrued
interest of Rs. 24,775 will be recognised as finance income.
The receivable would not be adjusted for any change in interest rates.

Emile Woolf International 281 The Institute of Chartered Accountants of Pakistan


Advanced accounting and financial reporting

5 Investment in 8.5% treasury stock 2018


This would be classified as to be subsequently measured at amortised cost.
On initial recognition, it will be recorded at fair value, the cost of Rs.
107,100.
Finance income will be credited to profit or loss using the gross redemption
yield of 5.9%.
Interest recognised in profit or loss will be Rs. 4,213 (Rs. 107,100 u 5.9% u
8/12).
The investment in the statement of financial position at 31 December 2016
will be at Rs. 107,100 plus Rs. 4,213 = Rs. 111,313. (No interest will have
been received to date as it is paid annually in arrears).
The market value is not reflected in the statement of financial position at 31
December 2016 but it would be disclosed in accordance with IFRS 7.
6 Investment in loan notes
The investment has been classified as held for trading so it is accounted for
as a financial asset at fair value through profit or loss.
On acquisition it will be recorded at its cost of Rs. 25,000.
At the reporting date the notes will be revalued to their fair value of Rs.
25,500 with the Rs. 500 uplift being recognised in profit or loss
7 Selling shares short
On initial recognition, the journal would be:
Rs. Rs.
Dr Cash (10,000 u Rs. 3.60) 36,000
Cr Financial liability 36,000
At the reporting date the financial liability must be revalued to its fair value
of Rs. 33,000:
Rs. Rs.
Dr Financial liability 3,000
Cr Statement of profit or loss 3,000

15.11 ARIF INDUSTRIES LIMITED


(a) Amortisation table
Interest income
Opening Expected Closing
@ 15.5%
balance cash flow balance
(Cr P&L)
A B = A x 15.5% C E=A+B+C
2016 104,641,483 16,219,430 (15,000,000) 105,860,913
2017 105,860,913 16,408,441 (15,000,000) 107,269,354
2018 107,269,354 16,626,750 (20,000,000) 103,896,104
2019 103,896,104 16,103,896 (120,000,000) -

Emile Woolf International 282 The Institute of Chartered Accountants of Pakistan


Answers

(b) Amortisation table with impairment


Interest
Impairmen
Opening income @ Expected Closing
t loss
balance 15.5% cash flow balance
(Dr P&L)
(Cr P&L)
A B=A x 15.5% C D E=A+B+C+D
2016 104,641,483 16,219,430 (15,000,000) - 105,860,913
2017 105,860,913 16,408,441 (15,000,000) (26,818,338) 80,452,016
2018 80,452,016 12,470,062 (15,000,000) - 77,922,078
2019 77,922,078 12,077,922 (90,000,000) - -

Computation of impairment loss:


Expected cash Discounted rate
Present value
flow (revised) (effective rate = 15.5%)
2018 15,000,000 0.8658 12,987,013
2019 90,000,000 0.7496 67,464,003
Revised present value (recoverable amount) 80,451,016
Existing present value (as at end of 2017) 107,269,354

Impairment loss 26,818,338


(c) Amortisation table with adjustment for future income estimate
Interest Effect of
Opening income @ Expected change in Closing
balance 15.5% cash flow estimate balance
(Cr P&L) (Cr P&L)
A B=A x 15.5% C D E=A+B+C+D
-
2016 104,641,483 16,219,430 (15,000,000) 105,860,913

2017 105,860,913 16,408,441 (15,000,000) 1,622,535 108,891,889


-
2018 108,891,888 16,878,243 (20,000,000) 105,770,132
-
2019 105,770,131 16,394,370 (20,000,000) 102,164,502
(118,000,00
-
2020 102,164,501 15,835,498 0) -

Computation of effect of change in estimate:

Expected cash Discounted rate


Discounted
flow (revised) (effective rate = 15.5%)
2018 (20,000,000) 0.8658 17,316,017
2019 (20,000,000) 0.7496 14,992,223
2020 (118,000,000) 0.6490 76,583,649

Revised present value 108,891,889


Existing present value (as at end of 2017) 107,269,354

Effect of change in estimate 1,622,535

Emile Woolf International 283 The Institute of Chartered Accountants of Pakistan


Advanced accounting and financial reporting

15.12 QASMI INVESTMENT LIMITED


Qasmi Investment Limited
Journal entries for 31 December 2011 and 2012

Debit Credit
Date Description
Rs. in million

31-Dec-2011 Accrued Interest written off (P&L) 12.00

Accrued Interest - 2010 12.00

(Accrued interest on 12%TFCs for 2010


is no more receivable, now written off.)

Financial assets W.3 (16.89


(12% TFCs) 12.00) 4.89

Interest income (P&L) 4.89

(Interest income on 12% TFCs at


4.426% for 2011)

Impairment loss (P&L) W.1 19.16

Financial assets (12%


TFCs) 19.16

(Impairment of financial assets (12%


TFCs) as interest for 2010 to 2013 is
no more receivable)

Financial W.1
31-Dec-2012
assets(12% TFCs) (88.5316.426%) 14.54

Interest income (P&L) 14.54

(Interest income for


2012)

Financial assets (12%


TFCs) W.2 10.31

Impairment reversal
(P&L) 10.31

(Reversal of impairment of financial


assets on rescheduling of payments
for TFCs)

W.1 Impairment

Carrying value of 12% TFCs on 31-12-2011 W.3 107.69

PV of future cash flows on 31-12-2011 120[(1.16426)2 ] 88.53

Impairment loss 19.16

Emile Woolf International 284 The Institute of Chartered Accountants of Pakistan


Answers

W.2 Impairment Reversal

Revised carrying amount on rescheduling at lower of (A) and (B)


below 113.38

(A) PV of the future cash flow as per the agreed revised


schedule 115.00

(B) Amortised cost on impairment reversal date of 31-12-2012


would have been had the impairment not been recognised.
W.3 113.38

Existing carrying amount at 31-12-2012 88.531.16426 (103.07)

Impairment reversal 10.31

W.3 Original amortisation schedule

Effective interest
Cash flow Cash flow
@ Amortised cost
dates (Interest @ 12%)
16.426%

--------------------------------Rs. in million--------------------------------

(10095%)
01-Jan-2009 95.00

31-Dec-2009 15.60 (12.00) 98.60

31-Dec-2010 16.20 (12.00) 102.80

31-Dec-2011 16.89 (12.00) 107.69

31-Dec-2012 17.69 (12.00) 113.38

Emile Woolf International 285 The Institute of Chartered Accountants of Pakistan


Advanced accounting and financial reporting

CHAPTER 16: FINANCIAL INSTRUMENTS: PRESENTATION AND


DISCLOSURE

16.1 SERRANO LTD


IAS 32 Financial Instruments: Disclosure and Presentation says that the issuer of
a compound (hybrid) instrument (i.e. one that contains both a liability debt and an
equity element) should classify the instruments components separately. Thus
the advice of Ancho Services is wrong; convertible loan stock cannot be
classified as pure equity. The proceeds of the issue have to be split between the
amount attributable to the conversion rights, which is then classed as equity, and
the balance of the proceeds being classed a liability/debt. There are several
methods of obtaining these amounts, but from the information given in the
question these can only be calculated on a residual value of equity basis:
Cash Factor at Present
flows 10% value
Rs.000 Rs.000
Year 1 interest 600 0.91 546
Year 2 interest 600 0.83 498
Year 3 interest 600 0.75 450
Year 4 interest and capital 10,600 0.68 7,208

Total value of debt component 8,702
Proceeds of the issue 10,000

Equity component (residual amount) 1,298

Statement of profit or loss: Rs.


Interest paid (6% of Rs. 10 million) 600,000
Provision for additional finance costs 270,000
((10% u 8.702m) 0.6m) 870,000
Statement of financial position:
Non-current liabilities:
6% Convertible Loan Stock (from above) 8,702,000
Provision for additional finance costs 270,000
8,972,000
Capital and reserves:
Option to convert to equity (from above) 1,298,000

16.2 POBLANO LTD


In the financial statements of Poblano Ltd for the year to 30 September 2016.
In the statement of profit or loss, the finance cost relating to the loan notes is Rs.
640,000.
In the statement of financial position:
Non-current liability for the loan notes = Rs. 9,384,000
Equity component of loan notes = Rs. 856,000.

Emile Woolf International 286 The Institute of Chartered Accountants of Pakistan


Answers

Workings: Rs. 10 million of loan notes


Discount
Annual factor at Present
Year Cash flows cash flow 7% value
Rs.000 Rs.000
1 Interest 400 0.93 372
2 Interest 400 0.87 348
3 Interest + Redemption 10,400 0.81 8,424
Value as straight loan notes 9,144
Issue price 10,000
Equity component (residual amount) 856

Finance cost: year to 30 September 2016


Rs.000
Total finance cost: 9,144 u 7% 640
Interest payable on 30 September 2016 (Rs. 10 million u 4%) 400
Accrual to add to carrying value of debt 240

Carrying value of loan notes: 30 September 2016


Rs.000
Initial valuation of debt element 9,144
Add accrued interest 240
Carrying amount at 30 September 2016 9,384

16.3 PIQUIN LTD


(a) Convertible instrument
A convertible instrument is considered part liability and part equity. IAS 32
requires that each part is measured separately on initial recognition. The
liability element is measured by estimating the present value of the future
cash flows from the instrument (interest and potential redemption) using a
discount rate equivalent to the market rate of interest for a similar
instrument with no conversion terms. The equity element is then the
balance, calculated as follows:
Rs.
PV of the principal amount Rs. 10m at 7% redeemable in 5 yrs 7,130,000
Rs. 10m x 0.713
PV of the interest annuity at 7% for 5 yrs 2,050,000
(5% x Rs. 10m) x 4.100
Total value of liability element 9,180,000
Equity element (balancing figure) 820,000
Total proceeds raised 10,000,000
The equity will not be remeasured, however the liability element will be
subsequently remeasured at amortised cost using the effective interest rate
of 7%. The total finance cost for the year ended 31 December 2016 is Rs.
642,600 (7% x 9,180,000). The coupon rate of interest of 5% has already
been charged to profit or loss in the year so a further Rs. 142,600 should
be recorded:

Emile Woolf International 287 The Institute of Chartered Accountants of Pakistan


Advanced accounting and financial reporting

Dr Finance costs Rs. 142,600


Cr Non-current liability Rs. 142,600
(b) Preference shares
The substance of the instrument is a debt instrument. IAS 32 requires that
any instrument that contains an obligation to transfer economic benefit be
classified as a liability. The cumulative nature of the returns on the
preference shares means that the outflow of benefit is inevitable. The
preference shares would then be classified as debt and would in fact
increase the gearing of the entity.

16.4 AJI LTD


(a) IAS 32 requires that the equity and liability elements within convertible
instruments be initially recognised separately. The initial carrying amount of
the liability is estimated by measuring the fair value of a similar liability that
has no equity element. This is achieved by calculating the present value of
the future cash flows associated with the instrument assuming that it is not
converted on redemption (ie: the interest and principal repayment cash
flows) discounted at the prevailing market rate for a similar instrument
without conversion rights. The difference between this amount and the
proceeds (ie: the residual) is recognised as equity.
The bonds are initially recognised as:
Rs.000 Rs.000
Dr Bank (proceeds of issue) 6,000
Cr Liability (W1) 5,609
Cr Equity (W2) 391
Working 1
Liability element Rs.000
PV of the principal (at 9% after 4 years) = (Rs. 6m x 0.708) 4,248
PV of interest of 7% on Rs. 6m for 4 years = (Rs. 6m x 0.07 x
3.24) 1,361
Total value of liability element 5,609
Working 2
Equity element Rs.00
0
Total proceeds raised on issue 6,000
Total value of liability element (5,609)
Value attributable to equity 391
(b) (i) In accordance with IAS 39, the liability element will be subsequently
measured at amortised cost using the effective interest rate (which in
this case is the interest rate used to discount the principal to PV, ie
9%). The equity element is not subsequently re-measured.
The interest of Rs. 420,000 (7% x Rs. 6m) has already been paid and
recorded. The additional finance cost is recorded as:
Rs.000 Rs.000
Dr Finance costs (W1) 85
Cr Liability element of bonds 85

Emile Woolf International 288 The Institute of Chartered Accountants of Pakistan


Answers

(ii) Extracts from statement of financial position Rs.000


Equity and Liabilities
Equity - Other component of equity 391

Liabilities (W1) 5,694

Working 1
Opening Finance cost at Interest paid 7% Closing balance
balance 9% Rs.000 Rs.000
Rs.000 Rs.000
5,609 505 (420) 5,694
Tutorial note:
The total finance cost for the year ended 31 December 2016 is Rs.
505K, however the interest paid of Rs. 420K has already been
recorded so only the difference of Rs. 85K is recognised.

16.5 CHILTEPIN LTD


The convertible bonds on issue will be recorded as:
Dr Bank Rs. 3,900,000
Cr Liability Rs. 3,729,400
Cr Equity Rs. 170,600
Workings:
Time Narrative Cash flow Discount Present
factor (7%) value
1 to 4 Interest 5% x Rs. 4m x 3.387 677,400
3.387
4 Repayment Rs. 4,000,000 0.763 3,052,000
Fair value of the liability 3,729,400
Fair value of equity component (balancing figure) 170,600
Total fair value (amount raised) 3,900,000
The liability will then be accounted for in accordance with IAS 39, i.e. at
amortised cost using the effective interest rate of 7%.
Opening Finance cost Interest paid Closing
carrying @ 7% carrying
value value
Rs. Rs. Rs. Rs.
y/e 30/9/2016 3,729,400 261,058 (200,000) 3,790,458
The interest paid of Rs. 200,000 has already been posted, so the additional Rs.
61,058 is recorded as:
Dr Finance costs Rs. 61,058
Cr Liability Rs. 61,058

Emile Woolf International 289 The Institute of Chartered Accountants of Pakistan


Advanced accounting and financial reporting

16.6 HABENERO LTD

(a) The preference shares will be classified as a liability despite being called
shares. IAS 32 requires us to consider the substance of the instrument in
order to determine whether it should be classified as debt or equity. In this
case the 5% dividend payable on the shares is cumulative which will
eventually result in an outflow of economic benefit for Habenero Ltd and
hence represents an obligation. It therefore meets the definition of a
liability. Once the principal amount is classed as a liability, it follows then
that any payment associated with this instrument (in this case the 5%
dividend) will be presented as a finance cost and be charged in arriving at
profit for the year.
The ordinary shares have no inherent obligation as they will not be repaid,
nor do they provide any fixed return to the shareholder. Indeed ordinary
shares contain only a residual interest in the profits of the entity (i.e.: after
all obligations have been settled) and hence will be classified as equity.
The associated dividend, when paid, will be presented in the statement of
changes in equity as a reduction in retained earnings.

(b) (i) Initial recognition of the HFT investment is at cost and transaction
costs are charged to the statement of profit or loss:
Dr HFT Investment Rs. 1,400,000
Cr Bank Rs. 1,400,000
Being recognition of investment (where Rs. 1,400,000 = Rs. 2.80 x
500,000 shares)
Dr Statement of profit or loss Rs. 7,000
Cr Bank Rs. 7,000
Being write off of transaction costs (where Rs. 7,000 = Rs. 1,400,000
x 0.5%), with the costs taken to profit or loss rather than included as
part of the initial investment (because of being classified as HFT).
(ii) Subsequent measurement is at fair value with the gain or loss taken
to profit or loss:
Dr HFT Investment Rs. 310,000
Cr Statement of profit or loss Rs. 310,000
Being the gain on HFT investment (where Rs. 310,000 = Rs.(3.42
2.80) x 500,000 shares), with the gain being recognised in profit for
the year.

Emile Woolf International 290 The Institute of Chartered Accountants of Pakistan


Answers

CHAPTER 18 IAS 12: INCOME TAXES

18.1 SHAKIR INDUSTRIES


COMPUTATION OF TAX EXPENSE
FOR THE YEAR ENDED DECEMBER 31, 2016
2016
Rs.in
million
Profit before tax 15.80
Add: Inadmissible expenses
Accounting depreciation (Rs. 1.1 million + Rs.0.7 million) 1.80
Financial charges on finance lease 0.15
Penalty paid to SECP 0.70
Provision for gratuity 2.40
5.05

Less: Admissible expenses Rs. m


Tax depreciation 1.65
Lease payments 0.65
Payment of gratuity 1.60
Borrowing cost capitalised 2.30
6.20
Taxable profit for the year 14.65
Current tax expense @ 35% 5.13

COMPUTATION OF DEFERRED TAX EXPENSE


FOR THE YEAR ENDED DECEMBER 31, 2016
Carrying Tax Temp
amount base difference
Rs. m Rs. m Rs. m
Fixed assets Owned 16.70 13.85 2.85
Fixed assets Leased 1.80 - 1.80
Capital work in progress 2.30 - 2.30
Provision for gratuity (0.7 + 2.4 1.6) (1.50) - (1.50)
Obligation against assets subject to
finance lease (1.20) - (1.20)
Total 4.25
Deferred tax expense @ 35% 1.49

Rs.in
million
Deferred tax liability (Opening) 0.55
Deferred tax expense for the year (balancing figure) 0.94
Deferred tax liability as at December 31, 2016 (Rs. 4.25 million x
35%) 1.49

Emile Woolf International 291 The Institute of Chartered Accountants of Pakistan


Advanced accounting and financial reporting

18.2 DWAYNE LTD (PART 1)


(a)
Carrying Tax Temporary To OCI
value base difference
Buildings 45,500 17,500
Revaluation 14,500  14,500
60,000 17,500 42,500

Plant 68,000 26,000 42,000


Investments 72,000 65,000 7,000 (1,000)
Dividend income 150 150 -
Loan 20,500 21,000 500
Defined Benefits 1,000  (1,000)
liability
91,000 13,500
@ 28% @28%
25,480 3,780

(b) Deferred taxation


liability
Rs. 000
Balance B/F 13,500
Due to change in rate (13,500 2/30) (900)
13,500 x 28/30 12,600
To OCI (28% x 13,500) 3,780
To statement of profit or loss (as a balancing figure) 9,100
25,480
(c) Journal
Debit Credit
Deferred tax liability 11,980
Movement due to rate change:
OCI ((24,000 30%) 2/30) 480
P&L (balancing figure) 420
Movement due temporary differences arising
during 2013
OCI 3,780
P&L 9,100

18.3 DWAYNE LTD (PART 2)

(a) Rs. 000


DWAYNE 25,480
LARRY FV Tax
base
Buildings 600 300 300

Emile Woolf International 292 The Institute of Chartered Accountants of Pakistan


Answers

Plant 56 25 31
Inventory 152 144 8
Retirement (100)  (100)
benefit
239
@28% 67
(b) 25,547

Deferred
taxation
liability
Rs. 000
(c) Balance B/F 13,500
2
Due to change in rate (13,500 /30) (900)
28
13,500 x /30 12,600
To OCI (28% x 13,500) 3,780
To statement of profit or loss (as a balancing figure) 9,100
Due to introduction of a new subsidiary 67
25,547
(d) Goodwill
Rs. 000
Cost 750
Less share of net assets
80% x (778 67) (569)
Goodwill arising 181

18.4 COHORT
Note for presentation to partner
Subject: Deferred Taxation
The calculation and presentation of deferred tax is considered by IAS 12 Income
taxes. A company is required to provide deferred tax on all material temporary
differences using the full provision method. Temporary differences arise because
there is a difference in timing between transactions being reflected in the financial
statements and the item being taxed.
In light of the recent acquisitions of Legion and Air, deferred tax must be
considered for the group accounts. Additional tax issues arise at the group level
that will not have been reflected in the individual entitys accounts and these
points are outlined below.
Once the temporary differences have been identified, deferred tax must be
provided at the tax rate expected to be effective at the date when the tax is
settled. Given this rate is not known when the differences arise, a provision is
made using the rates enacted at that time and the estimate is then confirmed as
tax changes arise.

Emile Woolf International 293 The Institute of Chartered Accountants of Pakistan


Advanced accounting and financial reporting

Air
(a) The acquisition of air mid-year gives rise to a number of issues:
(1) Intangible asset
There is some concern that the acquisition of the database of key
customers may not be allowed for tax purposes but it has
nevertheless been included in the tax calculation on the assumption
that a deduction will be allowed by the tax authorities. If this deduction
is not allowed, then an additional tax payment will need to be made to
the authorities, hence it would be prudent to recognise a liability for
this amount (probably classified as current taxation, rather than
deferred taxation).
(2) Inter-company sales
When goods are sold between group members, the profits made are
seen as unrealised in the group accounts until the items are sold
outside of the group. However, the tax authorities tax the individual
entities, not the group, and so the profit will be subject to tax at the
time of the inter-company sale. The unrealised profit represents the
temporary difference on which deferred tax must be provided. The
goods were sold at a margin of 33%. Goods sold for Rs. 1.8m
remain in inventory at the year end, and hence the unrealised profit,
and therefore temporary difference, is Rs.0.6m.
(3) Unremitted profits
Un-remitted earnings represent a temporary difference in the group
accounts. This is because the tax base is the cost of the investment,
yet in the consolidated accounts, this cost is uplifted by the post-
acquisition un-remitted profits. IAS 12 requires a provision to be made
on this timing difference unless the parent controls the timing of the
reversal and it is probable that the difference will not reverse for the
foreseeable future. The payment of dividends is under the control of
Cohort, but we understand that the intention is to realise these un-
remitted earnings through future dividends and hence a provision
must be made.
(b) Legion
The acquisition of Legion at the start of the year brings further deferred tax
issues in the group accounts as outlined below.
(1) Fair value through the profit or loss investments
The fair value adjustment has been reflected in the financial
statements, yet the gain is not taxed until the investments are sold.
Hence the fair value adjustment of Rs. 4m gives rise to a temporary
difference upon which deferred tax must be provided. As the gain has
been taken to profit or loss rather than other comprehensive income
or reserves, the deferred tax must also be expensed to profit or loss.
(2) General allowance
The allowance against the loan portfolio has reduced the carrying
value of the loans but the tax relief is not available until the loan is
written off, and hence a temporary difference is created on the
provision. As the tax relief will reduce future tax charges, the
temporary difference of Rs. 2m gives rise to a deferred tax asset. The
temporary difference is accounted for even though there is no
expectation that the difference will reverse in the immediate future.

Emile Woolf International 294 The Institute of Chartered Accountants of Pakistan


Answers

However, a deferred tax asset can only be reflected to the extent that
it is probable that there will be future taxable profits against which the
temporary difference can be relieved.
(3) Unrelieved tax losses
When Legion was acquired, it had unused tax losses brought forward
which could, in principle, give rise to a deferred tax asset. However, it
can only be recognised to the extent that it is believed that the loss
can be recovered. Given your belief that there will not be sufficient
future profits, the deferred tax can only be partially recognised. If the
fortunes of Legion change in the future, the deferred tax asset should
then be recognised, leading to a compensating amendment to
goodwill.

18.5 MODEL TOWN GROUP


Adjustment to
financial Tax Temporary
statements base difference
Rs.000 Rs.000 Rs.000 Rs.000
Property plant, and equipment 10,000 2,400 7,600
Goodwill 6,000 6,000
Other intangible assets 5,000 0 5,000
Financial assets (cost) 9,000 1,500 9,000 1,500

Total non-current assets 30,000

Trade receivables 7,000 7,500 (500)
Other receivables 4,600 5,000 (400)
Cash and cash equivalents 6,700 6,700

Total current assets 18,300

Total assets 48,300

Long term borrowings 10,000 (400) 10,000 400
Deferred tax liability 3,600 3,600
Employee benefits 4,000 5,000 1,000
Current tax liability 3,070 3,070
Trade and other payables 5,000 4,000 (1,000)

Total liabilities 25,670 13,600

Share capital 9,000
Other reserves 4,500 1,500
400
Retained earnings 9,130

Total equity 22,630

Rs.000
Deferred tax liability 15,500 @ 30% 4,650
Deferred tax asset (1,900) @ 30% (570)

Net deferred tax liability 13,600 @ 30% 4,080
Less existing liability (3,600)

Adjustment to deferred tax 480

Emile Woolf International 295 The Institute of Chartered Accountants of Pakistan


Advanced accounting and financial reporting

(i) The financial assets should be valued at fair value with the increase going
to OCI (Rs. 1.5 million).
(ii) The bond should be split into its equity and liability elements as per IAS 32.
(iii) As the development costs have been allowed for tax already, it will have a
tax base of zero. Goodwill is measured as a residual and, therefore, the
impact is not measured under IAS 12.
(iv) The accrual for compensation will not be allowed until a later period and,
therefore, will reduce the tax base relating to trade and other payables.

Emile Woolf International 296 The Institute of Chartered Accountants of Pakistan


Answers

CHAPTER 19: BUSINESS COMBINATIONS AND CONSOLIDATION

19.1 HELLO
Consolidated statement of financial position as at 31 December 2016
Rs.
Assets
Non-current assets
Property, plant and equipment (225 + 175 + 10 2) 408,000
Goodwill (W3) 8,000

Current assets (271 + 157) 428,000



844,000

Equity and liabilities
Shareholders equity
Called up share capital 100,000
Retained earnings (W5) 291,800

391,800
Non-controlling interest (W4) 79,200
Current liabilities 373,000

844,000

WORKINGS
(1) Group structure

Hello

60%

Solong
(2) Net assets of Solong Inc
Reporting Date of Post-
date acquisition acquisition
Rs. Rs.
Share capital 100,000 100,000
Retained earnings
Per the question 90,000
Less: Fair value adjustment
for depreciation (2/10 10,000) (2,000)
88,000 60,000
Fair value adjustment 10,000 10,000
198,000 170,000

Emile Woolf International 297 The Institute of Chartered Accountants of Pakistan


Advanced accounting and financial reporting

(3) Goodwill Rs.


Cost 110,000
Net assets acquired
60% u 170,000 (W2) (102,000)

8,000

(4) Non-controlling interest Rs.


40% u 198,000 (W2) 79,200

(5) Retained earnings Rs.


Hello 275,000
Solong (60% u (88,000 60,000 (W2)) 16,800

291,800

19.2 HASAN LIMITED


Hasan Limited
Consolidated statement of financial position as at 31 March 2016
Rs.000 Rs.000
Assets
Non-current assets
Property, plant and equipment (W1) 4,020
Goodwill (W4) 480
Software (W1) 1,440
Investments (65 + 210) 275

6,215
Current assets
Inventories (W2) 1,274
Trade receivables (524 + 328) 852
Cash and bank (20 + 55 cash in transit) 75

2,201

Total assets 8,416


Equity and liabilities


Capital and reserves
Equity capital 2,000
Reserves
Share premium 2,000
Retained earnings (W3) 2,420

4,420

6,420
Non-controlling interest (W5) 350

Emile Woolf International 298 The Institute of Chartered Accountants of Pakistan


Answers

Rs.000 Rs.000
Government grants (230 + 40) 270
Current liabilities
Trade payables (475 + 472) 947
Operating overdraft 27
Income tax liability (228 + 174) 402

1,376

Total equity and liabilities 8,416


Workings
(W1) Property, plant and equipment
Rs.000
Balance from question Hasan Limited 2,120
Balance from question Shakeel Limited 1,990
Fair value adjustment on acquisition (see below) (120)
Over-depreciation re fair value adjustment year to 31 March 2016 30

4,020

A fair value of the leasehold based on the present value of the future
rentals (receivable in advance) would be the next (non-discounted)
payment of the rental plus the final three years as an annuity at 10%:
Rs.000
PV of rental receipts: Rs. 80,000 + (Rs. 80,000 u 2.50) 280
Carrying value on acquisition is (400)

Fair value reduction of leasehold (120)


The depreciation of the leasehold in Shakeel Limiteds accounts would be


Rs. 100,000 per annum. However in the consolidated accounts it should be
Rs. 70,000 (Rs. 280,000/4). This would require a reduction in depreciation
of Rs. 30,000 in the consolidated accounts for the next four years.
Software:
Shakeel Consolidated Difference
Limiteds figures
accounts
Rs.000 Rs.000
Capitalised amount 2,400 2,400
Depreciation to
31 March 2015 (300) 8 year life (480) 5 year life

Value at date of 180 fair
acquisition 2,100 1,920 value adjustment
Depreciation to 180 additional
31 March 2016 (300) (480) amortisation

Carrying value
31 March 2016 1,800 1,440

Emile Woolf International 299 The Institute of Chartered Accountants of Pakistan


Advanced accounting and financial reporting

(W2) Inventories
Rs.000
Amounts given in the question (719 + 560) 1,279
Unrealised profit in inventories (25 u 25/125) (5)

1,274

(W3) Retained earnings


Rs.000
Retained profits of Shakeel Limited, 31 March 2016 1,955
Adjustments:
Excess charge for leasehold depreciation 30
Insufficient charge for Software amortisation (180)
Unrealised profit in inventory (W2) (5)

Adjusted retained profits at 31 March 2016 1,800


Retained earnings of Shakeel Limited at 1 April 2015 2,200

Shakeel Limited: loss for the year (post-acquisition loss) (400)


Rs.000
Parent company share of post-acquisition loss (90%) (360)
Hasan Limited reserves at 31 March 2016 2,900
Goodwill impairment (120)

Consolidated retained profits at 31 March 2016 2,420


(W4) Goodwill
Rs.000
At acquisition date
Shares of Shakeel Limited 1,500
Share premium of Shakeel Limited 500
Retained earnings of Shakeel Limited 2,200
Fair value adjustments:
Leasehold (W1) (120)
Software (W1) (180)

3,900

Acquired by Hasan Limited (90%) 3,510


Cost of investment 4,110

Goodwill at acquisition 600


Impairment 120

Goodwill at 31 March 2016 480


Emile Woolf International 300 The Institute of Chartered Accountants of Pakistan


Answers

(W5) Non-controlling interests


Rs.000
Share capital of Shakeel Limited 1,500
Share premium of Shakeel Limited 500
Adjusted retained earnings of Shakeel Limited, 31 March 1,800
2016 (W3)
Fair value adjustments:
Leasehold (120)
Software (180)

Total net assets at 31 March 2016 3,500


Non-controlling interests (10%) 350


(W6) Elimination of current accounts:


Rs.000
Shakeel Limiteds current account with Hasan Limited per 75
question
Deduct cash in transit regarding this balance (15)

Adjusted figure to cancel 60


(W7) Elimination of intra-group loan:


Rs.000
Investment in Hasan Limiteds books 200
Deduct repayment in transit (40)

Non-current liability in Shakeel Limiteds books 160


19.3 FLAMSTEED LTD AND HALLEY LTD


(a) An impairment loss is the amount by which the carrying amount of an asset
or a cash generating unit exceeds its recoverable amount.
(b) The following external sources of information may indicate that an asset is
impaired.
(i) There are observable indications that the assets value has declined
during the period significantly more than would be expected as a
result of the passage of time or normal use.
(ii) Significant changes with an adverse effect on the entity have taken
place during the period or will take place in the near future, in the
technological, market, economic or legal environment in which the
entity operates or in the market to which the asset is dedicated.
(iii) The carrying amount of the net asset of the entity is more than its
market capitalization.
(iv) The carrying amount of the investment in the separate financial
statements exceeds the carrying amount in the consolidated financial
statement of the investees net asset, including associated goodwill,

Emile Woolf International 301 The Institute of Chartered Accountants of Pakistan


Advanced accounting and financial reporting

or the dividend exceeds the total comprehensive income of the


subsidiary, joint venture or associates in the period the dividend is
declared.
(v) Market interest rate or other market rate of return on investment have
increased during the period and those increases are likely to affect
the discount rate used in calculating the asset value in use and
decrease the assets recoverable amount materially.
(c) Flamsteed Ltd group: extract of consolidated statement of financial position
as at 30 June 2016
Rs.000
Assets
Non-Current Assets
Property, Plant and Equipment (100,000 + 80,000) 180,000
Goodwill (WK) 13,468
Intangible-brand name 10,000
203,468
Current Assets
Inventory (6,000 + 16,000) 22,000
Receivables (32,000 + 14,000) 46,000
Cash (4,000 + 0 + 4,000) 8,000
76,000

Total Assets 279,468


Workings
(i) Goodwill
Rs.000 Rs.000
Consideration transferred 77,468
Fair value of NCI 18,000
Net Asset acquired as represented by: 95,468
Ordinary share capital 50,000
Revaluation surplus on acquisition 10,000
Retained earnings on acquisition 12,000
Intangible assets (brand name) 10,000 (82,000)
13,468
Note
The deferred consideration has been discounted at 7% for 2 years (1st July
2015 1st July 2017).

19.4 BRADLEY LTD


Consolidated statement of financial position as at 31 December 2016
Rs.Million Rs.Million
Non-current assets
Goodwill (working 1) 120
Land & building (630 + 556 + 140) 1,326
Machinery & equipment (570 + 440) 1,010
2,456

Emile Woolf International 302 The Institute of Chartered Accountants of Pakistan


Answers

Current assets
Inventory (714 +504 24) 1,194
Trade receivables (1,050 + 252 50) 1,252
Cash/Bank (316 + 60) 376 2,822
5,278
Ordinary shares of Rs. 1 each 3,000.0
Retained earnings (Working 3) 1,323.2
Non-controlling Int. (Working 4) 376.8
4,700
Current liabilities
Trade payables (440 + 188 - 50) 578
5,278
Workings:
Rs.
million
1. Calculation of goodwill:
Fair value of consideration 1,320
Plus fair value of NCI at acquisition 330
Less net acquisition fair value of
Assets acquired & liability:
Share capital 1,200
Retained Earning 190
Fair value adj at acquisition 140 (1,530)
Goodwill 120

2. Group structure
960 million
u 100
1,200 million 80%
3. Retained earnings:
As per question 1,160 424
Adjustment (unrealised profit) (24)
Pre-acquisition retained earnings (190)
234
Group share of post-acquisition retained
earnings:
(80% x 234) 187.2
1,323.2
4. Non-controlling interest: Rs.
million
Fair value of NCI at acquisition 330.0
Plus NCIs share of post-acquisition
retained earnings (20% x 234) 46.8
376.8
Alternative workings:

(W1) Fair value adjustment:


Dr: Consolidated land & building Rs. 140 million
Cr: Revaluation reserve Rs. 140 million

Emile Woolf International 303 The Institute of Chartered Accountants of Pakistan


Advanced accounting and financial reporting

(W2) Consolidation Schedule


Bliss Ltd Rs. M Bradley in NCI Post
Bliss 80% 40% Acq
(W3)
Rupees in millions
Ordinary share capital 1,200 960 240
Revaluation Res. (W1) 140 112 28
Retained earnings 424 152 84.8 187

Net assets acquired 1,224
Cost of acquisition (1,320)
Goodwill (partial value) (96)
Goodwill attribute to NCI (W5) (24) 24

Goodwill (fair value) ( 120)

Unrealised profit on inventory - (24)
NCI (fair value) 376.8

Retained earnings of Bradley Ltd 1,160
Consolidated retained earnings 1,323

(W3) Bradley in Bliss = = 80%


(W4) Bradleys share of Blisss pre-acquisition retained earnings = 80% x


= 80% x Rs. 190m = Rs. 152m

(W5) Goodwill attribute to NCI Rs. m


Fair value of NCI @ date of acquisition 330

Less: fair value of net assets attributable to


NCI (20% x (1,200 + 140 + Rs. 190m)) 306

Goodwill attributable to NCI 24

19.5 X LTD
Consolidated statement of financial position as at 31 December 2016
for the X Ltd Group
All workings in Rs.000
ASSETS Rs.000
Non-current assets
Property, plant and equipment (12,000 + 4,000 + 750(W1)) 16,750
Goodwill (W2) 208
Intangible asset (W1) 90
17,048
Current assets
Inventories (2,200 + 800 -30 (W3)) 2,970
Receivables (3,400 + 900) 4,300
Cash and cash equivalents (800 + 300) 1,100
8,370
Total assets 25,418

Emile Woolf International 304 The Institute of Chartered Accountants of Pakistan


Answers

EQUITY AND LIABILITIES


Equity
Share capital (Rs. 1 equity shares) 10,000
Retained earnings (W4) 7,893
Total equity attributable to parent 17,893
Non-controlling interest (W5) 1,741
Total equity 19,634
Non-current liabilities
Long term borrowings 2,700
Current liabilities (2,000 + 1,000 + 84) 3,084
Total liabilities 5,784
Total equity and liabilities 25,418
Workings
1. Fair value adjustments
At acquisition
Movement 31 Dec 2016
date
Rs.000 Rs.000 Rs.000
PPE 800 (50) 750
Inventories 200 (200) -
Intangible assets 150 (60) 90
Liabilities (210) 126 (84)

940 (184) 756

2. Goodwill
Rs.000 Rs.000
Consideration transferred 3,800
NCI at fair value 1,600

5,400
Net assets at fair value:
Share capital 1,000
Retained earnings 3,200
Fair value adjustments 940

(5,140)

Goodwill on acquisition 260
20% impairment (52)

Goodwill at 31 December 2016 208

3. Unrealised profit on inventories
Sales of Rs. 300k x 20% x 50% left in inventories at y/e = Rs. 30,000
4. Retained earnings
Rs.000 Rs.000
As per SOFP 7,500 4,000
Pre-acquisition reserves (3,200)
Adjustments arising from movement in
FV adjustments (184)

616

Emile Woolf International 305 The Institute of Chartered Accountants of Pakistan


Advanced accounting and financial reporting

Group share 75% 462


Unrealised profit on inventory transfer (30)
Goodwill impairment (75% x 52)(W2) (39)

Consolidated reserves 7,893

5. Non-controlling interests
Rs.000
NCI at acquisition (at fair value) 1,600
25% x post-acquisition retained earnings Rs. 616,000 (W4) 154
Goodwill impairment (25% x 52)(W2) (13)

1,741

19.6 KHAN LIMITED


(a) Khan Limited
Consolidated statement of financial position as at December 31, 2016
Rupees
ASSETS in million
Non-current assets
Property plant & equipment (W- 1) 14,800
Goodwill (W - 2) 100
14,900

Current assets (1,069+ 1,316) 2,385

17,285
EQUITY AND LIABILITIES
Equity
Share capital 6,800
General reserve (W5) 1,975
Retained earnings 3,844
12,619
Non-controlling interest (W8) 2,420
Total equity 15,039

Non-current liabilities
14% Term finance certificates (2,250-1,500) 750

Current liabilities
Accounts payable (445 + 190) 635
Dividend payable (W3) 861
17,285

Emile Woolf International 306 The Institute of Chartered Accountants of Pakistan


Answers

(b) Khan Limited


Consolidated statement of profit or loss and other comprehensive
income for the year ended December 31, 2016
Rupees in
million
Profit before tax and interest (W4) 4,315
Interest expense (315 - 210) (105)
Profit before tax 4,210
Taxation expense (650 + 474) (1,124)
Profit for the period 3,086
Other comprehensive income -
Total comprehensive income 3,086

Attributable to:
Equity holders of parent Balancing 2,894
Non-controlling interest (W-7) 192
3,086

(c) Khan Limited


Consolidated statement of retained earnings for the year ended
December 31, 2016
Rupees in
million
Balance as at January 1, 2016 (W- 6) 1,700
Total comprehensive income for the year 2016 2,894
Dividends (750)
Balance as at December 31, 2016 3,844
W1 Property, plant & equipment
Rupees in
million
Cost KL 16,250
Cost GL 25,000
Acc. depreciation KL (9,750)
Acc. depreciation GL (17,000)
14,500
Fair value adjustment 1,000
Less: Depreciation on increased
fair value (Rs. 1,000 x 10% x 7) (700)
14,800
W2 Goodwill
Purchase consideration 5,500
Less:
Share capital (75% of 5,000) (3,750)
Retained earnings (75% of 1,000) (750)
General reserve (75% of 200) (150)
FV increase in PPE (1,000 x 75%) (750)
100
W3 - Dividend payable
Ordinary dividend KL 750
Ordinary dividend - GL (300 x 25%) 75
Preference dividend - GL (60 x 60%) 36
861

Emile Woolf International 307 The Institute of Chartered Accountants of Pakistan


Advanced accounting and financial reporting

W4: Profit before tax and interest


KL 2,865
GL 1,550
Current year depreciation on increased
value of PPE (1,000 x 10%) (100)
4,315
W5: General reserve
General reserve KL 1,750
General reserve GL (500 200) x 75% 225
1,975
W6: Retained earnings
Rupees in
million
Retained earnings KL 2,000
Retained earnings GL (1,200 - 1,000) x 75% 150
Less: Depreciation charge on
increased FV (1,000 x 6 x 10% x 75%) (450)
1,700
W7: Non-controlling interest (for statement of comprehensive income)
Share from profit of GL (1,550+210-300-474120) x 25% 217
Less: Current year depreciation on
increased of PPE (100 x 25%) (25)
192
W8: Non-controlling interest (for statement of financial position)
Rupees in
million
Share capital (5,000 x 25%) 1,250
Preference shares (1,000 x 60%) 600
General reserve (500 x 25%) 125
Opening retained earnings (1,200 x 25%) 300
Comprehensive income for the year (W-7) 192
Increase in FV of building (1,000 x 25%) 250
Less: Depreciation charge on
increased FV (1,000 x 6 x 10% x 25%) (150)
Less: Dividend on ordinary shares (300 x 0.25) (75)
Less: Dividend on preference
shares (120 x 0.6) (72)
2,420

Emile Woolf International 308 The Institute of Chartered Accountants of Pakistan


Answers

CHAPTER 20: CONSOLIDATED STATEMENTS OF PROFIT OR LOSS AND


OTHER COMPREHENSIVE INCOME

20.1 MILLARD LTD


Millard Ltd: Consolidated profit and loss account for the year ended 31
December 2016
NOTE Rs.000
Revenue (1) 425,000
Cost of sales (2) (162,600)
Gross profit 262,400
Distribution costs (35,000)
Impairment losses (4) (7,000)
Administrative expenses (28,000)
Operating profit 192,400
Investment income (5) 3,750
Debenture interest (6) (58,750)
Profit before tax 137,400
Tax (52,500)
Profit after tax 84,900
Non-controlling Interests (7) (8,380)
Group profit 76,520
Preference dividends (13,750)
Ordinary dividends (20,000)
Retained profit 42,770
Retained profit 1 January 2016 (8) 72,350
Retained profit 31 December 2016 115,120

Workings:
1 Revenue: Rs.000
Millard 312,500
Fillmore 125,000
437,500
Less: inter-company group sales 12,500
425,000

2 Cost of Sales: Rs.000


Millard 125,000
Fillmore 50,000
Inter-company sales (12,500)
Unrealized profits (3) 100
162,600

Emile Woolf International 309 The Institute of Chartered Accountants of Pakistan


Advanced accounting and financial reporting

3 Provision for unrealised profits on inventory Rs.000


25 2000 400
2016 u
125 1
25 1500
2015 u (300)
125 1
Charged to profit and loss a/c 100

4 Goodwill on acquisition: Rs.000 Rs.000


Cost of investment 67,000
Net assets acquired:
- On ordinary share capital 62,500
- Profit and loss account 12,500
75,000
Percentage acquired 50/62.5 x 100 = 80%
80% of Rs. 75,000 (60,000)
Goodwill (written-off in 2016) 7,000

5 Investment income: Rs.000


As per draft account 7,950
Inter-company dividend 80% x Rs. 5,250 (4,200)
Investment income 3,750

6 Debenture interest Rs.000


Parents 47,500
Subsidiary 15,000
Inter-company amount 25% x Rs. 15,000 (3,750)
Investment income 58,750

7 Non-controlling interest:
Rs.000 Rs.000
Profit after tax 24,500
Less: Preference dividend (4,375) 4,375
20,125 x 20% 4,025
8,400
Non-Controlling Interest share of unrealized profit 20% x Rs. 100 (20)
8,380

8 Retained Profit b/f Rs.000 Rs.000


Parents 66,750
Subsidiary 19,500
Pre-acquisition (12,500)
Parents share 7,000 x
80% 5,600
72,350

Emile Woolf International 310 The Institute of Chartered Accountants of Pakistan


Answers

20.2 SHERLOCK
Sherlock Ltd: Consolidated statement of profit or loss and other comprehensive
income for the year ended 31 December 2016.
Rs. m
Revenue 538.0
Cost of sales (383.0)
Gross profit 155.0
Other income 29.0
Administrative costs (30.0)
Other expenses (72.6)
Operating profit 81.4
Finance costs (10.0)
Finance income 15.0
Profit before tax 86.4
Income tax expense (31.0)
Profit for the year 55.4
Other comprehensive income
Revaluation surplus 7.8
Remeasurement 2.0
Loss on cash flow hedge (3.0)
6.8
Total comprehensive income for year 62.2
Profit attributable to:
Rs. m
Owners of the parent (balancing figure) 43.8
Non-controlling interest (W1a) 11.6
55.4
Total comprehensive income attributable to:
Rs. m
Owners of the parent (balancing figure) 51.8
Non-controlling interest (W1a) 10.4
62.2
Workings
W1 Balances for inclusion in the consolidated statement of profit or loss and
other comprehensive income
Sherlock Mycroft Katie Ltd Adjustment Total
Ltd Ltd (6/12)
Rs. m Rs. m Rs. m Rs. m Rs. m
Revenue 400 115 35 (12) W3 538
Cost of sales (312) (65) (18) 12 W3 (383)
Gross profit
Other income 21 7 1 29
Administrative costs (15) (9) (6) (30)
Other expenses (35) (19) (4)
Goodwill impairment (3)

Emile Woolf International 311 The Institute of Chartered Accountants of Pakistan


Advanced accounting and financial reporting

Sherlock Mycroft Katie Ltd Adjustment Total


Ltd Ltd (6/12)
Rs. m Rs. m Rs. m Rs. m Rs. m
W2
Pension cost W4 (7.2)
Revaluation W5 (2.4)
Share Katie Ltd W6 (2.0) (72.6)
Operating profit
Finance costs (5) (6) (2)
Cash flow hedge W7 3.0 (10)
Finance income 6 5 4 15
Profit before tax
Income tax expense (19) (9) (3) (31)
Profit for the year 41 22 7
OCI
Revaluation
surplus 10
Revaluation W5 (2.2) 7.8
Remeasurement 2.0
W4 2
Cash flow hedge W7 (3.0) (3.0)
Total CI for year 50.8 19 7 6.8
W1a Non-controlling interests
Mycroft Ltd Katie Ltd Total
Profit for the year 22 7
NCI percentage holding 40% 40%
NCI 8.8 2.8 11.6

Mycroft Ltd Katie Ltd Total


Total CI for the year W1 19 7
NCI percentage holding 40% 40%
NCI 7.6 2.8 10.4
W2 Goodwill write off
Rs. m
Cost of investment 80
FV of NCI at acquisition 45
125
Fair value of identifiable net assets (110)
Goodwill on acquisition 15
Write off in previous year (20%) (3)
W3 Inter-company trading
The inter-company trading amounts must be eliminated (ie sales and
purchases).
There is no adjustment in respect of the loss. The question states that the sale is
at fair value. Therefore the loss is realised. Only unrealised amounts are
adjusted on consolidation.

Emile Woolf International 312 The Institute of Chartered Accountants of Pakistan


Answers

W4 Pension scheme
Amounts charged to profit or loss: Rs. m
Interest (10% of (Rs. 50m Rs,48m)) 0.2
Current service cost 4.0
Past service cost 3.0
7.2
Amount charged to OCI
Remeasurement 2.0
W5 Revaluation of plant
Rs. m
Original cost (1 January 2015) 12.0
Depreciation in year ended 31 December 2015 (1.2)
Carrying amount before revaluation at 31 December 2015 10.8
Revaluation recognised in year ended 31 December 2015 2.2
Value at 31 December 2015 13.0
Depreciation in year ended 31 December 2016 (9) (1.4)
Carrying amount before revaluation at 31 December 2016 11.6
Fall in value to be recognised 4.6
Value at 31 December 2016 7.0
Dr Cr
Rs. m Rs. m
Plant 4.6
Statement of profit or loss 2.4
Other comprehensive income 2.2
W6 Share Katie Ltd expense
Rs. m
Balance recognised in year ended 31 December 2015
8,000 Katie Ltds u Rs. 100 u 4 directors u 1/4 0.8

Balance recognised in year ended 31 December 2015


8,000 Katie Ltds u Rs. 100 u 7 directors u 2/4 2.8
Charge for the year ended 31 December 2016 2.0
Dr Cr
Rs. m Rs. m
Statement of profit or loss 2.0
Equity 2.0
W7 Cash flow hedge
Mycroft Ltds loss on the effective cash flow hedge has been treated incorrectly.
The effective part of any gain or loss on a cash flow hedge should be recognised
in other comprehensive income and accumulated in a cash flow hedge reserve
in equity.
The following corrective journal is necessary:
Dr Cr
Rs. m Rs. m
Loss on cash flow hedge
(in other comprehensive income) 3
Finance cost (profit or loss) 3

Emile Woolf International 313 The Institute of Chartered Accountants of Pakistan


Advanced accounting and financial reporting

20.3 FAISAL LIMITED


Faisal Limited
Consolidated statement of financial position as at 31 December 2016
Rs. in million
Non-current assets
Property, plant and equipment (W1) 31,926.00
Accumulated depreciation (W1) (7,491.00)
24,435.00
Goodwill (W6) 1,380.00
Other investments 11,100.00
36,915.00
Current assets
Inventory (W3) 23,740.00
Accounts receivable (6,240 + 2,460 + 6,580  800) 14,480.00
Cash and bank balances (4,920 + 660 + 2,700) 8,280.00
46,500.00
Total assets 83,415.00

Equity and liabilities


Share capital 30,000.00
Retained earnings (W8) 42,379.75
72,384.75
Non-controlling interest (W7) 5,655.25
78,035.00
Current liabilities
Accounts payable (2,760 + 1,980 + 1,440  800) 5,380.00
Total equity and liabilities 83,415.00

Faisal Limited
Consolidated statement of profit or loss
for the year ended 31 December 2016
Rs. in million
Sales (W4) 100,100.00
Cost of sales (W4) (80,991.00)
Gross profit 19,109.00
Operating expenses (3,600 + 2,100 + 5,400) (11,100.00)
8,009.00
Gain on sale of non-current assets (540 54) 486.00
Dividend income (1,080 (80% u 600)) 600.00
Profit for the year 9,095.00
Attributable to:
Ordinary shareholders of parent 8,599.75
Non-controlling interest (W9) 495.25
9,095.00

Emile Woolf International 314 The Institute of Chartered Accountants of Pakistan


Answers

Workings (all figures in millions of rupees)

(W1) Unrealised profit adjustments; Transfer of non-current asset


Figures in Figures if no
the transfer had Adjustment
accounts been made required
Against SLs figures:
Cost 144 150 6 Dr
Accumulated depreciation
(144/3 years) u 6/12 (24)
150 u 3 years/5 years (75)
(24) (75) 51 Cr
Carrying amount 120 75
Charge for the year 24 15 9 Cr

Against FLs figures:


Profit on disposal 54 54 Dr

Double entry in consolidated financial


statements Dr Cr
Profit on disposal 54
Depreciation charge for year 9
Property, plant and equipment (cost) 6
Accumulated depreciation 51

NCI in the statement of profit or loss 2.25


NCI in the statement of financial position 2.25
Being the NCI share of the depreciation adjustment (20% u 9)

Accumulated
Consolidated balances PP and E depreciation
Rs. millions Rs. millions
FL 22,500 5,760
SL 3,480 420
AIL 5,940 1,260
Adjustments for inter-company
transfer 6 51

31,926 7,491

Emile Woolf International 315 The Institute of Chartered Accountants of Pakistan


Advanced accounting and financial reporting

(W2) Unrealised profit adjustments: Inter-company trading


FL to SL to AIL to Total
AIL AIL FL
Sales 2,400 1,800 3,600 7,800

Inventory held 900 600 1,200


Gross profit percentage on
sale 20% 10% 30%
Unrealised profit 180 60 360 600
NCIs share (based on
selling companys NCI)
25% u 60 15
20% u 360 72

Double entry in consolidated financial


statements Dr Cr
Cost of sales (closing inventory) 600
Closing inventory in statement of financial
position 600

NCI in the statement of financial position 87


NCI in the statement of comprehensive income 87

Impact on consolidated retained earnings (600 97) 513

(W3) Consolidated inventory


Rs.
millions
FL 14,460
SL 4,200
AIL 5,680
(600)
23,740

Emile Woolf International 316 The Institute of Chartered Accountants of Pakistan


Answers

(W4) Consolidated sales and cost of sales


Cost of
Sales sales
Rs. Rs.
millions millions
FL 57,600 49,200
SL 16,500 18,000
AIL 33,800 21,000
Inter-company sales (7,800) (7,800)
Unrealised profit 600
Depreciation adjustment on inter-
company transfer of non-current assets (9)
100,100 80,991

(W5) Net assets of SL


At end of
At reporting
acquisition period
Rs. millions Rs. millions
Share capital 12,000 12,000
Retained earnings (3,600) -
8,400 12,000
Net assets of AIL
At end of
At reporting
acquisition period
Rs. millions Rs. millions
Share capital 6,000 6,000
Retained earnings (12,200) 5,400
18,200 11,400

(W6) Goodwill
SL AIL
Rs. millions Rs. millions
Cost of investment 9,000 10,500
NCI at acquisition
25% u 12,000 (W5) 3,000
20% u 11,400 (W5) 2,280
12,000 12,780
Net assets acquired (12,000) (11,400)
 1,380

Emile Woolf International 317 The Institute of Chartered Accountants of Pakistan


Advanced accounting and financial reporting

(W7) Non-controlling interest in statement of financial position


SL AIL
Rs. millions Rs. millions
NCI at acquisition
25% u 12,000 (W5) 3,000
20% u 11,400 (W5) 2,280
NCIs share of post-acquisition profit / (loss)
25% u (3,600) (W5) (900)
20% u (12,200 5,400) (W0) 1,360
2,100 3,640
Total (2,100 + 3,640) 5,740.00
Unrealised profit adjustments
on inter-company sale of inventory (W2) (87.00)
on inter-company sale of non-current asset (W1) 2.25
5,655.25
(W8) Retained earnings
Rs. millions
FL 40,200.00
Share of SL (75% u (3,600)) (2,700.00)
Share of AIL (80% u (12,200 5,400) (W5)) 5,440.00
42,940.00
Unrealised profit adjustments
Inter-company sale of inventory (W2) (600.00)
NCI share 87.00
(513.00)

Inter-company sale of non-current asset (W0)


Unrealised profit (W1) (54.00)
Depreciation adjustment (W1) 9.00
NCI share of depreciation adjustment (W1) (2.25)
(47.25)
42,379.25

(W9) Non-controlling interest in statement of profit or loss


SL AIL
Rs. millions Rs. millions
Sales 16,500 33,800
Cost of sales (18,000) (21,000)
Operating expenses (2,100) (5,400)
(3600) 7400
25% 20%
Non-controlling interest (900) 1,480

Emile Woolf International 318 The Institute of Chartered Accountants of Pakistan


Answers

SL AIL
Rs. millions Rs. millions
Total (1,480 900) 580.00
Unrealised profit adjustments
on inter-company sale of inventory (W2) (87.00)
on inter-company sale of non-current asset (W1) 2.25
495.25

20.4 GOLDEN LIMITED


Golden Limited
Consolidated statement of profit or loss for the year ended June 30, 2016
Rs. in million
Sales (875 + 350 - 40) 1,185.00
Cost of sales (567 + 206 - 33.6 (W1)) (739.40)
Gross profit 445.60
Selling expenses (33 + 11) (44.00)
Administrative expenses (63 + 40) (103.00)
Interest expenses (30 + 22) (52.00)
Other income (65 - 36) [20 x Rs. 2 x 90%) 29.00
Impairment losses
Goodwill (W2) (9.18)
Investment in associates (W3) (25.80)
Share of loss from associates [(Rs. 82 x 40%)+0.6] (33.40)
Profit before tax 207.22
Income tax expense (73 + 15) (88.00)
Profit for the year 119.22
Attributable to:
Ordinary shareholders of parent 114.26
Non-controlling interest (W4) 4.96
119.22
W1: Adjustment in cost of sales Rs. in million
Intra-group purchases (40.00)
Additional depreciation on machines 4.00
Unrealized profit in inventories 2.40
(33.60)
W2: Impairment on goodwill
Shares issued (18 x 4/5 x Rs. 20) 288.00
Less: Net assets acquired:
Share capital 200
Pre-acquisition reserves 24
Fair value adjustment (22 + 20 + 3) 45
269
Holding % 90% 242.10
Goodwill 45.90
20% Impairment in goodwill 9.18

Emile Woolf International 319 The Institute of Chartered Accountants of Pakistan


Advanced accounting and financial reporting

Rs. in
W3: Impairment in the value of investment in associates million

Cash paid (6 x 12) 72.00


Less: Post-acquisition losses:
Reserves on acquisition 40
Reserves at June 30, 2016 (108-82) 26
(14)
% holding 40% (5.60)
Elimination of unrealized gain to the extent of GL's share
(Rs. 11.5 x 0.15 / 1.15 x 40%) (0.60)
65.80
Fair value as per impairment testing 40.00
Impairment losses 25.80

W4: Non-controlling interests


Profit of YL 56.00
Less: Additional depreciation (4.00)
Unrealized profit in inventories (2.40)
49.60
Non-controlling interest % 10%
4.96

Emile Woolf International 320 The Institute of Chartered Accountants of Pakistan


Answers

CHAPTER 21: ASSOCIATES AND JOINT VENTURES

21.1 JOINT ARRANGEMENTS

(a) (i) Joint Operations


A joint operation is a joint arrangement whereby the parties that have
joint control of the arrangement have right to the assets and
obligations for the liabilities relating to the arrangement. Those
parties are called joint operators.
(ii) Joint venture
A joint venture is a joint arrangement whereby the parties that have
joint control of the arrangement have right to the net asset of the
arrangement. These parties are referred to as joint venturers.
(b) Elements to be recognised by a joint operator
(i) Its assets and share of any assets held jointly
(ii) Its liabilities and share of any liabilities incurred jointly
(iii) Its revenue from the sale of its share of the output arising from the
joint operation
(iv) Its share of the revenue from the sale of the output by the joint
operations
(v) Its expenses and share of any expenses incurred jointly.
(c) Characteristics of joint arrangements
(i) The parties are bound by a contractual arrangement
(ii) The contractual arrangement gives two or more of those parties joint
control of the arrangement.

21.2 Helium
Consolidated statement of financial position as at 31 December 2016
Rs.000
Assets
Non-current assets
Property, plant and equipment 500
Interest in associate (W6) 51
Goodwill 15
Current assets 605

Total assets 1,171

Equity and liabilities
Capital and reserves
Share capital 100
Retained earnings (W5) 737

837
Non-controlling interest 84
Long-term liabilities 250

Total equity and liabilities 1,171

Emile Woolf International 321 The Institute of Chartered Accountants of Pakistan


Advanced accounting and financial reporting

Workings
(1) Group structure

Helium

30%
60%

Arsenic
Sulphur

(2) Net assets


Sulphur
Balance Post
sheet date Acquisition acquisition
Rs.000 Rs.000 Rs.000
Share capital 30 30
Retained earnings 180 70 110

210 100

(3) Goodwill
Sulphuric
Rs.000
Cost of investment 75
Share of net assets acquired
(60% u 100 (W2)) (60)

15

(4) Non-controlling interest
Rs.000
Sulphur (40% u 210) 84

(5) Retained earnings
Rs.000
Helium 650
Sulphur (60% u 110 (W2)) 66
Arsenic (30% u (100 30)) 21

737

(6) Investment in associate
Rs.000
Cost 30
Share of post-acquisition profit (30% u (100 30)) 21

51

Emile Woolf International 322 The Institute of Chartered Accountants of Pakistan


Answers

21.3 HAMACHI LTD


(a)
Hamachi Ltd
Consolidated statement of financial position as at 31 March 2016
Rs.000 Rs.000
Non-current assets
Property, plant and equipment (8,050 + 3,600) 11,650
Goodwill (W2) 702
Licence (180 60) (W3) 120
12,472
Investments
Associate (W6) 717
Others (4,000 + 910 3,240 630 + 120 FV) 1,160
1,877
14,349
Current assets
Inventory (830 + 340) 1,170
Accounts receivable (520 + 290 40) 770
Bank (240 + 40) 280
2,220
Total assets 16,569
Equity and liabilities
Equity attributable to equity holders of the parent:
Ordinary shares of Rs. 1 each 5,000
Retained earnings (W5) 8,415
13,415
Non-controlling interest (W4) 374
13,789
Non-current liabilities
10% Loan notes (500 + 240) 740
Current liabilities
Accounts payable (420 + 960) 1,380
Taxation (220 + 250) 470
Overdraft 190
2,040
Total equity and liabilities 16,569
Workings
(W1) Net assets in subsidiary
At end of
At reporting
acquisition period
Rs.000 Rs.000
Share capital 1,200 1,200
Retained earnings 800 2,300
Fair value adjustment:
Investment property 120 120
Licence 180 180
Amortisation of licence 180/6 x 2yrs (60)
2,300 3,740

Emile Woolf International 323 The Institute of Chartered Accountants of Pakistan


Advanced accounting and financial reporting

(W2) Goodwill
Rs.000
Cost of investment (Rs. 3 u 1,200 u 90%) 3,240
Net assets acquired (90% u 2,300) (W1) 2,070
Goodwill 1,170
Less impairment (468)
702
(W3) Unrealised profit in inventory
((2/3 65,000) 30/130) 30% = Rs. 3,000
Parent sells to associate, therefore reduce group retained earnings
and Investment in associate

(W4) Non-controlling interest


10% u 3,740 = Rs. 374
(W5) Retained earnings
Rs.000
Hamachi Ltd 7,500
Saba Ltd group share post-acquisition
90% u (3,740 2,300) 1,296
Anogo Ltd group share post-acquisition
30% u (600 u 6/12) 90
Unrealised profit (W3) (3)
Less impairment (468)
8,415
(W6) Investment in associate
Rs.000
Investment at cost 630
Post-acquisition profit (30% u (600 u 1/2)) 90
Unrealised profit in inventory (3)
717
(b) IAS 28 Investments in Associates and Joint Ventures defines associates. In
order for an investment to be classified as an investment in an associate
the investor must have significant influence over the investee. Significant
influence is presumed to exist where there is a holding of 20% or more of
the voting power unless the investor can clearly demonstrate that this is not
the case. Conversely a holding of less than 20% is presumed not to be an
associate, unless it can be clearly demonstrated that the investor can
exercise significant influence. The voting rights can be held directly or
through subsidiaries.
IAS 28 says that a majority holding by one investor does not preclude
another investor having significant influence. An investing company owning
a majority holding in another company normally has control over the
investee and would thus class it as a subsidiary. In normal circumstances it
is difficult to see how a company could be controlled by one entity and be
significantly influenced by a different entity unless control was passive.
The 20% test is not definitive and the following other evidence should be
considered.

Emile Woolf International 324 The Institute of Chartered Accountants of Pakistan


Answers

Does the investing company:


have representation on the Board of the investee?
participate in the policy making processes (operational and financial);
have material transactions with the investee?
interchange managerial personnel with the investee; or provide
technical expertise to the investee?

21.4 HIDE
Hide
Consolidated statement of profit or loss for the year ended 30 June 2016
Rs.000
Revenue 15,131
Cost of sales and expenses (13,580)

Operating profit before tax 1,551
Tax (736)

Profit after tax 815
Share of profit of associate (30% of 594) 178

Profit for the year 993

Profit for the year attributable to members of Hide 963


Non-controlling interest (W2) 30

Profit for the year 993

Workings
(1) Group structure

Hide

30%
80%
Arrive

Seek

Emile Woolf International 325 The Institute of Chartered Accountants of Pakistan


Advanced accounting and financial reporting

(2) Consolidation schedule


Hide Seek (5/12) Adjustment Total
Rs. 000 Rs. 000 Rs. 000 Rs. 000
Revenue 12,614 2,567 (50) 15,131
Cost of sales
Per question (11,318) (2,302) 50
Unrealised profit
50 u (25/125) (10) (13,580)

Tax (621) (115) (736)


Profit for the year 150
Non-controlling
interest (%) 20%
Non-controlling
interest (Rs. 000) 30

21.5 HARK, SPARK AND ARK


Hark Group
Consolidated statement of financial position as at 31 March 2016
Rs.000 Rs.000
Non-current assets
Property, plant and equipment (working 1) 90,200
Goodwill (working 4) 23,000
Investment in associate (working 6) 9,500
Other investments 650
123,350
Current assets (working 5) 24,300
Total assets 147,650
Equity and liabilities
Equity shares of Rs. 1 each (working 3) 21,000
Share premium (working 3) 42,000
Retained earnings (working 8) 43,730
85,730
106,730
Non-controlling interests (working 7) 7,420
Total equity 114,150
Non-current liabilities
Deferred consideration for Spark shares 5,500
6% loan notes 10,000
7% loan notes 6,000
21,500
Current liabilities: 7,000 + 5,000 12,000
Total equity and liabilities 147,650

Emile Woolf International 326 The Institute of Chartered Accountants of Pakistan


Answers

Workings
1 Property, plant and equipment (PPE)
Rs.000 Rs.000
Hark 60,000
Spark 31,000
Profit on transfer of machines (3 million 2 million) 1,000
Less: Depreciation on this amount in accounts of Spark
(1,000/5 years) (200)
Unrealised profit in machines (800)
PPE in consolidated statement of financial position 90,200

2 Deferred consideration
The present value of the deferred consideration at 1 April 2015 is Rs. 6.05
million u 1/(1.10)2 = Rs. 5 million.
During the year to 31 March 2016 there is a finance charge of 10% (= Rs.
500,000) on this amount, reducing the parents share of the consolidated
profit.
The deferred consideration at 31 March 2016 is Rs. 5 million + Rs. 500,000
= Rs. 5,500,000. This is payable in just over 12 months and is included in
the consolidated statement of financial position as a non-current liability.

3 Share issues
The share issues to acquire the shares in Spark and Ark are not recorded
in the summary statement of financial position of Hark (as stated in the
question).
Share Share
Total capital premium
To acquire the shares in Spark Rs.000 Rs.000 Rs.000
Hark shares issued: (4 million at Rs.
9) 36,000 4,000 32,000
To acquire the shares in Ark
Hark shares issued: (1 million at Rs. 9) 9,000 1,000 8,000
Increase in share capital and share
premium of Hark 5,000 40,000
In summary statement of financial
position 16,000 2,000
In consolidated statement of financial
position 21,000 42,000

4 Goodwill
Hark has acquired 4 million/5 million = 80% of the shares of Spark.
At 1 April 2015 the fair value of the net assets of Spark was (share capital
plus reserves) = Rs.(5 + 4 + 16) million = Rs. 25 million
Rs.000
Purchase consideration paid by the parent company
Issue of 4 million shares at Rs. 9 36,000
Deferred consideration 5,000
41,000

Emile Woolf International 327 The Institute of Chartered Accountants of Pakistan


Advanced accounting and financial reporting

Fair value of parent company share of net assets


(80% u Rs. 25 million) 20,000
Purchased goodwill attributable to parent 21,000

Rs.000
Fair value of NCI at acquisition date (1 million shares u Rs. 7) 7,000
NCI share of net assets at this date (20% u Rs. 25 million) 5,000
Purchased goodwill attributable to NCI 2,000
There has been no impairment of goodwill during the year.
Rs.000
Purchased goodwill attributable to parent 21,000
Goodwill attributable to NCI 2,000
Total goodwill in consolidated statement of financial position 23,000
Alternatively, total goodwill could be calculated as follows:
Rs.000
Purchase consideration paid by the parent company 41,000
(see above)
Fair value of NCI at acquisition date 7,000
48,000
Net assets of the subsidiary at the acquisition date 25,000
(at fair value)
Total goodwill (parent and NCI) 23,000

5 Current assets
The cost of the goods sold by Spark to Hark was Rs. 3,600,000 u 100/150
= Rs. 2,400,000 and the profit was Rs. 1,200,000.
Since 75% of these goods are in closing inventory, the unrealised profit on
intra-group sales is 75% u Rs. 1,200,000 = Rs. 900,000. Current assets in
the consolidated statement of financial position (inventory) should be
reduced by this amount.
The question states that the transaction costs of the acquisition of Spark
have not yet been recorded. These costs reduce the consolidated profit,
and also (presumably) reduce the current assets of Hark.
Current assets on consolidation Rs.000
Hark 18,200
Spark 8,000
Less: unrealised profit in closing inventory (900)
Less: expenses of acquisition of Spark (1,000)
Current assets in consolidated statement of financial position 24,300

6 Investment in associate (Ark)


Since Hark owns 25% of the equity of Ark, it is assumed that Ark is an
associated entity.
Rs.000
Cost of investment: 25% u 6 million shares u Rs. 6 9,000
Share of post-acquisition retained profit: 25% u Rs. 2 million 500
9,500

Emile Woolf International 328 The Institute of Chartered Accountants of Pakistan


Answers

7 Non-controlling interests
Rs.000
Share of net assets of Spark at 31 March 2016 (20% u Rs. 28 5,600
million)
Goodwill attributable to NCI (working 4) 2,000
7,600
NCI share of unrealised profit in inventory (20% u Rs. 900,000) (180)
NCI at 31 March 2016: fair value method 7,420
8 Consolidated retained earnings
Rs.000 Rs.000
Hark retained earnings (36,000 + 8,000) 44,000
Spark
Profit for year ended 31 March 2016 3,000
Unrealised profit in closing inventory (900)
2,100
Parent company share (80%) 1,680
Share of post-acquisition retained profits of Ark 500
(25% u Rs. 2 million)
Costs of acquisition of Spark (expensed) (1,000)
Additional finance costs: deferred consideration (500)
Unrealised profit in machines (working 1) (800)
Loss on other (800 650) (150)
Consolidated retained earnings at 31 March 2016 43,730

21.6 P, S AND A
P Group
Consolidated statement of financial position as at 31 December Year 5
Assets
Non-current assets Rs.
Property, plant and equipment (450,000 + 240,000) 690,000
Goodwill (W3) 45,000
Investment in associates (W5) 168,800
903,800
Current assets
Inventory (70,000 + 90,000 10,000) 150,000
Other current assets (20,000 + 110,000 + 130,000) 260,000
Total assets 1,313,800
Equity and liabilities
Equity
Share capital 100,000
Share premium 160,000
Consolidated accumulated profits (W6) 711,300
Attributable to equity holders of the parent 971,300
Non-controlling interest in S (W4) 102,500
Total equity 1,073,800
Long-term liabilities (40,000 + 20,000) 60,000
Current liabilities (100,000 + 80,000) 180,000
Total equity and liabilities 1,313,800

Emile Woolf International 329 The Institute of Chartered Accountants of Pakistan


Advanced accounting and financial reporting

Workings
P owns 75% of the equity of S and 30% of the equity of A. Therefore S is a
subsidiary and A is an associate.
W1: Net assets summary
Calculate the net assets of S and A at the acquisition date and at the end of the
reporting period.
At this stage, make any fair value adjustments and eliminate the unrealised profit
in inventory.
At date of At date of Post-
Net assets of S consolidation acquisition acquisition
Rs. Rs. Rs.
Equity shares 200,000 200,000 -
Share premium 80,000 80,000 -
Accumulated profits (per question) 140,000 60,000 80,000
410,000 340,000

W2: Unrealised profit on inter-company trading


Sale by S to P: Rs. 40,000 33.33/133.33 = Rs. 10,000.
Dr Cr
Consolidated accumulated profits (75%) 7,500
Non-controlling interest (25%) 2,500
Consolidated inventory 10,000
Sale by P to A: Rs. 16,000 33.33/133.33 = Rs. 4,000.
Ps share: Rs. 4,000 u 30% = Rs. 1,200
Dr Cr
Consolidated accumulated profits (75%) 1,200
Investment in associate 1,200
W3: Goodwill on acquisition of S
Rs.
Cost of the acquisition 320,000
Non-controlling interest at acquisition (25% u 340,000 (W1)) 85,000
405,000
Less: Fair value of identifiable net assets at acquisition (W1) (340,000)
Goodwill at acquisition 65,000
Minus: Impairment to date (20,000)
Balance carried forward 45,000

Emile Woolf International 330 The Institute of Chartered Accountants of Pakistan


Answers

W4: Non-controlling interest in S


Rs.
Non-controlling interest at acquisition (25% u 340,000 (W1)) 85,000
Share of post-acquisition profits (25% u 80,000 (W1)) 20,000
Unrealised profit (W2) (2,500)
102,500

W5: Investment in associate


Rs.
Investment at cost 140,000
Ps share of post-acquisition accumulated profits 30,000
(30% u (250,000 150,000)
Unrealised profit (W2) (1,200)
168,800

W6: Consolidated accumulated profits.


Rs.
Accumulated profits of P 650,000
Ps share of post-acquisition profits of S
(75% Rs. 70,000 (W2)) 60,000
Unrealised profit (sale by S to P (W3)) (7,500)
Ps share of post-acquisition accumulated profits (W5) 30,000
Unrealised profit (W2) (1,200)
Impairment of goodwill (20,000)
Consolidated accumulated profits 711,300

21.7 H LTD GROUP


Consolidated statement of profit or loss and other comprehensive income for the
H Ltd group for the year ended 31 May 2016
Rs.000
Revenue (6,000 + 3,000) 9,000
Cost of sales (4,800 + 2,400) (7,200)
Gross profit 1,800
Distribution costs (64 + 32) (96)
Administrative expenses (336 + 168) (504)
Finance costs (30 + 15) (45)
Share of profit of associate (30% x 100) 30
Profit before tax 1,185
Income tax expense (204 + 102) (306)
PROFIT FOR THE YEAR 879

Emile Woolf International 331 The Institute of Chartered Accountants of Pakistan


Advanced accounting and financial reporting

Other comprehensive income:


Revaluation of PPE (200 + 100) 300
Actuarial gain on pension plan assets 40
Actuarial loss on pension plan liabilities (52)
Gain on AFS investment 14
Tax effect of other comprehensive income (42 + 21) (63)
Share of OCI of associate (net of tax) (30% x 24) 7
Other comprehensive income for the year, net of tax 246
TOTAL COMPREHENSIVE INCOME FOR THE YEAR 1,125

Profit for the period attributable to: Rs.000


Owners of the parent entity 822.4
Non-controlling interests (20% x 283) 56.6
879

Total comprehensive income attributable to:


Owners of the parent entity 1052.6
Non-controlling interests (20% x 362) 72.4
1,125

Emile Woolf International 332 The Institute of Chartered Accountants of Pakistan


Answers

CHAPTER 22: BUSINESS COMBINATIONS ACHIEVED IN STAGES

22.1 STEP ACQUISITION


The profits of AS since the investment was acquired (all retained) are Rs. 40
million (= Rs. 300m Rs. 260m). During this period, H held 25% of the equity of
AS and it is assumed that AS is an associate. Profits attributable to H for the year
are therefore Rs. 10 million (= 25% u Rs. 40 million).
(i) Total gain or profit attributable to the investment in AS
Rs. m
Initial investment in associate at cost 80
Share of post-investment retained profits 10
90
Fair value of investment at 30 June 95
Gain recognised when step acquisition occurs 5

The total gain/profit recognised for the year from the investment in AS is
therefore Rs. 10 million + Rs. 5 million = Rs. 15 million.

(ii) Total goodwill on acquisition


Rs. m
Fair value of shares that gave control (40%) 160
Fair value of previous investment (25%) 95
255
Fair value of NCI at acquisition 120
375
Net assets of AS at 30 June 300
Total goodwill 75

(iii) Goodwill attributable to NCI


Rs. m
Fair value of investment in 25% of AS (35% u 300) 105
Goodwill attributable to NCI (balancing figure) 15
Total NCI 120

22.2 A LTD
(a) Treatment of B Ltd
IFRS 3 Business combinations requires goodwill on acquisition to be
calculated at the date control is gained. The second acquisition gives A Ltd
a 75% holding and therefore control over B Ltd. The simple investment of
15% will be derecognised and the 75% holding will be fully consolidated as
a subsidiary in the group financial statements. The goodwill will be
calculated as the cost of the 60% acquired in the year plus the fair value of
the previously held interest of 15%, compared with the fair value of the net
assets at the date of acquisition (1 April 2016).

Emile Woolf International 333 The Institute of Chartered Accountants of Pakistan


Advanced accounting and financial reporting

(b) Consolidated statement of financial position for the A Ltd Group as at


31 September 2016.
A Ltd
ASSETS Rs.000
Non-current assets
Property, plant and equipment (22,000+5,000) 27,000
Goodwill (Working 1) 405
27,405
Current assets
Inventories (6,200+800 40 (Working 2)) 6,960
Receivables (6,600+1,900) 8,500
Cash and cash equivalents (1,200+300) 1,500
16,960
Total assets 44,365

EQUITY AND LIABILITIES


Equity
Share capital (Rs. 1 equity shares) 20,000
Retained earnings (Working 3) 8,629
Other components of equity (Working 6)
28,629
Non-controlling interest (Working 4) 1,604
Total equity 30,233

Non-current liabilities
5% Bonds 2015 (Working 5) 4,032
Current liabilities (8,100+2,000) 10,100
Total liabilities 14,132
Total equity and liabilities 44,365

Working 1 Goodwill Rs.000 Rs.000


Consideration transferred for the 60% 2,900
Fair value of 15% holding at 1 April 2016 800
Fair value of non-controlling interest 1,250
4,950
Net assets acquired:
Share capital 1,000
Retained earnings (5,000- 1,500) 3,500 (4,500)
450
Impaired by 10% (45)
Net value of goodwill 405

Working 2 Unrealised profit on inventories Rs.000


Sales from B Ltd to A Ltd 400
50% in inventories 200
Profit margin 20% - adjust Inventories and retained 40
earnings of B Ltd

Emile Woolf International 334 The Institute of Chartered Accountants of Pakistan


Answers

Working 3 Retained earnings A Ltd B Ltd


Rs.000 Rs.000
As at 30 September 2016 7,500 5,000
Pre-acquisition (5,000 (3,000 x 6/12)) (3,500)
Less unrealised profit of B Ltd (working 2) (40)
1,460
Group share 75% 1,095
Group share of impairment (75% x 45) (34)
Additional finance costs on bonds (working 5) (132)
Group profit on derecognition of AFS Investment
gain to date of deemed disposal 1 April
2016 (800 - 600) 200
Consolidated retained reserves 8,629

Working 4 Non-controlling interest Rs.000


Fair value at 1 April 2016 1,250
Plus 25% adjusted post-acquisition reserves 1,460 365
(working 3)
Less NCI share of goodwill impairment (25% x 45) (11)
NCI at 30 September 2016 1,604

Working 5 Bonds
amortised cost Rs.000 Rs.000 Rs.000 Rs.000
Opening Effective Interest paid Value at 30
value rate 8.5% 5% x Rs. 4m September
To 30 September 2016 3,900 332 (200) 4,032

The difference of Rs. 132,000 must be added to the value of the bond
liability and deducted from
A Ltds retained earnings.
Working 6 Other reserves and AFS investment
IFRS 3 requires that the 15% simple investment be derecognised and on
derecognition any gain/loss would be considered realised. The gain of Rs.
200,000 (FV of Rs. 800,000 at date of derecognition less the investment
cost of Rs. 600,000) represents the group gain and will be included in the
consolidated reserves.
The balance on other reserves again relates to the treatment of the
investment in the parents own accounts and the gains on the AFS
investment (B Ltd) and not relevant for the group accounts as the B Ltd
has been fully consolidated.

Emile Woolf International 335 The Institute of Chartered Accountants of Pakistan


Advanced accounting and financial reporting

22.3 X LTD GROUP


(a) (i) X Ltd group: Consolidated statement of profit or loss and other
comprehensive income for the year ended 31 December 2016

Rs.000
Revenue (1,200 + 290) 1,490
Cost of sales (810 + 110 + 4 (W1)) (924)
Gross profit 566
Operating expenses (100 + 40 + 9 (W2)) (149)
417
Investment income
(50 intra group dividend 40 (80% x 50)) 10
Finance costs (45 + 10) (55)
Share of associates profit (40% x 30) 12
Profit before tax 384
Income tax expense (80 + 30) (110)
Profit for the year 274
Other comprehensive income
Revaluation of property, net of tax (60 + 20) 80
Share of associates OCI (40% x 10) 4
Other comprehensive income for the year, net of tax 84
Total comprehensive income 358

Profit for the year attributable to:


Owners of the parent (274 17 (W3)) 257
Non-controlling interest 17
274
Total comprehensive income attributable to:
Owners of the parent (358 21 (W3)) 337
Non-controlling interest 21
358
(ii) Consolidated statement of changes in equity for the year ended
31 December 2016
X Ltd NCI Total
group
Rs.000 Rs.000 Rs.000
Equity at 1 January 2016 (W4)/(W5) 1,868 216 2,084
Total comprehensive income for the 337 21 358
year
Dividends (100) (100)
Dividend paid to NCI (20% x 50) (10) (10)
Equity at 31 December 2016 2,105 227 2,332

Emile Woolf International 336 The Institute of Chartered Accountants of Pakistan


Answers

Workings

(W1) Net assets of subsidiary


Acquisition 1 Jan 31 Dec
1 Jan 2010 2016 2016
Rs.000 Rs.000 Rs.000
Share capital 200 200 200
Retained reserves 420 640 (bal) 710
620 840 910
Fair value adjustment 60 60 60
Accumulated additional
depreciation on FV
adjustment (60/15 yrs = 4 per yr) (12) (16)
Accumulated impairment of
goodwill (W2) (30) (39)
Adjusted net assets 680 858 915
Post-acquisition retained
reserves to 1 Jan/31 Dec 178 235
(W2) Goodwill
Rs.000 Rs.000
Consideration transferred 620
NCI at fair value 180
800
Net assets acquired:
Share capital 200
Retained earnings 420
Fair value adjustment 60 (680)
120
Impairment 2015 (25%) (30)
90
Impairment 2016 (10% of carrying value) (9)
(W3) Non-controlling interest
PFY TCI
Rs.000 Rs.000
Profit for year/TCI of Y Ltd 100 120
Less impairment of goodwill in the year (W2) (9) (9)
Less depreciation on FV adjustment for the year
(W1) (4) (4)
87 107
20% NCI share 17 21
(W4) Group equity attributable to parent at 1 January 2016
Rs.000
Parents equity at 1 January 2016 as per SOCIE 1,700
Plus share of post-acquisition retained reserves of Y Ltd to 142
1 January 2016 (80% x 178 (W1))
Plus share of post-acquisition retained reserves of Z Ltd to
1 January 2016 (40% x(500-435)) 26
Equity attributable to parent at 1 January 2016 1,868

Emile Woolf International 337 The Institute of Chartered Accountants of Pakistan


Advanced accounting and financial reporting

(W5) Group equity attributable to NCI at 1 January 2016


Rs.000
At acquisition 180
Plus share of post-acquisition retained reserves to 1
January 2016 (20% x 178 (W1)) 36
Equity attributable to NCI at 1 January 2016 216
(b) (i) Additional acquisition of shares
The purchase of the additional 10% of Y Ltds share capital is treated
as a transaction between owners of the entity, as NCI reduces and
parents share increases. No additional goodwill is calculated as X Ltd
already controls Y Ltd and goodwill is only calculated when control is
attained. Any difference between the consideration paid by X Ltd and
the reduction in the NCI is adjusted through group retained earnings.
(ii)
Adjustment to parents equity Rs.000
Consideration transferred 120
Reduction in NCI at 1 January 2017 (50% x Rs. 227,000) (114)
Adjustment to retained earnings debit 6
(c) Additional investment in Z Ltd
The additional 20% investment will give X Ltd the majority holding of Z Ltds
ordinary shares. This gives the presumption of control, unless there is
evidence to the contrary and once control is attained Z Ltd will be treated
as a subsidiary and fully consolidated. Goodwill on acquisition is calculated
at 1 January 2017 and the existing investment will be restated to FV at the
date of acquisition

22.4 PLAIN LTD


Consolidated statement of financial position as at 31 March 2016
Rs. m
Assets
Tangible non-current assets (W4) 1,745
Intangible non-current assets goodwill 45 15 (W3) 30
Investment in associate (W5) 95
Held to maturity investment (W6) 50
Current assets (477 + 190 - 250) 417
2,337
Equity and liabilities
Share capital of Rs. 1 800
Share premium 150
Revaluation reserve 90
Retained earnings (W8) 169
1,209
Non-controlling interest (W9) 99
1,308

Emile Woolf International 338 The Institute of Chartered Accountants of Pakistan


Answers

Consolidated statement of financial position as at 31 March 2016


Rs. m
Non-current liabilities (640 + 30) 670
Current liabilities (214 + 130) 344
Pension liability (W7) 15
2,337
Workings

(W1) Retained profits of Stripes


Rs. m
Fair value of net assets at 1 April 2015 460
Fair value adjustment for land (25)
Carrying value of net assets 435
Share capital plus share premium (260)
Therefore retained earnings at 1 April 2015 175

Rs. m
Carrying value of net assets at 1 April 2013 325
Share capital plus share premium (260)
Therefore retained earnings at 1 April 2013 65
(W2) Gain or loss on acquiring control of Stripes
1 April 2015 Rs. m Rs. m
Fair value of initial investment in Stripes at 1 April 2015 150
Initial cost of investment 120
Share of retained earnings 1 April 2013 1 April 2015 33
(= 30% u (175 65) see W1
Carrying value of investment in associate 153
Loss recognised on gaining control of Stripes (3)
This loss has not yet been recognised in the individual financial statements
of Plain; it must therefore be included in the calculation of group reserves
(see Working 8).

(W3) Goodwill in Stripes at acquisition


Rs. m
Fair value of initial investment at acquisition 150
Cost of additional shares 260
Total cost 410
Fair value of net assets acquired (80% u 460) 368
Goodwill at acquisition attributable to Plain 42
Goodwill attributable to NCI 3
Total goodwill at acquisition date 45

Emile Woolf International 339 The Institute of Chartered Accountants of Pakistan


Advanced accounting and financial reporting

Goodwill in statement of financial position: There has been impairment of


Rs. 15 million in goodwill. This is apportioned between the interests of the
equity owners of Plain and NCI in the ratio 80:20.

Impairment of goodwill attributable to parent = Rs. 15m u 80% = Rs. 12


million

Impairment of goodwill attributable to NCI = Rs. 15m u 20% = Rs. 3 million.

(W4) Tangible non-current assets


Rs. m
Plain 1,280
Stripes 440
Fair value adjustment 25
1,745
(W5) Investment in associate Spots
Rs. m
Cost 60
Group share of post-acquisition profit
(324 (200 16)) 25% 35
95

Or Rs. m
Share of net assets (25% 324) 81
Fair value adjustment (25% 16) 4
Goodwill [60 (200 25%)] 10
95
(W6) Held to maturity investment
Rs. m
Amortised cost
Cost 54
Less: Discount (20/5) (4)
50
Tutorial note: It is not correct to recognise interest on a straight line basis. It
is used here as a simplification. IAS 39 requires the recognition of interest
using the effective rate.

Emile Woolf International 340 The Institute of Chartered Accountants of Pakistan


Answers

(W7) Pension
Rs. m
Scheme assets
Cash 250
Expected return 26
Actuarial gain (bal fig) 26
Fair value of scheme assets 302
Scheme liabilities
Current service cost 276
Interest cost 41
Present value of obligation 317

Pension scheme liability (317 302) 15


Expense in profit or loss
Current service cost 276
Interest cost 41
Expected return (26)
Expense in profit or loss 291
(W8) Consolidated retained earnings
Rs. m
Plain (given) 390
Stripes post-acquisition retained earnings
(210 175 (W1)) 80% 28
Loss on acquiring control (W2) (3)
Goodwill impairment attributable to parent (W3) (12)
Share of post-acquisition profits of associate (W5) 35
Discount on investment (W6) (4)
Pension cost (W7) (291)
Actuarial gain (W7) 26
169
(W9) Non-controlling interest in Stripes
Rs. m
Book value (20% 470) 94
Fair value adjustment (20% 25) 5
Goodwill (3 impairment 3) (W3) 0
99

Emile Woolf International 341 The Institute of Chartered Accountants of Pakistan


Advanced accounting and financial reporting

22.5 MANGO LTD


Mango Ltd
Consolidated statement of financial position as at 31 March 2016
Rs. m
Non-current assets
Non-current assets (3,295 + 2,000 + 226 + 2) 5,523.0
Goodwill (W2) 89.0
Current assets (1,685 + 861) 2,546.0
Total assets 8,158.0

Equity and liabilities


Share capital 850.0
Retained earnings (W4) 3,405.9
Other components of equity (W5) 257.0
4,512.9
Non-controlling interest (W3) 649.1
5,162.0
Non-current liabilities (1,895 + 675) 2,570.0
Current liabilities (320 + 106) 426.0
Total equity and liabilities 8,158.0
Workings
(W1a) Net assets in subsidiary at acquisition before measurement
period adjustments
At end of At
reporting period acquisition
Rs. m Rs. m
Share capital 1,020 1,020
Retained earnings 980 900
Other components of equity 80 70
1,990

Fair value adjustments:


Contingent liability (6)
Property 266
260
Fair value of net assets (given) 2,250
The total fair value adjustment of Rs. 260 million above is taken as a
balancing figure as is the fair value adjustment that relates to
property.
The amount in respect of the contingent liability and an amount within
the property adjustments is subsequently found to be incorrect. This
information came to light in the measurement period. Therefore, they
retrospectively adjust the carrying amount of goodwill. In this case the
easier approach is to calculate goodwill using the corrected figures.

Emile Woolf International 342 The Institute of Chartered Accountants of Pakistan


Answers

(W1b) Net assets in subsidiary at acquisition after measurement


period adjustments
At At
consolidation acquisition
Rs. m Rs. m
Share capital 1,020 1,020
Retained earnings 980 900
Reduction of depreciation
recognised on the buildings
(Rs. 40/20 years) 2
Adjustment re recognition of the
provision 5
987 900
Other components of equity 80 70
1,067 1,990
Fair value adjustments:
Contingent liability (6 1) (5) (5)
Property (266 40) 226 226
Fair value of net assets 2,308 2,211
The contingent liability at acquisition is recognised for consolidation
purposes as a deduction from the net assets of the subsidiary. It is
not recognised in the subsidiarys own accounts as it does not pass
the IAS 37 recognition criteria.

The contingent liability has evolved into a provision by the date of


consolidation. This means that it is recognised as a liability and the
amount has been expensed in the subsidiarys own financial
statements. The adjustment made above (Dr Fair value adjustment
and Cr Retained earnings) is made because the expense which has
been recognised by the subsidiary since the date of acquisition
relates to an amount that has already been recognised in the
consolidation workings at acquisition.
(W2) Goodwill
Rs. m
Cost of investment 975
Fair value of initial holding 705
Fair value of NCI at date of acquisition 620
2,300
Net assets acquired (W1b) (2,211)
89
(W3) Non-controlling interest
Rs. m
Fair value of NCI at date of acquisition 620.0
NCIs share of post-acquisition growth in:
Retained earnings (30% of (987 900)) 26.1
Other components in equity (30% of (80 70)) 3.0
649.1

Emile Woolf International 343 The Institute of Chartered Accountants of Pakistan


Advanced accounting and financial reporting

(W4) Retained earnings


Rs. m
Mango Ltds balance as per the question 3,340.0
fair value adjustment re initial holding (705 700) 5.0
Share of post-acquisition growth (70% of (987 900)) 60.9
3,405.9
(W5) Other components of equity
Rs. m
Mango Ltds balance as per the question 250.0
Share of post-acquisition growth (70% of (80 70)) 7.0
257.0

Emile Woolf International 344 The Institute of Chartered Accountants of Pakistan


Answers

CHAPTER 23: COMPLEX GROUPS

23.1 PARVEZ LTD

(a) Parvez Ltd: Consolidated statement of profit or loss for the year
ended 31 December 2016
Rs. 000
Revenue (W4) 92,360.0

Cost of sales (W4) (28,123.0)


Gross profit 64,237.0
Distribution costs (3,325 + 2,137 + 1,900) (7,362.0)
Administrative expenses (3,475 + 950 + 1,900) (6,325.0)
Operating profit 50,550.0
Interest paid (325.0)
Profit before tax 50,225.0
Tax (17,931.0)
Profit after tax 32,294.0

Attributable to:
Ordinary shareholders of parent 28,580.8
Non-controlling interest (W9) 3,713.2
32,294.0

(b) Parvez Ltd: Consolidated statement of financial position as at 31


December 2016
Rs. 000
Goodwill (W6) 3,963.5
Property, plant and equipment
(35,483 + 24,273 + 13,063) 72,819.0
Current assets (W3) 19,446.0
96,228.5

Share capital 8,000.0


Retained earnings (W8) 56,641.3
Non-controlling interest (W7) 8,453.2
Current liabilities (13,063 + 10,023 + 48) 23,134.0
96,228.5

Emile Woolf International 345 The Institute of Chartered Accountants of Pakistan


Advanced accounting and financial reporting

(W1) Group structure

(W2) Unrealised profit adjustments: Inter-company trading


Vazir to Saad to Total
Saad Parvez
Sales 480,000 260,000 740,000

Inventory held 75,000 60,000


Unrealised profit adjustment u 25/125 u 33.3/133.3
Unrealised profit 15,000 15,000 30,000

NCIs share (based on selling


companys NCI)
10% u 15 1,500
28% u 15 4,200

Double entry in consolidated financial


statements Dr Cr
Cost of sales (closing inventory) 30,000
Closing inventory in statement of financial
position 30,000

NCI in the statement of financial position 5,700


NCI in the statement of comprehensive income 5,700

Impact on consolidated retained earnings (30 5.7) 24,300

(W3) Consolidated current assets


Rs. 000
Parvez 1,568
Saad 9,025
Vazir 8,883
Unrealised profit (W2) (30)
19,446

Emile Woolf International 346 The Institute of Chartered Accountants of Pakistan


Answers

(W4) Consolidated sales and cost of sales


Cost of
Sales sales
Rs. 000 Rs. 000
Parvez 45,600 18,050
Saad 24,700 5,463
Vazir 22,800 5,320
Inter-company sales (740) (740)
Unrealised profit 30
92,360 28,123

(W5) Net assets summary


At date of At date of Post-
Saad Ltd consolidation acquisition acquisition
Rs. 000 Rs. 000
Share capital 3,000 3,000
Retained earnings 24,075 1,425 22,650

27,068 4,425

At date of At date of Post-


Vazir Ltd consolidation acquisition acquisition
Rs. 000 Rs. 000
Share capital 2,000 2,000
Retained earnings 19,898 950 18,948

21,898 2,950

(W6) Goodwill

In Saad Ltd In Vazir Ltd


Rs. 000 Rs. 000
Cost 6,650.0
90% 3,800 3,420.0
NCI at acquisition
10% 4,425 W5 442.5
28% 2,950 W5 826.0
7,092.5 4,426.0
Net assets at acquisition (4,425.0) (2,950.0)

2,667.5 1,296.0

Total 3,963.5

Emile Woolf International 347 The Institute of Chartered Accountants of Pakistan


Advanced accounting and financial reporting

(W7) Non-controlling interest (statement of financial position)


Rs. 000 Rs. 000
NCI at acquisition
Saad Ltd: 10% 4,425 (W5) 442.5
Vazir Ltd: 28% 2,950 (W5) 826.0
NCIs share of post-acquisition profits
10% 22,650 (W5) 2,265.0
28% 18,948 (W5) 5,305.4
NCI in Saad Ltd s share of net assets of Vazir Ltd
10% 3,800 (380.0)

2,327.5 6,131.4

Total 8,458.9
Unrealised profit (W2) (5.7)

8,453.2

(W8) Consolidated retained earnings carried forward


Rs. 000
All of Parvez Ltd
Per the question 22,638.0
Share of Saad Ltd
90% u 22,650 (W5) 20,385.0

Share of Vazir Ltd


72% 18,948 (W5) 13,642.6
Unrealised profit (W2) (24.3)

56,641.3

(W9) Non-controlling interest (statement of profit or loss)


Saad Ltd Vazir Ltd
Rs. 000 Rs. 000
Profit after tax 10,760 9,439
10% 28%

Non-controlling interest 1,076 2,642.9

Total (1,076 + 2,642.9) 3,718.9


Unrealised profit (W2) (5.7)

3,713.2

Emile Woolf International 348 The Institute of Chartered Accountants of Pakistan


Answers

23.2 HASAN, RIAZ AND SIDDIQ

Hasan Limited: Consolidated statement of financial position as at 31


December 2016
Rs.
Goodwill (W6) 26,250
Property, plant and equipment
(1,102,500 + 271,950 + 122,550) 1,497,000

Inventories (W4) 783,520


Receivables
(241,920  8,000 W2) + 129,680 + 29,750 17,500 W2) 375,850
Cash and bank balances
((88,200 + 8,000 W2) + 4,725 + 8,105) 109,030
2,791,650
Share capital 1,750,000
Retained earnings (W8) 181,795
Other reserves (W9) 402,500
Non-controlling interest (W7) 191,625
Payables
(95,480 + (86,645 + 12,500 W2) + 88,605 17,500 W2) 265,730
2,791,650

(W1) Group structure

Non-controlling
Hasans interest interest (balance)

In Riaz Ltd 75% 25%

In Siddiq Ltd
(40% + (75% u 20%)) 55% 45%

Emile Woolf International 349 The Institute of Chartered Accountants of Pakistan


Advanced accounting and financial reporting

(W2) Individual company adjustments: Transaction before the year-end not yet
accounted for
Books of Riaz
Purchase of inventory from Hasan Dr Cr
Closing inventory 12,500
Payable (to Hasan) 12,500

Books of Hasan
Cash received from Riaz Dr Cr
Cash 8,000
Receivable (from Hasan) 8,000

The inter-company balances can be reconciled as follows after these


adjustments have been processed:
Hasans Riazs
financial financial
statements statements
Receivable Payable
Given in the question
Receivable from Riaz (note 5 in the
question) 25,500
Payable to Hasan (note 7 in the
question) 5,000
Cash from Riaz (8,000)
Purchase from Hasan 12,500
17,500 17,500
These balances must be cancelled out on consolidation as follows:
Dr Cr
Consolidated payables 17,500
Consolidated receivables 17,500
(W3) Unrealised profit adjustments: Inter-company trading
Inventories held by Riaz purchased from Hasan
Rs.
Purchased 30 December 12,500
Purchased previously 10,400
Purchased previously 22,900
Mark up adjustment u 25/125
Unrealised profit 4,580

Emile Woolf International 350 The Institute of Chartered Accountants of Pakistan


Answers

Double entry in consolidated financial


statements Dr Cr
Cost of sales (closing inventory) 4,580
Closing inventory in statement of financial
position 4,580

There is no double entry for the NCI as all sales were from the parent.
(W4) Consolidated inventories
Rs.
Hasan 526,610
Riaz (163,290 + 12,500 (W2) 175,790
Vazir 85,700
Unrealised profit (W3) (4,580)
783,520

(W5) Net assets summary


At date of At date of Post-
Riaz Ltd consolidation acquisition acquisition
Rs. Rs.
Share capital 420,000 420,000
Accumulated profits 17,500 35,000 (17,500)
Other reserves 70,000 nil 70,000
437,500 455,000

At date of At date of Post-


Siddiq Ltd consolidation acquisition acquisition
Rs. Rs.
Share capital 175,000 175,000
Accumulated losses (17,500) (35,000) 17,500
157,500 140,000
(W6) Goodwill
In Riaz In Siddiq
Ltd Ltd
Rs. Rs.
Cost 367,500 49,000
75% 24,500 18,375
67,375
NCI at acquisition
25% 455,000 W5 113,750
45% 140,000 W5 63,000
481,250 130,375
Net assets at acquisition (455,000) (140,000)
26,250 (9,625)
The balance for Siddiq is a gain on a bargain purchase.

Emile Woolf International 351 The Institute of Chartered Accountants of Pakistan


Advanced accounting and financial reporting

(W7) Non-controlling interest (statement of financial position)


Rs. Rs.
NCI at acquisition
25% 455,000 W5 113,750
45% 140,000 W5 63,000
NCIs share of post-acquisition profits
25% (17,500) (W5) (4,375)
45% 17,500 (W5) 7,875
NCIs share of other reserves
25% 70,000 (W5) 17,500
NCI in Riaz Ltd s share of net assets of Siddiq Ltd
25% 24,500 (6,125)
120,750 70,875
Total 191,625
(W8) Consolidated retained earnings carried forward
Rs.
All of Hasan Ltd
Per the question 180,250
Unrealised profit (W2) (4,580)
175,670
Share of Riaz Ltd
75% (17,500) (W5) (13,125)
Share of Siddiq Ltd
55% 17,500 (W5) 9,625
Gain on bargain purchase (W6) 9,625
181,795
(W9) Consolidated other reserves
Rs.
All of Hasan Ltd
Per the question 350,000
Share of Riaz Ltd
75% 70,000 (W5) 52,500
402,500

Emile Woolf International 352 The Institute of Chartered Accountants of Pakistan


Answers

CHAPTER 24: DISPOSAL OF SUBSIDIARIES

24.1 PATCHE LTD


(a) (i) Statement of profit or loss and other comprehensive incomes for the
year ended 30 June 2016
Rs.m
Operating profit 390
Profit on disposal of shares (W1) 210
Profit before tax 600
Income tax (120)
Profit for the year 480
Other comprehensive income 60
540
(ii) Statement of changes in equity for the year ended 30 June 2016
Share Retained Total
capital earning
Rs.m Rs.m Rs.m
Balance b/f (opening Balance) 1,500 660 2,160
Profit for the year - 480 480
Balance c/d 1,500 1,140 2,640
(b) Consolidated statement of profit or loss and other comprehensive
incomes for the year ended 30 June 2016
Rs.m
Profit before tax (390 + 180) 570
Income tax expenses (120 + 60) (180)
Profit for the year 390
Other comprehensive income (60 + 30) 90
Total comprehensive income 480
Attributable to
Non-controlling interest (W2) 30
Members of the parent (480 30) 450
480
(c) Consolidated statement of financial position as at 30 June 2016
Non-current assets: Rs.m

Property plant and equipment (1605 + 534) 2,139


Goodwill (W3) 240
2,379
Current assets:
Inventories (960 + 570) 1,530
Trade receivables (750 + 525) 1,275
Cash and bank (240 + 267) 507

Emile Woolf International 353 The Institute of Chartered Accountants of Pakistan


Advanced accounting and financial reporting

3,312
5,691
Equity attributable to owners of the parent
Share capital 1,500.0
Reserves (W4) 1,432.5
2,932.5
Non-controlling interest (W6) 472.5
3,405
Current liabilities:
Trade payables (885 + 513) 1,398
Income tax payables (240 + 180 + 90 (W1)) 510
Provisions (285 + 93) 378
2,286
5,691
Workings:
1. Gain on disposal of shares in parents separate financial
statement:-
Rs.m
Fair value of consideration received 480
Less: Original cost of shares (765 x 20%/85%) (180)
Parent gain 300
Less tax on parents gain (30%) (90)
210
2. Non-controlling interest (NCI)
Profit for the year: Rs.m
9
Pre-disposal periods = /12 x 120m x 15% = 13.5
3
Post-disposal periods = /12 x 120m x 35% = 10.5 24
Other comprehensive income
Pre-disposal periods = 9/12 x 30m x 15% = 3.375
3
Post-disposal periods = /12 x 30m x 35% = 2.625 6
30

3. Goodwill
Rs.m
Consideration transferred (285 + 480) 765
Non-Controlling interest at fair value 135
Less:
Fair value of identifiable net assets
at acquisition:
Share capital 600
Pre-acquisition reserve 60 (660)
240

Emile Woolf International 354 The Institute of Chartered Accountants of Pakistan


Answers

4. Consolidated reserves
Rs.m
All of Patche
Per question at year-end 930.00
Adj. to equity on disposal (W5) 217.50
Tax on parent gain (W1) (90.00)
1,057.50
Groups share of post-acquisition reserve of Somers:
85% (412.5 (see below) x 85%) 350.63
65% (37.5 (see below) x 65%) 24.37
1,432.50

Post-acquisition profits of Somers


Rs.m
Per question at year-end 510.0
Profit since the date of disposal (when Somers is a
65% subsidiary): Rs. 150m u 3/12 (37.5)
Balance on reserves at the date of disposal 472.5
Balance on reserves at the date of acquisition (60.0)
Post-acquisition reserves up to the date of disposal
(for which period Somers was an 85% subsidiary) 412.5

5 Adjustment to equity on disposal of shares in group financial


statement
Rs.m
Fair value on consideration received 480.00
Increase in NCI in net asset and goodwill at disposal
(196.8 x 20%/15%) (262.50)
217.50

6 Non-controlling interest
Rs.m
NCI @ acquisition 135.00
NCI share of post-acquisition reserve:
Somers (412.5 x 15%) 61.88
Somers (37.5 x 35%) 13.12
Increase in NCI (W5) (214.5 + 48) 262.50
472.50

Emile Woolf International 355 The Institute of Chartered Accountants of Pakistan


Advanced accounting and financial reporting

24.2 DISPOSAL
Rs. Rs.
million million
Consideration from sale of shares 960
Fair value of retained shares in Spool 100
1,060
Net assets of Spool at carrying value 800
Minus: non-controlling interest de-recognised (10% u 800) (80)
720
Gain on sale of shares 340
None of the assets of Spool have been re-valued, therefore there is no balance
on a revaluation reserve; therefore none of this gain should be transferred
directly to retained earnings and not reported in profit or loss.
There is no information to suggest that a reclassification adjustment is required to
reclassify income previously reported as other comprehensive income as profit or
loss.
The total gain of Rs. 340 million on disposal of the shares should therefore be
recognised in profit or loss for the period.
Hoo will recognise an investment in Spool in its statement of financial position in
accordance with the requirements of IAS 39. On initial recognition, this
investment should be valued at Rs. 100 million.

24.3 PART DISPOSAL

The disposal of 10% of the shares in S leaves P with a controlling interest;


therefore the disposal of the shares should be accounted for as an equity
transaction between owners of the group. No gain or loss is recognised in the
consolidated financial statements of P.
It is assumed that the profits of S for the year were Rs. 200 million (all retained;
therefore Rs. 900 million - Rs. 700 million). At 30 June it is assumed that profits
for the year to date were Rs. 100 million (= Rs. 200 million u 6/12); therefore the
net assets of S at this date were Rs. 800 million.

P NCI

Rs. m Rs. m

Before the share sale (80% u 800) 640 (20%) 160

After the share sale (70%) 560 (30%) 240

Change in interest in S - 80 + 80

The shares were sold for Rs. 94 million adding to the assets in Ps statement of
financial position. The transaction should therefore be accounted for in equity as
follows:
Debit: Cash Rs. 94 million
Credit: NCI Rs. 80 million
Credit: Reserves attributable to P (= gain = balance) Rs. 14 million

Emile Woolf International 356 The Institute of Chartered Accountants of Pakistan


Answers

Rs. Rs.
million million
Post-acquisition profit attributable to S (see above) 200
Less: Impairment of goodwill (8)
Recognised profit 192
Attributable to equity owners of P
1 January 30 June (80% u 200 u 6/12) 80
1 July 31 December (70% u 200 u 6/12) 70
Goodwill impairment (8)
Attributable to NCI 142
1 January 30 June (20% u 200 u 6/12) 20
1 July 31 December (30% u 200 u 6/12) 30
50
192

24.4 THE A GROUP


As original investment in C was 90% of Cs 400,000 shares (360,000 shares).
During the year A has disposed of 350,000 of these shares, which reduces the
investment from subsidiary status to that of a simple investment.
A Group
Consolidated statement of financial position
as at 31 December Year 4
Rs.000
Goodwill (W2, B only) 428
Investment in C at fair value 44
Other net assets (W4) 6,661
7,133
Equity
Share capital 1,500
Accumulated profits attributable to owners of A (working 1) 5,347
Equity attributable to owners of A 6,847
Non-controlling interest: 20% u (1,260 + 170) 286
Total equity 7,133
Statement of profit or loss for year ended 31 December Year 4
Working
A B Group
Rs.000 Rs.000 Rs.000
Operating profit 1,200 250 1,450
Minus: Dividend from B (16) nil (16)
1,184 250 1,434
Gain on disposal of C (W2) 237 237
Profit before tax 1,421 250 1,671
Tax (360) (60) (420)
Profit after tax 1,061 190 1,251

Emile Woolf International 357 The Institute of Chartered Accountants of Pakistan


Advanced accounting and financial reporting

Attributable to:
Equity owners of A (1,061 + 80% u 190) 1,213
Non-controlling interest: 20% 190 38
1,251
Workings
(1) Movement on consolidated reserves attributable to owners of parent
A B C Group
Rs.000 Rs.000 Rs.000 Rs.000
At 31 December Year 3 (W5) 3,300 272 612 4,184
Profit for year attributable to A 1,213
Dividends paid by A (50)
At 31 December Year 4 5,347
(2) Disposal of shares in C, with loss of control
Gain to parent Rs.000 Rs.000
Net assets of C at date of disposal: de-recognised 1,400
Purchased goodwill in C de-recognised
(see working 3) 472
1,872
Minus: Non-controlling interest de-recognised
(10% u 1,400) (140)
Assets attributable to A de-recognised 1,732
Fair value of investment retained 44
Sale proceeds 1,925
1,969
Total gain on disposal of shares 237
Since there has been no revaluation of non-current assets and there is no
information about any reclassification adjustments that might be required, it
is assumed that this entire gain should be included in profit or loss for the
year.

(3) Calculation of goodwill


B C
Rs.000 Rs.000
Cost of Investment 1,164 1,120
Less: Group share of the fair value of the net
assets at acquisition
80% (500 + 420) (736)
90% (400 + 320) (648)
428 472

Emile Woolf International 358 The Institute of Chartered Accountants of Pakistan


Answers

(4) Other net assets


Rs.000
As net assets as 1 January Year 4 2,516
Bs net assets at 1 January Year 4 1,260
As retained profit year ended 31 December Year 4 790
Bs retained profit year ended 31 December Year 4 170
Proceeds of disposal of C 1,925
6,661
(5) Calculation of post-acquisition retained profits b/f attributable to A
Rs.000
A As given in the question 3,300
B and Group share of post-acquisition
C
B 80% u (760 - 420) 272
C 90% u (1,000 - 320) 612
Total 4,184

24.5 BARTLETT LTD


Consolidated statement of profit or loss for the year ended
31 December 2016
Total
Rs.
Revenue (1,926,500 + 396,200 + 260,800) 2,583,500

Cost of sales (1,207,200 + 202,950 + 193,100) (1,603,250)


Gross profit 980,250
Net operating expenses (400,100 + 152,650 + 52,650) (605,400)
Operating profit 374,850
Profit on disposal of
operations (W1) 66,360
Profit on ordinary
activities before
taxation 441,210
Tax on profit on
ordinary activities (110,000 + 4,750 + 13,750) (128,500)
Profit for the year 312,710

Profit attributable to: Rs.


Owners of the parent 286,162
Non-controlling interests (W2) (11,458)
Profit for the year 312,710

Emile Woolf International 359 The Institute of Chartered Accountants of Pakistan


Advanced accounting and financial reporting

Workings
(1) Profit on disposal of Lymon
Recognise: Rs.
Proceeds 212,000
Derecognise:
Net assets of subsidiary
Net assets at January 2016 140,000
Profit to 1 July 2016 (6/12 x 20,600) 10,300
150,300
Non-controlling interest (20%) (30,060)
(120,240)
Unimpaired goodwill (25,400)
Profit on disposal 66,360
(2) Non-controlling interests
In Lymon Inc 20% x (6/12 x Rs. 20,600) 2,060
In Zeigler Inc 35% x (6/12 x Rs. 53,700) 9,398
11,458

Emile Woolf International 360 The Institute of Chartered Accountants of Pakistan


Answers

CHAPTER 26: FOREIGN CURRENCY

26.1 DND LIMITED


Date Description Dr. Cr.
Rs. Rs.
1-Jul-16 Advance to suppliers 1,210,000
Cash 1,210,000
(Amount paid on signing the contract.
Exchange rate was Rs. 60.5/US$)
30-Sep-16 Advance to suppliers 3,050,000
Cash 3,050,000
(Amount paid on delivery. Exchange rate
was Rs. 61/US$)
30-Sep-16 PPE in transit/ CWIP 6,090,000
Advance to suppliers 4,260,000
Payable to suppliers 1,830,000
(Recording of asset on the delivery date as
risk and rewards are transferred to the
company)
31-Dec-16 Exchange loss 6,000
Payable to suppliers 6,000
(Adjustment of exchange rate as of
reporting date. Exchange rate was Rs.
60.5/US$)
31-Jan-17 Property, plant and Equipment 6,090,000
PPE (In transit/ in progress) 6,090,000
(Transfer the new plants and machineries to
Property, Plant and Equipment)
31-Jan-17 Payable to suppliers 1,836,000
Exchange loss (Bal.) 9,000
Cash 1,845,000
(Final payment to supplier. Exchange rate
was Rs. 61.5/US$1)

26.2 STARLIGHT LIMITED


(a) Starlight Limited. Translated Profit and Loss Account
QR000 Rate Rs.000
Turnover 344,880 32 10,778
Cost of Sales (249,710) 32 (7,804)
Gross Profit 95,170 2,974
Expenses (29,490) 32 (922)
Profit Before Tax 65,680 (2,052)
Taxation (17,325) 32 (541)
Profit After Tax 48,355 1,511
Interim dividend (16,300) 32 (509)
Retained profit for the year 32,055 1,002

Emile Woolf International 361 The Institute of Chartered Accountants of Pakistan


Advanced accounting and financial reporting

(b) (i) Calculation of goodwill


QR000 QR000
Cost of investment:
(Rs. 2,500,000 x 30 QR) 75,000
Less net assets acquired:
Share capital 20,250
Preacquisition reserves 49,300
69,550
Group share 80% thereon (55,640)
Goodwill in QR 19,360
Translated to rupees at the closing rate:
QR 19,360,000 33 = Rs. 586,667
(ii) Noncontrolling Interest in statement of profit or loss:
20% x Rs. 1,511,000 = Rs. 302,200

26.3 PERCEPT LTD


(a) Translation of financial position of Trint Ltd as at 31 December 2016
YEN YEN RATE
Rs.000 Rs.000 Rs.000
Property plants and equipment 12,375 5 2,475
Financial assets 1,250 5 250
Current assets 8,250 5 1,650
21,875 4,375

Share capital 5,000 6 833


Retained earnings 4,500 6 750
Fair value adjustment 2,875 6 479
Post-acquisition (Balancing figure) 3,000 1,013
Non-current liabilities 4,000 5 800
Current liabilities 2,500 5 500
21,875 4,375

Note: Fair value of property,


plant and equipment: = 12,375 Yen 5,000 yen (equity)
- 4,500 Yen (pre-acquisition profit) = 2,875 Yen
(b) Goodwill
Yen Rate Rs.
000 000
Cost of acquisition 6,900 6 1,150
Fair value (NCI) 6,250 6 1,042
13,150 2,192
Net asset acquired 12,375 6 (2,063)
775 6 129

Emile Woolf International 362 The Institute of Chartered Accountants of Pakistan


Answers

Retranslated at the closing rate 775 5 155


Exchange gain on retranslation of goodwill 26

Parents share (70% x 26) = 18.2


Non-controlling interest share (30% x
= 7.8
26)
26
(c) Exchange rate difference arising on re-translation of Trint Ltds net
assets
Rs.000
Difference from translation of opening net assets
Opening rate: 12,375,000 Yen at 6 2,062.5
Closing rate: 12,375,000 Yen at 5 2,475.0
Exchange gain 412.5
Difference arising from translation of profit
Average rate (2,000 Yen at 5.5) 364.0
Closing rate (2,000 Yen at 5) 400.0
Exchange gain 36.0
Total exchange gain 448.5

Rs.000
Parents share of the exchange gain (70% of 448.5) 314.0
Non-controlling interest share of the exchange gain (30% x 448.5) 134.5
448.5

26.4 ORLANDO
(a) Year to June Year 4
The revenue and the receivable for the sale of 96,000 should be
translated at the spot rate of 0.8 = $120,000
The capital expenditure of 1m should also be translated at the spot rate of
0.8:
Debit Property, plant and equipment $1,250,000
Credit: Payables $1,250,000.
The receipt on 12 June relating to the receivable is translated at the rate at
that date of 0.9. This generates cash of $106,667 to settle a receivable of
$120,000. Hence an exchange loss of $13,333 is recognised in profit or
loss.
The non-current asset is not re-translated at the year end, but the
outstanding payable (a monetary item) must be re-stated to the year end
exchange rate of 0.7. This gives a year-end payable balance of $1,428,571.
This has increased from the initial $1,250,000; therefore an exchange loss
of $178,571 will be recognised in profit or loss.

Emile Woolf International 363 The Institute of Chartered Accountants of Pakistan


Advanced accounting and financial reporting

(b) Year to June Year 5


When the payable is settled after the year end at the spot rate of 0.8, it
results in a payment of $1,250,000. There is an exchange gain of $178,571
compared with the carrying value at the end of Year 4.

26.5 MANCASTER AND STOCKPOT


Part A
(1) (a) Functional currency
Functional currency is the currency of the primary economic
environment in which the entity operates. It reflects the underlying
transactions, events and conditions that affect the company. It is not
simply the currency of the country where the company is based. For
example, if a company is incorporated and listed in Pakistan but
operates in the South African mine fields, then its functional currency
will be the South African rand, not rupees, because the rand is the
currency in which it generates and spends the most cash.
(b) Presentation currency
Presentation currency is the currency in which the financial
statements are presented. Continuing the example of the company
referred to above, since the company is listed in Pakistan it will
present and file its accounts in rupees.
Transactions and balances measured in a currency other than the
presentation currency will have to be translated into the presentation
currency for reporting purposes. So the sales and operating costs
incurred in South Africa will need to be translated from Rand into
Sterling.
(2) Factors to be considered when determining the functional currency of
an overseas subsidiary
The primary economic environment in which an entity operates is defined in
IAS 21 as the one in which it primarily generates and expends cash.
Primary indicators
An entity must consider the following factors in determining its functional
currency:
the currency that mainly influences sales prices for goods and
services
the country whose competitive forces and regulations mainly
determined the sales prices of its goods and services
the currency that mainly influences labour, material and other costs of
goods or services.
Additional indicators
The following factors may also provide evidence:
the currency in which funds from financing activities are generated
(i.e. currency used for issuing debt and equity)
the currency in which surplus cash is invested.

Emile Woolf International 364 The Institute of Chartered Accountants of Pakistan


Answers

Part B
(a) Translation: Statement of financial position of Stockpot at 31 March
Year 4
EU000 Rate $000
Property, plant and equipment 30,000 2.2 13,636
Inventories 18,000 2.2 8,182
Trade receivables 15,000 2.2 6,819
Trade payables (10,400) 2.2 (4,727)
Bank overdraft (7,600) 2.2 (3,455)
Non-current liabilities (20,000) 2.2 (9,091)
25,000 11,364
Issued capital 15,000 3.0 5,000
Pre-acquisition reserves 5,000 3.0 1,667
20,000 6,667
Post-acquisition reserves 5,000 balancing figure 4,697
25,000 11,364
Mancaster Group: Consolidated statement of financial position at
31 March Year 4
$000 $000
Non-current assets
Goodwill (see workings) 682
Property, plant and equipment (20,000 + 13,636) 33,636
Current assets:
Inventories (10,000 + 8,182) 18,182
Trade receivables (10,000 + 6,819) 16,819
35,001
69,319
$000
Capital and reserves:
Issued capital 9,000
Accumulated profits (see workings) 16,205
25,205
Non-controlling interest (see workings) 2,841
28,046
Non-current liabilities:
Loans (10,000 + 9,091) 19,091
Current liabilities:
Bank overdraft (6,100 + 3,455) 9,555
Trade payables (7,900+4,727) 12,627
22,182
69,319

Emile Woolf International 365 The Institute of Chartered Accountants of Pakistan


Advanced accounting and financial reporting

(b) Translation: Statement of profit or loss of Stockpot for year ended 31


March Year 4
EU000 Rate $000
Revenue 60,000 2.3 26,087
Cost of sales (30,000) 2.3 (13,043)
Gross profit 30,000 13,044
Operating expenses (16,000) 2.3 (6,957)
Operating profit 14,000 6,087
Interest payable (2,000) 2.3 (870)
Profit before tax 12,000 5,217
Tax (4,200) 2.3 (1,826)
Profit after tax 7,800 3,391
The statement of profit or loss has been translated at the average rate as
an approximation to the actual (historical) rate. The closing rate is not
allowed under IAS 21.
Mancaster Group: Consolidated statement of profit or loss for the
year ended 31 March Year 4
$000
Revenue (50,000 + 26,087) 76,087
Cost of sales (25,000 + 13,043) (38,043)
Gross profit 38,044
Operating expenses (15,000 + 6,957) (21,957)
Operating profit 16,087
Interest payable (1,000 + 870) (1,870)
Profit before tax 14,217
Tax (3,600 + 1,826) (5,426)
Profit after tax 8,791
Attributable to
Equity holders of the parent 7,943
Non-controlling interest (25% 3,391) see translation 848
8,791
Workings
(1) Goodwill at date of acquisition
$000 Rate EU000
Cost of investment 5,500 u3 16,500
Minus: Share of net assets acquired:
Share capital (translated at 3.0) 5,000
Accumulated profits (translated at 3.0) 1,667
6,667
Group share (75%) 5,000 u3 (15,000)
Goodwill 500 1,500
Re-stated to closing rate: (1,500/2.2) 682
Translation gain on goodwill to group
reserves 182

Emile Woolf International 366 The Institute of Chartered Accountants of Pakistan


Answers

(2) Consolidated accumulated profits


$000
Mancaster: 12,500
Stockpot: group share of post-acquisition profits (75%
4,697) see translation of statement of financial
position 3,523
Translation gain on goodwill 182
16,205
(3) Non-controlling interest
$000
Non-controlling share of net assets at 31 March
Year 4 : (25% 11,364) see translation of Stockpot
statement of financial position 2,841

26.6 A, B AND C
A group: Summarised consolidated statement of profit or loss and other
comprehensive income for the year ended 30 September 2016
Rs.000
Revenue (4,600 +3,385(W1)) 7,985
Costs and expenses (3,700+2,462(W1)) (6,162)
Share of associates profit (W3) 160
Profit before tax 1,983
Income tax expense (200+231(W1)) (431)
Profit for the year 1,552

Other comprehensive income


Revaluation gains net of tax (200+185(W1)) 385
Share of associates OCI (W3) 28
Forex gain in year (W4) 803
Total other comprehensive income 1,216
Total comprehensive income 2,768

Profit for year attributable to:


Equity holders of the parent 1,414
Non-controlling interest (W5) 138
1,552
Total comprehensive income attributable to:
Equity holders of the parent 2,432
Non-controlling interest (W5) 336
2,768

Emile Woolf International 367 The Institute of Chartered Accountants of Pakistan


Advanced accounting and financial reporting

Consolidated statement of financial position for the A group as at


30 September 2016
Rs.000
Assets
Non-current assets
Property, plant and equipment (7,000 + 6,349 (W1)) 13,349
Goodwill (W2) 635
Investment in associate (W6) 1,220
15,204

Current assets (3,000 + 3,175 (W1)) 6,175


Total assets 21,379
Equity and liabilities
Equity attributable to the parent
Share capital 2,000
Retained reserves (W8) 13,522
15,522
Non-controlling interest (W7) 1,476
Total equity 16,998
Current liabilities (2,000 + 2,381(W1)) 4,381
Total equity and liabilities 21,379
Rate @ avge
W1 Translation of B
A$000 rate Rs.000
Statement of profit or loss and other
comprehensive income
Revenue 2,200 Rs./A$0.65 3,385
Cost of sales and expenses (1,600) Rs./A$0.65 (2,462)
Profit before tax 600 923
Income tax (150) Rs./A$0.65 (231)
Profit for year 450 692
Other comprehensive income:
Revaluation gains on PPE Total OCI 120 Rs./A$0.65 185
120 185
Total comprehensive income 570 877
Statement of financial position
Non-current assets
Property, plant and equipment 4000 @CR A$0.63 6349
Current assets 2,000 @CR A$0.63 3,175
Total assets 6,000 9,524
Share capital 1000 @HR A$0.50 2000
Pre-acquisition reserves 1800 @HR A$0.50 3600
Post-acquisition reserves 1,700 Bal fig 1,543
Total equity 4,500 7143
Current liabilities 1,500 @CR A$0.63 2,381
Equity and liabilities 6,000 9524

Emile Woolf International 368 The Institute of Chartered Accountants of Pakistan


Answers

W2 Goodwill A$000 Rate Rs.000


Consideration transferred 2,600 Rs./A$0.50 5,200
NCI @ FV 600 Rs./A$0.50 1,200
Net assets acquired:
Share capital (1,000) Rs./A$0.50 (2,000)
Retained earnings (1,800) Rs./A$0.50 (3,600)
Goodwill at 1 October 2013 400 800
Forex loss (balancing figure) (237)
Goodwill at 30 September 2015 400 Rs./A$0.71 563
Forex gain (balancing figure) 72
Goodwill at 30 September 2016 400 Rs./A$0.63 635

W3 Share of associates profit/OCI Rs.000


Share of associates PFY (40% x Rs. 400,000) 160
Share of associates other comprehensive income (40% x Rs. 70,000) 28

W4 FOREX gains/losses in the year Rs.000


Closing net assets @ CR (A$4,500,000/0.63) or from W1 7,143
Less opening net assets @ OR ((A$4,500,000 less TCI (5,535)
A$570,000)/0.71)
Less TCI for year @ average rate (A$570,000/0.65) (877)
Forex gain on translation of subsidiarys net assets 731
Plus Forex gain on translation of goodwill 72
Total Forex gains on translation of subsidiary 803
W5 NCI share of Profit/Total comp income PFY TCI
Rs.000 Rs.000
Subsidiarys PFY/TCI (W1) 692 877
20% share 138 175
Forex gain on translation of subsidiary (20% x
Rs. 803,000) 161
138 336

W6 Investment in associate Rs.000


Investment at cost 900
Plus share of post-acquisition reserves 40% x
(Rs. 1,500,000 -Rs. 700,000) 320
1,220

Emile Woolf International 369 The Institute of Chartered Accountants of Pakistan


Advanced accounting and financial reporting

W7 Non-controlling interest Rs.000


NCI on acquisition (W2) 1,200
NCI share of post-acquisition reserves of subsidiary
(20%xRs. 1,543,000(W1)) 309
NCI share of net FOREX losses on translation of goodwill
(20% x Rs.(237,000-72,000)) (33)
NCI at 30 September 2016 1,476

W8 Reserves A B
Rs.000 Rs.000
As per SOFP 12,100 5,143

Less pre-acquisition reserves (W1) (3,600)


1,543
Group share 80% x Rs. 1,543,000 1,234
Group share of associates post-acquisition reserves 320
(W6)
Group share of net FOREX losses on translation of
goodwill (80% x Rs.(237,000-72,000)) (132)
Group reserves 13,522

26.7 OMEGA LIMITED


Omega Limited
Extract from Statement of comprehensive income for the year ended 31
December 2013
Profit for the year: Rupees
Dividend received from
AWL (IFRS
9,B5.7.5.1) (20,000*10*15%*26.5) 795,000
Transfer of FV gain reserve of 31-12-2012, on
derecognition of AWL investment W.1 500,000
FV / exchange gains on valuation of AWL shares on
1-6-2013 W.1 2,124,000
Loss on de-recognition of AWL' shares W.1 ( 308,000)
Other comprehensive income:
FV gain/(loss) on investment available for sale W.1 693,000
Exchange gain on investment available for sale W.1 225,225

Emile Woolf International 370 The Institute of Chartered Accountants of Pakistan


Answers

W-1 FV per Gain /


Investment
No. of share (loss) Remar
Date
shares Conv. ks
AED AED Rupees Rupees
@
1-May-
2012 20,000 12.00 240,000 25.00 6,000,000
31-
Dec-
2012 20,000 13.00 260,000 25.00 6,500,000 500,000 FV gain
1-Jun- 22,000 14.00 308,000 28.00 8,624,000 2,124,000 Gain on
(20,000x1.1)
2013 valuation
of AWL
on its
acquisitio
n by HL
1-Jun- 16,500 18.00 297,000 28.00 8,316,000 Loss on
(22,000/4*3)
2013 (308,000) de-
recognitio
n of
AWL
shares
31- 16,500 19.50 321,750 28.00 9,009,000 693,000
Dec-
2013 FV gain
31- 16,500 19.50 321,750 28.70 9,234,225 225,225 Exchange
Dec- gain
2013
3,234,225

26.8 PARENT COMPANY LIMITED


(i) Parent Company Limited
Consolidated statement of financial position as at 30 June 2014
Rs. in
million
Assets
Non-current assets
Property, plant and
equipment 4,200+3,500+25017.3 12,025.00
Intangible assets (W-1) 796+1,730 2,526.00
Current assets 3,500+4,000+45017.3 15,285.00
29,836.00
Equity and liabilities
Equity attributable to owners of
PL:
Ordinary shares capital 6,000.00
Retained earnings (W-4) 5,565.15
[(W-1) 253+(W-2)
Exchange reserve 813.20]] 75% 799.65
12,364.80
Non-controlling interest W-4 (731.20+2,050) 2,781.20
15,146.00
Current liabilities 4,700+4,800+30017.3 14,690.00
29,836.00

Emile Woolf International 371 The Institute of Chartered Accountants of Pakistan


Advanced accounting and financial reporting

(ii) Parent Company Limited


Consolidated statement of other comprehensive income
For the year ended 30 June 2014
Rs. in
million
Other comprehensive income:
Items that may be translated to profit or
loss:
Exchange gain on translation of
goodwill W-1 55.00
Exchange gain on translating of
foreign operations W-2 195.80
250.80

W-1: Goodwill and exchange gain


LS FS
thereon
---------- Rs. in million ----------
(300
Purchase consideration 2,000 15) 4,500
NCI fair value on acquisition date 540 (90 15) 1,350
2,540 5,850

Net assets on acquisition (120+260)


date (1,800/1.2+250) (1,750) 15 (4,200)
Estimated liability for a pending claim 6 -
Goodwill on acquisition date i.e. 1 July
2012 1,650
(10
Impairment on 30 June 2014 - 17.3) (173)
Goodwill as at 30 June 2014 1,477
Goodwill as at 30 June 2014 @ Rs. (10017.3
17.30 796 ) 1,730
Exchange gain reserve as 30 June
2014 253
Exchange gain reserve as 30 June (148.5
2013 75%) (198)
Exchange gain for the year 55

W-2: Exchange reserve on translation of FS foreign operations


CU in Conversio Rs. in
million n@ million
Net assets as at 30 June 2014 400.00 17.30 6,920.00
Net assets as at 30 June 2013
(400-30+18) 388.00 16.80 6,518.40
Profit for the year 30.00 17.00 510.00
Dividend paid during the year
(12015%) (18.00) 16.90 (304.20)
400.00 6,724.20

Emile Woolf International 372 The Institute of Chartered Accountants of Pakistan


Answers

Exchange gain for the year ended 30


Jun 2014 195.80
(463.05/75
Exchange reserve as at 30 June 2013 %) 617.40
Exchange gain on foreign operations as
at 30 June 2014 813.20

W-3: FS retained earnings CU in Conversio Rs. in


million n@ million
Net assets as at 30 June 2014 400.00 17.30 6,920.00

(4,200.0
Net assets on acquisition date (280.00) 15.00 0)
Post-acquisition retained earnings as at
30 June 2014 including OCI item of
exchange gain 120.00 2,720.00
Exchange gain to be classified to OCI W-2 (813.20)
Post-acquisition retained earnings as at
30 June 2014 1,906.80

W-4: Consolidated retained earnings Retaine


and NCI d
NCI - LS NCI - FS
earning
s
----- Rs. in million -----
3,500.0
Balance as at 30 June 2014 0 - -
NCI fair value on acquisition date - 540.00 1,350.00
Post-acquisition profit LS
(65080%) 520.00 130.00 -
Post-acquisition profit FS 1,430.1
(1,906.875%) 0 - 476.70
LS earnings used for bonus issue
240.00
(30080% /20%) 60.00 -
Liability paid in May 2014 booked on
4.80
acq. (680%/20%) 1.20 -
Exchange gain on translation of FS
- 266.55
(W-2) 1,066.2025% -
Goodwill impairment
(17375%/25%) (129.75) - (43.25)
5,565.1
5 731.20 2,050.00

Emile Woolf International 373 The Institute of Chartered Accountants of Pakistan


Advanced accounting and financial reporting

CHAPTER 27: IAS 7: STATEMENTS OF CASH FLOWS

27.1 EVERNEW LTD


Consolidated statement of cash flows for the year ended 31 December 2016
Rs.000 Rs.000
Profit before taxation 138,960
Adjustment for non-cash items:
Depreciation charges 72,720
Profit on disposal of subsidiary (W.1) (5,040)
Interest expenses (payable) 10,080
Operating profit before working
Capital changes 216,720
Changes in working capital
Increase in inventory (W2) (28,800)
Increase in Receivables (W2) (32,400)
Increase in Creditors (W2) 25,200
(36,000)
Cash generated from operations 180,720
Income tax paid (W.3) (37,080)
Net cash flow from operating activities 143,640
Cash flow from investing activities:
Purchases of non-current assets (W4) (111,240)
Sales of Pastit Limited (W5) 41,040
Net cash used in investing activities (70,200)
Cash flow from financing activities:
Redemption of 10% debenture (W6) (18,000)
Dividend paid to non-controlling interest (W7) (3,600)
Interest paid (10,080)
Net cash used in financing activities (31,680)
Net increase in cash & cash equivalent 41,760
Cash & cash equivalent b/f (14,400 36,000) (21,600)
Cash & cash equivalent c/f 20,160
Cash & cash equivalent c/f is represented by:
Cash in hand 63,360
Bank overdraft (43,200)
20,160

Emile Woolf International 374 The Institute of Chartered Accountants of Pakistan


Answers

Workings
(W1) Profit on disposal of subsidiary:
The entire 80% shareholding was sold.
Rs.000
Net asset of subsidiary sold (shown in the question) 43,200
Sales proceeds 39,600
Less Net asset sold x 80% = (80% x Rs. 43,200) 34,560
Profit on disposal of subsidiary 5,040
(W2) Movement in Working Capital
31/12/16 Add Less Bal. Cash flow
Rs.000 disposal 31/12/15 statement
Rs.000 Rs.000 Rs.000
Inventory 180,000 14,400 165,600 28,800
Receivables 151,200 18,000 136,800 32,400
Trade creditors (108,000) (10,800) (93,600) (25,200)
(W3) Income Tax Paid
Taxation
Rs.000 Rs.000
Tax on disposal 2,160 Balance b/f 39,240
Cash/Bank 37,080 Tax for the year P & L 46,800
Balance c/f 46,800
86,040 86,040
(W4) Non-current assets
Non-current assets
Rs.000 Rs.000
Balance b/f 360,000 Disposal 28,800
Cash/Bank 111,240 Depreciation (P & L) 72,720
Balance c/f 369,720
471,240 471,240
W5 Cash flow from sale of Pastit Limited
Rs.000
As per question 39,600
Add Bank overdraft of Pastit Limited on disposal 1,440
41,040
(W6) Movement on debenture
Rs.000
Balance b/f at 01/01/2015 90,000
Disposal of subsidiary (3,600)
Cash paid (bal. figure) (18,000)
Balance c/f at 31/12/2016 68,400

Emile Woolf International 375 The Institute of Chartered Accountants of Pakistan


Advanced accounting and financial reporting

(W7) Non-controlling interest


Rs.000 Rs.000
Disposal 8,640 B/d 41,400
Dividend paid to NCI 3,600 P&L 7,200
B/d 36,360
48,600 48,600

27.2 BELLA

Statement of cash flows for the year ended 31 March Year 6


Rs.000 Rs.000
Cash flows from operating activities
Profit before taxation 4,617
Adjustments for:
Depreciation (Working 1) 300
Loss on disposal of non-current asset (800 700) 100
Interest expense 60

5,077
Increase in inventories (280 100) (180)
Decrease in trade and other receivables (1,350 1,290) 60
Increase in trade payables (430 275) 155

Cash generated from operations 5,112
Interest paid (Working 3) (45)
Income taxes paid (Working 4) (185)

Net cash from operating activities 4,882
Cash flows from investing activities
Purchase of property, plant and equipment (6,000)
Proceeds from the sale of property, plant and equipment 700
Purchase of intangible assets (800 300) (500)

Net cash used in investing activities (5,800)
Cash flows from financing activities
Proceeds from the issue of share capital (Working 2) 865
Issue of long-term loan (600 500) 100
Dividends paid (350)

Net cash inflow from financing activities 615

Net decrease in cash and cash equivalents (303)
Cash and cash equivalents at the beginning of the period 45

Cash and cash equivalents at the end of the period (55 313) (258)

Emile Woolf International 376 The Institute of Chartered Accountants of Pakistan


Answers

Workings
(1) Property, plant and equipment (PPE)
Rs.000 Rs.000
PPE at net book value (NBV) at end of year 12,900
PPE at NBV at beginning of year 8,000
Disposals during the year at NBV (800)
(7,200)
5,700
Depreciation charge for the year (balancing figure) 300
PPE acquired during the year 6,000
(2) Share capital and premium
Share Share Total
capital premium
Rs.000 Rs.000 Rs.000
At end of year 1,900 95 1,995
At beginning of year 1,100 30 (1,130)
Cash receipts from share issue 865
(3) Interest payable
Rs.000
Accrued interest at beginning of year 25
Interest charge in profit and loss 60
85
Accrued interest at end of year (40)
Interest payments in the year 45
(4) Current tax payable
Rs.000
Tax payable at beginning of year 325
Tax charge in profit and loss 400
725
Tax payable at end of year (540)
Tax payments in the year 185

27.3 BISHOP GROUP

(a)
Statement of cash flows for year ended 31 December 20X2
Rs.000 Rs.000
Cash flows from operating activities (Note 1) 2,282
Interest paid (120 + 205) (325)
Dividends received 90
Taxation paid (W2) (117)

Net cash flows from operating activities 1,930

Emile Woolf International 377 The Institute of Chartered Accountants of Pakistan


Advanced accounting and financial reporting

Statement of cash flows for year ended 31 December 20X2


Rs.000 Rs.000

Cash flows from investing activities:


Payments to acquire tangible non-current assets (W3) (4,996)
Receipts from sale of tangible non-current assets 810
Purchase of investments (300)

Net cash used in investing activities (4,486)

Cash flows from financing activities:


Proceeds of share issue 3,824
Additional loans (1,200 800 25) 375
Capital payments under finance leases (W4) (150)
Dividends paid to NCI (W1) (295)
Equity dividends paid (600)

Net cash provided by financing activities 3,154

Net increase in cash and cash equivalents 598
Effect of exchange rate movements 53

651
Cash and cash equivalents brought forward 169

Cash and cash equivalents carried forward 820

Notes to the statement of cash flows
Reconciliation of operating profit to net cash inflow from operating activities
Rs.000
Operating profit 2,849
Depreciation 1,200
Profit on sale of non-current assets (810 720) (90)
Increase in inventories (6,135 5,740 117) (278)
Increase in receivables (5,720 4,380 339) (1,001)
Decrease in payables (1,420 1,760 58) (398)
Net cash flows from operating activities 2,282
Workings
(1)
Non-controlling interest
Rs.000 Rs.000
Dividend paid to NCI 295 Balance b/fwd 2,500
Balance c/fwd 2,800 Statement of profit or 420
loss
Exchange gain
(20% 875) 175
3,095 3,095

Emile Woolf International 378 The Institute of Chartered Accountants of Pakistan


Answers

(2)
Tax
Rs.000 Rs.000
Tax paid 117 B/fwd current tax 167
C/fwd current tax 700 B/fwd deferred tax 400
C/fwd deferred tax 550 Statement of profit or 800
loss
1,367 1,367
(3) Non-current assets
Rs.
Opening NBV 7,520
Depreciation (1,200)
Disposals at NBV (720)
New finance leases 700
Exchange rate gains 424
Purchase for cash 4,996
Closing NBV 11,720
(4)
Obligations under finance leases
Rs.000 Rs.000
Cash paid 355 Balance b/f < 1 year 50
Balance c/f < 1 year 110 Balance b/f > 1 year 250
Balance c/f > 1 year 740 Finance charge in profit or loss 205
Non current asset additions 700
1,205 1,205
The payment of Rs. 355,000 is split as Rs. 205,000 interest and Rs.
150,000 capital as payments are made in arrears and hence the year end
payment pays off the years finance cost.
(b) The statement of profit or loss and statement of financial position are based
on the accruals concept whereas the statement of cash flows is based on
the cash concept. Cash is the 'life blood' of the company and is therefore
critical to an entitys survival. Without cash to pay suppliers, the work force
and other payables, the company will cease to operate, irrespective of how
profitable it is.
Shareholders need to know that a company is viable and has the resources
to continue, and perhaps expand, operations. Suppliers need to know they
will be paid and customers need to know the company is in a position to
continue operations.
Profit may be significantly affected by the choice of accounting policies
made by a company. This means it is more subjective than cash and more
open to manipulation. However, the statement of cash flows itself may be
subject to window dressing, for example by delaying payment of suppliers
until after year end. The auditor needs to be involved in this respect to
ensure the shareholders and other users receive meaningful information.

Emile Woolf International 379 The Institute of Chartered Accountants of Pakistan


Advanced accounting and financial reporting

The statement of cash flows gives additional information not provided by


the other financial statements.

27.4 THE GRAPE GROUP


Group statement of cash flows for the year ended 31 March Year 4.
Rs.000
Cash flows from operating activities
Net profit before taxation 9,550
Adjustments for:
Depreciation (Note 1) 1,176
Loss on sale of assets 18
Income from associate (139)
Interest expense 552
Operating profit before working capital changes 11,157
Increase in inventories (1,127 139) (988)
Increase in receivables (273 85) (188)
Increase in payables (203 68) 135
Cash generated from operations 10,116
Interest paid (552)
Income taxes paid (W3) (2,400)
Net cash from operating activities 7,164
Cash flows from investing activities
Acquisition of subsidiary net of cash acquired (346 3) (343)
Purchase of property, plant and equipment (1,875 315) (1,560)
Proceeds from sale of property, plant and equipment (W1) 156
Dividends received from associate (W2) 93
Net cash used in investing activities (1,654)

Cash flows from financing activities Rs.000


Proceeds from issuance of share capital (675 + 519 - 152) 1,042
Repayment of loan notes (990)
Dividends paid (2,100)
Net cash used in financing activities (2,048)
Net increase in cash and cash equivalent 3,462
Cash and cash equivalents at beginning of period 1,728
Cash and cash equivalents at end of period 5,190
Notes to the statement of cash flows
(1) Major non-cash transactions
During the year the group purchased a subsidiary undertaking. Part of the
consideration for the acquisition was in the form of shares. Further details
of the acquisition are given below.

Emile Woolf International 380 The Institute of Chartered Accountants of Pakistan


Answers

(2) Purchase of subsidiary undertaking


Rs.000
Net assets acquired:
Property, plant and equipment 315
Inventories 139
Receivables 85
Cash at bank and in hand 3
Payables (68)
474
Goodwill 24
498
Satisfied by:
Shares allotted 152
Cash 346
498
Workings
(1) Proceeds from sale of property, plant and equipment
Rs.000
Cost of assets sold 429
Accumulated depreciation (255)
Loss on sale (18)
Proceeds 156
(2) Dividends received from associate
Interest in associate
Rs.000 Rs.000
Balance b/d 1,920 Dividends received 93
from associates
Share of associates' 139 Balance c/d 1,966
profit after tax
2,059 2,059
(3) Taxation
Taxation
Rs.000 Rs.000
Cash paid 2,400 Balance b/d 2,400
Balance c/d 2,950 Statement of profit or loss 2,950
5,350 5,350

Emile Woolf International 381 The Institute of Chartered Accountants of Pakistan


Advanced accounting and financial reporting

CHAPTER 28: IAS 33: EARNINGS PER SHARE

28.1 AIRCON LTD


(a) Earnings Per Share

=


1,854
2016 = Rs. = Rs. 1.01
1,818
1,584 6.06
2015 = x = Rs. 1.69
900 6.30
Workings
1. Calculation of theoretical ex-rights price
1 share at Rs. 6.30 each 6.30
2 rights issue for every 1 at Rs. 5.94 11.88
3 shares for 18.18

Price per share = = Rs. 6.06

2. Weighted average number of shares



1 April 30 Sept. 2015 = 900m x 6/12 x = 467.8

1 Oct. 31 March 2016 = 2,700 x 6/12 = 1,350
1,818

(b) Report
To: Mr Hamad
From: Management Accountant
Date: 15 April 2016
Subject: Evaluating the changes in EPS of Aircon Ltd
The key factors which has led to changes in the EPS of Aircon Ltd. are as
follows:
Revenue and profitability. Revenue increased by Rs. 2,700 million (18%)
last year, but the gross profit and net profit ratios have not increased
proportionately.
The gross profit percentage fell from 40% to 37% in 2016, while the net
profit percentage remained constant at 10%.
Factors responsible for the decline might be due to the inability of the entity
to maintain good profit margin coupled with the failure to also maintain
good control over operating expenses.
The more funds realised from the rights issue did not lead to any significant
increase in return on capital employed which fell from 43% (2,880/6,606) in
2015 to 25% (3,240/12,780) in 2016.
Capital employed: raising over Rs. 5,760 million of new finance was
largely used to acquire intangible assets.

Emile Woolf International 382 The Institute of Chartered Accountants of Pakistan


Answers

It is hoped that this asset will start generating substantial returns in the near
future.
EPS has therefore fallen from Rs. 1.69 in 2015 to Rs. 1.01 in 2016.
Signed
Management Accountant
APPENDIX TO THE REPORT
The ratios that are relevant to discussion and evaluation of changes in EPS
of Aircon Ltd are those that relate to profitability and return on capital
employed.
The effect of the rights issue should also be considered in the discussion in
relation to how the funds raised through the shares were employed.
TABLE OF RATIOS
(i) Change in revenue 18,000  15,300
= x 100 = 18% Increase
18,000

2016 2015

(ii) Costs of sales/revenue 11,340 6,120


= 63% = 40%
18,000 15,300

(iii) Gross profit % 6,600 6,120


= 37% = 40%
18,000 15,300

(iv) Net profit % 1,854


= 10% = 10%
18,000

(v) Operating expenses % 3,420 3,420


= 19% = 22%
18,000 15,300

(vi) Interest payable/sales 540 576


= 3% = 4%
18,000 15,300

(vii) Taxation/sales 846 720


= 5% = 5%
18,000 15,300

(viii) Capital employed 3,240 2,880


=25% = 43%
9,180  3,600 3,006  3,600

(ix) Assets/turnover 18,000 15,300


= 1.41 = 2.32
12,780 6,606

Emile Woolf International 383 The Institute of Chartered Accountants of Pakistan


Advanced accounting and financial reporting

Relevance of EPS to shareholders


(i) The EPS is used to compute the price earning (P/E) ratio, a major
market indicator to determine how successful a company has been
operating.
(ii) The price earning figure is a multiple of the EPS, where the multiple
represents the number of years earnings required to recoup the price
paid for the share.
(iii) Rising trend in EPS is a more accurate performance indicator than
rising trend in profit after tax. The investor should consider the future
economic conditions of an entity with some other ratios such as
dividend cover and ROCE.
(iv) EPS is a measure of performance from the existing and potential
investors perspective.
(v) EPS show the amount available to each ordinary shareholder thereby
indicating the potential returns on individual investment.
(vi) EPS is used to compare the activities of two entities in the same
industry.

28.2 CACHET LTD


BASIC EPS DILUTED
EPS
i. No change in share capital
PAT - Pref Div 69,000 - 1,380
= 20,700
No. of shares
= 3.27 N/A

ii. Bonus issue on 30 Sept. 2016:
No. of shares before bonus issue 20,700
Bonus (1 for 4) 5,175

No. of shares after bonus issue 25,875

PAT - Pref Div 69,000 - 1,380
=
No. of shares
25,875
= 2.61 N/A

iii. Rights issue on 1 Oct. 2016
Before rights issue 5 shares 1.80 20,700 9.00 37,260
Rights issue (1 for 5) 1 share 1.20 4,140 1.20 4,968

After rights issue 6 shares 3.00 24,840 10.20 42,228

Theoretical ex-right price (Rs. 1.70 1.70
10.20/6)

Bonus element of issue increases shares to 20,000 u 1.8/1.7 = 21,176


Full price element of issue increases shares to 20,700 u 6/5 = 24,840
Weighted average number of shares in issue
21,176 u 9/12 15,882
24,840 u 3/12 6,210
22,092

Emile Woolf International 384 The Institute of Chartered Accountants of Pakistan


Answers

EPS
PAT - Pref Div 69,000 - 1,380
= =.
No. of shares
22,

28.3 MARY
Rs.
2 existing shares have a cum rights value of (2 u Rs. 4) 8
1 new share is issued for 1

3 new shares have a theoretical value of 9

Theoretical ex-rights prices = Rs. 9/3 = Rs. 3
Weighted
Number Time Bonus Rights average
Date of shares factor fraction fraction number of
shares
1 January Brought
forward 5,000,000 1/12 6/5 4/3 666,667
1 February Bonus issue
(1 for 5) 1,000,000

6,000,000 2/12 4/3 1,333,333
1 April Rights issue
(1 for 2) 3,000,000

9,000,000 2/12 1,500,000
1 June Issue at full
market price
800,000

31 December Carried 9,800,000
forward 7/12 5,716,667

9,216,667

Earnings for Year 5 are (3,362,000 600,500 800,000) Rs. 1,961,500

EPS Year 5 = 1,961,500/9,216,667 = Rs.0.21 or 21 paisa

EPS Year 4 (adjusted) = Rs.0.32 3/4 5/6 = Rs.0.20 or 20 paisa

28.4 MANDY
Adjusted total earnings
Rs. Rs.
Reported earnings 2,579,000 1,979,000
Add back interest saved
(1,000,000 u 7%) 70,000
(1,000,000 u 7% u 9/12) 52,500
Minus tax at 30% (21,000) (15,750)
49,000 36,750
Adjusted total earnings 2,628,000 2,015,750

Emile Woolf International 385 The Institute of Chartered Accountants of Pakistan


Advanced accounting and financial reporting

Number of shares
Year 4 Number of
shares
1 January Brought forward 5,000,000
Dilutions:
Share options (W) 200,000
Convertible shares (1,000,000 100 u 30) 300,000

31 December 5,500,000

Year 3 Weighted
average
Number of Time number
Date shares factor of shares
1 January Brought forward 5,000,000
Share options: dilution (W) 125,000
5,125,000 3/12 1,281,250
1 April Convertibles: dilution 300,000
5,425,000 9/12 4,068,750
5,350,000
Diluted EPS
Year 4 = 2,628,000/5,500,000 = Rs.0.48 or 48 paisa
Year 3 = 2,015,750/5,350,000 = Rs.0.38 or 38 paisa
Working
Cash receivable on exercise of all the options = 500,000 Rs. 3 = Rs. 1,500,000

Year 4
Number of shares this would buy at full market price in Year 4 = Rs. 1,500,000/5
= 300,000 shares
Shares
Options 500,000
Minus number of shares at fair value (300,000)

Net dilution 200,000

Year 3
Number of shares this would buy at full market price in Year 3 = Rs. 1,500,000/4
= 375,000 shares
Shares
Options 500,000
Minus number of shares at fair value (375,000)

Net dilution 125,000

Emile Woolf International 386 The Institute of Chartered Accountants of Pakistan


Answers

28.5 AAZ LIMITED


a) Step 1: Ranking in order of dilution

Increase in Earnings
Increase
no. of per
in Rank
ordinary incremental
earnings
shares shares
Rs. Rs.
Convertible Debentures
Increase in earnings
(Rs. 7.5m x 70%) 5,250,000 1.75 3
Increase in shares 3,000,000
Convertible Preference
Shares
Increase in earnings
2,450,000 0.61 2
Increase in shares 4,000,000
Options
Increase in earnings -
Increase in shares - 1
(1.5m x 1.1 / 11) 150,000

Step 2: Testing for dilutive effect


Profit from
operations
attributable to
ordinary Ordinary
shareholders Shares EPS Effect
Rs. Rs.
Basic Earnings per share *125,380,000 85,220,000 1.471 -
Options (Rank 1) - 150,000
125,380,000 85,370,000 1.469 Dilutive

Convertible preference
shares (Rank 2) 2,450,000 4,000,000
127,830,000 89,370,000 1.430 Dilutive

Convertible debentures
(Rank 3) 5,250,000 3,000,000
Anti-
133,080,000 92,370,000 1.44 Dilutive
*Rs. 127,830,000 Rs. 2,450,000 = Rs. 125,380,000

Emile Woolf International 387 The Institute of Chartered Accountants of Pakistan


Advanced accounting and financial reporting

b) AAZ Limited
Notes to the financial statements for the year ended December 31,
2016

EARNINGS PER SHARE


2016
Basic alternative to ordinary share holders
Profit (Rupees) 125,383,000

Weighted average number of ordinary shares


outstanding during the year 85,220,000

Earnings per share - basic (Rupees) 1.47

Diluted
Profit after taxation (Rupees) 127,833,000

Weighted average number of ordinary shares, options


and convertible preference shares outstanding during
the year 89,370,000

Earnings per share - diluted (Rupees) 1.430

Because diluted earnings per share is increased when taking the


convertible preference shares into account (from Rs. 1.430 to Rs.
1.44), the convertible debentures are anti-dilutive and are ignored in
the calculation of diluted earnings per share.

28.6 ABC LIMITED


ABC Limited
Notes to consolidated financial statements for the year ended March 31,
2016
2016
Rs. in
'000
Earnings per share basic
Profit after tax and non-controlling interest (15,000-2,000) 13,000
Dividend paid during the year to ordinary shareholders
-
(Rs. 4,000)
10% Cumulative preference dividend for 2015 (Rs. 2,000) -
10% Cumulative preference dividend for 2016 (2,000)
Dividend declared on 12% non-cumulative preference shares
for 2016 (2,400)
Profit available for distribution to ordinary share holders 8,600

Emile Woolf International 388 The Institute of Chartered Accountants of Pakistan


Answers

Diluted earnings per share


Profit available for distribution to ordinary share holders 8,600
Effect of dividend declared on 12% non-cumulative preference
shares convertible into ordinary shares on or before
December 31, 2017 2,400
11,000
Weighted average number of ordinary shares W1 13,146
12% Non-cumulative preference shares convertible to
ordinary shares on or before December 31, 2017 W3 1,771
Weighted average number of ordinary shares - diluted 14,917
Antidiluted earning per share Rs. 0.74
W1: Weighted average ordinary shares outstanding for "Basic EPS"

Bonus Weighted
Number of Time average
Date fractions
shares factor number of
(W3) shares
1 April 2015 to 30 June u 6/5
2015 10,000,000 3/12 u1.00833 3,024,990
1 July
Conversion of
cumulative prefs at a
premium of Rs. 2 per
share
(500,000 u 10/12) 416,667
u 6/5 u
1 July to 30 September 10,416,667 3/12 1.00833 3,151,031
1 October
Rights issue 1,200,000

30 September to 31
December 11,616,667 3/12 u 6/5 3,485,000
1 January
Bonus issue (20%) 2,323,333
1 January to 31 March 13,940,000 3/12 3,485,000
Weighted average 13,146,021

W2: Calculation of bonus adjustment factor


No. of Rs. in
@ Rs.
shares '000
Bonus element with right issue
Outstanding shares before the exercise of
rights at fair value 10,417 12.50 130,213
Rights issued at a premium of Rs. 1.5 1,200 11.50 13,800
11,617 144,013

Emile Woolf International 389 The Institute of Chartered Accountants of Pakistan


Advanced accounting and financial reporting

Rs.
Actual cum rights price per share 12.5000
Theoretical ex-right value per share (144,013/11,617) 12.3967
Adjusting factor 1.00833

Bonus issued on January 01, 2016 (20%)


Adjusting factor (6 shares for 5 shares) 1.2
W3: Diluted EPS

Number Earnings
of shares (Rs.) EPS (Rs.)
Basic EPS 13,146,021 8,600,000 0.65
Dilution:
Non-cumulative prefs in issue for
the year (W4)at a premium of Rs. 2
per share (for the whole year)
2,000,000 u 10/12 u 12/12 1,666,667
Add back dividend paid to non-
cumulative prefs in issue at the
year-end 2,400,000

Non-cumulative prefs actually


converted in the year (for the part
of the year before conversion)
(500,000 u 10/12) u 3/12
i.e. 416,667 u 3/12 104,167
1,770,834
Adjusted figures 14,916,855 11,000,000 0.74

Diluted EPS: Rs. 11,000,000/14.917 million = Rs.0.74 per share


The non-cumulative preference shares are anti-dilutive

W4: Non-cumulative prefs in issue at the year-end


This can be found from the information about the dividend.
Rs. 2,400,000 is 12% of the nominal value of the shares.
Therefore, the nominal value is Rs. 20,000,000 (Rs. 2,400,000/0.12).
Therefore the number of shares (at Rs. 10 per share) is 2,000,000

Emile Woolf International 390 The Institute of Chartered Accountants of Pakistan


Answers

28.7 ALPHA LIMITED


Alpha Limited
Extracts from consolidated profit and loss account for the year ended 31
December 2013
Rs. in '000
Profit for the year W.1 (49,462.16+26,950) 76,412.12
Profit attributable to
Owners of Alpha Limited 76,412.12-5,390 71,022.12
Non-controlling interest 26,950*20% 5,390
76,142.12
Earnings per share: Rupees
Basic W.2 72.10
Diluted W.2 53.39

W-1 Profit for the year AL ZL


(Rs. in '000)
Profit after tax 60,000.00 25,000.00
Cash dividend received from ZL (net of tax)
Final dividend for 2012 (35,000*15%*80%)*9
0% (3,780.00)
Interim dividend for (35,000*1.16*12%*80
2013 %)*90% (3,507.84)
FV gain on ZL's investment [67,000-
property (40.35) (59,000+5,000)]*65
% 1,950
Cost of defined benefit gratuity (8,000-
sch. (19.120) 3,000)*65% (3,250.00)
49,462.16 26,950

W-2 Basic / diluted EPS:


Basic
Weighted Basic/
/Diluted
average Diluted
earnings
shares in EPS
(Rs. in
'000 (Rs.)
'000)
Weighted average No. of shares:
1-Jan- Balance
2013 80,000/100, 8007/12 800
1-Jan- Bonus issue at 20%
2013 (800*20%), 1607/12 160
960

Emile Woolf International 391 The Institute of Chartered Accountants of Pakistan


Advanced accounting and financial reporting

1-Aug- Shares issued under


2013 employees' share option
scheme
(60*5/12) 25
(960+60)5/12
Basic earnings per share (EPS) 985 71,022 72.10
Shares from assumed conversions:
1-Aug- Convertible 12% bonds (5
2013 shares for 4 bonds)
(30,000/100*5/4),
(30,000*0.12*0.65) 375 2,340
1-Aug- Shares for no
2013 consideration issued
under employees' share
option.
(250-150)/250*60*7/12
(IAS 33.45) 14 -
Diluted earnings per share (EPS) 1,374 73,362 53.39

Emile Woolf International 392 The Institute of Chartered Accountants of Pakistan


Answers

CHAPTER 29: ANALYSIS AND INTERPRETATION OF FINANCIAL


STATEMENTS

29.1 Alpha Limited and Omega Limited


(a)
Alpha Limited Omega Limited
(i) Current ratio
Current assets 620,000 504,000
Current liabilitie s 240,000 440,000
= 2.58:1 = 1.15:1
(ii) Acid test
Current assets - stock 340,000 332,000
Current liabilitie s 240,000 440,000
= 1.42:1 = 0.75:1
(iii) Creditors ratio
Average creditors 180,000 344,000
u 365 u 365 u 365
Purchases or cost of sale 1,120,000 1,342,000
= 59 days = 94 days
Collection period/Receivables
(iv)
Ratio
Average debtors 310,000 300,000
u 365 u 365 u 365
Sales 1,440,000 1,720,000
= 79 days = 64 days
(v) Earnings per share
PAT 60,000 48,000
No. of ord. shares 600,000 200,000
= 0.1/share = 0.24/share
(b) Comments on comparative analysis of the two companies.
Based on the ratios computed above:
(i) In terms of working capital and liquidity, Alpha Limited is in a better
position to honour its obligations as they fall due because its current
ratio and acid test ratio are higher than those of Omega Limited.
(ii) Omega Limiteds payment period is better than that of Alpha Limiteds
because Omega Limited uses suppliers funds to finance its
operation.
(iii) Omega Limiteds collection period is also better than that of Alpha
Limited. It extends shorter credit period to its customers than Alpha
Limited.
(iv) Omega Limiteds credit policy is better than that of Alpha Limited.
This is because there is 30 days difference between its payments
period and collection periods compared with Alpha Limited that had a
longer collection period than its payment period.

Emile Woolf International 393 The Institute of Chartered Accountants of Pakistan


Advanced accounting and financial reporting

(v) Omega Limiteds EPS is better than that of Alpha Limited by 0.14
(0.24 0.10). Omega Limiteds shareholders will be happier than
those of Alpha Limited.

29.2 COOK LIMITED


(a) The ratios computed from Cook Limited financial statement for the years
ended 31 December, 2015 and 2016 revealed that the companys
profitability has fallen in 2016 in comparison with the performance in 2015.
(i) The gross profit percentage of 60% in 2015 has decreased to 40% in
2016. The net profit percentage has also fallen from 33.6% in 2015
to 11.07% in 2016. The fall in the profitability of the company might be
due to the new sales managers price-reduction policy.
(ii) The return on capital employed also dropped from 30.5% in 2015 to
17.5% in 2016. The increased market resulting from the new sales
policy did not translate into increased profitability as hoped for.
(iii) The company appears solvent. There is no shortage of liquid assets.
Considerable amount of fund is being tied down with 76 days credit period
given to receivables under the new sales policy adopted in 2016 as
compared to 34 days in 2015.
Profitability ratios 2016 2015
(i) Gross profit percentage 2,240 1,080
u 100 u 100
Gross Profit 5,600 1,800
u 100 40% 60%
Sales
(ii) Net profit percentage 620 604
u 100 u 100
Net Profit 5,600 1,800
u 100 11.1% 33.6%
Sales
(iii) Return on capital employed
620  384 604  24
Profit before loan interest u 100 u 100
(ROCE) 2,534  3,200 1,858  200
Shareholde r' s funds plus
17.5% 30.5%
long  term loans
(iv) Net Profit 620  100 604
Return on equity or u 100 u 100
Equity 2,534 1,858
28.4% 32.5%

Liquidity ratios 2016 2015


(i) Current ratio
1,642 250
Current Assets 4.41: 1 3.5 : 1
372 72
Current Liabilitie s
(ii) Acid test ratio
1,642  476 250 - 60
Current Assets - Stock 3 :1 2.6 : 1
372 72
Current Liabilitie s
(iii) Receivables collection period
1,166 166
Trade Debtors x 365 76 days u 365 33.6%
x 365 5,600 1,800
Credit Sales

Emile Woolf International 394 The Institute of Chartered Accountants of Pakistan


Answers

(iv) Creditors payment period


350 72
Trade Creditors u 365 38 days u 365 36.5 days
u 365 3,360 720
Credit Purchase
(b) Amount of cash that should be released
1,166
Receivables collection period u 365 76 days
5,600
If the collection period is reduced to 45 days,
The new receivables figure would be:-
45
u Rs. 1,166m = Rs. 690m
76
The amount that would be released
= Rs. (1,166 - 690)m = Rs. 476m
or
Receivables collection period in 2016
1,166
= u 365 75.99 days 76 days
5,600
Imposition of 45 days
x x
45 days = 365
5,600 1
365x
45 days =
5,600
365x = 252,000
252,000
x=
365
x = Rs. 690m
Amount of cash to be released will be
= (Rs. 1,166 Rs. 690)m = Rs. 476m

29.3 FITZROY LIMITED


Statement of financial position as at 31 December, 2016
Rs.000 Rs.000 Rs.000
Non-current assets
Land & Building 5,000
Furniture & Fittings 1,450
Motor vehicles 2,300
8,750
Current assets
Inventory 1,800

Emile Woolf International 395 The Institute of Chartered Accountants of Pakistan


Advanced accounting and financial reporting

Rs.000 Rs.000 Rs.000


Receivables (W3) 1,650
Cash (W5) 300
(W1) 3,750
Less: Current liabilities
Creditors (W9) 900
Dividend (W10) 600
1,500
Net current assets 2,250
Total assets less current liabilities (balance) 11,000
Financed by: Rs.000
Ordinary shares (W11) 5,100
7 Debenture (W12) 3,800
Reserves:
Retained profits (balance) 2,100
11,000

Profit and loss accounts for the year ended 31 December 2016
Note Rs.000
Turnover (W5) 12,925

Operating profit before interest & tax (W6) 2,585


Interest expense (W13) (285)
Profit before Tax 2,300
Taxation (given) (400)
Profit after tax 1,900
Dividend ordinary (W10) (600)
Retained profit for the year 1,300
Retained profit b/f (given) 800
Retained profit c/f 2,100
Workings
All figures are in thousand
(W1) Non-current assets = 70% of total assets

8750 100
? Total assets = x
70 1

= Rs. 12,500
Current assets = Rs. 12,500 8,750 = Rs. 3,750

Current Assets 3750 2 .5


(W2) Current ratio = = =
Current Liabilitie
s CL 1

Emile Woolf International 396 The Institute of Chartered Accountants of Pakistan


Answers

3750 100
? Current liabilities = x = Rs. 1,500
2 .5 1

Cash 0 .2 Cash
(W3) Cash ratio = = =
CL 1 1500
? Cash = Rs. 300
(W4) Receivables = (3,750 1,800 300) = Rs. 1,650

Debtorsx365
(W5) Average collection period =
Sales

1650x365
46.596 =
Sales

1650x 365
Sales = = Rs. 12,925
46.576

(W6) Profit margin = 1/5 ? Gross profit = Rs. 12,925 x 1/5


= Rs. 2,585
(W7) Cost of sales = Rs. 12,925 Rs. 2,585 = Rs. 10,340
(W8) Cost of sales = Opening inventory + purchases closing
inventory
Rs. 10,340 = 0 + purchases 1800
Purchases = 10,340 + 1800 = Rs. 12,140

Trade creditors 365


(W9) Creditors payment period = u
Purchases 1

27.06x12140
Trade creditors = = Rs. 900
365

(W10) Current liabilities = Trade creditors + Dividend


? Dividends = 1,500 900 = Rs. 600

600
(W11) Ordinary shares = = Rs. 5,100
0.1176

Totalcash flow
(W12) Cash flow ratio =
Totaldebts

Total cash flow = profit + depreciation

2585  972
i.e. 0.6711 =
Totaldebts

3557
Total debts = = 5,300
0.6711

? 7% debenture = 5,300 1,500 = Rs. 3,800

Emile Woolf International 397 The Institute of Chartered Accountants of Pakistan


Advanced accounting and financial reporting

(W13) ? Interest expenses 7% x 3800 = Rs. 285

29.4 TRAVELWELL LTD


(a) Ratios

2016 2015

Return on year-end capital employed (24 + 9.75)


/(144 + 130) 12.3% 8.5%

Net asset turnover 310 1.2 1.5 times


/(144 + 130)
times

Gross profit margin (given) 20% 16.7%

Net profit margin (before tax) 24 8.0% 5.7%


/300
Current ratio 46 1.3 3.0
/36
Inventory holding period (29/240) u 365 44 days 35 days

Trade receivables collection period (17/300) u 365 21 days 16 days

Trade payables payment period (28/240) u 365 43 days 31 days

Gearing ratio 130 47.4% nil


/(144 + 130)
(b) The acquisition of Rondel Ltds net assets
At the beginning of the year Travelwell Ltd acquired the net assets of
Rondel Ltd. The revenue contributed by the purchase of these assets was
Rs. 90 million, which explains the entire increase of 43% in sales revenue
for Travelwell Ltd for the year.
The acquisition also added Rs. 40 million to gross profit, and without this
the gross profit of the company would have fallen in 2016 compared with
the previous year. The gross profit margin would have been (60,000
40,000)
/(300,000 90,000) = 9.5%, substantially lower than the 16.7% achieved in
the previous year.
Clearly, the Chief Executive Officer has selected ratios and other
performance measurements that suggest excellent performance; however,
his report is misleading because it fails to explain the effects of the
acquisition of Rondel Ltds net assets. The acquisition cost Rs. 130 million
which was the equivalent of 92% of Travelwell Ltds capital employed just
before the acquisition occurred.
A more appropriate analysis of performance and financial position should
consider a wider range of ratios and should also allow for the effects of the
acquisition of Rondel Ltds business.
Return on capital employed
ROCE improved from 8.5% in the previous year to 12.3% in the current
year, but the improvement is attributable to the acquisition of the net assets
of Rondel Ltd. There were no disposals of non-current assets during the

Emile Woolf International 398 The Institute of Chartered Accountants of Pakistan


Answers

year. This means that the year-end net assets of Rondel Ltds business that
have been incorporated in the Travelwell Ltd statement of financial position
can be estimated as follows:
Rs.000
Net assets at acquisition (balancing figure) 118,000
Goodwill 12,000
Cost of the acquisition (given) 130,000
Plus post-acquisition increase in net assets:
Pre tax profit for the year (given) 29,000
Taxation (@25%) (7,250)
21,750
Net assets at 30 September 2016 151,750

This suggests that the ROCE from Rondel Ltds business was 19.1% (=
29
/151.75). The high ROCE from Rondel Ltds business will explain the rise on
ROCE for the company s a whole.
Profitability
As indicated earlier, gross profit would have fallen in 2016 but for the gross
profit contributed by the net assets of Rondel Ltd and the gross profit from
the original business of Travelwell Ltd was lower in 2016 than in 2015
(9.5% compared with 16.7%).
There would also have been a loss of Rs. 5 million before tax except for the
profit of Rs. 29 million contributed by Rondel Ltds business.
It is likely however that the finance costs of Rs. 9.75 million in the current
year, resulting from the issue of the loan notes, were due to a need to
borrow to acquire the assets of Rondel Ltd. If so, it would be more
appropriate to assess the profit before tax from Rondel Ltds business as
Rs. 19.25 million (= Rs. 29 million Rs. 9.75 million finance charge) and
the profit before tax from Travelwell Ltds other business as Rs. 4.75 million
(= Rs. 5 million loss + Rs. 9.75 million).
Using these adjusted figures, this suggests that the profit before tax/sales ratio for
Travelwell Ltds other business was 2.3% (= 4.75/(300 90), which is much
worse than the previous year.
Net asset turnover
Net asset turnover fell in 2016 to 1.1 times compared with 1.5 times in
2015. The net asset turnover from the business of Rondel Ltd was only
0.59 times (= Rs. 90 million/Rs. 151.75 million), which means that the
acquisition of the net assets of Rondel Ltd contributed significantly to the
fall.
Financial position
The change in the financial position of Travelwell Ltd can be assessed by
looking at the gearing ratio and working capital ratios.
Gearing
At 30 September 2015, Travelwell Ltd had no gearing. Gearing was 47.4%
one year later. This is due to the issue of the loan notes, presumably to

Emile Woolf International 399 The Institute of Chartered Accountants of Pakistan


Advanced accounting and financial reporting

contribute towards financing the acquisition of Rondel Ltds not assets.


Higher gearing creates greater financial risk, in the sense that any fall in
profits before interest will have a much greater proportional effect on
earnings per share.
Liquidity
The current ratio has fallen from 3.0 times to 1.3 times. The fall is
attributable largely to the change from having a bank balance of Rs. 28
million at 30 September 2015 to a bank overdraft of Rs. 2 million one year
later. This net cash outflow of Rs. 30 million is exactly equal to the cash
used to acquire the net assets of Rondel Ltd (= Rs. 130 million cost minus
loan notes issued Rs. 100 million). The fall in liquidity is therefore possibly
not a matter of concern.
Working capital
There has been an increase in the inventory turnover period from 35 to 44
days, but this is largely offset by the increase in the trade creditors payment
period from 31 to 44 days (since the values of inventory and trade payables
in the statement of financial position are roughly equal). There has been a
slight increase in the average collection period by trade receivables. On
balance, the change in working capital has not affected the financial
position of the company significantly.
Dividends
The company paid a dividend for the year of Rs. 15 million, up from Rs. 12
million the previous year (Rs. 15 million u 100/125). Since profit after tax is Rs.
18 million, dividend cover is just 1.20 times, which is quite low, and retained
profits are only Rs. 3 million. It might therefore be argued that the 25%
increase in dividends was perhaps excessive.
Conclusion
The acquisition of the net assets of Rondel Ltd appears to have contributed
very favourably to the financial performance of Travelwell Ltd. However the
contribution from Rondel Ltds assets should not hide the fact that there has
been some deterioration in the performance of Travelwell Ltds other
business. This is a problem that management need to consider.

29.5 SACHAL LIMITED

(a) To: Board of Directors

From: Chief Financial Officer

Date: December 8, 2017

Subject: Financial and Operating Performance of Waris Limited

As requested, I have analyzed the financial performance of Waris


Limited (WL) with the industry with a view to evaluate the feasibility of
launching a takeover bid. My analyses of each category of ratios is as
follows:

Profitability Ratios

Emile Woolf International 400 The Institute of Chartered Accountants of Pakistan


Answers

The gross profit ratio is near to the highest while the operating profit is
near to the lowest as compared to similar companies. It indicates that
key issue which is affecting WLs profitability is its lack of control over
operating expenses. The positive aspect of this situation is that we may
be able to improve the profitability just by controlling the operating
expenses without being required to make significant changes in the
current operations of WL.

Return on shareholders equity is around the average prevailing in the


industry. This ratio is obviously, related to operating profit and as
discussed above it can be improved by exercising greater control over
operating expenses, after take over.

Working Capital Ratios

WLs working capital ratios specially the current ratio indicates that the
companys liquidity position is in line with the industry average. Hence,
it seems that the companys working capital is being appropriately
managed although there may be some room for improvement.

The inventory turnover is among the lowest in the industry which shows
that sound inventory management policies are in place.

However, the level of receivables is among the highest in the industry.


The possible causes of the situation may be as follows:

Poor efforts in making collections

Lack of proper credit control policies or slackness in their


implementation.

Chances of bad debts which may not have been provided.

Sales to related parties.

Fictitious sales.

We need to seek appropriate explanations and investigate the matters


if possible.

Gearing Ratios

The debt equity ratio is on the higher side but can be restructured after
acquisition. However, the interest cover is only 1.3. It is among the
lowest in the industry and is indicative of a high degree of risk as the
profits are barely able to cover the interest charges. Even a slight
decline in the profitability of the company may have highly adverse
impact on the companys bottom line.

Investor Ratios

Earnings per share is on the lower side. However, it can be improved


by improving profits as discussed while comparing performance ratios.
WLs dividend payout is the lowest (22.2%) in terms of percentage
among other similar companies. Generally, past history of dividend

Emile Woolf International 401 The Institute of Chartered Accountants of Pakistan


Advanced accounting and financial reporting

payouts is not relevant to our bid decision. However, low dividend may
also be on account of liquidity problems and we should consider this
aspect.

Conclusion

The companys performance indicates a mixed trend. However, it may


be concluded that below average performance, (wherever applicable)
can be improved by revisiting the situation and bringing about
necessary changes in the policies.

(b) Following additional information could have been useful for a better
analysis of the situation:

(i) Any recent audited or management accounts.


(ii) Comparison of accounting policies following by the companies
in the same industry and the possible impact thereof on the
above ratios.
(iii) Expected growth in future earnings
(iv) Alternative investment opportunities
(v) Effect of synergy
(vi) WLs market reputation;
(vii) Quality of human resource within the company;
(viii) Research and development activities
(ix) Legal framework and industry risks
The figures given in the question suggest that company had the funds
in addition to sale proceeds to pay for cost associated with PIB
investment. Therefore, Present Value has been taken as Rs.
104,641,483 (Rs. 100,000,000 + Rs. 4,641,483).

29.6 OPAL INDUSTRIES LIMITED

Opal Industries Limited

Accounting treatment for various investments in first separate financial


statements

In accordance with IAS 27, in separate financial statements, investments in


subsidiaries, joint ventures and associates should be valued either at cost or fair
value in accordance with IFRS 9.

As OIL is preparing its first separate financial statements for the year ended 30
June 2014, there is a change in accounting policy as investments in AL, an
associate company, would be treated in 2014 at cost as against the previous
basis of equity accounting. Accordingly, comparative figures would be restated to
incorporate this change in accounting policy.

Emile Woolf International 402 The Institute of Chartered Accountants of Pakistan


Answers

(i) Opal Industries Limited

Statement of changes in equity for the year ended 30 June 2014


Retained
earnings

Rs. in million

Balance as at 30 June 2012 465.00

Profit for the year - restated W-1 1,251.60


Final dividend for the year ended 30 June
2012:

- Cash dividend at 20% 2,50020% (500.00)

- Bonus issue at 15% 2,50015% (375.00)

Balance as at 30 June 2013 - restated 841.60

Profit for the year W-1 1,454.80


Final cash dividend at 25% for the year
ended 30 June 2013 2,87525% (718.75)

Balance as at 30 June 2014 1,577.65

(ii) Opal Industries Limited

Notes to the financial statements or the year ended 30 June 204

1 - Long term investments:


2013
2013 2014 2014
(Restated)
Description
Number of shares Rs. in million

Subsidiary and associated


companies-at cost

- 975,000 GL 195.00 -

240,000 240,000 AL 50.00 50.00

Others - Available-for sale


BL
70,000 70,000 7.70 11.20
(70,000110), (70,000160)

310,000 1,285,000 252.70 61.20

The company holds 65% and 30% and 10% ownership interest in GL, AL
and BL respectively.

Emile Woolf International 403 The Institute of Chartered Accountants of Pakistan


Advanced accounting and financial reporting

W-1: OIL profit for the year after taking effect of investee companies:

2013
2014
(Revised)

Rs. in million

Profit for the year 1,450.00 1,260.00

AL - Associated company:

Reversal of previously booked profit (2830%) - (8.40)

Dividend for the year ended 30 June 2013

(8030%16%) 3.84 -

BL - Available for sale:

Dividend for the year ended 30 June 2013

(7010%18%) 1.26

(7.7-8) OR (11.2-8)-(7.7-
Investment impairment 11.2) (0.30) -

1,454.80 1,251.60

Emile Woolf International 404 The Institute of Chartered Accountants of Pakistan


Answers

CHAPTER 30: SUNDRY STANDARDS AND INTERPRETATIONS

30.1 GUJRANWALA FOODS LIMITED


(a)
Chickpea Apricot Dates Onion Total
Rs.000 Rs.000 Rs.000 Rs.000 Rs.000
Sales: 200 300 750 250 1,500
Add: Subsidies 40 80 60 60 240
Own
consumption 45 40 75 20 180
285 420 885 330 1920
Cost of sales
Opening
inventory 40 40 70 - 150
Add: Purchases 75 150 300 80 605
Closing inventory (60) (110) (300) (60) (530)
55 80 70 20 225
Gross output
value 230 340 815 310 1,695

Less expenses:
Casual labour 16
Regular workers 24
Land preparation 64
Hire of tractors 48
Depreciation: irrigation 80
Depreciation: farms equipment 60
(292)
1,403
Workings:
12
Casual labour /15 x Rs. 20,000 = Rs. 16,000
12
Regular workers /15 x Rs. 30,000 = Rs. 24,000
12
Land preparation /15 x Rs. 80,000 = Rs. 64,000
12
Hire of tractors /15x Rs. 60,000 = Rs. 48,000

Depreciation:
00,000
Irrigation cost

Farm equipment 20% x 400,000 x 9/12 = Rs. 80,000
(b) (i) Biological assets are living plants and animals.
(ii) Biological transformation relates to the process of growth and
degeneration that can cause changes of a quantitative or qualitative
nature in a biological asset.
(iii) Harvest is the detachment of produce from a biological asset or
cessation of a biological assets life process.

Emile Woolf International 405 The Institute of Chartered Accountants of Pakistan


Advanced accounting and financial reporting

30.2 WAH AGRIPROD LTD


(a) Wah Agriprod Ltd: Statement of profit or loss and other comprehensive
income for the year ended 31 December 2016.
Notes Rs.000
Revenue (wi) 851,400
Cost of sales (wii) (495,532)
Gross profit 355,868
Distribution cost (31,950)
Administrative expenses (wiii) (79,100)
Profit from operations 244,818
Investment income 18,250
Finance cost (1,020)
Fair value loss on financial instrument
(40,500 39,700) (800)
Profit before tax 261,248
Income tax expenses (86,750)
Profit for the year 174,498
Other comprehensive income:
Fair value gain on leasehold property 12,333
Total comprehensive income for the year 186,831
Working Notes:
Rs.
Revenue
(i) Rs. 855,000 Rs. 3,600 value of returnable goods = 851,400
(ii) Cost of sales:
Opening inventory 85,075
Purchases 503,600
Depreciation of plant and equipment
(0.15 of Rs. 98,800 Rs. 28,800) 10,500
Amortization of leased property
(Rs. 3,833 + Rs. 4/208 of Rs. 99,000) 5,237
Closing inventory
(Rs. 106,000 + 0.8 of Rs. 3,600) (note iv) (108,880)
495,532

(iii) Admin expenses .


Per the question 104,400
Less dividend = Rs. 115,00 x 2 x Rs. 2.2 x Rs.0.05 (25,300)
79,100

(iv) Closing inventory: As per question 106,000


Sales return at cost (80% of 3,600) 2,880
108,880

Emile Woolf International 406 The Institute of Chartered Accountants of Pakistan


Answers

(b) Statement of changes in equity for the year ended 31 December 2016
Revaluat
Share Share ion Retained
capital premium surplus earnings Total
Rs.000 Rs.000 Rs.000 Rs.000 Rs.000
Balance as 105,000 6,400 - 55,600 167,000
at 1/1/2016
Issue of 10,000 2,000 - - 12,000
shares
Profit for the - - - 174,498 174,498
year
Fair value - - 12,333 - 12,333
gain
Realised - - (237) 237 -
during the
year
Dividend - - - (25,300) (25,300)
paid
Balance 115,000 8,400 12,096 205,035 340,531
31/12/2016

Revaluation of leased property Rs.000 Rs.000


Cost as at 1/1/2016 125,000
Accumulated depreciation as at 1/1/2016 35,000
For the eight months to 1/9/2016:
8/12 of 125,000 25 years 3,333 38,333
86,667
Value of leased property 99,000
(12,333)
(c) (i) A financial asset is any asset that is cash, an equity instrument of
another entity or a contractual right to receive cash or another
financial asset from another entity; or to exchange financial assets or
financial liabilities with another entity under conditions that are
potentially favourable to the entity.
(ii) The classes of financial assess and how each is valued are as
follows:
Classification Measurement
x Financial assets at fair value Fair value through profit or loss
through profit or loss
x Held-to-maturity investments Amortised cost
x Loans and receivables Amortised cost
x Available-for-sale Fair value through other
comprehensive income

Emile Woolf International 407 The Institute of Chartered Accountants of Pakistan


Advanced accounting and financial reporting

(d) A biological asset is defined as a living animal or plant.


Entities are required to recognise biological assets or agricultural produce
when and only when, all of the following conditions are met:
(i) The entity controls the asset as a result of past events;
(ii) It is probable that future economic benefits associated with the asset
will flow to the entity; and
(iii) The fair value or cost of the asset can be measured reliably.

30.3 HELIOS GROUP


(a) Helios Ltd
Consolidated statement of financial position as at 31 December, 2016
Non-current assets Rs.000 Rs.000

Biological assets - Dairy livestock: Immature 40,000


- Dairy livestock: mature 50,000
- Plantation 20,000 110,000

Property, plant and equipment
(600,000 + 450,000 + 60,000 40,000 50,000) 1,020,000
Investments (800,000 600,000) 200,000
Goodwill (W3) 260,000

1,590,000
Current assets
Inventories (160,000 + 150,000 20,000) 290,000
Trade receivables (120,000 + 280,000) 400,000
Cash and cash equivalents (20,000 + 50,000) 70,000 760,000

2,350,000

Equity and liabilities
Equity
Ordinary shares capital 160,000
Share premium 40,000
Group reserves (W5) 730,000

Parent equity 930,000
Non-controlling interests (W4) 220,000

Total equity 1,150,000

Non-current Liabilities
Loan notes (600,000 + 170,000) 770,000

Current liabilities
Trade payables (310,000 + 120,000) 430,000 1,200,000

Total equity and liabilities 2,350,000

Emile Woolf International 408 The Institute of Chartered Accountants of Pakistan


Answers

Workings (W)
(1) Group Structure
Helios Ltd
70%
NCI = 30%
Sol Ltd

(2) Net assets of Sol Ltd


Acquisition Reporting Post-
date date acquisition
Rs.000 Rs.000 Rs.000
Ordinary shares 120,000 120,000 -
Share premium 20,000 20,000 -
Reserves 300,000 500,000 200,000
Fair value adjustment
(land) 60,000 60,000
Fair value of net assets 500,000 700,000 200,000
(3) Goodwill in Sol Ltd
Rs.000
Purchase consideration 600,000
Fair value of NCI at acquisition 160,000
760,000
Fair value of net assets at acquired (500,000)
Goodwill 260,000
(4) NCI at reporting date
Rs.000
Fair value of NCI at acquisition 160,000
Share of post-acquisition reserves
(30% x Rs. 200,000) 60,000
220,000
(5) Group reserves
Rs.000
Helios Ltd 590,000
Share of post-acquisition
(70% x Rs. 200,000) 140,000
730,000
(b) Measurement of harvested agricultural products. Agricultural products
harvested from an entitys biological assets shall be measured at its fair
value less costs to sell at the point of harvest. Such measurement is the
cost at that date when applying it as inventories or another applicable
standard.

Emile Woolf International 409 The Institute of Chartered Accountants of Pakistan


Advanced accounting and financial reporting

30.4 FASHION BLUE ENTERPRISES


(a) Fashion Blue Enterprises: Statement reconciling the inventory balance
Recorded Physical
Balance Count
Rs. 000 Rs. 000
Balance prior to adjustment 73,410 71,400
Add: Goods sold, but not dispatched by 31
December 2017 300
Less: Goods held on behalf of third parties (200)
Add: Unrecorded purchases 410
Add: Goods purchased, in transit at 30 December
2017 400
Add: Goods at Sialkot undercasted 90
Less: Unrecorded purchase return (2,500)
Add: Adjustment of sales to Saleem now recorded
at cost (780 u 30/130) 180
Less: Inventory shortfall / loss (balancing figure) (110)
(b) Value of inventory at 31 December 2017 71,690 71,690

No adjustment required for goods costing Rs. 310,000 sold on credit to Skims
Industries Ltd.
The value of inventory that should be recorded in the Statements of financial
position is Rs. 71,690 thousand

(c) Adjustments in the books of Fashion blue enterprises


Debit Credit
Rs. 000 Rs. 000
(i) Inventory 300
Cost of goods sold 300

(iii) Inventory 410


Account payable Mustafa & Co. 410
Being: Correction of unrecorded purchases

(iv) Account payable (Ali Garments) 2,500


Inventory 2,500
Being: Correction of unrecorded purchases
return

(viii) Inventory account 180


Cost of sales 180
Being: Sales to Saleem recorded at sale
price instead of cost now adjusted

Part (a) Inventory losses / write downs (P&L) 110


Inventory 110
Being: Unexplained difference / inventory
theft / inventory damage

Emile Woolf International 410 The Institute of Chartered Accountants of Pakistan


Answers

30.5 KHAN LIMITED


Khan Limited: Inventory Reconciliation Statement
Rs. Rs.
Inventory as per physical inventory at July 14, 2017 185,000
(i) Less: Purchases for July 1 to 14, 2017 included in the
physical inventory:

Payment against purchases 48,000


Adjustments to the above

Last year purchases (5,000)

Purchased and not delivered yet (6,000)


Purchased and returned July 07 (2,000) (35,000)
(ii) Add: Sales for July 1 to 14, 2017 excluded from the
physical inventory:

Collection against sales 60,000


Adjustments to the above
Last year sales (1,500)
Sold and not delivered (2,800)
Sold and returned (760)
54,940
(v) Sales on account 10,000
64,940
(iii) Profit element (25% of 64,940) (16,235)
Cost of sales during 1-14 July 48,705
(iv) No adjustment is required for purchase of goods on
June 28 for Rs., 6,000, as already included in inventory
on 14 July, 2017 
(vi) Adjustment of net realizable value not to be accounted for
at June 30, as damage pertains to subsequent period.
Cost of goods over and above NRV (9,000 6,000) 3,000
(vii) Error in carry forward of page total (1,000)
Error in casting 200
(viii) Goods held on consignment (2,200)
198,705

Emile Woolf International 411 The Institute of Chartered Accountants of Pakistan


Advanced accounting and financial reporting

30.6 AFRIDI
Statement showing the amount of physical inventory as on March 31, 2017
Rs.
Inventory as on December 31, 2017 (W1) 140,025
Add: Purchases for the quarter (W2) 145,360
285,385
Less: Adjusted Cost of sales (W3) (100,345)
100
Less: goods given in charity ( /120 of Rs. 2,100) (1,750)
Physical inventory balance as on March 31, 2017 183,290
Working - 1
Inventory as on December 31, 2016
Inventory as valued previously 140,525
Add: Cost of 1,000 items recorded at Re. 0.50 per item instead of Rs.
10 per item. 9,500
150,025
Less: error in carry forward of a page total (10,000)
Actual inventory as on December 31, 2016 140,025
Working - 2
Purchases for the quarter ended March 31, 2017
Total of invoices from Jan. 01 to Mar. 31, 2017 as per purchased day
book 138,560
Add: Goods purchased before march 31, 2017 but recorded in April
2017 37,000
Less : Invoices pertaining to Goods received before December 31, 2017 (28,000)
Less : Purchase of ceiling fan (2,200)
145,360
Working - 3
Cost of sales for the quarter
Total of sales invoices raised from January 01 to March 31, 2017 151,073
Add: Goods dispatched before March 31, 2017 but invoiced in April
2017 25,421
Less: Goods dispatched before December 31, 2016 but invoiced during
the quarter ended March 31, 2017 (38,240)
Less: sales return during the quarter (12,800)
Less: Sale invoice recorded twice (5,760)
Net sales 119,694
Add: Discount allowed (6,000 1.20 = 7,200 10%) 720
Sales before discount 120,414
Less: gross margin of 20% on cost (120,414*20/120) (20,069)
100,345

Emile Woolf International 412 The Institute of Chartered Accountants of Pakistan


Answers

CHAPTER 31: IFRS 1: FIRST TIME ADOPTION OF IFRS

31.1 IFRS 1
(a) The first IFRS reporting period was the year ended 31 December 2016, and
the date of Transition to IFRS was 1 January 2015.

(b) The procedures which must be followed in order to prepare the


Financial Statements for the year ended 31 December 2016, are as
follows:

(i) Choice of accounting policies to be included as part of notes to the


Financial Statements

(ii) Preparation of the opening IFRS Statements of Financial Position by


applying the following rules, except in cases where IFRS grants
exemptions and /or prohibits retrospective application:

Recognise all assets and liabilities required by IFRS

Not recognise assets and liabilities not permitted by IFRS

Reclassify all assets and liabilities and equity in accordance with


IFRS

Measure all assets and liabilities in accordance with IFRS

Any gains and losses arising from this exercise should be recognised
immediately in retained earnings as at January 2016

(iii) Since IAS 1 requires that at least one year of comparative prior period
financial information be presented, the opening Statement of
Financial Position will be 1 January 2015, if not earlier.

(iv) Preparation of full IFRS Financial Statements for the year ended 31
December 2016, which should include:
x three statements of financial position
x two statements of comprehensive income
x two separate statements of profit or loss (if presented)
x two statements of cash flows
x two statements of changes in equity
x related notes, including comparative information
(c) The reconciliation which the company must include in its financial
statements for the year ended 31 December 2102, to explain how the
transition from previous GAAP to IFRS affect the reported financial position,
financial performance and cash flows are as follows:

(i) Reconciliation of equity reported under previous GAAP to equity


under IFRS both (a) at the date of the opening IFRS Statement of
Financial Position and (b) the end of the last annual period reported
under the previous GAAP.

Emile Woolf International 413 The Institute of Chartered Accountants of Pakistan


Advanced accounting and financial reporting

(ii) Reconciliation of Total Comprehensive Income under IFRS for the


last annual period reported under the previous GAAP to Total
Comprehensive Income under IFRS for the same period.

(iii) Explanation of material adjustments that were made, in adopting


IFRS for the first time, to the Statement of Financial Position,
Statement of profit or loss and Statement of Cash Flows.

(iv) If errors in previous GAAP financial statements were discovered in


the course of transaction to IFRS, those must be separately
disclosed.

(v) If the entity recognised or reversed any impairment losses in


preparing its opening IFRS Statement of Financial Position, these
must be disclosed.

(vi) Appropriate explanations if the entity has elected to apply any of the
specific recognition and measurement exemptions permitted under
IFRS 1, for example, if the entity used fair values as deemed cost.

(d) Contents of a typical statement of changes in equity are as follows:

(i) Total comprehensive Income for the period, showing separately


amounts attributable to owners of the parent and to non-controlling
interests.

(ii) For each component of equity, the effect of retrospective application


or retrospective restatement recognised in accordance with IAS 8.

(iii) For each component of equity, a reconciliation between the carrying


amount at the beginning and end of the period, separately disclosing

Profit or loss

Other comprehensive income

Transactions with owners in their capacity as owners showing


separately, contributions by and distributions to owners, and
changes in ownership interests in the subsidiaries that do not
result in a loss of control.

Emile Woolf International 414 The Institute of Chartered Accountants of Pakistan


Answers

CHAPTER 32 SPECIALISED FINANCIAL STATEMENTS

32.1 IFRS for SMEs


(a) (i) IFRS, were not designed specifically for a class of companies
quoted or unquoted. IFRSs give financial statements enhanced
reliability, relevance and credibility and result in fair presentation.
(ii) SMEs would wish to comply with IFRSs for consistency and easy
comparability with similar SMEs both within their own country and
internationally.
(iii) The objectives of general purpose Financial Statements are basically
appropriate for SMEs and bigger publicly quoted companies alike.
Given this, one set of IFRS that would be used nationally and
internationally is desirable.
(iv) The cost burden of applying the full set of IFRSs may not be justified
on the basis of user needs. The purpose and usage of the Financial
Statements, and the nature of the accounting expertise available to
the SMEs, will not be the same as for listed companies. These may
provide justification for a separate set of IFRS for SMEs.
(v) Where attention is devoted to adopt local Generally Accepted
Accounting Principle (GAAP) for SMEs on a national basis and IFRSs
for listed companies, this practice of adopting GAAP for SMEs may
differ between countries when applied by such SMEs thus making
comparability of Financial Statements difficult across national
boundaries.
(b) (i) Most SMEs have a narrower range of users than publicly quoted
companies. Users of SMEs Financial Statements are basically
owners, lenders and suppliers of the merchandize and tax authorities.
(ii) In deciding the form of modification to be made to IFRSs, the needs
of these users must be taken into consideration. Also the financial
burden imposed by the compliance with IFRSs by these SMEs must
be identified.
(iii) There will have to be a relaxation of some of the measurement and
recognition criteria in IFRSs in order to achieve the reduction in the
costs and burden of implementation.
(iv) Some disclosures, such as Earnings Per Share, segment reporting,
etc, may not be relevant to SMEs and therefore may not be needed.
(v) A review of these disclosures requirements in IFRSs will be required
to assess their appropriateness for SMEs.
(c) IFRSs for SMEs would not necessarily deal with all the recognition and
measurement issues facing an entity but the key issues should revolve
around the nature of the recognition, measurement and disclosure of the
transactions of SMEs. In the case where the item is not dealt with by the
standards, there are three alternatives:
(i) the entity can look to the full IFRSs to resolve the issue.
(ii) management judgement can be used with reference to the framework
and consistency with other IFRSs for SMEs.
(iii) existing practice could be used.

Emile Woolf International 415 The Institute of Chartered Accountants of Pakistan


Advanced accounting and financial reporting

32.2 AKMAL GENERAL INSURANCE LIMITED

Emile Woolf International 416 The Institute of Chartered Accountants of Pakistan


Answers

32.3 MAHFOOZ GENERAL INSURANCE LIMITED


Mahfooz General Insurance Limited
Statement of claims for the year ended June 30, 2016
Business underwritten inside Pakistan

Rupees in million

Reinsuranc

Net claims expense


e and the

Reinsurance and other

Reinsurance and other


recoveries received

recoveries revenue
Outstanding recoveries in
Total claims paid

Claims expenses
claims respect of

Class outstanding
claims
Opening

Opening
Closing

Closing
2016

Direct and
facultative
Fire and
property
damage 900 600 500 800 600 500 350 450 350
Marine,
aviation &
transport 450 400 450 500 300 300 400 400 100
Motor 1,150 900 750 1,000 850 700 550 700 300
Accident
and health 250 300 150 100 160 150 80 90 10

Total 2,750 2,200 1,850 2,400 1,910 1,650 1,380 1,640 760
Treaty
Proportional 13 10 12 15 - - - - 15

2,763 2,210 1,862 2,415 1,910 1,650 1,380 1,640 775

Emile Woolf International 417 The Institute of Chartered Accountants of Pakistan


Advanced accounting and financial reporting

32.4 DEE GENERAL INSURANCE LIMITED


Dee General Insurance Limited
Consolidated statement of expenses for the year ended 31 December 2016
Deferred

Net commission

from reinsurers
Commissions

Underwriting
management

underwriting
Commission
commissions

expense
expense

expense
Other
Class

Net
Opening

Closing
Direct and
(Rs. in million)
Facultative
Fire and
property 321.41 148.79 160.43 309.77 165.28 475.07 270.44 204.61
damage
Marine,
aviation and 126.87 11.31 5.68 132.50 139.96 272.46 5.70 266.76
transport
Motor 215.00 128.50 114.23 229.27 499.93 729.20 12.72 716.48
Miscellaneous 90.94 38.59 35.17 94.36 172.70 267.06 82.40 184.66
1,372.
754.22 327.19 315.51 765.90 977.87 1,743.77 371.26
51
Treaty
Proportional 0.30 - - 0.30 0.13 0. 43 - 0.43
1,372.
Grand total 754.52 327.19 315.51 766.20 978.00 1,744.20 371.26
94

32.5 BANK LATEEF BANK LIMITED


8 Lendings to financial institutions
Notes 2016 2015
Rs. in million

Call money lending 8.2 850 1,200


Repurchase agreement lending
(reverse repo) 8.3 2,100 2,850
2,950 4,050

8.1 Particulars of lending


In local currency 2,840 3,900
In foreign currencies 110 150
2,950 4,050
8.2 These are unsecured lendings to financial institutions, carrying mark up
ranging from 15% to 17% (2015: 10% to 12 % and will mature latest by
October 2016.

Emile Woolf International 418 The Institute of Chartered Accountants of Pakistan


Answers

8.3 These are short term lendings to various financial institutions and are
secured against government securities shown in note 8.4 below. These
carry mark up at rates ranging from 9.5% to 13.2 % (2015:8% to 10.5 %)
and will mature on various dates, latest by October 2016.

8.4 Securities held as collateral against lending to financial institutions


Rs. in million
2016 2015
Held Further Held Further
by given as by given as
bank collateral Total bank collateral Total
Market
treasury bills 1,650 - 1,650 1,850 - 1,850
Pakistan
investment
bonds 450 - 450 1,000 - 1,000
2,100 - 2,100 2,850 - 2,850

Market value of the above as at September 30, 2016 amounted to


Rs. 2,250 million 2015: 2,930 million).

32.6 SECURED BANK LIMITED


Secured Bank Limited
Notes to the financial statements for the year ended December 31, 2016
2016 2015
Rs. in million
9. INVESTMENTS BY SEGMENTS
Federal government securities
Market treasury bills 366 309
Pakistan investment bonds 69 61
Government of Pakistan bonds (USD/Euro) 26 30
Investments in associated undertakings 9 8
Fully paid ordinary shares
Listed companies 6 5
Unlisted companies 2 3
Bonds, participation term certificates & term
finance certificates
Listed securities 19 30
Unlisted securities 260 210
Other investments
Overseas government securities 60 52
Investments of mutual funds 32 28
Others 19 29
868 765
Less: Provisions for diminution in value of
9.1 (45) (39)
investments
Net investments 823 726

Emile Woolf International 419 The Institute of Chartered Accountants of Pakistan


Advanced accounting and financial reporting

2016 2015
9.1 Particulars of provision for diminution in value of
Rs. in million
investments
Opening balance 39 28
Charge for the year 17 12
Impairment / (reversals) (6) 2
Amounts written off (5) (3)
6 11
Closing balance 45 39

32.7 AL-AMIN BANK LIMITED


Al-Amin Bank Limited
Notes to the Financial Statements for the year ended XX/XX/XX

1.1 Particulars on non-performing advances


Advances include Rs. 5,000 million which, as detailed below, have been
placed under non-performing status:
2016
Provisions required and
Category of Amount outstanding
held
classification
Domestic Overseas Total Domestic Overseas Total
Rs. in million
Other assets
especially
mentioned 100 - 100 5 - 5
Sub-standard 400 260 660 70 50 120
Doubtful 840 - 840 530 - 530
Loss 3,400 - 3,400 3,345 - 3,345
Total 4,740 260 5,000 3,950 50 4,000

1.2 Particulars of provision against non-performing advances


2016
Specific General Total
Rs. in million
Opening balance 3,320 65 3,385
Charge for the year 802 40 842
Amounts written off (50) - (50)
Reversals (90) - (90)
Exchange adjustments 18 - 18
Closing balance 4,000 105 4,105

32.8 BLUE-CHIP ASSET MANAGEMENT LIMITED


Rs. in '000'
Net assets at beginning of the year 350,050
Cash received / receivable on issuance of 765,900 units 85,015
Cash paid / payable on redemption of 717,480 units (77,488)
7,527
357,577
Element of (income) / loss and capital gains included in
prices of units issued less those in units redeemed net (2,685)
Net income for the year (recognized income for the year) 65,325
Net assets at end of the year 420,217

Emile Woolf International 420 The Institute of Chartered Accountants of Pakistan


Answers

Working
Sold Redeemed
No. of Units 765,900 717,480

Rupees in 000
Par value of units @ Rs. 100 76,590 71,748
Sale proceed / redemption value 85,015 77,488
Element of (income) / loss (8,425) 5,740
Net element (2,685)

32.9 A-ONE ASSET MANAGEMENT FUND LIMITED


A-One Asset Management Fund Limited
Statement of movement in unit holders' fund for the year ended March
31, 2016
2016
Rs. in
million
Net assets at the beginning of the year 27,000
Issue of 100 million units 3,500
Redemption of 95 million units (3,277)
223
27,223
Element of income and capital gains included in prices of units
issued / redeemed transferred to income statement (173)
Net unrealized appreciation of re-measurement of investments
classified as available for sale (1,800-1,200-480) 120
Capital gains 400
Net unrealized appreciation on re-measurement of investments
classified as financial assets at fair value through profit or loss
(2,500-2,200) 300
Other net income for the year 3,000
Final distribution for the year ended March 31, 2015 at Rs. 4 per
unit (900*4) (3,600)
100
Net assets at the end of the year 27,270

32.10 IAS 26
(a) The differences between 1AS 26 - Accounting and Reporting by Retirement
Benefit Plan and IAS 19 - Employee Benefits are:
(i) IAS 26 addresses the financial reporting considerations for the benefit
plan itself as the reporting entity while IAS 19 deals with employers
accounting for the cost of such benefits as they are earned by the
employees
(ii) These standards are thus somewhat related, but there will not be any
direct inter-relationship between the amounts reported in benefit plan
financial statements and amounts reported under IAS 19 by
employers.
(iii) IAS 26 differs from IAS 19, Employee Benefits, in allowing a choice of
measurement based either on current salary levels or projected
salary levels. IAS 19 requires an actuarial valuation to be based on
the latter, whereas IAS 26 requires valuation based on present value
of promised retirement benefits.

Emile Woolf International 421 The Institute of Chartered Accountants of Pakistan


Advanced accounting and financial reporting

(b) Defined Benefit Plan (DBP)


Defined benefit plans are retirement benefit plans under which amounts to
be paid as retirement benefits are determined by reference to a formula
usually based on employees earnings and/or years of service.
(c) Defined Contribution Plan (DCP)
Defined contribution plans are retirement benefit plans under which
amounts to be paid as retirement benefits are determined by contributions
to a fund together with investment earnings thereon.
(d) Actuarial present value of promised retirement benefits: This is the present
value of the expected payments by a retirement benefit plan to existing and
past employees attributable to the service already rendered.

32.11 SOGO LIMITED


(a) SOGO Limited
Staff Gratuity Fund
Statement of net assets available for benefits as at December 31, 2016
Note 2016
Rupees
ASSETS
Investments 3 159,033,144
Receivable from SOGO Limited 1,147,150
Cash at bank in current accounts 17,930,120
178,110,414
LIABILITIES
Due to outgoing members 4,301,017
Accrued expenses 3,822
Withholding tax payable 61,251
4,366,090
NET ASSETS 173,744,324
REPRESENTED BY:
Members' Fund
(Rs. 142,472,122 + Rs. 27,712,441) 170,184,563
Surplus on re-measurement of investments available
for sale 3,559,761
173,744,324
(b) SOGO Limited
Staff Gratuity Fund
Statement of changes in net assets available for benefits for the year
ended December 31, 2016
Income
Contribution during the year 10,623,106
Profit from investments 23,389,251
Dividend income 2,696,399
Liabilities no more payable 3,450,000
40,158,756
Expenditure
Transferred / paid to outgoing members (12,432,973)
Bank charges (3,342)
Audit fee (10,000)
(12,446,315)
Net Income for the year 27,712,441

Emile Woolf International 422 The Institute of Chartered Accountants of Pakistan


Profit / Profit /
Balance Fair Principal Balance
Addition interest interest
as at value realized as at
during the accrued realized
July 01, gain / during the June 30,
year during the during the

Emile Woolf International


2015 (loss) year 2016
year year

HELD TO MATURITY

Government Securities
Defense Saving Certificates 87,812,855 - 21,376,809 - (1,600,000) (5,456,000) 102,133,664
Unlisted Securities and
Deposits
-
Term Finance Certificates 19,943,656 5,000,000 1,655,223 (12,873,068) (1,893,722) 11,832,089

423
Term Deposit 6,414,058
-
119,341,142 5,000,000 23,389,251 (19,773,068) (7,577,514) 120,379,811
AVAILABLE FOR SALE
Listed Securities

SUN Ltd. 8,220,957 9,373,936 (784,518) - 16,810,375

PEACE Ltd. 587,169 - 317,728 - 904,897


NIT Units 16,911,510 - 4,026,551 - - 20,938,061

25,719,636 9,373,936 - 3,559,761 - - 38,653,333

145,060,778 14,373,936 23,389,251 3,559,761 (19,773,068) (7,577,514) 159,033,144

The Institute of Chartered Accountants of Pakistan


Answers
Advanced accounting and financial reporting

CHAPTER 35: ETHICAL ISSUES IN FINANCIAL REPORTING

36.1 ETHICAL ISSUES


The range of comments made by Arif raises questions over his ethical behaviour
and professional standards.
A chartered accountant should be unbiased when involved in preparing and
reviewing financial information. A chartered accountant should prepare financial
statements fairly, honestly, and in accordance with relevant professional
standards and must not be influenced by considerations of the impact of reported
results.
Arifs failings
Arif appears to be influenced by the need to achieve a specified level of profit.
This is not appropriate and calls his integrity into question.
In addition Arifs professional competence seems to be suspect. His comment on
not being up to date on all of the little technicalities in IFRS s suggests that he
has not maintained a level of professional competence appropriate to his
professional role.
ICAP members have a responsibility to engage in continuing professional
development in order to ensure that their technical knowledge and professional
skills are kept up to date. Arif should seek continuing professional development
activities and improve his knowledge on ethical standards. Furthermore, it might
be expected that as Waheeds superior he should set an example to Waheed
and guide him in his responsibilities. Clearly this is not happening.
As a member of ICAP Arif should be aware of the ICAP code of ethics. Arif
should know of the danger of self-interest threats and intimidation threats to
himself and to others. His attempt to influence the outcome of a fellow
professional by applying such a threat to that individual is very unprofessional.
Waheeds ethical issues
Waheed faces a self-interest threat, in that there is the possibility of a bonus
provided the earnings per share figure remains the same as last year. Arif has
also suggested that he can influence the Boards decision over employing him as
a replacement finance director another self-interest threat to Waheed. Both of
these threats must be ignored.
Arifs comments imply that his application of professional responsibility is lacking.
This may extend into the way in which the current financial statements have been
prepared. Waheed must be very careful (as always) to carry out the review with
all due care.
Waheed should first discuss his recommendations with Arif and remind him of his
professional responsibilities to ensure that the accounting standards are correctly
followed. If the financial statements are found to contain errors or incorrect
accounting treatment then they must be amended. If Arif refuses to amend the
draft financial statements if necessary Waheed should discuss the matter with
other board members (including non- executives and the audit committee, if
possible). Further action might include consulting with ICAP.

Emile Woolf International 424 The Institute of Chartered Accountants of Pakistan


Answers

36.2 SINDH INDUSTRIES LTD


(a) Financial reporting issues
Revenue
IAS 18 Revenue sets out the rules to be followed in recognising revenue.
The fact that the customer cannot cancel the contract is not relevant to the
recognition of revenue. Revenue from providing a service is recognised
according to the stage of completion subject to satisfying criteria set out in
IAS 18. In the absence of other information the revenue in this contract
should be recognised over the life of the contract as time progresses. As
the contract was only signed just before the year end, none of the revenue
can be recognised in 2016.
The credit for the amount received should be recognised as a liability. This
represents the obligation that the company has to provide the service over
the next two years.
The fact that the customer cannot cancel the contract is not relevant to the
recognition of revenue. If Sindh Industries failed to provide the service they
would be sued for restitution. Therefore the revenue can only be
recognised as the service is provided.
New factory
Borrowing costs directly attributable to construction of an asset which
necessarily takes a substantial period to get ready for its intended use
should be capitalised as part of the cost of that asset under IAS 23
Borrowing Costs. IAS 23 states that the capitalisation of borrowing costs
should commence when three conditions are all met for the first time:
borrowing costs are being incurred, expenditure is being incurred and
activities to prepare the asset are being undertaken. Although borrowing
costs were incurred throughout the year and expenditure was incurred from
1 February 2016 (the date the land was purchased), construction only
started on 1 June 2016. Therefore this is the date on which capitalisation
commences.
Capitalisation ceases when substantially all of the activities required to
make the asset ready for use/sale have been completed, that is on 30
September 2016. (The actual date on which the factory was brought into
use is irrelevant.) Therefore the period of capitalisation should be four
months.
Where construction is financed from general borrowings, the calculation of
the amount to be capitalised should be based on the weighted average cost
of borrowings. This is:
(Rs. 1,000,000 9.75%) + (Rs. 1,750,000 10%) + (Rs. 2,500,000 8%)/
(Rs. 1,000,000 + Rs. 1,750,000 + Rs. 2,500,000) = 9%
Therefore the amount capitalised should be 9% Rs. 4.5 million (land Rs.
1.8 million plus construction costs Rs. 2.7 million) 4/12 = Rs. 135,000.
The total cost of the factory should be measured at Rs. 4,635,000 (Rs. 1.8
million plus Rs. 2.7 million, plus Rs. 135,000). The amount that has been
recognised in the statement of financial position should be reduced by Rs.
315,000 (Rs. 450,000 Rs. 135,000). Finance costs recognised in profit or
loss should be increased by Rs. 315,000.
Land should not be depreciated because it has an indefinite life. Under IAS
16 Property, Plant and Equipment depreciation charges should start when

Emile Woolf International 425 The Institute of Chartered Accountants of Pakistan


Advanced accounting and financial reporting

the asset becomes available for use, from 1 October 2016 in this case.
Depreciation of Rs. 35,000 ((Rs. 2.7 million, plus (Rs. 135,000 2.7/4.5)
20) 3/12) should be recognised in profit or loss for the year ended 31
December 2016 and the carrying amount of the asset reduced by the same
amount to Rs. 4.6 million.
Useful life of the blast furnace
Depreciation of the blast furnace has been based on an estimated useful
life of 20 years. This is at variance with a report by a qualified expert. The
asset valuation specialist treats the furnace as being made up of two
components, the main structure and the lining, which must be replaced at
regular five yearly intervals over the life of the asset. This is the approach
required by IAS 16. The uncertainties inherent in business mean that many
items in financial statements cannot be measured with certainty, but
estimates should always be made using the most up to date and reliable
information. Where estimates have been prepared by professionals with
relevant qualifications, then it is nearly always most appropriate to use
those estimates. Therefore in accordance with the valuers report the main
structure of the furnace should be depreciated over 15 years from 1
January 2016 and the lining should be depreciated over five years from that
date.
The reassessment of the estimated lives of assets is a change in
accounting estimate, rather than a change in accounting policy (IAS 8
Accounting Policies, Changes in Accounting Estimates and Errors).
Changes in accounting estimate should be dealt with on a prospective
basis. This is achieved by including the effect of the change in profit or loss
in current and future periods. The additional depreciation should be
calculated as:

Rs.000

Revised depreciation: main structure 140


((Rs. 3.5m Rs. 1.4m)/15 years)
lining (Rs. 1.4m/5 years) 280

420

Current depreciation (Rs. 3.5m/20 years) (175)

Additional depreciation 245

IAS 8 requires the disclosure of the nature and amount of the effect of the
change in the estimate of useful lives on the profit for the year.
(b) Revised financial statements
Statement of profit or loss extract for the year ended 31 December 2016

Borrowing Blast
Draft Revenue costs furnace Revised
Rs.000 Rs.000 Rs.000 Rs.000 Rs.000
Profit before tax 2,500 (1,000) (315)+ (35) (245) 905

Emile Woolf International 426 The Institute of Chartered Accountants of Pakistan


Answers

Statement of financial position at 31 December 2016


Borrowing Blast
Draft Revenue costs furnace Revised
Rs.000 Rs.000 Rs.000 Rs.000 Rs.000
Non-current
assets
Property, plant
and equipment 12,000 (315) + (35) (245) 11,405
Current assets 3,500 3,500
Total assets 15,500 14,905
Share capital 2,000 2,000
Retained
earnings 6,000 (1,000) (315) + (35) (245) 4,405
Equity 8,000 6,405
Non-current
liabilities 5,000 500 5,500
Current liabilities 2,500 500 3,000
Total equity and
liabilities 15,500 14,905
(c) Ethical issues
It is noticeable that all the adjustments required reduce profit. This and the
background to the previous finance directors resignation suggest serious
problems.
It is not clear who actually prepared the draft financial statements. If they
were prepared by more junior staff in the absence of a finance director,
some of the adjustments (for example, the calculation of borrowing costs to
be capitalised) could be the result of genuine errors or lack of accounting
knowledge. However, it seems reasonably clear that the managing director
has attempted to influence the treatment of the revenue and the estimated
useful life of at least one significant non-current asset. (Note: the directors
have reviewed the useful lives of several items of plant and machinery and
it is possible that other assets besides the furnace are being depreciated
over unrealistically long periods.)
It seems almost certain that the previous finance director resigned as a
result of pressure from the managing director (and possibly from other
members of the Board) to present the financial statements in a favourable
light. The directors intend to seek a stock market listing in the near future.
Therefore they have clear motives for manipulating the profit figure and
also (perhaps) for making controversial decisions before the financial
statements come under much greater scrutiny as a result of the listing. The
job title of financial controller is also significant. It suggests that the role has
been downgraded and that the person holding it has less authority than the
rest of the Board.
Possible courses of action:
Discuss with the managing director the financial reporting standards
that apply to the transactions and explain the implications of non-
compliance. If the managing director is himself a member of a
professional body then it might be worth pointing out to him that he
himself is bound by an ethical code.
Advise him that as a Chartered Accountant you are bound by the
ICAP code of ethics, and that you would not be prepared to

Emile Woolf International 427 The Institute of Chartered Accountants of Pakistan


Advanced accounting and financial reporting

compromise your views of the figures he has prepared for career


advancement.
Consider speaking to the other directors (or audit committee if there is
one) and seeking their support.
If all of these actions produce a negative response then it would be
appropriate to consult the ICAP ethical handbook and/or the Institute.
If all else fails then consider seeking alternative employment.

36.3 SOHAIB AND OMAR


The ICAP Ethical Code sets out fundamental principles which should guide the
behaviour of an accountant in all of his or her professional and business
activities.
The code applies to full members and to student members who required to
conduct themselves to the highest standards in all professional engagements.
The fact that no fee is involved is irrelevant. Omar is calling upon Sohaib's
professional expertise as an accountant, and therefore this is a professional
engagement. The Ethical Code therefore applies.
It is important that accountants are guided not only by the specific terms and
examples provided in the Code, but also by the underlying spirit of the Code.
A key fundamental principle of the Code is that of integrity: a professional
accountant must be straightforward and honest in all professional and business
relationships. Following the spirit of the Code, the source of the financial
information about Gethsemene is questionable, and Sohaib should not produce a
report that uses it without further investigation.
It is important in professional relationships with clients that the chartered
accountant should avoid the threat of familiarity. This does not preclude a
professional relationship with someone who is a friend, but care must be taken
that the friendship does not present a familiarity threat in the context of the work
undertaken.
Sohaib should explain to Omar that he cannot use Gethsemene's draft financial
statements without obtaining permission directly from Gethsemene.
Also he should establish a proper, professional relationship with Omar for the
purposes of this work. This could involve setting out the nature of the work to be
done, and Sohaib's obligations in respect of it, in an engagement letter.
If Sohaib is in need of advice, he could seek it from a more experienced
chartered accountant in his firm (if applicable; no details of Sohaib's current
employment are supplied in the scenario) or he could contact the ICAP to discuss
the matter confidentially.

36.4 ABBAS AND BASHIR


As a chartered accountant, Abbas is expected to follow the guidance set out in
the ICAP's Code of Ethics. This is a principles-based code, and chartered
accountants are expected to be able to identify threats to compliance with the
code's principles, to evaluate the significance of the threats and to implement
safeguards to eliminate the threats or to reduce them to an acceptable level. It
appears that Abbas has identified a potential threat to compliance, but is
currently struggling to evaluate the significance of the threat and to formulate a
plan to address any significant threat.

Emile Woolf International 428 The Institute of Chartered Accountants of Pakistan


Answers

The fundamental principles in the Code of Ethics include objectivity and


confidentiality. It is expected that a chartered accountant should not allow bias,
conflict of interest or undue influence by others to override his or her judgement.
Also, a chartered accountant is expected to keep their business dealings with
clients strictly confidential.
In the current situation, there are possible threats to objectivity and
confidentiality. In his role as a consultant to BigShop Limited and Nourish Limited
who have an interdependent business relationship, Abbas may be party to
confidential and commercially sensitive information about both companies. The
extent to which this is a problem depends upon the nature of the work that Abbas
does for these clients.
If it is limited to the analysis of publicly available information (as in the analysis
conducted in the above report) there is unlikely to be a problem. However, if he is
party to confidential information, as seems likely, a conflict of interest may exist.
Abbas could find it difficult to retain objectivity if he is dealing with both clients,
and he would have to be very careful about confidentiality. The next step for
Abbas is to assess the significance of the threats. It is not possible to determine
conclusively from the information given whether or not these threats exist. Abbas
must do this for himself, but he may be able to obtain advice from other chartered
accountants in his organisation.
Because Insight Ltd employs multi-disciplinary teams of professionals, it is
possible that the firm has experience of these conflicts, and policies to deal with
them. Abbas should make himself familiar with the firm's policies and assess the
extent to which they do, or do not address, the threats he has identified.
In the absence of firm's policies to deal with such issues, Abbas may be obliged
to decline involvement in the work relating to one or other of the clients. He
should discuss the ethical issue with his team leader, Bashir. Because he is not
himself a chartered accountant, he may not be familiar with the Ethical Code
governing Abbas's activities as a chartered accountant, and he may need to
explain it to him.
If Abbas is experiencing difficulties because of an absence of policies within his
firm, or because there is no one available with whom he can discuss ethical
issues, it would be appropriate to call the ICAP's ethics helpline for the
opportunity to discuss the issues and obtain advice.

Emile Woolf International 429 The Institute of Chartered Accountants of Pakistan


Advanced accounting and financial reporting

Emile Woolf International 430 The Institute of Chartered Accountants of Pakistan

Das könnte Ihnen auch gefallen